You are on page 1of 456

Second Year Engineering

Semester – III

According to New Revised Credit System Syllabus of Savitribai


Phule Pune University, Pune.
EFFECTIVE FROM ACADEMIC YEAR JUNE 2016

M.Sc. (Mathematics), M.Phil (Mathematics), M.Sc.(Mathematics ), Ph.D.(Mathematics )


Ph.D (Mathematics), Head and Professor of Mathematics
Assistant Professor Sandip University, Nashik, Maharashtra
Sandip University, Nashik, Maharashtra

M.Sc.(Maths.), Ph.D (Mathematics),. M.Sc. (Maths.)


Professor, Assistant Professor
Sinhgad Institute of Technology & Science, Zeal College Of Engineering and Research,
Nare, Pune. Pune

Download your free e-Book here

Gigatech Publishing House


Igniting Minds
Engineering Mathematics - III
Second Year Engineering (Semester – III)

Dr. Naveen Mani, Dr. S. M. Bhati, Dr. B. S. Waghe, Prof. S. R. Sakhare

First Edition : 2018


Published By : Gigatech Publishing House
631/32, Budhwar Peth, Office No. 105, First Floor,
Shan Bramha Complex, Pune – 411 002.
Phone No. 7757042853

Copyright Gigatech Publishing House


All rights reserved. No part of this publication can be stored in any retrieval system or
reproduced in and form or by any means without prior permission of the Publisher.

LIMITS OF LIABILITY AND DISCLAIMER OF WARRANTY :


The Authors and Publisher of this book have tried their best to ensure that the program,
procedure and function described in the book are correct. However the author and publisher
make no warranty with regard to the program and documentation contained in the book.

ISBN :
Price : `450 /-

Available at All Leading Book Stalls


Distributor
PRADEEP BOOK DISTRIBUTORS
ST
102/3/4 1 Floor, Shan Bramha Complex, Pune.
Telephone No. 020 – 24458333, 942 208 7031 / 844 699 5959
Email : pbd0909@gmail.com

Note : For any queries, doubts, suggestions and complaints regarding the subject, please feel free
to contact on the below e-mail or phone number. We will be more than thankful for your feedback.
Ph: 7757042853 E-mail : info.gigatech1@gmail.com
What is an e-Book?

1. Open any QR Code reader on your phone from the Google Play Store/Apple Store.
2. Hold your device over a QR Code so that it’s clearly visible on your smartphone’s screen.
Two things can happen when you correctly hold your smartphone over a QR Code.
i) The phone automatically scans the code.
ii) On some readers, you have to press a button to snap a picture, not unlike the button on
your smartphone camera. If necessary, press the button.
3. Your smartphone reads the code and navigates to the intended destination, which doesn't
happen instantly. It may take a few seconds on most devices.

Download your free e-Book here


Preface

This Book Engineering Mathematics-III is intended to be a textbook for


students of Second Year Engineering In most sciences, one generation years
down what another has built and what one has established another undoes. In
Engineering Mathematics-III, each generation adds a new story to the old
structure. Keeping this in mind, this book is written to have a better introduction
of the Engineering Mathematics-III. This book is presented with simple but exact
explanation of subject matter, application of each topic to real life, engineering
problems, large number of illustrative examples followed by well graded exercise.
We have tried to be rigorous and precise in presenting the concepts in very
simple manner. We hope that the students will not only learn some powerful
concepts, but also will develop their ability to understand the concept and apply
it properly to solve engineering problems. We feel that faculty member will also
enjoy reading this book which is enriched with application of each topic.

Acknowledgment
The authors acknowledge the help of colleagues and friends for the warm
relationship which provides a source of energy in our endeavours.
We are grateful to our family members for the encouragement and constant
cooperation and assistance in creation of this book.
We are pleased to acknowledge the encouragement from Management and
other authorities during the write up of this book.
We are certainly thankful to the students of engineering who are a constant
source of our enthusiasm and encouragement in our endeavours.
We are also thankful to Gigatech Publishing House TEAM for their
continuous support, hard work and patience in preparing this book.
SYLLABUS

Unit - I : Linear Differential Equations (LDE) and Applications (09 Hrs.)


LDE of nth order with constant coefficients,Method of variation of parameters, Cauchy’s & Legendre’s
DE, Simultaneous & Symmetric simultaneous DE. Modeling of Electrical circuits.

Unit - II : Laplace Transform(LT) (09 Hrs.)


Definition of LT, Inverse LT, Properties & theorems, LT of standard functions, LT of some special
functions viz. Periodic, Unit Step, Unit Impulse. Applications of LT for solving Linear differential
equations.

Unit – lll : Fourier and Z - transforms (09 Hrs.)


Fourier Transform (FT): Complex exponential form of Fourier series, Fourier integral theorem,
Fourier Sine & Cosine integrals, Fourier transform, Fourier Sine and Cosine transforms and their
inverses.
Z - Transform (ZT): Introduction, Definition, Standard properties, ZT of standard sequences and their
inverses. Solution of difference equations.

Unit – IV : Vector Differential Calculus (09 Hrs.)


Physical interpretation of Vector differentiation, Vector differential operator, Gradient, Divergence and
Curl, Directional derivative, Solenoidal, Irrotational and Conservative fields, Scalar potential, Vector
identities.

Unit - V : Vector Integral Calculus and Applications (09 Hrs.)


Line, Surface and Volume integrals, Work-done, Green’s Lemma, Gauss’s Divergence theorem, Stoke’s
theorem. Applications to problems in Electro-magnetic fields.

Unit - VI : Complex Variables (09 Hrs.)


Functions of Complex variables, Analytic functions, Cauchy-Riemann equations, Conformal mapping,
Bilinear transformation, Cauchy’s integral theorem, Cauchy’s integral formula, Laurent’s series and
Residue theorem.
Recommended by SPPU Text Books and Reference Books
Text Books :
1. Erwin Kreyszig, “Advanced Engineering Mathematics”, 9e,(Wiley India).
2. Peter V. O'Neil, “2. Advanced Engineering Mathematics”, 7e,(Cengage Learning).
Reference Books :
1. M. D. Greenberg, “Advanced Engineering Mathematics”, 2e, Pearson Education.
2. Wylie C.R. & Barrett L.C. “Advanced Engineering Mathematics”, McGraw-Hill, Inc.
3. B. S. Grewal, “Higher Engineering Mathematics”, Khanna Publication, Delhi.
4. P. N. Wartikar& J. N. Wartikar, “Applied Mathematics (Volumes I and II)”, Pune
VidyarthiGrihaPrakashan, Pune.
5. B.V. Ramana, “Higher Engineering Mathematics”, Tata McGraw-Hill.
6. Thomas L. Harman, JamesDabney and Norman Richert, “Advanced Engineering
Mathematics with MATLAB”, 2e, Brooks/Cole, Thomson Learning.


LIST OF REQUIRED FORMULAE

1. Natural numbers = N = {1, 2, 3, 4, ….. }


2. Integers = I = {0, ± 1, ± 2, ± 3, ± 4, ….. }
p 
3. Rational numbers = Q =   q ≠ 0 p q  I 
q 

4. Irrational numbers = In = { n
a  where a is a rational numbers }
5. Real numbers = {Q} Ս {Ir}
6. i= –1 , i2 = – 1, i3 = – i, i4 = 1
7. Any equation of nth degree always possesses n roots.
8. Nature of roots of algebraic equations
i) Roots may be real and distinct
ii) Roots may be real and repeated
iii) Roots may be Imaginary and distinct
iv) Roots may be Imaginary and repeated
9. Roots of quadratic equation ax2 + bx + c = 0
-b ± b2 - 4ac
are x = 2a
10. a2 – b2 = (a + b) (a – b)
11. a3 + b3 = (a + b) (a2 – ab + b2)
12. a3 – b3 = (a – b) (a2 + ab + b2)
13. a2 + b2 = (a + ib) (a – ib)
14. x3 = 0  x = 0, 0, 0
15. x3 – ax2 = 0  x2 (x – a) = 0  x = 0, 0, 9
16. (a + b)2 = a2 + 2ab + b2
17. (a – b)2 = a2 – 2ab + b2
18. (a + b)3 = a3 + 3a2b + 3ab2 + b3
19. (a – b)3 = a3 – 3a2b + 3ab2 – b3
1
20. = (1 + p)-1 = 1 – p + p2 – p3 + - -
1+p
1 -1 2 3 4
21. 1 - p = (1 - p) = 1 + p + p + p + p + + +
d
22. Differentiation : where D =
dx
i) D (cosx) = – sinx
ii) D (sinx) = cosx
1 Gigatech Publishing House
Igniting Minds
iii) D (tanx) = sec2x
iv) D (cotx) = – cosec2x
v) D (secx) = secx tanx
vi) D (cosecx) = – cosecx cotx
1
vii) D (tan-1x) =
1 + x2
1
viii) D (cot-1x) = –
1 + x2
1
ix) D (logx) = x

x) d (eax) = a eax
xi) D (uv) = udv + vdy
u  vdu - udv
xii) D
 v  = v2
xiii) D (ax) = ax log a
xiv) D (cosh x) = sinh x
xv) D (sinh x) = cosh x
d
xvi) D [f (x)] = f  (x) dx where D =
dx
23. Integration :
i)  cos hx dx = sinh x + C
ii)  sin hx dx = cosh x + C
emx
iii)  emxdx = +C
m
iv)  sinx dx = -cosx + C
v)  cosx dx = sinx + C
vi)  secx dx = log (secx + tanx) + C
vii)  tanx dx = log secx + C
viii)  cosecx dx = log (cosecx – cotx) + C
ix)  cotx dx = log (sinx) + C
x)  secx tanx dx = secx + C
xi)  cosecx cotx dx = – cosecx + C
xii)  logx dx = (x logx – x) + C
xiii)  uv dx = uv1 – uv2 + uv3 – uv4 + ….
eax
xiv)  eax cos (bx + c) dx = [a cos (bx + c) + b sin (bx + c)]
a + b2
2

2 Gigatech Publishing House


Igniting Minds
eax
xv)  eax sin(bx + c) dx = [a sin (bx + c) – b cos (bx + c)]
a + b2
2

xvi)  cosec2x dx = – cotx + C


xvii)  sec2x dx = tanx + C
1
xviii) D f (x) =  f (x) dx + C
f  (x)
xiv)  f (x) dx = log f (x) + C
[f (x)]n + 1
xv)  [f (x)]n f  (x) dx = n+1 +C
24. Hyperbolic functions / trigonometric functions
ex + e-x ex  ex
i) cosh x = vii) tan hx =
2 e x + e x
ex - e-x [eix  eix]
ii) sinh x = viii) tan x =
2 i [e ix + e ix]
eix + e-ix
iii) cosx =
2
1
iv) sinx = 2i (eix – e-ix)

v) cosh (0) = 1
vi) sinh (0) = 0
25. Binomial expansion :
(a + b)n = an + n c1 an-1 b + n c2 an-2 b2 + n c3 an-3 b3 + + + bn
26. Even and odd functions :
i) Even function if f (– x) = f (x) x
ii) odd function if f(–x) = – f(x) x
iii) (odd function) (odd function) = even function
iv) (even function) (even function) = even function
v) (odd function (even function) = odd function
vi) f(x) = sinx, sin3x, sin5x, ….. odd functions
vii) f(x) = cosx, cos2x, cos3x, cos4x, ….. even functions
viii) f(x) = sin2x, sin4x, sin6x, ….. even functions
a
ix)  f(x) dx = 0, f(x) = odd function
-a
a a
x)  f(x) dx = 2  f(x) dx, f(x) = even function
-a 0

3 Gigatech Publishing House


Igniting Minds
27. Series :
x3 x5
i) sinx = x - + + ….
3 5

x2 x4
ii) cosx = 1 - + + ….
2 4

x3 x5
iii) sinhx = x + + + ….
3 5

x2 x4
iv) coshx = 1 + 2! + 4! + ….

x x2 x3
v) ex = 1 + + + + +
1 2 3

x x2 x3
vi) e-x = 1 - + - + + +….
1 2 3

x2 x3 x4
vii) log (1 + x) = x - + - + -
2 3 4
x2 x3 x4 x5
viii) log (1 - x) = - x - 2 - 3 - 4 - 5 -

a
ix) Geometric series, a + ar + ar2 + ar3 + + + arn – 1 + +  = , |r|<1
1-r
x) If x is a real number, |x| = x, x ≥ 0
= -x, x < 0
xi) |x| < a  -a < x < a
xii) |x| > a  - < x < -a and a < x < 
28. Trigonometry :
i) sin2x = 2sinx cosx
1 + cos2x 1 - cos2x
ii) cos2x = 2 sin2 x = 2
iii) 2 cosA cosB = cos (A + B) + cos (A – B)
iv) 2 sinA sinB = cos (A – B) – cos (A + B)
v) 2 sinA cosB = sin (A + B) + sin (A – B)
vi) sin (A + B) = sinA cosB + cosA sinB
vii) sin (A – B) = sinA cosB – cosA sinB
viii) cos (A – B) = cosA cosB + sinA sinB
ix) cos (A + B) = cosA cosB – sinA sinB
x) sin (0) = 0, cos (0) = 1, tan (0) = 0
xi) sin (-x) = -sinx

4 Gigatech Publishing House


Igniting Minds
xii) cos (-x) = cosx
xiii) sin2 + cos2 = 1
xiv) sec2 = 1 + tan2
xv) cosec2 = 1 + cot2
29. Geometry :
i) y = 0, X – axis
ii) x = 0, Y – axis
iii) x = a, a line parallel to x = 0
iv) y = b, a line parallel to y = 0
where a, b are some constants.
v) y = mx is a equation of line through (0,0)
vi) A line through (x1,y1) and (x2, y2 ) is
y2 - y1 
y – y1 = 
 x2 - x1  (x – x1)
vii) x2 + y2 = a2 is a circle whose centre (0, 0) radius a units
viii) (x – h)2 + (y – k)2 = r2, a circle with centre (h, k) radius r units
ix) y2 = 4ax, y2 = – 4ax, x2 = 4ay, x2 = – 4ay represents parabolas through (0,0)
x) ax + by + c = 0, represents a general line
x2 y2
xi) a2 + b2 = 1, a ellipse
30. Complex numbers :
i) |z| = 1  x2 + y2 = 1, a circle with centre (0,0) rad = 1
ii) |z + 1| = 1  (x + 1)2 + (y – 0)2 = 12 , a circle with centre (-1,0), rad = 1
iii) |z + i| = 1  (x – 0)2 + (y + 1)2 = 12, a circle with centre (0, -1), rad = 1
iv) z = x + iy, cartesion form
v) |z| = x2 + y2

vi) z = x – iy
vii) z = rei, polar form
viii) ei = cos + i sin
ix) e-i = cos - i sin
x) |ei| = 1
xi) |e-i| = 1
xii) |z| < a, is an interior of circle |z| = a
xiii) |z| ≥ a, is an exterior of circle |z| = a

5 Gigatech Publishing House


Igniting Minds
31. Proper and Partial Fractions :
P P
i) If in a fraction Q , degree of Q > degree of P, then Q is called proper fraction otherwise
improper fractions.
ii) Proper fraction can be resolved into partial fractions as below
P A B
a) = +
(x - m) (x + n) x - m x + n
P A B C D E F
b) (x - m)2 (x + n) (x + r)3 = x - m + (x - m)2 + x + n + x + r + (x + r)2 + (x + r)3
P A Bx + C
c) = + 2
(x + n) (x2 + 1) x + n x +1
32. Factorial : for n  N, set of natural numbers
i) n = n! = n (n – 1) (n – 2) ……. 3  2  1 = 1 2  3  4 …. (n – 2) (n – 1) n
ii) 0! = 1
iii) n c1 = 1
n (n - 1)
iv) n c2 =
2!
n(n - 1) (n - 2)
v) n c3 = 3!
n!
vi) n cr =
r! (n - r)!



6 Gigatech Publishing House


Igniting Minds
CONTENTS

Unit : I Linear Differential Equations (LDE) and Applications........... 1.1 – 1.53


1.1 Introduction
1.2 Linear Differential Equations
1.3 The Operator D
1.3.1 Auxiliary Equation (AE)
1.4 Complementary Function (CF)
1.4.1 Rules for Finding CF
1.4.2 Formulas for Factorization
1.5 Particular Integral (PI)
1.6 Complete Solution (CS)
1.7 Methods for Finding Particular Integral
1.7.1 Shortcut Method for Finding Particular Integral
1.7.2 General Method for Finding PI
1.7.3 Method of Variation of Parameter for Finding PI
1.8 Cauchy’s Homogeneous Linear Differential Equation
1.9 Legendre’s Linear Differential Equations
1.10 System of Simultaneous Differential Equations
1.11 Symmetrical Simultaneous Differential Equations
1.12 Modeling of Electrical Circuits
1.12.1 Kirchhoff’s Voltage Law

Unit : II Laplace Transform (LT) ................................................................ 2.1 – 2.88


2.1 Introduction
2.2 Preliminaries
2.2.1 Piecewise Continuous Function
2.2.2 Functions of Exponential Order
2.3 Laplace Transforms
2.3.1 Definition
2.3.2 Sufficient Conditions or Existence
2.4 Linearity of Laplace Transform
2.5 Laplace Transforms of Some Basic Functions
2.6 First Shifting Property (Multiplication by eat)
2.7 Change of Scale Property
2.8 Second Shifting Property
2.9 Laplace Transforms of Derivatives
2.10 Laplace Transforms of Integrals
2.11 Laplace Transforms of Function Multiplied by tn
2.12 Laplace Transforms of Function Divided by t
2.13 The Convolution Theorem (without proof)
2.13.1 Definition
2.13.2 Theorem on Laplace Transform : (without proof)
2.14 Initial Value Theorem (without poof)
2.15 Evaluation of Integrals of Special Kind using Laplace Transforms
2.16 Laplace Transform of Special Function
2.17 Inverse Laplace Transform
2.18 Inverse Laplace transform of Basic Function
2.19 Partial Fraction
2.20 Inverse Laplace Transform using Partial Fraction
2.21 Other Methods of Findings Inverse Laplace Transform
2.23 Applications of Laplace Transforms
2.24 Solution of Initial Value Problems
2.25 Solution of Boundary Value Problem
2.26 Solution of Integro differential Equations

Unit : III Fourier and Z Transforms ............................................................. 3.1 – 3.64


3.1 Fourier Transform
3.1.1 Introduction
3.1.2 Integral Transform
3.1.3 Fourier Transform and Inverse Fourier Transform
3.2 Prerequisites
3.2.1 Sequence
3.2.2 Linearly Property
3.3 Modulus of a Real Number
3.4 Complex Number
3.5 Series
3.6 Geometric Series
3.7 Z – Transform
3.8 Z– Transforms of Standard Sequences
3.9 Theorems on Z – Transforms
3.9.1 Theorem 1 : Change of Scale
3.9.2 Theorem 2 : Multiplication by k
3.9.3 Theorem 3 : Recurrence Formula
3.9.4 Theorem 4 : Shifting Property
3.10 Inverse Z Transforms
3.10.1 Inverse Z–T by Using Standard Formulae
3.10.2 Method of Partial Fractions
3.10.3 Residue Method
3.10.4 Residue Theorem
3.10.5 Working Procedure of Residue Theorem
3.11 Difference Equations

Unit : IV Vector Differential Calculus ......................................................... 4.1 – 4.35


4.1 Physical Interpretation of Vector Differentiations
4.2 The Vector Differential Operator Del. (∇)
4.3 Gradient
4.4 Directional Derivative
4.5 Divergence and Curl
4.5.1 Divergence of a Vector Field
4.5.2 Physical Interpretation of Divergence
4.5.3 Curl of a Vector Field
4.5.4 Physical Interpretation of Curl
4.5.5 Properties of Divergence and Curl
4.6 Irrotational and Solenoidal Fields
4.7 Scalar Potential

Unit : V Vector Integral Calculus and Applications ................................ 5.1 – 5.52


5.1 Line Integral
5.2 Green’s Theorem
5.3 Surface Integral
5.4 Stoke’s Theorem
5.5 Gauss Divergence Theorem
5.7 Electromagnetic Fields (Maxwell’s Equations)
5.7.1 Gauss’s Law of Electric flux
5.7.2 Gauss’s Law for Magnetic Fields
5.7.3 Electromagnetic Induction (Faraday’s Law)
5.7.4 Magnetic Field Due to a Current : Ampere’s Law

Unit : VI Complex Variables........................................................................... 6.1 – 6.88


6.1 Introduction
6.1.1 Prerequisites
6.1.2 Complex Functions / Complex Variables / Function of
Complex Variables
6.1.3 Single Valued Function / Multivalued Function
6.2 Analytic Functions
6.3 Cauchy Riemann (C–R) Equations : (Cartesian Form )
6.4 Cauchy Riemann (C–R) Equations : (Polar Form )
6.5 Argands Diagram
6.6 Transformation
6.6.1 Conformal Transformation
6.6.2 Some Standard Transformations
6.7 Bilinear Transformation /Mobius Transformation
6.8 Complex Integration
6.8.1 Line Integral or Complex Integral
6.9 Cauchy′s Integral Theorem
6.10 Cauchy′s Integral Formula
6.11 Residue Theorem
6.12 Application of Cauchy′s Residue Theorem, to evaluate the

integrals, over the circles, of the type ∫ f(sinθ, cosθ) dθ :
0
6.13 Laurent’s Series

Solved University Question Papers


Dec. 2016 ............................................................................................ Q.1 – Q.15
May 2017 ............................................................................................ Q.1 – Q.26
May 2018 ............................................................................................ Q.1 – Q.18
♦♦♦
Unit
Linear Differential
1 Equations (LDE) and
Applications

Syllabus :
LDE of nth order with constant coefficients, Method of variation of parameters, Cauchy’s &
Legendre’s DE, Simultaneous & Symmetric simultaneous DE. Modeling of Electrical
circuits.

An equation containing derivatives or differential is called differential equation.


Differential equations arise from many problems such as mass spring system, electrical
circuits, heat conduction, chemical reactions. The higher order differential equations appear in
problems involving series of electrical circuits containing several loops, mechanical spring
system. The differential equations can be used to explain and predict new facts about every
thing that changes continuously. In this chapter we study solution of linear differential
equations of second and higher order.

Linear differential equation of the n th order is given by


dny dn-1y dn-2y d2y dy
n + P1 n-1 + P2 n-2 + – – – – – – – +Pn–2 + Pn–1 + Pny= F(x)
dx dx dx dx2 dx
where P1, P2, P3, – – – – – ,Pn–2, Pn–1, Pn, F(x) are functions of x alone.
d3y d2y 2 dy
e. g. dx 3 + 4x
dx2 + x dx + y = sinx
Linear differential equation with constant coefficients of the nth order is given by
dny dn-1y dn-2y d2y dy
n + a1 n-1 + a2 n-2 + – – – – – – – +an–2 + an–1 +an y = F(x) …(1)
dx dx dx dx2 dx

Gigatech Publishing House


Igniting Minds
Engineering Mathematics - III 1.2 Linear Differential Equations (LDE.....

where a0, a1, a2, –––––––, an–2, an–1, an are constants and F(x) is a function of x alone or
constant.
d3y d2y dy
e.g. 4 dx3 3 dx2 + dx + 7 y = ex

d
The operator D stands for dx
d
i.e. D 
dx
dy
Hence, Dy =
dx
2 3
dy dy d n - 1y dny
D2y = dx2 , D3y = dx3 , Dn – 1y = dxn - 1 , Dny = dxn

Using above in equation (1.1),


We get,
Dny + a1 Dn –1 y + a2 Dn –2 y + ------------ + an –2 D2y + an – 1Dy + any = F(x)
 (Dn + a1 Dn –1y + a2 Dn – 2 + ------------ + an –2D2 + an–1 D + an) y = F(x)
Take Dn + a1 Dn –1 + a2 Dn – 2 + ------------ + an –2D2 + an–1 D + an = (D)
So that, we get
 (D) y = F(x)
which is standard form of LDE with CC.
d2y dy
e.g dx2 + 5 dx + 6 y = cos x
become D2y + 5Dy + 6y = cos x
(D2 + 5D + 6) y = cos x
which has type  (D) y = F(x)
where  (D) = D2 + 5D + 6
F (x) = cos x
Thus every linear differential equation with constant coefficients (LDE with CC) can be
put in the form
d
 (D) y = F(x) with D
dx

d
For LDE with CC of type  (D) y = F(x) with D  , auxiliary equation (AE) is given
dx
by  (D) = 0.
For (D2 + 5D + 6) y = cos x,
AE is D2 + 5D + 6 = 0
Gigatech Publishing House
Igniting Minds
Engineering Mathematics - III 1.3 Linear Differential Equations (LDE.....

d
For LDE with CC of type  (D) y = 0 with D  dx , solution is given as
y = G (x)
The function G (x) is called complementary function. The complementary function
depends on nature of root of the auxiliary equation (AE).

Case (I) : If roots of the AE are real and distinct (say m1, m2, m3, – – – – , mn) then
CF = c1em1x + c2em2x+ c3em3x + – – – – + cnemnx
where c1, c2, c3, – – – –, cn are arbitrary constants.
Case (II) : If two roots of the AE are real and repeated (say m1, m1, m3, – – – – , mn) then
CF = (c1 + c2x) em1x + c3 em3x + c4 em4x + – – – + cn emnx
If roots are m1, m1, m1, m4 then,
CF = (c1 + c2x + c3x2 ) em1x + c4 em4x
If roots are m1, m1, m3, m3 then,
CF = (c1 + c2 x) em1x + (c3 + c4 x ) em3x
Case (III) : If roots are distinct and imaginary say   i (where  and  are real numbers and
i is imaginary unit, i = -1 ), then
x
CF = e [c1 cos x + c2 sin x]
Case (IV) : If roots are repeated imaginary say   i and   i then
CF = ex [(c1 + c2 x) cos x + (c3 + c4 x) sin x]

1. a2 – b 2 = (a – b) (a + b)
2. a 2 + b2 = (a – ib) (a + ib)
3. a3 – b 3 = (a – b) (a2 + ab + b2)
4. a 3 + b3 = (a + b) (a2 – ab + b2)
5. Roots of the equation
aD2 + bD + c = 0 are
– b  b2 – 4ac
D =
2a

Illustrative Examples

Example : 1

d2y dy
Find complementary function of +9 + 14 y = e3x
dx2 dx

Gigatech Publishing House


Igniting Minds
Engineering Mathematics - III 1.4 Linear Differential Equations (LDE.....

d
Solution : Use D  in given (LDE with CC)
dx
D2y + 9 Dy + 14 y = e–3x
(D2 + 9D + 14) y = e–3x
which has type  (D) y = F(x)
AE is  (D) = 0
2
D + 9D + 14 = 0
(D + 7) (D + 2) = 0
D = 7 , 2
CF = c1 e 7x + c2 e 2x or
CF = c1 e 2x + c2 e 7x
Example : 2
d2y
Find complementary function of dx2 – 9y = sin 3x
d
Solution : Use D  dx
D2y – 9y = sin 3x
(D2 – 9) y = sin 3x
AE is  (D) = 0
D2 – 9 = 0
D2 – (3)2 = 0
(D – 3) (D + 3) = 0
D = 3, – 3
CF = c1e3x + c2e3x
Example : 3
d2y
Find complementary function of dx2 + 9y = sin 3x
d
Solution : Use D 
dx
D2y + 9y = sin 3x
(D2 + 9) y = sin 3x
AE is D2 + 9 = 0
D + (3)2
2
= 0
(D – 3i) (D + 3i) = 0
D =  3i
= 0  3i
CF = e0x [c1 cos3x+ c2 sin3x]
CF = c1 cos3x+ c2 sin3x

Gigatech Publishing House


Igniting Minds
Engineering Mathematics - III 1.5 Linear Differential Equations (LDE.....

Example : 4

d2y dy
Solve dx2  dx + 6y = 0
Solution :
D2y – Dy + 6y = 0
(D2 – D + 6) y = 0
which has type  (D) y = 0
So its solution is y = CF
AE is  (D) = 0
D2 – D + 6 = 0
(D – 3) (D + 2) = 0
D = 3, – 2
CF = c1 e3x + c2 e2x
So solution is y = c1 e3x + c2 e2x
Example : 5

d
Solve (D3 – 9D) y = 0, where D 
dx
Solution :
AE is D3 – 9D = 0
D (D2 –9) = 0
D (D – 3) (D + 3) = 0
D = 0, 3, 3
CF = c1e0x + c2e3x + c3e2x
Solution is y = CF
y = c1 + c2e3x + c3e2x
Example : 6

d3y d2y dy
Find complementary function of 3+4 + 2 = xe–x
dx dx2 dx
Solution :
D3y + 4D2y + 4Dy = xe–x
(D3 + 4D2 + 4D)y = xe–x
AE is D3 + 4D2 + 4D = 0
D (D2 + 4D + 4) = 0
D (D + 2)2 = 0
D = 0, 2, 2
CF = c1e0x + (c2 + c3 x) e–2x
CF = c1 + (c2 + c3x) e–2x
Gigatech Publishing House
Igniting Minds
Engineering Mathematics - III 1.6 Linear Differential Equations (LDE.....

Example : 7

Find complementary function of (D2 + 2D + 5) y = sin2x


Solution :
AE is D2 + 2D + 5 = 0
2  (2)2 - (4) (1) (5)
D = (2)(1)
2  16
= 2
2  4i
= 2
= 1  2i
CF = e–x [c1 cos2x + c2 sin 2x]

Example : 8

Solve (D3 – 5D2 + 8D – 4) y = 0


Solution :
AE is D3 – 5D2 + 8D – 4 = 0
(D – 1) (D – 2) (D – 2) = 0
D = 1, 2, 2
CF = c1ex + (c2 + c3x) e2x
Solution is y = c1ex + c2 + c3x) e2x

Self-Assessment Exercise 1.1

A) Find complementary function of following differential equations :


d2y
Ex.1 : dx2 – y = e3x Ans. : c1ex + c2e–x

Ex.2 : (D2 + D) y = sin3x Ans. : c1 + c2e–x


Ex.3 : (D2 + 3D + 2) y = xe–x Ans. : c1e–2x + c2e–x
Ex.4 : (D2 + 6D + 8) y = 0 Ans. : c1e–4x + c2e–2x
Ex.5 : (D – 1)3 y = cos3x Ans. : (c1 + c2x + c2x2)ex
x
–   3   3 
Ex.6 : (D3 + 1) y = sinx Ans. : e 2  c1cos  2 x + c2sin 2 x
    
Ex.7 : (D5 – D) y = e4x Ans. : c1 + c2ex + c3 e–x + c4cosx + c4sinx
Ex.8 : (D3 – D2 + 3D + 5) y = 4 Ans. : c1e–x + ex [c2cos2x + c3sin3x

Gigatech Publishing House


Igniting Minds
Engineering Mathematics - III 1.7 Linear Differential Equations (LDE.....

B) Solve the following equations :


d2y dy
Ex.1 dx2 + 5 dx + 4y = 0 Ans. : y = c1e–x + c2e–4x

  7   7 
Ex.2 (D2 – D – 2) y = 0 Ans. : y = ex/2  c1cos  x + c sin x
  2  2  2 
Ex.3 (D2 + 16) y = 0 Ans. : y = c1cos4x + c2 sin4x
Ex.4 (D2 + 6D + 9) y = 0 Ans. : y = (c1 + c2x) e–3x
Ex.5 (D2 + 6D + 10) y = 0 Ans. : y = e–3x (c1 cosx + c2 sinx)
Ex.6 (D3 – D2 + 3D + 5) y = 0 Ans. : y = c1e–x + ex (c2 cos2x + c3 sin2x)
d4y
Ex.7 dx4 – y = 0 Ans. : y = c1ex + c2e–x + c3cosx + c4sinx
Ex.8 (D4 + 8D2 + 16) y = 0 Ans. : y = (c1 + c2x) cos2x + (c3 + c4x) sin2x

For linear differential equation with constant coefficients of the type  (D)y = F(x) with
d
D  dx , particular integral (PI) is defined as
1
PI = F(x)
 (D)
e.g. For (D2 + 5D +6) y = e–4x
1
PI = D2 +5D +6 e–4x

Note : If a and b are any constants then


1 1 1
[a F(x) + b G(x)] = a F(x) + b G(x)
 (D)  (D)  (D)

For differential equation of type  (D)y = F(x)


Complete solution (CS) is given by,
y = CF + PI

There are three methods :


1. Short cut method
2. General method
3. Method of variation of parameters.

d
Consider the differential equation of type  (D) y = F(x) with D  dx . Apply short cut
method to find PI if F(x) has the following type or types.

Gigatech Publishing House


Igniting Minds
Engineering Mathematics - III 1.8 Linear Differential Equations (LDE.....

Type I :
F (x) = eax
where a is any number or constant.
1
PI = F(x)
 (D)
1
= eax
 (D)
1
= eax if  (a)  0
 (a)
If  (a) = 0, then above formula fails and we use the following formula.
1
PI = x eax , if (a)  0
 (a)
If (a) = 0, then above formula fails and we use the following formula.
1
PI = x2 eax , if (a)  0
 (a)
If (a) = 0, then above formula fails and we use the following formula.
1
PI = x3 eax , if (a)  0
 (a)
and so on.
Type II :
F (x) = sin (ax + b) or cos (ax + b)
where a and b are any constant.
Consider F(x) = sin (ax + b)
1
PI = F(x)
 (D)
1
= sin (ax + b)
 (D, D2)
1
= sin (ax + b), if  (D, –a2)  0
 (D, -a2)
If  (D, –a2) = 0, then above formula fails and we use the following formula.
1
PI = x sin (ax + b)
(D, D2)
1
= x sin (ax + b)
(D, -a2)
if  (D , –a2)  0 and so on.
Note : That formula for F(x) = cos (ax + b) is same as that for F(x) = sin (ax + b) with sin
replace by cos.
Type III :
F (x) = sinh (ax + b) or cosh (ax + b)
Gigatech Publishing House
Igniting Minds
Engineering Mathematics - III 1.9 Linear Differential Equations (LDE.....

Formulas for F (x) = sinh (ax + b) are same as that for


F (x) = sin (ax + b) with change of sin by sinh and –a2 by a2
Formulas for F (x) = cosh (ax + b) are same as that for
F (x) = cos (ax + b) with change of cos by cosh and –a2 by a2
Type IV : F (x) = xm
where m = 0. 1, 2, 3, 4, ––––––––––––
1 1
In this type write as , by taking the lowest degree term out. Use the formulas.
(D) 1 G(D)
1
= 1 + A + A2 + A3 + ––––––
1A
1
= 1 – A + A2 – A3 + –––––
1+A
Neglecting higher powers of D, we can get PI.
Type V : F (x) = eax V
V is any function of x and a is any non zero number.
In this type, we use the following formula
1 1
eaxV = eax  V
 (D)  (D + a) 
Type VI :
F (x) = xV
Here V is any function of x.
In this type we use the following formula
1 1  1
xV = x  (D) V
 (D)  (D)  (D)

Illustrative Examples

Example : 1

d2y dy
Solve dx2+3
dx + 2y = e–4x.

Solution :
d
Use D 
dx
D2y + 3Dy + 2y = e–4x
(D2 + 3D + 2) y = e–4x
Which has type  (D) y = F(x)
So C.S is
y = CF + PI

Gigatech Publishing House


Igniting Minds
Engineering Mathematics - III 1.10 Linear Differential Equations (LDE.....

AE is  (D) = 0
D2 + 3D + 2 = 0
(D + 2) (D + 1) = 0
D = 2, 1
CF = c1e2x + c2ex
1
PI = F(x)
 (D)
1
= e4x
D2 + 3D + 2
1
= e4x
(4)2 + (3) (4) + 2
1
= e4x
16  12 + 2
1 4x
PI = e
6
So C.S is
1
y = c1e2x + c2e–x + e4x
6

Example : 2
2
Solve (D2 1) y = (1 + e–x)
Solution :
(D2 1) y = 1 + 2e–x + e–2x
AE is  (D) = 0
D2 1 = 0
(D  1) (D + 1) = 0
D = 1, 1
CF = c1ex + c2e–x
1
PI = F(x)
 (D)
1
= 2 [1 + 2e–x + e–2x]
D 1
1
= 2 [e0x + 2e–x + e–2x]
D 1
1 1 1
= e0x + 2 2 ex + 2 e2x
D2 1 D 1 D 1
1 1 1
= e0x + 2 ex + e2x
(0)2  1 (1)2  1 (2)2  1

Gigatech Publishing House


Igniting Minds
Engineering Mathematics - III 1.11 Linear Differential Equations (LDE.....

1 0x 1 1
= e + 2 ex + e2x
01 0 4-1
1 1 –x 1 –2x
= 1 + 2x e + e
-1 2D 3
1 1
= –1 + 2x e–x + e–2x
2 (-1) 3
1
PI = –1 – xe–x + 3 e–2x

C.S, is
y = CF + PI
1 –2x
y = c1 ex + c2 e–x – 1 – x e–x + e
3

Example : 3

7
Solve (D – 1)3 = 3x + 2

Solution :
Type of DE is
 (D)y = F(x)
AE is  (D) = 0
3
(D – 1) = 0
D = 1, 1, 1
CF = (c1 + c2x + c3x2) ex
1
PI = F(x)
 (D)
1  x 7
= 3 3 +
(D  1)  2
1 exlog3 + 7 e0x
=
(D  1)3  2 
1 7 1
= exlog3 + e0x
(D  1)3 2 (D  1)3
1 7 1
= 3x – 2 e0x
(log 3  1)3 (0  1)3
1 7
PI = 3x – 2
(log 3  1)3
C.S is, y = CF + PI
1 7
y = (c1 + c2x + c3x2) ex + 3x –
(log 3  1)3 2

Gigatech Publishing House


Igniting Minds
Engineering Mathematics - III 1.12 Linear Differential Equations (LDE.....

Example : 4

Solve (D2 + 3D + 2)y = sin 3x


Solution :
AE is  (D) = 0
2
D + 3D + 2 = 0
(D + 2) (D + 1) = 0
D = –2, –1
CF = c1e–2x + c2 e–x
1
PI = F(x)
 (D)
1
= D2 + 3D + 2 sin 3x
1
= sin 3x
-9 + 3D + 2
1
= sin 3x
3D  7
(3D + 7)
= (3D - 7)(3D + 7) sin 3x
3D + 7
= sin 3x
9D2  49
3D + 7
= (9)(-9) - 49 sin 3x
-1
= 130 (3D + 7) sin 3x
1
= – 130 (3D sin3x + 7 sin 3x)
1
= – (3  3 cos3x + 7 sin 3x)
130
1
PI = – 130 (9 cos3x + 7 sin 3x)

C.S is y = CF + PI
1
y = c1e–2x + c2 e–x – (9 cos3x + 7 sin 3x)
130

Example : 5

Solve (D2 + 8D + 16)y = cos 4x


Solution :
AE is D2 + 8D + 16 = 0
(D + 4)2 = 0
Gigatech Publishing House
Igniting Minds
Engineering Mathematics - III 1.13 Linear Differential Equations (LDE.....

D = –4, –4
CF = (c1 + c2x) e–4x
1
PI = F(x)
 (D)
1
= D2 + 8D + 16 cos 4x
1
= -16 + 8D + 16 cos 4x
1
= cos 4x
8D
1
=
8
 cos 4x dx
1 sin 4x
= 8 4
1
PI = 32 sin 4x
C.S. is
y = CF + PI
1
y = (c1 + c2x) e–4x + sin 4x
32

Example : 6

Solve (D3 + D) y = cosx


Solution :
AE is D3 + D = 0
D (D2 + 1) = 0
D (D – i) (D + i) = 0
D = 0,  i
CF = c1 + c2 cosx + c3 sinx
1
PI = F(x)
 (D)
1
= D3 + D cosx
1
= D2 D + D cosx
1
= cosx
(-1) D + D
1
= 0 cosx

Gigatech Publishing House


Igniting Minds
Engineering Mathematics - III 1.14 Linear Differential Equations (LDE.....

1
= x 2 cosx
3D + 1
1
= x (3)(-1) + 1 cosx
1
PI = -2 x cosx
C.S. is y = CF + PI
1
y = c1 + c2 cosx + c3 sinx – x cosx
2
Example : 7
Solve (D3 + 2D2 + D) y = sin2x
Solution :
AE is
D3 + 2D2 + D = 0
D (D2 + 2D + 1) = 0
D (D + 1)2 = 0
D = 0, –1, –1
CF = c1 + (c2 + c3x) e–x
1
PI = sin2x
D3 + 2D2 + D
1 1 - 1 cos2x
=
D3 + 2D2 + D 2 2 
1 1 0x 1 1
= 2 D3 + 2D2 + D e – 2 D3 + 2D2 + D cos2x
1 1 0x 1 1
= 2 0 + 0 + 0 e – 2 - 4D - 4 + Dcos2x
1 1 0x 1 1
= 2 x 3D2 + 4D + 1 e + 2 3D + 4 cos2x
1 1 1 (3D - 4)
= x e0x – cos2x
2 0+0+1 2 (3D + 4) (3D - 4)
1 1 3D - 4
= 2 x + 2 9D2 - 16 cos2x
1 1 3D - 4
= 2 x + 2 (9) (-4) - 16 cos2x
1 1 1
= 2 x + 2 -52 (3D – 4) cos2x
1 1
PI = x– (–6 sin2x – 4 cos2x)
2 104
C.S. is
1 1
y = c1 + (c2 + c3x) e–x + x– (–6 sin2x – 4 cos 2x)
2 104
Gigatech Publishing House
Igniting Minds
Engineering Mathematics - III 1.15 Linear Differential Equations (LDE.....

Example : 8

Solve (D3 – 9D) y = 2 cosh 3x


Solution :
AE is
D3 – 9D = 0
D (D – 3)(D + 3) = 0
D = 0, 3, –3
CF = c1 + c2 e3x + c3 e–3x
1
PI = F(x)
 (D)
1
= D3 - 9D 2 cosh 3x
1
= 2 2 cosh 3x
D D - 9D
1
= 2 9D - 9D cosh 3x
1
= 2 0 cosh 3x
1
= 2x 3D2 - 9 cosh 3x
1
= 2x cosh 3x
(3) (9)9
1
= 2x cosh 3x
18
1
PI = x cosh 3x
9
1
y = c1 + c2 e3x + c3 e–3x + x cosh 3x
9

Example : 9

Solve (D2 – 1) y = ex + x3
Solution :
D2 – 1 = 0
(D – 1) (D + 1) = 0
D = 1, –1
CF = c1ex + c2e–x
1 x 3
PI = D2 - 1 [e + x ]

Gigatech Publishing House


Igniting Minds
Engineering Mathematics - III 1.16 Linear Differential Equations (LDE.....

1 1
= 2 ex + 2 x3
D -1 D -1
1 x 1 3
= 1 - 1 e + (-1) (-D2 + 1) x
1 x 1 3
= 0 e – 1 - D2 x
1 2 3
= x 2D ex – [1 + (D2) + (D2) + (D2) + ––––––] x3
1
= x ex – [1 + D2 + D4 + D6 + –––––––] x3
2
1
= x 2 ex – [1 + D2] x3
( D4x3 = 0)
1 x 3 2 3
PI = 2 x e – [x + D x ]
1 x 3
PI = 2 x e – [x + 6x]
Solution is
1
y = c1ex + c2e–x + x ex – [x3 + 6x]
2
Example : 10

Solve (D2 + 2D + 2) y = x3 – 4x
Solution :
D2 + 2D + 2 = 0
-2  (2)2 - (4) (1) (2)
D = 2 (1)
D = –1  i
CF = e–x [c1cosx + c2sinx]
1
PI = D2 + 2D + 2 (x3 – 4x)
1 1
= (x3 – 4x)
2 D2 + 2D
1+ 2
1  D2 + 2D D2 + 2D2 D2 + 2D3
PI = 1- + - + - - - -  (x3 – 4x)
2  2   2   2  
Now,
D (x3 – 4x) = 3x2 – 4
D2 (x3 – 4x) = 6x
D3 (x3 – 4x) = 6
D4 (x3 – 4x) = 0
Gigatech Publishing House
Igniting Minds
Engineering Mathematics - III 1.17 Linear Differential Equations (LDE.....
4 5 6
So we neglect terms containing D , D , D and higher powers of D
1 1 1 1
PI = 1 - (D2 + 2D) + (D4 + 4D3 + 4D2) - (D6 + 6D5 + 12D4 + 8D3) + --- (x3 – 4x)
2 2 4 8 
1 1 2 1 1
PI = 2 1 - 2 (D + 2D) + 4 (4D3 + 4D2) - 8 (8D3) (x3 – 4x)
 
1 1 2 3
= 2 1 - D + 2 D (x – 4x)
 
1 3 1
= (x - 4x) - D(x3 - 4x) + D2 (x3 - 4x)
2 2 
1 1
= 2 (x3 - 4x) - (3x2 - 4) + 2 (6x)
 
1 3
PI = 2 [x – 3x2 – x + 4]
1
y = e–x [c1cosx + c2sinx] + [x3 – 3x2 – x + 4]
2
Example : 11
Solve (D2 – 4D + 4) y = e2x sin 3x
Solution :
AE is
D2 – 4D + 4 = 0
(D – 2)2 = 0
D = 2, 2
CF = (c1 + c2x) e2x
1
PI = F(x)
 (D)
1
= e2x sin3x
(D  2)2
1
= e4x  V
  (D + a) 
1
= e2x  sin 3x
  (D + 2) 
1
= e2x  sin 3x
 (D + 2  2)2 
1
= e2x  D2 sin 3x
 
1
= e2x  sin 3x
9 
1 2x
PI = – e sin3x
9
1
y = (c1 + c2x) e2x – e2x sin3x
9

Gigatech Publishing House


Igniting Minds
Engineering Mathematics - III 1.18 Linear Differential Equations (LDE.....

Example : 12

1
Solve (D2 + 6D + 9) y = x3 e–3x

Solution :
D2 + 6D + 9 = 0
(D + 3)2 = 0
D = –3, –3
CF = (c1 + c2x) e–3x
1 1 –3x
PI = D2 + 6D + 9 x3 e
1 1
= e–3x 3
(D + 3)2 x
1 1
= e–3x (D - 3 +3)2 x3
 
1 1
= e–3x D2 x3
 
1 1
= e–3x D D x-3
 
1
= e–3x D  x-3 dx
 
-3 + 1
1 x
= e–3x D -3 + 1
 
-2
1 x
= e–3x  
D 2 
x2 
= e–3x  dx
 2 
1 x2 + 1
= e–3x
2  2 + 1
1
PI = e–3x 2x
e-3x
y = (c1 + c2x) e–3x +
2x

Example : 13

d2y
Solve dx2 + 4y = x sinx

Solution :
(D2 + 4) y = x sinx
AE is

Gigatech Publishing House


Igniting Minds
Engineering Mathematics - III 1.19 Linear Differential Equations (LDE.....

D2 + 4 = 0
(D – 2i) (D + 2i) = 0
D =  2i
CF = c1 cos2x + c2 sin2x
1
PI = F(x)
 (D)

= x - (D) 1  1 V
 (D)  (D)

= x - 2D 21  21 sinx
 D + 4 D + 4

= x - 2D 21  1 sinx
 D + 4 - 1 + 4
1 1 
= 3 x sinx - 2D D2 + 4 sinx
1 1
= x sinx - 2D sinx
3 -1 + 4 
1 2
PI = x sinx - cosx
3 3 
1 2
y = c1 cos2x + c2 sin2x + x sinx - cosx
3 3 
Example : 14

Solve (D2 + 1) y = x coshx


Solution :
D2 + 1 = 0
(D – i) (D + i) = 0
D = i
CF = c1 cos x + c2 sinx
1
PI = x coshx
 (D)

= x - (D) 1  1 V
 (D)  (D)

= x - 2D 21  21 coshx
 D + 1 D + 1

= x - 2D 21  1 coshx
 D + 1 1 + 1
1 1 
= 2 x coshx - 2D D2 + 1 coshx
1 1 
= 2 x coshx - 2D 1 + 1 coshx
Gigatech Publishing House
Igniting Minds
Engineering Mathematics - III 1.20 Linear Differential Equations (LDE.....

1
2[
PI = x coshx - sinhx]
1
y = c1 cos x + c2 sinx + [x coshx - sinhx]
3

Example : 15

Solve (D2 – 2D + 1) y = x ex cosx


Solution :
D2 – 2D + 1 = 0
(D – 1)2 = 0
D = 1, 1
CF = (c1 + c2x) ex
1
PI = (D - 1)2 ex (x cosx)
1
= ex (D + 1 - 1)2 xcosx
 
1
= ex 2 x cosx
D
1  1
= ex x - (D) V
 (D)  (D)
where  (D) = D2,  (D) = 2 D, V = cosx
1 1
PI = ex x - 2D cosx
 D2  D2
1 1
= ex x - 2D D2  -1 cosx
 
1 1
= -1 ex x cosx - 2 D cosx
 
PI = – ex [x cosx – 2 sinx]
Solution is
y = CF + PI
y = (c1 + c2x) ex + – ex [x cosx – 2 sinx]

Self Assessment Exercise 1.2

Solve the following differential equations :


d2y dy e2x
Ex. 1 dx2 – 7dx + 6y = e2x Ans. : y = c1e6x + c2ex – 4
x4 x
Ex.2 (D3 – 4D2 + 4D) y = e2x + ex + 1 Ans. : y = c1 + (c2 + c3x)e2x + 4 e2x + ex + 4

Gigatech Publishing House


Igniting Minds
Engineering Mathematics - III 1.21 Linear Differential Equations (LDE.....
2
dy 2 x e2x
Ex.3 – 4y = (1 + ex)2 + 3 Ans. : y = c1e2x + c2e–2x – 1 – e –
dx2 3 4
1
Ex.4 (D2 – 3D + 2) y = sinx Ans. : y = c1e2x + c2ex + 10 (sinx + 3cosx

Ex.5 (D2 – 2D + 5) y = 10 sinx Ans. : y = ex (c1 cosx + c2sinx)+ 2 sinx + cosx


1 x
Ex.6 (D2 + 1) y = cos2x cosx Ans. : y = c1 cosx + c2 sinx – 16 cos3x + 4 sinx

Hint : 2 cosA cosB = cos(A + B) + cos(A – B)


1 x
Ex.7 (D2 + 1) y = sin3x cos2x Ans. : y = c1 cosx + c2 sinx – 48 sin5x – 4 cosx
1 x
Ex.8 (D2 + 1) y = sinx sin2x Ans. : y = c1 cosx + c2 sinx + 16 cos3x + 4 sinx
x
Ex.9 (D3 – 4D) y = 2 cosh2x Ans. : y = c1 + c2 e2x + c3 e–2x + 4 cosh 2x
2
Ex.10 (D2 – 2D + 5) y = 25x2 Ans. : y = ex (c1 cos2x + c2 sin2x + 5x2 + 4x –
5
  3   3 
Ex.11 (D2 – D + 1) y = x3 – 3x2 + 1 Ans.: y = ex/2 c1 cos  x + c2 sin  x 
 2   2 
+ x3 – 6x – 5
Ex.12 (D2 – 4D + 3) y = x3 e2x Ans.: y = c1 ex + c2 e3x – e2x [x3 + 6x]
8
Ex.13 (D3 – 3D2 + 3D – 1) y = x1/2 ex Ans.: y = (c1 + c2 x + c2x2) ex + 105 ex x7/2
d2y dy e-x
Ex.14 +3 + 2y = e–x sin 2x Ans. : y = e–x + c2e–2x – [2 cos2x + 4 sin2x]
dx2 dx 20
1
Ex.15 (D2 – 2D + 1) y = x cosx Ans.: y = (c1 + c2 x) e–x + 2 (x sinx + cosx – sinx
d2y 1 ex
Ex.16 dx2 – y = x sinx + (1 + x2) ex Ans. : y = c1ex + c2e–x – 2 [x sinx + cosx] + 2
3 2
x – x + 3x
3 2 2 
x
e
Ex.17 (D2 – 2D + 1) y = xex sin 2x Ans.: y = (c1 + c2 x) ex – 4 (x sin2x + cos2x)
x2ex x3 3x2
Ex.18 (D3 – D2) y = 3x + xex Ans. : y = c1 + c2 x + c2 ex – 2xex + 2 – 2 – 2
d2y ex x2 1
Ex.19 dx2 + 4y = ex + x2 Ans. : y = c1 cos2x + c2 sin2x + 5 + 4 – 8
1 2
Ex.20 (D2 – 4) y = xsinhx Ans. : y = c1 e2x + c2 e–2x – [ xsinhx + coshx]
3 3

Gigatech Publishing House


Igniting Minds
Engineering Mathematics - III 1.22 Linear Differential Equations (LDE.....

Consider differential equation of type  (D) y = F(x). In general method, we use the
following formula.
1
F(x) = ex  e–x F(x) dx
D–
for any number .
Note : 1. If  = 0 then we have
1
D F(x) =  F(x) dx
2. If short cut method fails, then we may try general method.

Illustrative Examples

Example : 1
x
Solve (D2 + 3D + 2) y = ee
Solution :
AE is
D2 + 3D + 2 = 0
(D + 2) (D + 1) = 0
D = –2, –1
CF = c1e–2x + c2e–x
1
PI = F(x)
 (D)
1 ex
= (D + 2) (D + 1) e
1  1
ee 
x
=
D+2D+1 
1
=
D+2
[e-x  ex eexdx]
Put ex = t
 ex dx = dt in integral
1
D + 2 [e  e dt]
-x t
PI =
1
= [e–x et]
D+2
1 –x ex
= D+2e e
x
= e–2x  e2x e–x ee dx

Gigatech Publishing House


Igniting Minds
Engineering Mathematics - III 1.23 Linear Differential Equations (LDE.....
x
= e–2x  ex ee dx
= e–2x  et dt
= e–2x et
x
PI = e–2x ee
C.S. is y = CF + PI
x
y = c1e–2x + c2e–x + e–2x ee

Example : 2

Solve (D2 – 1) y = e–x sin e–x + cos e–x


Solution :
AE is
D2 – 1 = 0
(D – 1) (D + 1) = 0
D = 1, –1
CF = c1ex + c2e–x
1
PI = F(x)
 (D)
1
= (D – 1) (D + 1) (e–x sin e–x + cos e–x)·
1
=
(D – 1)
[e-x  ex (e-x sin e-x + cos e-x) dx]
1
= (D – 1) [e-x  ex (cos e-x + e-x sin e-x) dx]
1
PI = (D – 1) [e-x ex cos e-x]
x
··· e [f(x) + f (x)] dx = ex f(x)
1 –x
PI = (D – 1) cos e
= ex  e–x cos e–x dx
–x
Put e = t
–x
– e dx = dt
e–x dx = – dt
PI = ex  cost (– dt)
= – ex sint
PI = – ex sin e–x
C.S is y = CF + PI
y = c1ex + c2e–x – ex sin e–x
Gigatech Publishing House
Igniting Minds
Engineering Mathematics - III 1.24 Linear Differential Equations (LDE.....

Example : 3

d2y dy 1
Solve dx2 + dx = 1 + ex
Solution :
1
(D2 + D) = 1 + ex
AE is
D2 + D = 0
D = 0, –1
CF = c1 + c2 e–x
1 1
PI = D (D + 1) 1 + ex
1 1 1
= D (D + 1) 1 + ex
1  -x x 1 
= D e  e 1 + ex dx
1 –x
= [e log (1 + ex)]
D
PI =  [log(1 + ex)] [e–x] dx
By using integration by parts
ex
PI = [log(1 + ex)] [– e–x] –  1 + ex [–e–x] dx
 
1
= – e–x log (1 + ex) +  dx
1 + ex
(1 + ex) – ex
= – e–x log (1 + ex) +  dx
1 + ex
ex
= – e–x log (1 + ex) +  1 – 1 + ex dx
 
PI = – e–x log (1 + ex) + [x – log (1 + ex) ]
C.S is y = CF + PI
y = c1 + c2 e–x – e–x log (1 + ex) + [x – log (1 + ex)]

Self Assessment Exercise 1.3


Solve the following differential equations :
Ex.1 (D2 + 3D + 2) y = sin ex Ans. : y = c1 e–2x + c2 e–x – e–2x sin ex)
x x
Ex.2 (D2 + 3D + 2) y = ee + cos ex Ans. : y = c1 e–2x + c2 e–x + e–2x (ee – cos ex)
ex 1
Ex.3 (D2 – 1) y = ex + 1 Ans. : y = c1 ex + c2 e–x + 2 [(e–x + ex) log (ex + 1)

+ xex – e–x – 1]
Gigatech Publishing House
Igniting Minds
Engineering Mathematics - III 1.25 Linear Differential Equations (LDE.....

d
Consider differential equation of second order of type  (D) y = F(x) with D  dx . In
this case of CF will be of the form CF = c1y1 + c2y2, where y1 and y2 are some functions of x.
We define Wronskian ‘W’ as;
 y1 y2 
W =  
  
 y1 y2
By method of variation of parameters, PI can be given by the following formula
PI = u y1 + v y2
where u and v are function of x. We have to find value of u and v by using the following
formulas.
y2 F(x)
u = –  W dx
y1 F(x)
v =  W dx

Illustrative Examples

Example : 1

Solve (D2 + 4) y = tan 2x by the method of variation of parameters.


Solution :
AE is
D2 + 4 = 0
(D – 2i) (D + 2i) = 0
D =  2i
CF = c1 cos2x + c2 sin2x
which is of form CF = c1y1 + c2y2
So, y1 = cos 2x , y2 = sin2x
y1 = –2 sin2x , y2 = 2 cos2x
 y1 y2 
W =  
  
 y1 y2
cos2x sin 2x 
= 
 –2 sin 2x 2 cos2x
= 2 cos22x + 2 sin22x

Gigatech Publishing House


Igniting Minds
Engineering Mathematics - III 1.26 Linear Differential Equations (LDE.....

= 2 (cos22x + sin22x)
W = 2
By method of variation of parameters
PI = uy1 + vy2
y2 F(x)
u = – dx
W
sin2x tan2x
= – 2 dx
1 sin2x
= – 2  sin2x cos2x dx
1 sin22x
= – 2  cos2x dx
1 1 – cos22x
= –  dx
2 cos2x
1
= – 2  [sec2x – cos2x] dx
1 1 1
= – 2 2 log (sec2x + tan2x) – 2 sin2x
 
1
= 4 [sin 2x – log (sec 2x + tan 2x)]
y1 F(x)
V =  W dx
cos2x tan2x
=  dx
2
1 sin2x
= 2  cos2x cos2x dx
1
= 2  sin2x dx
1 – cos2x
= 2  2 
 
1
V = – cos2x
4
1 1
PI = 4 [sin2x – log(sec2x + tan2x)] cos2x – 4 cos2x sin2x
1
PI = – 4 cos2x log (sec2x + tan2x)

Complete solution (CS) is


y = CF + PI
1
y = c1 cos2x + c2 sin2x – cos2x log (sec2x + tan2x)
4

Gigatech Publishing House


Igniting Minds
Engineering Mathematics - III 1.27 Linear Differential Equations (LDE.....

Example : 2

d2y dy
Solve dx2 – 4 dx + 4y = e2x sec2x by the method of variation of parameters.
Solution :
(D2 – 4D + 4) y = e2x sec2x
AE is  (D) = 0
2
D – 4D + 4 = 0
(D – 2)2 = 0
D = 2, 2
CF = (C1 + C2x) e2x
which has the form
CF = C1y1 + C2y2
So that,
y1 = e2x , y2 = xe2x

y = 2e2x , y = e2x + 2xe2x
1 2
y y
  1 2 2x
 e 2x 2x xe 2x 
2x
W =    =  2e e + 2xe 
 y 1 y2 
W = 2x e4x + e4x – 2xe4x
W = e4x
By the method of variation of parameters
PI = uy1 + vy2
y2 F(x)
u = –  W dx
xe2x e2x sec2x
= – e4x dx

= –  x sec2x dx
= – [(x) (tanx) – (1) (log secx)]
 by integration by parts
u = log secx – x tanx
y F(x)
v =  1 W dx
e2x e2x sec2x
=  e4x
dx

=  sec2x dx
v = tanx
PI = (log secx – xtanx) e2x + (tanx) (x e2x)
Gigatech Publishing House
Igniting Minds
Engineering Mathematics - III 1.28 Linear Differential Equations (LDE.....
2x
PI = e log secx
C.S. is y = CF + PI
y = (c1 + c2x) e2x + e2x log (secx)

Example : 3

Solve (D2 + 4) y = sec2x space by the method of variation of parameters.


Solution :
AE is
D2 + 4 = 0
(D – 2i ) (D + 2i) = 0
 D = 2i
CF = c1 cos2x + c2 sin2x
CF = c1y1 + c2y2
 y1 = cos2x , y2 = sin2x
 
y = – 2sin2x , y = 2 cos2x
1 2
y y
 1 2 
W =  
y1 y

2

=  cos2x sin2x 
 – 2sin2x 2 cos2x
= 2
By method of variation of parameters.
PI = uy1 + vy2
y2 F(x)
u = –  W dx
sin2x sec2x
= – 2 dx
1
= –  tan2x dx
2
11
u = – log sec2x
22
y1 F(x)
v =  W dx
cos2x sec2x
=  2 dx
1
=  2 dx

Gigatech Publishing House


Igniting Minds
Engineering Mathematics - III 1.29 Linear Differential Equations (LDE.....

x
v =
2
1 x
PI = – 4 cos2x log sec 2x + 2 sin2x

C.S. is y = CF + PI
1 x
y = c1 cos2x + c2 sin2x – cos2x log sec 2x + sin2x
4 2

Example : 4

d2y dy
Solve dx2 – 2 dx + 2y = ex tanx by the method of variation of parameters.

Solution :
(D2 – 2D + 2) y = ex tanx
AE is D2 – 2D + 2 = 0
2 4-8
D = 2
D = 1i
CF = ex (c1cosx + c2 sinx)
= c1y1 + c2y2
y1 = ex cosx , y2 = ex sinx
y y
 1 2 
W =  
y1 y

2
x x
=  x e cosxx e sinx 
 e cosx – e sinx ex sinx + ex cosx
= e2x cos2x + e2x sinx cosx + e2x sin2x – e2x sinx cosx
W = e2x
PI = uy1 + vy2
y2 F(x)
where u = –  W dx
exsinx ex tanx
= – e2x dx
sin2x
= – dx
cosx
1 – cos2x
= – dx
cosx
= –  (secx – cosx) dx
u = – log (secx – tanx) + sinx

Gigatech Publishing House


Igniting Minds
Engineering Mathematics - III 1.30 Linear Differential Equations (LDE.....

y1 F(x)
v =  W
dx
excosx extanx
=  e2x dx

=  sinx dx
v = – cosx
PI = – log (secx + tanx) excosx + sinx ex cosx
+ (– cosx) ex sinx
PI = – ex cosx log (secx + tanx)
Complete solution is
y = CF + PI
y = ex (c1cosx + c2 sinx) – ex cosx log (secx + tanx)

Self Assessment Exercise 1.4

Solve the following differential equations by using the method of variation of parameters.
d2y 2
Ex.1 dx2 – y = 1 + ex Ans. : y = c1ex + c2e–x + (ex – e–x) log(1 + ex)– xex – 1

Ex.2 (D2 + 1) y = cosec x Ans. : y = c1 cosx + c2 sinx – x cosx + sinx log(sinx)


e3x
Ex.3 (D2 – 6D + 9) y = x2 Ans. : y = (c1 + c2x) e3x – e3x (1 + logx)
1
Ex. 4 (D2 + 9) y =
1 + sin3x
1 1
Ans. : y = (c1 cos3x + c2 sin3x) + 9 (–1 + sin 3x – 3x cos3x) + 9 sin3x log(1 + sin3x))
d2y
Ex.5 dx2 + y = secx tanx Ans. : y = c1cosx + c2sinx + xcosx – sinx + sinx log (secx)


An equation of type
dny dn-1y d2y dy
a0 xn dxn + a1 xn–1 dxn-1 + – – – – – – – + an–2 x2 dx2 + an–1 x dx + any = F(x)

where a0, a1, –––––––, an are constants, is called Cauchys homogeneous linear differential of
nth order.
d3y d2y dy
e.g 1. x3 dx3 – 4x2 dx2 + x dx + 7y = x6
d2y dy
2. x2dx2 + 4x dx + 7y = cos (logx)

Above types of equations can be converted into LDE with CC by using the following
substitutions.

Gigatech Publishing House


Igniting Minds
Engineering Mathematics - III 1.31 Linear Differential Equations (LDE.....

x = ez
 log x = z
d
Use D  dz
dy
So that, x dx = Dy
d2y
x2 = D (D – 1) y
dx2
d3y
x3 dx3 = D (D – 1) (D – 2) y

Illustrative Examples

Example : 1

d3y d2y 1
Solve : x3 dx3 + x2 dx2 – 2y = x + x2

Solution :
Put, x = ez
 log x = z
d
Use D 
dz
dy
So that, x dx = Dy
d2y
x2 dx2 = D (D – 1) y
d3y
x3 dx3 = D (D – 1) (D – 2) y

Given equation 
1
D (D – 1) (D – 2) y + D (D – 1) y – 2y = ez + e2z

 (D3 – 2D2 + D – 2) y = ez + e–2z


AE is D3 – 2D2 + D – 2 = 0
D2 (D – 2) + (D – 2) = 0
(D – 2) (D2 + 1) = 0
(D – 2) (D – i) (D + i) = 0
D = 2, i
CF = c1 e2z + c2 cosz + c3 sinz
1
PI = (ez + e–2z)
D3 – 2D2 + D – 2

Gigatech Publishing House


Igniting Minds
Engineering Mathematics - III 1.32 Linear Differential Equations (LDE.....

1
= 3 2 ez
D – 2D + D – 2
1
+ D3 – 2D2 + D – 2 e–2z
1 z
= 1–2+1 –2e
1
+ e–2z
(–2)3 – 2 (–2)2 + (–2) – 2
1 z 1 –2z
PI = e – e
–2 4
C.S. is y = CF + PI
1 1
y = c1 e2z + c2 cosz + c3 sinz – 2 ez – 4 e–2z
1 1
 y = c1 x2 + c2 cos (logx) + c3 sin (logx) – x –
2 4x2

Example : 2
d2y dy
Solve : x2 dx2 – 3x dx + 5y = x2 sin (logx)

Solution :
Given is Cauchys DE
Put x = ez
 log x = z
d
Use D  dz
dy
So that, x dx = Dy
d2y
x2 = D (D – 1) y
dx2
2
D (D – 1) y – 3D y + 5y = (ez) sinz
(D2 – 4D + 5) y = e2z sinz
 AE is D2 – 4D + 5 = 0
4  16 – 20
D =
2
4  2i
= 2
D = 2i
CF = e2z (c1 cosz + c2 sinz)
1 2z
PI = D2 – 4D + 5 e sinz
Gigatech Publishing House
Igniting Minds
Engineering Mathematics - III 1.33 Linear Differential Equations (LDE.....

1
= e2z 2 sinz
(D + 2) – 4 (D + 2) + 5
1
= e2z D2 + 1 sinz
1
= e2z z 2D sinz
z
= e2z  sinz dz
2
1
PI = – z e2z cosz
2
C.S. is y = CF + PI
ze2z
y = e2z (c1 cosz + c2 sinz) – 2 cosz
x2
y = x2 (c1 cos logx + c2 sin logx) – logx cos logx
2

Self Assessment Exercise 1.5

Solve the following differential equations by using the method of variation of parameters.
d2y dy 1
Ex.1 x2 dx2 – 3xdx + 3y = x2 sin logx Ans. : y = c1x + c2x3 – 2 x2 sin logx
d2y dy c2
Ex.2 x2 2 + x – y = 2 sinh logx Ans. : y = c1x + + logx cosh logx
dx dx x
d3y d2y 1
Ex.3 x3 3 + 2x2 2 + 2y = 10x + 
dx dx  x
c1 2
Ans. : y = x + x [c2 cos logx + c3 sin logx]+ 5x + x logx
d2y dy 1
Ex.4 x3 dx2 + 3x2 dx + xy = sin logx Ans. : y = x [c1 + c2 logx – sin logx]


An equation of type
d2y dy
a0 (ax + b)2 2 + a1 (ax + b) + a2y = F(x)
dx dx
where a , b, a0, a1, a2 are constants, is called Legendres LDE of second order.
d2y dy
e.g. (3x + 1)2 dx2 + 4 (3x + 1) dx – 3y = 6x + 1

Above type of equations can be converted into LDE with CC by using the following
substitutions
Gigatech Publishing House
Igniting Minds
Engineering Mathematics - III 1.34 Linear Differential Equations (LDE.....

ax + b = ez
 log (ax + b) = z
d
Use D  dz
dy
So that, (ax + b) dx = aDy
d2y
and (ax + b)2 = a2 D (D – 1) y
dx2

Illustrative Examples

Example : 1
d2y dy
Solve : (2x + 3)2 dx2 + (2x + 3) dx – 2y = 24 x2

Solution :
Put 2x + 3 = ez
log (2x + 3) = z
d
Use D  dz
dy
(2x + 3) dx = 2 Dy
d2y
(2x + 3)2 dx2 = (2)2 D (D – 1) y

Given equation 
ez – 3
4D (D – 1) y + 2Dy – 2y = 24  2 
 
ez – 3
·· · x = 2
(4D2 – 2D – 2) y = 6 e2z – 36 ez + 54
(2D2 – D – 1) y = 3 e2z – 18 ez + 27
AE is 2D2 – D – 1 = 0
2D2 – 2D + D – 1 = 0
2D (D – 1) + (D – 1) = 0
(D – 1) (2D + 1) = 0
–1
D = 1,
2
1
– z
CF = c1ez + c2 e 2
1
PI = (3e2z – 18ez + 27)
2D2 – D – 1

Gigatech Publishing House


Igniting Minds
Engineering Mathematics - III 1.35 Linear Differential Equations (LDE.....

1 1 1
= 2 3e2z – 2 18ez + 2 27e0z
2D – D – 1 2D – D – 1 2D – D – 1
1 2z 1 z 1 0z
= 2(2)2 – 2 – 1 3e – 2 – 1 – 1 18e + 0 – 0 – 1 27e
3 2z 1 z 1
= 5 e – z 4D – 1 18e + – 1 27
3 2z 1
= e –z 18ez – 27
5 4–1
3 2z
PI = e – 6z ez – 27
5
C.S. is y = CF + PI
1
–2 z 3
y = c1 ez + c2 e + 5 e2z – 6z ez – 27
1
– 3
 y = c1 (2x + 3) + c2 (2x + 3) 2
+ (2x + 3)2 – 6 (2x + 3) log (2x + 3) – 27
5

Example : 2
d2y dy
Solve : (2x + 1)2 dx2 + 2 (2x + 1) dx + 4y = 4 sin[2log(2x + 1)]

Solution :
Put (2x + 1) = ez
 log (2x + 1) = z
d
Use D  dz
dy
So that (2x + 1) dx = 2 Dy
2
dy
And (2x + 1)2 dx2 = (2)2 D (D – 1) y

Given equation 
4D (D – 1) y + 2 (2 Dy) + 4y = 4 sin2z
2
(4D + 4) y = 4 sin2z
2
(D + 1) y = sin2z
2
AE is, D +1 = 0
 D = i
CF = c1 cosz + c2 sinz
1
PI = sin2z
D2 + 1
1
= sin2z
–4+1

Gigatech Publishing House


Igniting Minds
Engineering Mathematics - III 1.36 Linear Differential Equations (LDE.....

1
PI = sin2z
–3
C.S. is y = CF + PI
1
y = c1 cosz + c2 sinz – sin2z
3
1
y = c1 cos [log (2x + 1)] + C2 sin [log (2x + 1)] – sin [2log (2x + 1)]
3

Self Assessment Exercise 1.6

Solve the following differential equations by using the method of variation of parameters.
d2y dy
Ex.1 (2x + 1)2 2 – 2 (2x + 1) – 12y = 6x
dx dx
3 1
Ans. : y = c1 (2x + 1)3 + c2 (2x + 1)–1 – 16 (2x + 1) + 4 )

d2y dy
Ex.2 (x + 2)2 dx2 + 3 (x + 2) dx + y = 4 sin log (x + 2)

1
Ans. : y = [c + c2 log (x + 2)] – 2 cos [log (x + 2)]
x+2 1
d2y dy
Ex.3 (1 + x)2 + (1 + x) + y = 2 sin log (x + 1)
dx2 dx
Ans. : y = c1 cos log(1 + x) + c2 sin log (1 + x) – log (1 + x) cos log (1 + x)
2
dy dy 3x + 2a
Ex.4 (x + a)2 dx2 – 4 (x + a) dx + 6y = x Ans. : y = c1 (x + a)3+ c2 (x + a)2 + 6

In this section we find the solution of a system of differential equations which involve
two or more dependent variables and one independent variable.
Consider system :
1 (D) x + 2 (D) y = F1 (t)
3 (D) x + 4 (D) y = F2 (t)
d
where D  dt
The method of solving above system consists of eliminating one of the variable x or y,
solving the resulting equation and then substituting the value of that variable in any of the
equation. The complete solution contains two independent equations.

Gigatech Publishing House


Igniting Minds
Engineering Mathematics - III 1.37 Linear Differential Equations (LDE.....

Illustrative Examples

Example : 1
dx dy
Solve : dt + 4x + 3y = t, dt + 2x + 5y = et

Solution :
d
Writing D for dt , given equation become

(D + 4) x + 3y = t …(1)
t
2x + (D + 5) y = e …(2)
To eliminate y, operating on both sides of (1) by (D + 5) and on both sides of (2) by 3
and subtracting, we get
[(D + 4) (D + 5) – 6] x = (D + 5) t – 3t
(D2 + 9D + 14) x = 1 + 5t – 3et
AE is D2 + 9D + 14 = 0
(D + 7) (D + 2) = 0
D = – 7, – 2
CF = c1 e–2t + c2 e–7t
1
PI = (1 + 5t – 3et)
D2 + 9D + 14
1 1 1
= D2 + 9D + 14 e0t + 5 D2 + 9D + 14 t – 3 D2 + 9D + 14 et
1 1 1
= 0 + 0 + 14 e0t + 5 9D D2 t–3
1 + 9 + 14 e
t

14 1 + 14 + 14 
 
2
1 5  9D D 1
= + 1 –  +  + ----- t – et
14 14   14 14  8
1 5  9 1
= + t –  – et
14 14  14  8
5 31 1
PI = 14 t – 196 – 8 et

x = CF + PI
5 31 et
x = c1 e–2t + c2 e–7t + 14 t – 196 – 8 …(3)
t
dx 5 e
= – 2 c1 e–2t – 7 c2 e–7t + –
dt 14 8
dx
Put value of x and dt in (1), we get

Gigatech Publishing House


Igniting Minds
Engineering Mathematics - III 1.38 Linear Differential Equations (LDE.....

dx
3y = t– – 4x
dt
5 et
=t + 2 c1 e–2t + 7 c2 e–7t – 14 + 8 + 4 c1 e–2t
20 31 1
+ 4 c2 e–7t + 14 t + 49 + 2 et
1 3 27 5 t
y = – 2c1 e–2t + 3c2 e–7t – t + + e …(4)
3 7 98 8 
The equation (3) and (4) together gives solution for given system.

Example : 2
dx dy
Solve the system : dt – wy = a cospt, dt + wx = a sinpt

Solution :
d
Writing D for , we get,
dt
Dx – wy = a cospt …(1)
wx + Dy = a sinpt …(2)
To eliminate, (1)  D + (2)  w
D2x – w Dy = – ap sinpt
w2x + w Dy = aw sinpt
(D2 + w2) x = a (w – p) sinpt
AE is D2 + w2 = 0
(D – iw) (D + iw ) = 0
D =  iw
CF = c1 cos wt + c2 sin wt
1
PI = D2 + w2 a (w – p) sinpt
1
= – p2 + w2 a (w – p) sinpt
a (w – p)
= (w – p)(w + p) sinpt
a
PI = sinpt
w+p
C.S. is x = CF + PI
a
x = c1 coswt + c2 sinwt + w + p sinpt …(3)

From (1) wy = D x – a cospt


a
= D  c1 coswt + c2 sinwt + sinpt – a cospt
 w+p 
Gigatech Publishing House
Igniting Minds
Engineering Mathematics - III 1.39 Linear Differential Equations (LDE.....

ap
= – w c1 sinwt + w c2 coswt + cospt – a cospt
w+p
a
 y = – c1 sinwt + c2 coswt – cospt …(4)
w+p
Equation (3) and (4) provide solution for given system.

Example : 3

The current x and y in the coupled circuits are given by


dx dy
L dt + R x + R (x – y) = E ; L dt + Ry – R (x – y) = 0. Find x and y if x = y = 0 at t = 0.

Solution :
d
Use D  dt
L Dx + Rx + Rx – Ry = E
L Dy + Ry – Rx + Ry = 0
 (LD + 2R) x – Ry = E …(1)
– Rx + (LD + 2R) y = 0 …(2)
To eliminate y, (1)  (LD + 2R) + (2)  R
 (LD + 2R)2 x – R (LD + 2R) y = 2ER
2
– R x + R (LD + 2R) y = 0
2 2
[(LD + 2R) – R ] x = 2ER
2 2 2
(L D + 4 L R D + 3R ) x = 2ER
So C.S. is x = CF + PI
2 2 2
AE is L D +4LRD+3R = 0
– 4 L R  16 L2 R2–4L2 3R2
D = 2 L2
– 4 L R  2 LR
= 2 L2
– R – 3R
= L , L
Rt 3Rt
–L –
CF = c1 e + c2 e L
1
PI = 2 ERe0t
L2D2 + 4 L R D + 3R2
1 0t
= 0 + 0 + 3R2 2 ERe
2E
PI = 3R
Gigatech Publishing House
Igniting Minds
Engineering Mathematics - III 1.40 Linear Differential Equations (LDE.....
Rt 3Rt
– L – L 2E
C.S is x = c1 e + c2 e + 3R …(3)

From (1), Ry = (LD + 2R) x – E


Ry = L Dx + 2 Rx – E
Rt 3Rt
 R – 3R – 
= L – c 1 e L – c 2 e L + 0
 L L 
Rt 3Rt
 – – 2E 
+ 2R e L + c2 e L + 3R  – E
 

Rt 3Rt
– L – L E
 Ry = R c1 e – R c2 e +3
Rt 3Rt
– L – L E
 y = c1 e – c2 e + 3R …(4)

Given that x = 0 when t = 0 and y = 0 when t = 0 so (3) and (4) 


2E
0 = c1 + c2 + 3R
E
And 0 = c1 – c2 + 3R
–E –E
 c1 = 2R and c2 = 6R
– E – RtL E – 3Rt 2E
So (3) and (4)  x = e – e L +
2R 6R 3R
– E – Rt E – 3Rt E
y = e L+ e L +
2R 6R 3R

Self-Assessment Exercise 1.7

Solve the following differential equations by using the method of variation of parameters.
du dv
Ex.1 dx + v = sinx, dx + u = cosx Ans. : u = c1 ex + c2 e–x & v = sinx – c1ex + c2e–x
dx dy
Ex.2 dt + y = et, dt + x = e–t
1 1
Ans. : x = c1 cost + c2 sint + 2 (et – e–t)y = c1 sint – c2 cost + 2 (et – e–t)
dx
Ex.3 Solve for x and y if 4 =y–x
dt
t t
1 dy
Ans. : y = c1 + c2 e 4 , x = c1 + 2 c2 e 4 = 2 dt given x = 20, y = 100 when t = 0
dx dy
Ex.4 dt + wy = 0, dt – wx = 0 Ans. : x = C1 coswt + C2 sinwt, y = C1 sinwt – C2 coswt

Gigatech Publishing House


Igniting Minds
Engineering Mathematics - III 1.41 Linear Differential Equations (LDE.....

A symmetrical simultaneous differential equation is given by


dx dy dz
= =
P Q R
where P, Q, R are functions of x, y, z or constant.
The solution of such differential equation contains two independent relations.
There are two methods for solving such differential equation.
1. Method of Grouping :
dx dz
Observe if it is possible to take two ratios, P = R from which y can be eliminated or it
is absent leaving equation in x and z alone.
If so, integrate it we get ,
 (x , z) = c1 …(1)
Again set if one variable say x is absent or can be eliminated may be with the help of
dy dz
(1), from the equation Q = R.
Then integrate it, we get
 (y,z) = c2 …(2)
(1) and (2) taken together constitute complete solution.
2. Method of Multipliers :
By a proper selection of multipliers l, m, n (which are not necessarily constants) we
write
dx dy dz ldx + mdy + ndz
= = =
P Q R lP + mQ + nR
such that lP + mQ + nR = 0
But then we get,
ldx + mdy + ndz = 0
Integrating, we get
 (x, y, z) = c1 …(1)
Again find (if possible) another set of multipliers , ,  so that
P + Q + R = 0
  dx +  dy +  dz = 0
On integrating we get,
 (x, y, z) = c2 …(2)
(1) and (2) together from solution.

Gigatech Publishing House


Igniting Minds
Engineering Mathematics - III 1.42 Linear Differential Equations (LDE.....

Illustrative Examples

Example : 1

dx dx dx
Solve : z2y = z2x = y2x .

Solution : Taking first two ratios and cancelling z2, we find


dx dy
=
y x
x dx – y dy = 0
Integrating we get,
 x dx –  y dy = c1
x2 y2
2– 2 = c1

 x2 – y2 = 2c1 …(1)
Now taking second and third ratios and cancelling x, we get,
dy dz
z2 = y2
y2 dy = z2 dz
y3 z3
Integrating we get, 3 = 3 + c2
y3 = z3 + 3c2 …(2)
(1) and (2) taking together form solution for given system.

Example : 2

dx dy dz
Solve : = =
y –x 2x – 3y
dx dy
Solution : y = –x
 x dx = – y dy
x2 y2
 2 = – 2 + c1

x2 = – y2 + 2 c1 …(1)
Select multipliers 3, 2, 1
3 dx + 2 dy + dz 3 dx + 2 dy + dz
Each ratio = 3y – 2x + 2x – 3y = 0
which implies 3 dx + 2 dy + dz = 0
 3 dx +  2 dy +  dz = c2
3x + 2y + z = c2 …(2)

Gigatech Publishing House


Igniting Minds
Engineering Mathematics - III 1.43 Linear Differential Equations (LDE.....

Example : 3

dx dy dz
Solve : 2x = – y = 4xy2 – 2z

Solution : Consider first and second ratio


dx dy
=
2x –y
1
 logx = – logy + log c1
2
log x = – 2 logy + 2 log c1
2
log x = log y–2 + log c1
c21 
log x = log  2 
y 
2
c1 2 2
 x = y2  xy = c1 …(1)
dx dz
Consider, 2x = 4xy2 – 2z
dx dz
= 2
2x 4 c1 – 2z
1 1 2
2 logx = – 2 log (4 c1 – 2z ) + log c2
2
logx = – log (4 c1 – 2z ) + 2 log c2
 c22 
logx = log  2 
4c1 – 2z
2
c2
x = 2
4 c1 – 2z
2
x (4xy2 – 2z) = c2 …(2)

Example : 4

dx dy 2 dz
Solve : xz = yz = (x+y)2

Solution :
dx dy dx dy
=  =
xz yz x y
 log x = logy + logc1  x = c1y …(1)
dx + dy 2 dz
xz + yz = (x + y)2
Gigatech Publishing House
Igniting Minds
Engineering Mathematics - III 1.44 Linear Differential Equations (LDE.....

(x + y) (dx + dy) = 2z dz
p dp = 2z dz  x +y = p, dx + dy = dp
p2 z2
 2 = 2 2 + c2

(x + y)2 = 2z2 + 2c2 …(2)

Example : 5

dx dy dz
Solve : x (y2 – z2 ) = – y (z2 + x2) = z (x2 + y2)

Solution :
Applying the multipliers x, y , z.
x dx + y dy + z dz
(each ratio) = x2 (y2 – z2 ) – y2 (z2 + x2) + z2 (x2 + y2)
x dx + y dy + z dz
= 0
Hence, x dx + y dy + z dz = 0
On, integration
x2 y2 z2
+ + = c1
2 2 2
x2 + y2 +z2 = 2 c1 …(1)
1 –1 –1
Applying multipliers x , y , z , we have
1 1 1
dx + dy + dz
x y z
(each ratio) = (y2 – z2 ) + (z2 + x2) – (x2 + y2)
1 1 1
x dx + y dy + z dz
= 0
which implies
1 1 1
x dx – y dy – z dz = 0

On integrating, we get
logx – logy – logz = log c2
x
log yz = log c2
 
x
 = c2
yz
 x = c2 yz …(2)

Gigatech Publishing House


Igniting Minds
Engineering Mathematics - III 1.45 Linear Differential Equations (LDE.....

Example : 6

dx dy dz
Solve : x (2y4 – z4 ) = y (z4 – 2x4) = z (x4 – y4)

Solution :
Applying the multipliers x3, y3, z3
x3 dx + y3 dy + z3 dz
(each ratio) = x x (2y – z ) + y3y (z4 – 2x4) + z3z (x4 – y4)
3 4 4

x3 dx + y3 dy + z3 dz
= 0
which implies,
x3 dx + y3 dy + z3 dz = 0
x4 y4 z4
+ + = c1
4 4 4
x4 + y4 + z4 = 4 c1 …(1)
1 1 2
Applying multipliers x , y , z , we get,
1 1 2
dx + dy + dz
x y z
(each ratio) = (2y – z ) + (z – 2x ) + 2 (x4 – y4)
4 4 4 4

1 1 2
x dx + y dy + z dz
= 0
1 1 2
which implies dx + dy + dz = 0
x y z
On integrating, we get
1 1 2
 x dx +  y dy +  z dz = c2

log x + log y + 2log z = log c3


log (xyz2) = log c3
xyz2 = c3 …(2)

Self Assessment Exercise 1.8

Solve the following differential equations by using the method of variation of parameters.
dx dy dz z
Ex.1 y = –x = x2 + y2 Ans. : x2 + y2 = 2 c1, y + x2 + y2 = c2
xe e
dx dy dz 3 3
Ex.2 2 = 2 = 2 2 2 Ans. : x3 – y3 = c1, y3 + = c2 or x3 + = c3
y x x y z z z
dx dy dz 1
Ex.3 x = y = – (x + z) Ans. : x = c1y , 2 xy + yz = c2

Gigatech Publishing House


Igniting Minds
Engineering Mathematics - III 1.46 Linear Differential Equations (LDE.....

dx dy dz
Ex.4 1 = 3 = 5z + tan (y – 3x ) Ans. : y – 3x = c1 , 5x = log (5z

+ tan (y – 3x) + c2
dx dy dz xy
Ex.5 1 = 1 = exy + sinxy Ans. : x – y = c1 , e – cos xy = z + c2
dx dy dz
Ex.6 y – z = z – x = x – y Ans. : x + y + z = c1 , x2 + y2 + z2 = c2

(Hint : Multipliers are 1, 1, 1 and x, y, z)


dx dy dz
Ex.7 3x – 4y = 4x – 2z = 2y – 3x Ans. : x2 + y2 + z2 = c1 , 2x + 3y + 4z = c2

Hint : Multipliers are x, y, z and 2, 3 4


dx dy dz
Ex.8 y + zx = – x – yz = x2 – y2 Hint : Multipliers are x, y, – z and y, x, 1
dx dy dz 1 1 1
Ex.9 x (y2 + z) = – y (x2 + z) = z (x2 – y2) Hint : Multipliers are x, y, – 1 and x , y , z

Consider a closed and simple electrical circuit containing inductor, resistor, capacitor
and some e.m.f. source.
q = Total charges in circuit at time t
I = Current flowing in circuit at time t
dq
Then, I = dt

Applying Kirchhoffs Voltage Law to the above circuit we get linear differential
equation which can be solved. We can find value of arbitrary constants by using initial
conditions. Physical interpretation can be drawn from solution. This procedure is called
modeling of electrical circuit.

The algebraic sum of all the voltage drops around an electric loop or circuit is equal to
the resultant electromotive force (e.m.f.) in the circuit. We need the following laws.
dI
1. (Voltage drop across inductance L) = L
dt
2. (Voltage drop across resistance R) = R I
q
3. (Voltage drop across capacitance C) = C

1. LC Circuit :
dI q
By KVL, L + = E(t)
dt C
dq d2q q
But, I = , so we get, L + = E(t) Fig.: 1.1
dt dt2 C
Gigatech Publishing House
Igniting Minds
Engineering Mathematics - III 1.47 Linear Differential Equations (LDE.....

2. LCR Circuit :
By KVL,
dI q
L dt + RI + C = E(t)
dq
But, I = dt , so we get,
d2q dq q Fig.: 1. 2
L dt2 + R dt + C = E(t)

Illustrative Examples

Example : 1

An inductor of 0.25 henries is connected in series with negligible resistance and a


capacitor of 0.04 farads, a generator having alternative voltage given by 12 sin 10 t. Set up the
differential equation and find the charge and current.
Solution :
Given is LC circuit, By KVL we get,
dI q
L dt + C = E
d2q q dq
L dt2 + C = E  I = dt
d2q 1 E
dt2 + LC q = L
D2 + 1  q = E
 LC L
Given that, L = 0.25 henries
C = 0.04 F
E = 12 sin 10 t
(D2 + 100) q = 48 sin 10 t
AE is, D2 + 100 = 0
(D – 10i ) (D + 10i) = 0
D =  10 i
CF = C1 cos10 t + C2 sin10 t
1
PI = D2 + 100 48 sin10 t
1
= 48 sin10 t
– 100 + 100
1
= 48 t 2D sin10 t

Gigatech Publishing House


Igniting Minds
Engineering Mathematics - III 1.48 Linear Differential Equations (LDE.....

= 24 t  sin10 t dt
– cos10 t
= 24 t 10
PI = – 2.4 t cos 10 t
C.S is q = CF + PI
q = c1 cos10 t + c2 sin10 t – 2.4 t cos10 t …(1)
dq
I = dt
= – 10 c1 sin10t + 10 c2 cos10 t
– 2.4 (– 10t sin10 t + cos10 t)
I = (– 10 c1 + 24t) sin10 t + (10 c2 – 2.4) cos10 t …(2)

Example : 2

An uncharged condenser of capacity C charged by applying an e.m.f. of value


t
E sin through the leads of inductance L and of negligible resistance. The charge Q on the
LC
d2Q Q E t
plate of condenser satisfies the differential equation : dt2 + LC = L sin , prove that the
LC
EC  t t t 
2  
charge at any time t is given by Q = sin – cos
LC LC LC
Solution :
d
Use, D  dt in given differential equation we get
D2 + 1  Q = E
sin
t
 LC L LC
1
Take w2 =
LC
E
(D2 + w2) Q = L sin wt
AE is D2 + w2 = 0
(D – wi) (D + wi ) = 0
D =  wi
CF = c1 cos wt + c2 sin wt
1 E
PI = D2 + w2 L sin wt
E 1
= L D2 + w2 sin wt
E 1
= sin wt
L – w2 + w2

Gigatech Publishing House


Igniting Minds
Engineering Mathematics - III 1.49 Linear Differential Equations (LDE.....

E 1
= t sin wt
L 2D
E 1
= L t 2  sin wt dt
E t – cos wt
= L2 w 
Et
PI = – 2Lw cos wt

The complete solution is


Q = CF + PI
E
Q = c1 cos wt + c2 sin wt – 2Lw cos wt …(1)
dQ
I = dt
= – c1 w sin wt + c2 w cos wt –
E
[– wt sin wt + cos wt] …(2)
2Lw
At t = 0, I = 0 and Q = 0
(1)  0 = c1 + 0 – 0  c1 = 0
E
(2)  0 = – 0 + c2 w – 2 Lw [– 0 + 1]
E
 c2 =
2 Lw2
Put value of c1 and c2 in (1)
E Et
Q = sin wt – cos wt
2 Lw2 2 Lw
E
Q = 2 Lw2 [sin wt – wt cos wt]
1
Now w2 = LC
EC  t t t 
2  
Q = sin – cos
LC LC LC
Example : 3

A capacitor of 10–3 farads is in the series with e.m.f. of 20 volts and an inductor of 0.4
Henry. At t = 0, the charge Q and current I are zero, find Q at any time t.
Solution :
By KVL,
dI Q
L dt + C = E

Gigatech Publishing House


Igniting Minds
Engineering Mathematics - III 1.50 Linear Differential Equations (LDE.....

dI 1 E
+ Q =
dt LC L
Given that , C = 10–3 farads
E = 20 volts
L = 0.4 Henry
dQ
And, I = dt
d2Q
dt2 + 2500 Q = 50

d
Use, D  dt
(D2 + 2500) Q = 50
2
AE is, D + 2500 = 0
(D – 50i ) (D + 50i) = 0
D =  50i
CF = c1 cos50 t + c2 sin50 t
1
PI = 50
D2 + 2500
1 0t
= D2 + 2500 50 e
1 0t
= 0 + 2500 50 e
PI = 0.02
The complete solution is,
Q = CF + PI
Q = c1 cos50 t + c2 sin50 t + 0.02 …(1)
dQ
I = dt
= – 50 c1 sin50 t + 50 c2 cos50 t …(2)
Given that at t = 0, Q = I = 0
(1)  0 = c1 + 0 + 0.02, c1 = – 0.02
(2)  0 = 0 + 50 c2 ,  c2 = 0
(1)  Q = – 0.02 cos50 t + 0.02
Q = 0.02 [1 – cos50 t]

Gigatech Publishing House


Igniting Minds
Engineering Mathematics - III 1.51 Linear Differential Equations (LDE.....

Example : 4

In an L – C – R circuit, the charge q on a plate of a condenser is given by


2
dq dq q 1
L dt2 + R dt + C = E sin pt. The circuit is tuned to resonance so that p2 = LC . If initially the
R
current j and the charge q be zero, show that for small value of L , the current in the circuit at
Et
time t is given by sin pt.
2L
Solution :
d
Take D  dt
1
L D2 q + R D q + C q = E sin pt

LD2 + RD + 1  q = E sin pt
 C
1
AE is, LD2 + RD + C = 0

1  4L R R2 1
D = 
2L – R  R2 – C  = – 2L  4 L2 – LC

R R
As L is small, so , to the first order in L .
R 1
D = – 2L  i
LC
R
D = – 2L  ip
– Rt
2L
CF = e [c1 cos pt + c2 sin pt]

CF = 1 – Rt  ( c cos pt + c sin pt)


 2L 1 2

R 2
 reject term in  L and higher powers
 
1 1
PI = E sin pt = E sin pt
1 1
LD2 +RD + C – Lp2 +RD + C
E
= R  sin pt dt
E
PI = – Rp cos pt

The complete solution is,


q = CF + PI

Gigatech Publishing House


Igniting Minds
Engineering Mathematics - III 1.52 Linear Differential Equations (LDE.....

q = 1 – Rt  (c cos pt + c sin pt) – E cos pt …(1)


 2L 1 2
Rp
dq
j = dt

= 1 – Rt  ( – c p sin pt + c p cos pt) – R


 2L 1 2
2L
E
(c1 cos pt + c2 sin pt) + R sin pt …(2)

Given that at t = 0, q = j = 0
E E
(1)  0 = c1 – Rp  c1 = Rp
Rc1 Rc1 E
(2)  0 = c2 p – ,  c2 =  c2 =
2L 2Lp 2Lp2
Put value of c1 and c2 in (2)
Rt E E R E E E
j = 1 – 2L – Rp sin pt + 2Lp2 cos pt p–2L  Rp cos pt + 2Lp2 sin pt + R sin pt
    
Et
j = sin pt
2L

Self Assessment Exercise 1.9

Ex.1 A circuit of an inductance L and condenser of capacity C is in series. An alternating


e.m.f. Esin nt is applied to it at time t = 0, the initial current and charge on the condenser
being zero. Find the current flowing in the circuit at any time t for : (i) w  n, (ii) w = n,
1
where w2 = LC .

Ans. :
En
(i) I = (w2 – n2) L [cos nt – cos wt]
E
(ii) I = t sin wt
2L
Ex.2 A circuit consists of an inductance L, resistance R and condenser of capacity C in series.
1
An alternating e.m.f. E sin wt is applied to it at time t = 0 where w2 = LC . The initial
4L
current and charge on the condenser being zero. Find the charge at any time t if R 2 < C
E E E
Ans. : q = e–t  cost – sint – cos wt
Rw 2Lw  Rw
R 1 4L 2
where  = 2L and  = 2L C –R

Gigatech Publishing House


Igniting Minds
Engineering Mathematics - III 1.53 Linear Differential Equations (LDE.....

Ex.3 An e.m.f E sinpt is applied at t = 0 to a circuit containing a condenser C and inductance


dI 1 dq
L in series the current I satisfies the equation : L dt + C  I dt = E sin pt, where I = – dt .
1
If p2 = LC and initially the current and the charge are zero, find current at any time t.
dq
Hint : Taking I = – dt in given differential equation we get,
d2q 1 dq
– L 2 +  – dt = E sin pt
dt C dt
d2q 1
– L dt2 – C q = E sin pt
E
(D2 + p2) q = – L sin pt
Et
Finally, I = 2L sin pt

Ex.4 A resistance of 50 ohms, an inductor of 2 henries and a 0.005 farad capacitor are in
series with an e.m.f. of 40 volts and an open switch. Find the instantaneous charge and
current after the switch is closed at t = 0, assuming that at that time the charge on the
capacitor is 4 coulomb.
Ans. :
Q = 5.07 e–5t – 1.27 e–20t + 0.2
I = 25.4 (e–20t – e–5t)

Descriptive Questions

Q. 1 (D3 + 3D2 – 4)y = 6e-2x + 4x2


Q. 2 (D2 – 4D + 4)y = 8(e2x + sin2x + x2)
Q. 3 (D2 – 4)y = e3x x2.
Q. 4 (D2 + 1)2 y = 24x cosx
Q. 5 (D2 – 2D + 4)2 y = xex cos[ 3 + ]
dx dy dz
Q. 6 1 = 3 = 5z + tan(y - 3x)
dx dy dz
Q. 7 2 = =
x + y2 2xy (x + y)3z
dx dy dz
Q. 8 = =
y -x 2x - 3y


Gigatech Publishing House
Igniting Minds
Unit
2
Laplace Transform (LT)

Syllabus :
Definition of LT, Inverse LT, Properties & theorems, LT of standard functions, LT of some
special functions viz. Periodic, Unit Step, Unit Impulse. Applications of LT for solving
Linear differential equations.

In elementary calculus, we studied that differentiation and integration of a function


means transformation of a function into another function. Moreover, these two transforms also
possess the linearity property.
For example : If f(x) = x2 + sinx, then
d d 2 d d
f(x) = [x + sinx] = x2 + sinx = 2x + cosx
dx dx dx dx
x3
f(x) dx =  [x2 + sinx] dx = x2 dx + sinx dx = 3 – cosx.
In this chapter, we study a particular type of integral transform known as Laplace
transform. The Laplace transform is named in respect of the French mathematician and
astronomer Pierre–Simon Marquis de Laplace (1749 – 1827).
Laplace transform are invaluable for any scientists and engineers due to its simplicity in
solving linear differential equations – ordinary as well as partial, and related initial value
problems. The key inspiration for learning about Laplace transforms is that :
 The process of solving ordinary differential equations is simplified to an algebraic
problem.
 It directly gives the solution of differential equation with given initial conditions without
the necessity of first finding the general solution and then particular solution.
 We can analyze system containing short impulses using Laplace transforms.
 We can examine response of different electrical circuits by using Laplace transforms.

Gigatech Publishing House


Igniting Minds
Engineering Mathematics – III 2.2 Laplace Transformation (LT)

A function F(t) is said to piecewise continuous on [0, ] if, in any interval 0  a  t  b,


there exist at most finite number of points ti, i = 1, 2,..., n (ti–1 < ti) at which F has finite
discontinuities and is continuous on each open interval (ti–1, ti)

Fig.: 2.1 – Graph of piecewise continuous function F(t)

A function F(t) is said to be exponential order if, there exist constants , M > 0, and T >
0 such that |F(t)|  Met for all t > T.
The functions F(t) = t, F(t) = e–t, F(t) = t3 sin t all are of exponential order.

Fig.: 2.2 – Graph of exponential order functions


3
Note : Function F(t) = et is not of exponential order. Because we cannot find largest M > 0,
3 3
 > 0 such that |et | < Met  t > N. That is, e(t – t)cannot be made less than to any given
constant by increasing t.

Let F(t) be a function of t defined for all t  0. Then the Laplace transform of the
function F(t), denoted by L[F(t)], is defined by

L[F(t)] =  e–st F(t) dt
0

Provided the integral exists, where s is a parameter which may be real or complex.
Clearly, L[F(t)] is a function of s and is briefly written as f(s) i.e. L[F(t)] = f(s)
Note : Throughout the chapter, we use the notation L[F(t)] = f(s).
Gigatech Publishing House
Igniting Minds
Engineering Mathematics – III 2.3 Laplace Transformation (LT)

If F(t) is continuous for all t  0 or if F(t) is piecewise continuous in every finite interval

for t  0 and lim [e–at F(t)] is finite then,


t

L[F(t)] =  e–st F(t) dt exists for s > a
0
Note : It should be noted that above conditions for existence are sufficient but not
necessary for the existence of Laplace transforms.

If C1 and C2 are any arbitrary constant and F(t) and G(t) are functions of t, then
L[C1F(t) + C2G(t)] = C1 L[F(t)] + C2L[G(t)]
Verification : By definition of Laplace transform, we get

L[C1F(t) + C2G(t)] =  e–st [C1F(t) + C2G(t)] dt
0

=  [C1e–st F(t) dt + C2 e–st G(t)]dt
0
 
= C1  e F(t) dt + C2 
–st
 e–st G(t)dt
0 0 0
= C1 L[F(t)] + C2 L[G(t)]
Note : This result can be easily generalized for any number of functions.

Illustrative Examples

Example : 1

Let F(t) = 1 for all t  0. Then by definition of Laplace transform, find f(s)
Solution : We know by definition that,
 
f(s) = L[F(t)] = L[1] =  e–st 1dt =  e–st dt
0 0
–st 
=  e  = – 1 [e– – e–0]
 –s  0 s
1 1
= – s [ 0 – 1] = s , provided s > 0.

1
i.e. f(s) = ;s>0
s

Gigatech Publishing House


Igniting Minds
Engineering Mathematics – III 2.4 Laplace Transformation (LT)

Example : 2

Let F(t) = eat for all t  0 and a is any constant. Then by definition of Laplace transform,
find f(s).
Solution :
We know by definition

f(s) = L [f(t)] = L[e ] =  e–st eat dt
at

0

e–(s – a)t  
=  e–(s – a)t dt = 
0
–(s – a) 0
1 –1
= – s – a [e– – e–0] = s – a [0 – 1]
1
= s – a , provided s > a
1
i.e. f(s) =  if s > a
s–a

Example : 3

Let F(t) = tn for all t  0 and n is a positive integer. Then by definition of Laplace
transform, find f(s).
Solution :
We know by definition

f(s) = L[f(t)] = L[t ] =n
 e–st tndt .... (1)
0
z
Put st = z, then sdt = dz and t =
s
Also, if s > 0, then t = 0 implies z = 0
and t =  implies z = 
Thus from (1), we get
 
z n dz 1
f(s) =  e  s  s = sn+1  e–z zn dz
–z

0
  0

n+1
= sn + 1 , provided s > 0 and n + 1 > 0 .... (2)


By definition of gamma function n + 1 =  e–z zn dz
0

Gigatech Publishing House


Igniting Minds
Engineering Mathematics – III 2.5 Laplace Transformation (LT)

Since n is a +ve integer, then we know that


n+1 = n!
n!
Hence from (2) f(s) = sn + 1 , provided s > 0
Thus, we obtained

 n+1
; if s > 0 & n > –1
f (s) =  n!
sn + 1

 sn + 1
; if s > 0 & n is +ve integer

Example : 4

Using the definition of Laplace transform, show that :


a
i) If F(t) = sin at for all t  0, then f(s) = s2 + a2
s
ii) If F(t) = cos at for all t  0, then f(s) = s2 + a2

Solution :
We know by definition,

f(s) = L[F(t)] =  e–st F(t) dt
0
i) If F(t) = sin at, then

f(s) =  e–st sin at dt
0
–st 
=  2e 2 (–s sin at – a cos at) 
s +a  0
1 a
= [e– – e–0 (0 – a)] = 2 , provided s > 0
s2 + a2 s + a2
a
i.e. f(s) =  if s > 0
s2 + a2
ii) If F(t) = cos at, then
 
e–st
f(s) =  e–st cos at dt =  s2 + a2 (–s cos at – a sin at) 
0
  0

1 – –0 s
= s2 + a2 [e – e (–s + 0)] = s2 + a2 , s > 0
s
i.e. f(s) =  if s > 0
s2 + a2

Gigatech Publishing House


Igniting Minds
Engineering Mathematics – III 2.6 Laplace Transformation (LT)

Example : 5

Using the definition of Laplace transform, show that :


a
i) If F(t) = sinh at for all t  0, then f(s) = s2 – a2 for s > | a |.

s
ii) If F(t) = cosh at for all t  0, then f(s) = for s > | a |
s – a2
2

Solution :
We know by definition

f(s) = L[F(t)] =  e–st F(t) dt
0

i) If F(t) = sinh at, then


we know that
at –at (eat – e–at)
e –e sinh at =
f(s) = L(sinh at) = L   2
 2  (eat + e–at)
cosh at = 2

Using linearity property, we obtain


1
L [eat] – L [e–at] ]
2 [
f(s) =

1  1 1 
= 2 s–a–s+a Using Example 2

1 2a a
= 2 s2 – a2 = s2 – a2
a
i.e. f(s) = s > |a| ,
s 2 – a2

ii) If F(t) = cosh at, then by using linearity property we obtain


eat + e–at 
f(s) = L[coshat] = L
 2 
1
= 2 (L[eat] + L[e–at]) Using example 2
1 1 1 1 2s s
= 2  s – a + s + a  = 2 s2 – a2 = s2 – a2
 
s
i.e. f(s) =  s > |a|
s2  a2

Gigatech Publishing House


Igniting Minds
Engineering Mathematics – III 2.7 Laplace Transformation (LT)

Table 2.1 : Laplace Transforms of Elementary (basic) Functions


F(t) ; t  0 f(s) = L [F (t)]
1 1
;s>a
s
1
eat s–a
;s>0

 n + 1 ; s > 0 and n > –1


n+1
tn  n!s
 s ; s > 0 and n is +ve integer
n+1

a
sin at s2 + a2 ; s > 0
s
cos at s2 + a2 ; s > 0
a
sinh at ; s>|a |
s2 – a2
s
cosh at ;s>|a|
s2 – a2

Example : 6

Find the Laplace transforms of :


i) e4t ii) e–2t + 4 iii) 2e–t + 5e–2t
iv) t2 + 4 v) t3/2 + t1/2 vi) e2t + t + 4 vii) (1 + t)3
Solution :
1
i) L [e4t] = , s > 4.
s–4
1 1 4
ii) L [e–2t + 4] = L[e–2t] + L[4] = + 4 L[1] = +
s+2 s+2 s
s + 4s + 8 5s + 8
= s(s + 2) = s(s + 2) , s > 0
iii) L [2e–t + 5e–2t] = L[2e–t] + L[5e–2t]
= 2L[e–t] + 5L[e–2t]
2 5 7s + 9
= + = , s>0
s + 1 s + 2 (s + 2) (s + 1)
iv) L[t2 + 4] = L[t2] + L[4]
2! 4 2 4 4s2 + 2
= s3 + s = s3 + s = s3 , s>0

Gigatech Publishing House


Igniting Minds
Engineering Mathematics – III 2.8 Laplace Transformation (LT)

v) L [t3/2 + t1/2] = L [t3/2] + L [t1/2]


we know that
3/2 + 1 1/2 + 1 (a) n+1 =n n
= s 5/2 + s 3/2
1
(b) 2 = 

31 1 1 1
22 2 2 2
= + 3/2
s5/2 s
 3 1 
 4 2  3 
=  5/2 + 3/2   = 5/2 + 3/2 , s > 0.
s s  4s 2s
vi) L[e2t + t + 4] = L[e2t] + L[t] + L[4]
1 1 1
= + + , s>0
s – 2 s2 s
we know that
vii) L[(1 + t)3] = L [1 + t3 + 3t2 + 3t] (a + b)3 = a3 + b3 + 3a2b + 3ab2
= L[1] + L[t3] + 3L[t2] + 3L[t]
1 3! 3  2! 3 s3 + 3s2 + 6s + 6
= + + 3 + 2 = , s>0
s s4 s s s4

Example : 7

Find Laplace transform of :


i) sin2t ii) sin2t iii) cos32t
iv) sint cos2t v) cos (at + b) vi) sinh 2t
2t 2
vii) sin2t + cos3t viii) e + t + sin4t + cosht ix) cost cos3t
Solution :
2
i) L[sin2t] = 2 , s>0
s +4
1 – cos2t 1
ii) L[sin2t] = L = [L[1] – L[cos2t]]
 2  2
1 1 s 2
= 2 s – s2 + 4  = , s>0
  s(s2 + 4)
cos6t + 2cos2t
iii) L [cos32t] = L [ ... cos3 = 4cos3 – 3cos
 4 
1
= 4 [L[cos6t] + 3L [cos2t]]
1 s 3s  s(s2 + 28)
= + 2 = 2 , s>0
4 s + 36 s + 4 
2
(s + 4) (s2 + 36)
Gigatech Publishing House
Igniting Minds
Engineering Mathematics – III 2.9 Laplace Transformation (LT)

1
iv) L[sint  cos2t] = L  (2sint cos2t) 
2 
1 1
= 2 L [sin3t – sint] = 2 [L[sin3t] – L[sint]]
1 3 1 s2 – 3
= 2 s 2 + 9 – s 2 + 1  = 2 ; s > 0.
  (s + 9) (s2 + 1)
v) L[cos (at + b)] = L[cos at cosb – sin at sinb]
= cosb L[cos at] – sinb L[sinat]
s a
= cosb s2 + a2 – sinb s2 + a2
s cosb – a sinb
= ; s > 0.
(s2 + a2)
2
vi) L[sinh at] = 2 , s > | 2|
s –4
2 s
vii) L[sin2t + cos3t] = L[sin2t] + L[cos3t] = 2 + , s > 0.
s + 4 s2 + 9
viii) L[e2t + t2 + sin4t + cosht] = L[e2t] + L[t2] + L[sin4t] + L[cosht]
1 2 4 s
= + + + , s > |1|
s – 2 s3 s2 + 16 s2 – 1
1
ix) L[cost cos3t] = L  (2 cost cos3t) 
2 
1
= 2 L[cos4t + cos2t]
1
= 2 [L[cos4t] + L[cos2t]]
1 s s
= 2  s2 + 16 + s2 + 4 
 
1 2s (s2 + 20)
=
2 (s2 + 16) (s2 + 4)
s(s2 + 20)
= , s > 0.
(s + 16) (s2 + 4)
2

Example : 8

Find the Laplace transform of F(t) defined as


 t  if 0 < t < T
F(t) =  T
 1  if t > T
Solution :
Since function F(t) is discontinuous so we use definition of Laplace transform.

Gigatech Publishing House


Igniting Minds
Engineering Mathematics – III 2.10 Laplace Transformation (LT)

 T 
L[F(t)] =  –st
e F(t) dt = e –st
F(t) dt +  e–st F(t) dt
0 0 T
T 
t
=  e–st T dt +  1 e dt
–st

0 T
T
1 t e–st T e–st  e–st  
= T  –s  0 +  s dt  +  –s  T
 0 
1 –T –sT 1 e–st T  1 – –st
= T s e + s –s – [e – e ]
  0 s
1 –T e–sT 1 e–sT
= T  s e–sT – s2 + s2  + s
 
e–sT e–sT 1 e–sT
= – – 2 + 2 +
s Ts Ts s
1 – e–sT
= , s>0
Ts2

Example : 9

 1  if 0  t < 1
Find Laplace transform of F(t) =  t  if 1  t < 2
 t2  if 2  t < 
 1 2 
Solution : L[F(t)] =  –st
e F(t) dt = e –st
1 dt + te –st
dt +  t2 e–stdt
0 0 1 2
2

–st 1 –st 2 –st –st 
–st 2 –st
=  e  +  t e  –  e dt  + t e – 2t e 2 + 2e 3
 s  0  –s 1 1 –s   –s (–s) (–s)  2
–s –2s –2s –s –s –2s
e – 1 2e e e e   4e 4e–2s 2e–2s
= + – 2 – + + 0 + + s2 + s3 
–s  –s s –s s  2
s
e–s 1 2 –2s 3 –2s 2 –2s
= + + e + 2e + 3e , s>0
s2 s s s s

Example : 10

Find Laplace transforms of the following functions.


t  0<t<4 sint  0<t<
i) F(t) =  ii) F(t) = 
5  t>4 0  t>
Solution :

i) L[F(t)] =  e–st F(t) dt
0

Gigatech Publishing House


Igniting Minds
Engineering Mathematics – III 2.11 Laplace Transformation (LT)
4 
= e –st
t dt +  5 e–st dt
0 4
–st –st 4 –st 
= –te – e 2  + 5 e 
 s s 0  –s  4
1 (s – 1)
= + e–4s 2
s2 s
–4s
1 + e (s – 1)
= , s > 0.
s2
  
ii) L [F(t)] =  –st
e F(t) dt = e –st
sint dt +  0 dt
0 0 
–st 
=  2e [–s sint – cost] 
s + 1 0
e–s 1 ... eax sinbx = eax
= 2 (1) – 2 (–1) a2 + b2
[a sinbx – b cos bx
s +1 s +1
1
= (e–s + 1), s>0
s2 + 1

Self-Assessment Exercise 2.1

Ex. 1 Find the Laplace transforms of following functions.


9s + 2
i) 4e2t + 5e–3t Ans. : (s – 2) (s + 3) , s > 2

27 s3 + 54s2 + 72s + 48
ii) (2t + 3)3 Ans. : ,s>0
s4
5s3 + 8s2 + 80s + 8
iii) 2sin4t + 5cost Ans. : (s2 + 16) (s2 + 1)

2(s2 – 5)
iv) sin2t cos3t Ans. : (s2 + 25) (s2 + 1)

48
v) sin32t Ans. :
(s2 + 4) (s2 + 36)
1 120 3 – 2s
vi) e3t + 5t4 – 2cost + 3sint Ans. : s – 3 + s5 + s2 + 1
s2 + 8
vii) cos22t Ans. : s(s2 + 16)
2a2s
viii) cosh at – cos at Ans. : 4 4
s –a

Gigatech Publishing House


Igniting Minds
Engineering Mathematics – III 2.12 Laplace Transformation (LT)

Ex.2 Obtain the Laplace transform of following functions :


24 5040
i) 4t3 + tn Ans. : s4 + s8
5
ii) 5 sin (2t + 3) Ans. : s2 + 4 [2cos3 + s sin3]
s (s2 – 28)
iii) cosh32t Ans. : (s2 – 4) (s2 – 36)
s (s2 + 13)
iv) cos3t cos2t Ans. : 2
(s + 1) (s2 + 25)
4s 10 2
v) 4cos2t – 5t2 + 2e3t Ans. : s2 + 4 – s3 + s – 3

 cos + s sin
vi) sin (t + ) Ans. :
s2 + 2
Ex.3 Obtain the Laplace transform of F(t) defined as :
 cost  if 0  t < 2 s
i) F(t) =  Ans. : 2 [1 – e–2s]
0  t > 2 s +1
2
 t  0t<2
2 e–2s e–3s
ii) F(t) =  t – 1  2  t < 3 Ans. : 3 – 3 (2 + 3s + 3s2) + 2 (5s – 1)
s s s
 7  t>3
4  0t1 4 – e–s
iii) F(t) =  Ans. :
3  t>1 s
Ex.4 Find Laplace transform of the function :
1 3  3 6 12
F(t) =  t +  Ans. : + + + 8s1/2
 t 4 s5/2 s3/2 s1/2 
Ex.5 Find Laplace transform of the functions :
i) F(t) = |t – 1| + |t + 1| + |t + 2| + |t – 2|, t  0
Hint : F(t) =  2  0  t  1  Ans. :
2
(3s + e–s + e–2s)
  2t  t > 1  s2
2 e–s
ii) F(t) = |t – 1| + |t + 1| , t  0 Ans. : s  1 + s 
 
Ex.6 Find Laplace transform of function F(t) = [t], where [ ] stands for the greatest
integer function.
0  0<t<1
 
1  1<t<2 
 Hint : F(t) =
2  2<t<3
.   Ans. :
1

 
s
s(e – 1)

.

 .
 
Gigatech Publishing House
Igniting Minds
Engineering Mathematics – III 2.13 Laplace Transformation (LT)

If L[F(t)] = f(s) exist. Then L[eat F(t)] = f(s – a)


Proof : By definition,

at
L[e  F(t)] =  e–st eat F(t) dt
0
 
=  e t F(t) dt =  e–rt F(t) dt
–(s – a)
where r = s – a
0 0
= f(r) = f(s – a)
Similarly, L[e–at F(t)] = f(s + a)
Application of this property leads us to following important results.
n!
1. L[eat tn]
 (s – a) n+1
 where n is +ve integer
 n+1
 (s – a) n+1
 n > –1  s > a

2. L[eat sinbt] b
, s>a
(s – a)2 + b2

3. L[eat cos bt] s–a


, s>a
(s – a)2 + b2

4. L[eat sinh bt] b


, s>a
(s – a)2 – b2

5. L[eat cosh bt] s–a


, s>a
(s – a)2 – b2

Note : In order to apply above result, we have to first find the L [F(t)] = f(s), and then replace
ss–a

1 s
If L[F(t)] = f(s), then L[F(at)] = f  
a a

Proof : By definition, L[F(at] =  e–st F(at) dt .... (3)
0
dz
Put at = z  adt = dz and dt = a

Hence from (3)


Gigatech Publishing House
Igniting Minds
Engineering Mathematics – III 2.14 Laplace Transformation (LT)

 –sz  –sz
dz 1
L[F(at)] =  e a F(z) =  e a F(z) dz
a a
0 0

1 s
a  e F(z) dz , where  = a
–z
=
0

1 1 s 1 s
= a f() = a f  a  , i.e. L[f(at)] = a f  a 
 

F(t – a)  t > a

If L[F(t)] = f(s) exist and G(t) = 
0  t<a
then L[G(t)] = e–as f(s)
Proof : By definition,
 a 
L[G(t)] =  –st
e G(t) dt = e –st
G(t) dt +  e–st G(t) dt
0 0 a
a 
=  e–st 0 dt +  e–st F(t – a) dt
0 a

=  e–s(z + a) F(z) dz Put t – a = z
 dt = dz
a
 
=  e –sz
e –sa
F(z) dz = e –as
 e–sz F(z) dz
a a

i.e. L[G(t)] = e–as f(s)

Illustrative Examples

Example : 1

Obtain Laplace transform of :


i) e2t sin4t ii) e–3t cosh 2t iii) et (t2 + 1)
iv) (1 + te–t)3 v) e–3t cos2t vi) et sint cost
Solution :
4
i) Since L[sin4t] = s2 + 16 exist, then
4 4
L[e2t sin4t] = (s – 2)2 + 16 = s2  4s + 20

Gigatech Publishing House


Igniting Minds
Engineering Mathematics – III 2.15 Laplace Transformation (LT)

s
ii) Since L[cosh 2t] = 2 exist, then
s –4
s+3 s+3
L[e–3t cosh 2t] = (s + 3)2 – 4 = s2 + 6s + 5
2 1 2 + s2
iii) Since L[t2 + 1] = s3 + s = s3 exist, then
2 2
2 + (s – 1) s – 2s + 3
L[et (t2 + 1)] = =
(s – 1)3 (s –1)3
iv) We know (1 + t e–t)3 = 1 + 3t e–t + 3t2 e–2t + t3e–3t
Therefore,
L[(1 + t e–t)3] = L[1 + 3t e–t + 3t2e–2t + t3e–3t]
= L[1] + 3 L [e–t t] + 3L[e–2t t2] + L[e–3t t3]
1 3 3  2! 3!
= s + (s + 1)2 + (s + 2)3 + (s + 3)4
1 3 6 6
= + + +
s (s + 1)2 (s + 2)3 (s + 3)4
1
v) Since L[cos2t] = L  (1 + cos2t) 
2 
1 1
= 2 L [(1 + cos2t)] = 2 [L[1] + L[cos2t]]
1 1 s 
= 2  s + s2 + 4
1 1 s+3 
then L[e–3t cos2t] = 2 s + 3 + (s + 3)2 + 4
1 1 s+3 
= 2 s + 3 + s2 + 6s + 13
s2 + 6s + 11
=
(s + 3) (s2 + 6s + 13)
1
vi) Since L[sint cost] = L  sin2t 
2 
1 2  1
= 2  s2 + 4  = s2 + 4
1 1
then, L[et sint cost] = (s – 1)2 + 4 = s2 – 2s + 5

Example : 2

1 – cost  1 s2 + 4 1 – cos3t 
If L  = log  2  , then find L 
 t  2  s   t 
Solution :
2
1 – cost  1  s +2 4 
Given that L = log
 t  2  s 
Gigatech Publishing House
Igniting Minds
Engineering Mathematics – III 2.16 Laplace Transformation (LT)

By change of scale property


2
1 – cos3t  11  (s/3) +2 4  
L = log
 3t  32  (s/3)  
2
1  1 – cos3t  11  s +2 36  
 3L = log
t  32  s 
2
1 – cos3t  1 s + 36 
 L = log 
 t  2  s2 
Example : 3

2s
If L[t sint] = (s2 + 1)2 , find L[t sint]

Solution :
2s
Given that L[t sint] =
(s2 + 1)2
By change of scale property
1  2(s/) 
L[t sint] = 
  ((s/) + 1)2 

1  2s 4 
  L[t sint] =  
2  (s2 + 2)2 
2s
 L[t sint] =
(s2 + 2)2

Example : 4

s t
If L[F(t)] = , find L  F   
s2 + 4   
2
Solution :
s
Given that L[F(t)] = s2 + 4
By change of Scalar property
s
t 1 1/2  Here a = 1
LF2 =
   (1/2)  s 2  2
+4
1/2
2s
= 2 (2s)2 + 4
4s
=
4s2 + 4
t s
LF  =
 2 s2 + 1

Gigatech Publishing House


Igniting Minds
Engineering Mathematics – III 2.17 Laplace Transformation (LT)

Example : 5

sint 1 sinbt
If L  t  = tan–1  s  , Find L  t 
     
Solution :
sint 1
Given that L t  = tan–1  s 
   
By change of scale property
sinat  1 –1  a 
L = tan
 at  a s
1  sinat  1 –1  a 
 aL t  = a tan  s 
sinat  a
 L = tan–1  
 t  s
Example : 6

Find the Laplace transform of :


 

sin  t –   t >
 3 3 (t – 2)3  t > 2
i) G(t) =  ii) G(t) = 
 0  t<2

0  t<3
2 2

cos  t – 3   t > 3
 
iii) G(t) =  2

0  t< 3

Solution :
 
i) Here F(t – a) = sin  t – 3  and a = 3
 
therefore, F(t) = sint
1
Now, L[F(t) = L[sint] = s2 + 1 = f(s)

then by second shifting theorem, and using a = 3 we obtain
–s
1
L[G(t)] = e–as f(s) = e 3 2
s +1
ii) Here F(t – a) = (t – 2)3 and a = 2
3! 6
 F(t) = t3 and L[F(t)] = s4 = s4

Hence by second shifting theorem,

Gigatech Publishing House


Igniting Minds
Engineering Mathematics – III 2.18 Laplace Transformation (LT)

6
L[G(t)] = e–as f(s) = e–2s
s4
2  2
iii) Here F(t – a) = cos  t – with t =
 3  3
s
 F(t) = cos t and L[F(t)] = = f(s)
s2 + 1
Hence by second shifting theorem,
L[G(t)] = e–as f(s)
–2s
3 s
= e
s2 + 1

Self Assessment Exercise 2.2

Ex.1 Obtain the Laplace transforms of following functions :


(–2s2 + 8s – 2)
i) e2t sin3t Ans. :
(s – 4s + 5)(s2 – 4s + 13)
2


ii) e3t t Ans. :
2(s – 3)3/2
s–4
iii) e4t cosh 5t Ans. : s2 – 8s – 9
6
iv) e–3t t3 Ans. : (s + 3)4
2s2 – 16s + 38
v) e4t sin2t cost Ans. : (s2 – 8s + 25) (s2 – 8s + 17)

Ex.2 Find the Laplace transform of :


2
i) e–t sin2t Ans. :
(s + 1)(s2 + 2s + 5)
(s2 + 2s + 3)
ii) e–t cos2t Ans. : (s + 1) (s2 + 2s + 5)
30 (s + 3)
iii) e–3t sin5t sin3t Ans. : (s2 + 6s + 13) (s2 + 6s + 73)
t
Ex.3 If L [F(t)] = f(s), show that L  F  a   = a f(as)
  
Ex.4 Obtain the Laplace transform of :
(t – 1)4  t > 1 e–s  24
i) G(t) =  Ans. :
0  t<1 s5
 

5sin3  t – 4   t > 4
  15
ii) G(t) =  
Ans. : e–s/4 
s2 + 9
0   t<4

Gigatech Publishing House


Igniting Minds
Engineering Mathematics – III 2.19 Laplace Transformation (LT)

1 5
Ex.5 If L[sin t] = 2 , find L [sin 5t] Ans. : 2
s +1 s + 25
e–1/s e–3/(s + 1)
Ex.6 If L[F(t)] = s , find L[e–t F(3t)] Ans. : s + 1

2 t 1 1  1
Ex.7 If L   = s3/2 , show that L   = s
   t 
1 4
Ex.8 If L [f(t)] = s (s2 + 1) , find L[e–t f(2t)] Ans. : (s – 1) (s2 – 2s + 5)

Ex.9 Verify change of scale property for L[e3t  cos3t] [Hint : take F(t) = et cost]
s2 – s + 1 s2 – 2s + 4
Ex.10 If L[F(t)] = , find L [F(2t)] Ans. :
(2s + 1)2 (s – 1) 4(s + 1)2 (s – 2)

 Theorem : 1
If F (t) be continuous and L[F(t)] = f(s), then

L[F(t)] = sf(s) – F(0), provided Lt est F(t) = 0


t  
Proof : We know by definition,

L[F(t)] =  e–st F(t) dt
0


= –st
[ e F(t) ]0 –  –se–st F(t) dt
0

= Lt e–st F(t) – F(0) + s  e–st F(t) dt
t 0
= 0 – F(0) + s f(s)
 L[F(t)] = sf(s) – F(0)
Remark : The above theorem can be generalized for nth derivative.
If F(t) and its first (n – 1) derivatives be continuous, then
L[Fn(t)] = sn f(s) – sn – 1 F(0) – sn – 2 F (0) – sn – 3 F (0) ... Fn – 1 (0).
In particular, when n = 2, 3, 4, ..., we have
L [F (t)] = s2 f(s) – sF(0) – F (0)
L[F (t)] = s3 f(s) – s2 F(0) – sF (0) – F(0)
L[FIV (t)] = s4 f(s) – s3 F(0) – s2 F(0) – sF(0) – F(0).
Note : The above results will be further used in the solution of differential equations.

Gigatech Publishing House


Igniting Minds
Engineering Mathematics – III 2.20 Laplace Transformation (LT)

 Theorem : 2
If L [F(t)] = f(s), then
t
  f(s)
L  F(u) du  = s
0 
t 0
Proof : Let  (t) =  F(u) du, then  (t) = F(t) and (0)  F(u) du = 0
0 0
 L[ (t)] = s  (s) –  (0)
 L [F(t)] = s  (s)
 f(s) = s  (s)
f(s)
  (s) = s

f(s) t  f(s)
 L[ (t)] =
s
 L  F(u)du = s
0 

 Theorem : 3
If L[F(t)] = f(s) , then
n
d
L[tn F(t)] = (–1)n dsn [f(s)] .... (4)

Proof : We prove the theorem by induction on n.


d
Put n = 1, then we claim L [t F(t)] = – [f(s)] .... (5)
ds
By definition, we know
L[F(t)] = f(s)

  e–st F(t) dt = f(s) .... (6)
0
Differentiate equation (6) both sides w.r.to s, we get

d d
ds 
e–st F(t) dt = [f(s)]
ds
0

d –st d
  ds
[e F(t)] dt =
ds
[f(s)]
0

d
  –te–st F(t) dt = ds [f(s)]
0
Gigatech Publishing House
Igniting Minds
Engineering Mathematics – III 2.21 Laplace Transformation (LT)


d
  e–st [t F(t)] dt = –
ds
[f(s)]
0
d
 L [t F(t)] = – ds [f(s)]

Thus we get (5), so result is true for n = 1


Suppose that : equation (4) holds for n = m,
dm
i.e. L[tmF(t)] = (–1)m m [f(s)]
ds

dm
 L  e–st tm F(t) dt = (–1)m [f(s)]
dsm
0
Differentiating both sides w.r.to s, we have

d dm + 1
ds  e t F(t) dt =
–st m
(–1)m dsm + 1 [f(s)]
0

d –st m dm + 1
  ds
[e t F(t)] dt = (–1)m
dsm + 1
[f(s)]
0

dm + 1
  – t e–st tm F(t) dt = (–1)m dsm + 1 [f(s)]
0

dm + 1
  e–st [tm + 1 F(t)] dt = (–1)m + 1 dsm + 1 [f(s)]
0
dm + 1
 L[tm + 1 F(t)] = (–1)m + 1 [f(s)] Hence Proved
dsm + 1

 Theorem : 4
If L [F(t)] = f(s), then

F(t) 
L =  f(s) ds
 t 
s
Proof : By definition, we know

f(s) = L[F(t)] =  e–st F(t) dt
0
Integrating both sides w.r.to s to , we get
   
 f(s) ds =   e–st F(t) dt ds
s s 0 
Gigatech Publishing House
Igniting Minds
Engineering Mathematics – III 2.22 Laplace Transformation (LT)

Since s and t are independent, on changing order of integration on the right – hand side,
we have
   
 f(s) ds =   e–st ds F(t) dt
s 0 s 

F(t) F(t)
=  e–st dt = L 
t  t 
0

F(t)
 L t  =  f(s) ds
 
s

Note : The above result further can be generalized for tn as


 
F(t)
L  tn  =   ...  f(s) ds ds ... ds
 
s s s

Illustrative Examples

Example : 1

Find the Laplace transform of following functions :


d2x dx
i) dt2 + dt + x = 0 , where x (0) = 1 and x(0) = 0
d3x
ii) = 0, where x(0) = x(0) = 1, x(0) = 0
dt3
2
dx dx
iii) +5 + 6x = 0 , given that x(0) = 0 ; x(0) = 1
dt2 dt
Solution :
d2x dx
i) Given that dt2 + dt + x = 0 , where x (0) = 1 and x(0) = 0

taking Laplace on both side, we obtain


d2x dx
L + +x = L[0] = 0
 dt2 dt 
d2x dx
 L  dt2  + L  dt  + L[x] = 0
   
 [s2x (s) – sx (0) – x (0)] + [sx(s) – x(0)] + x(s) = 0
2
 (s + s + 1) x(s) – s – 1 = 0
s+1
 x(s) =
s2 + s + 1
Gigatech Publishing House
Igniting Minds
Engineering Mathematics – III 2.23 Laplace Transformation (LT)

ii) Taking Laplace on both side, we obtain.


d3x
L  dt3  = L [0] = 0
 
 [s3 x (s) – s2 x (0) – sx (0) – x (0)] = 0
3 2
 s x(s) – s – s = 0
s (s + 1) s + 1
 x(s) = = 2
s3 s
iii) Taking Laplace on both side, we obtain
d2x dx
L  dt2 + 5 dt + 6x  = L[0] = 0
 
d2x dx
 L  dt2  + 5L  dt  + 6L [x] = 0
   
 [s2x(s) – sx(0) – x(0)] + 5[sx(s) – x(0)] + 6x(s) = 0
(s2 + 5s + 6) x(s) – 1 = 0
1
 x(s) =
s2 + 5s + 6

Example : 2

If F(t) = sint cost then find L[F(t)].


Solution :
Given F(t) = sint cost
Clearly F(0) = 0
1
and L [F(t)] = f(s) = L [sint cost] = 2 L[sin 2t]
1 2 1
= 2 s2 + 4 = s2 + 4
s
 L[F (t)] = s f(s) – F(0) =
s2 + 4

Example : 3

Find the Laplace transform of :


t t t
i)  (x + sinx) dx ii) e x
sinx dx iii)  x2ex dx
0 0 0
Solution :
i) Here F(t) = t + sint, then
1 1 2s2 + 1
L[F(t)] = 2 + 2 = 2 2
s s +1 s (s + 1)

Gigatech Publishing House


Igniting Minds
Engineering Mathematics – III 2.24 Laplace Transformation (LT)

t  f(s) 2s2 + 1
 L  (x + sinx) dx  = s = s3 (s2 + 1)
0 
ii) Here F(t) = et sint , then
1
L[F(t)] = L[et sint] = (By Ist shifting property)
(s – 1)2 + 1
1
=
s2 + 2 – 2s
t x  1
 L  e + sinx dx  = 2
0  s(s – 2s + 2)
iii) Here F(t) = t2et, then
2
L[F(t)] = L[et t2] = (s – 1)3 (By Ist shifting property)

t  2
 L  x2 ex dx  =
0  s(s – 1)3

Example : 4

Find Laplace transform of :


i) t sin23t ii) t3e–3t iii) t cos at
iv) t2 sinbt v) te–t sin3t vi) sint – 2t cos2t
2
vii) t sin4t viii) te3t cos2t ix) (1 + tet)2
x) t sin3t cos2t xi) t2 e–2t cost
Solution :
1 – cos6t  1  1 s 
i) Since L[sin23t] = L = –
 2  2  s s2 + 36 
18
= s(s2 + 36)
d 18
 [t sin23t] = (–1) ds s(s2 + 36)

d 3
= – 18 (s + 36s)–1
ds
= –18 (–1) (s3 + 36s)–2 (3s2 + 36)
54 (s2 + 12)
=
(s3 + 36s)2
1
ii) Since L[e–3t] = s+3 , s > –3

d3  1 
 L [t3e–3t] = (–1)3
ds3  s + 3 

Gigatech Publishing House


Igniting Minds
Engineering Mathematics – III 2.25 Laplace Transformation (LT)
3 3
(–1) (–1) 3!
=
(s + 3)4
6
=
(s + 3)4
s
iii) Since L[cos at] = s2 + a2
d s (s2 + a2 – 2s2)
 L[t cos at] = (–1) ds  s2 + a2  = (–1) (s2 + a2)2
 
s 2 – a2
=
(s2 + a2)2
b
iv) Since L [sin bt] = s2 + b 2
d2 b
 L[t2 sinbt] = (–1)2 ds2  s2 + b2 
 
d2 2 d –2bs
= b ds2 (s + b2)–1 = b ds  (s2 + b2)2 
 
2 2
2b(3s – b )
=
(s2 + b2)3
3 3
v) Since L[e–t sin3t] = =
(s + 1)2 + 9 s2 + 2s+ 10
d 3
L[te–t sin3t] = (–1)  2 
ds  s + 2s+ 10 
d
= (–3) ds ( (s2 + 2s + 10)–1 )
6(s + 1)
=
(s2 + 2s + 10)2
1
vi) Since L[sin t] =
s2 + 1
s
and L [cos 2t] =
s2 + 4
 L [sint – 2t cost] = L[sint] – 2L [t cost]
1 d  s 
= s2 + 1 – 2(–1) ds  s2 + 4 
1 (s2 – 4)
= 2 –2 2
s +1 (s + 4)2
4
vii) Since L[sin4t] =
s2 + 16
d2 4 
 L[t2 sin4t] = (–1)2 2  2
ds  s + 16 
d2 1
= 4 ds2  s2 + 16 
 
Gigatech Publishing House
Igniting Minds
Engineering Mathematics – III 2.26 Laplace Transformation (LT)
2
d  –2s  8(3s – 16)
= 4 = 2
ds  (s2 +16)2  (s + 16)3
s–3
viii) Since L [e3tcos 2t] = (s – 3)2 + 4
s–3
= s2 – 6s + 13
d s–3
 L[te3t cos2t] = (–1) ds  s2 – 6s + 13 
 
[(s2 – 6s + 13) 1 – (s – 3) (2s – 6)]
= –
(s2 – 6s + 13)2
(s2 – 6s + 5)
=
(s2 – 6s + 13)2
ix) We know (1 + tet)2 = 1 + t2e2t + 2tet
1 1
and L[et] = 2t
s – 1 L[e ] = s – 2
 L[1 + tet]2 = L[1 + t2e2t + 2tet]
= L[1] + L[t2e2t] + 2L[tet]
2
1 2 d  1  d  1 
= s + (–1) ds2  s – 2  + 2(–1) ds  s – 1 
1 2 2
= + +
s (s – 2)3 (s – 1)2
1
x) Since L[sin3t cos2t] = 2 L[sin5t + sint]
1 5 1 
= 2  s2 + 25 + s2 + 1 
d 1 5 1 
 L[t sin3t cos2t] = (–1)   2 +
ds  2  s + 25 s2 + 1  
1 d 5  d  1 
= –   2 +
2  ds  s + 25  ds  s2 + 1  
5s s
= +
(s2 + 25)2 (s2 + 1)2
s+2
xi) Since L[e–2t cost] = s2 + 4s + 5
d2 s+2
 L[t2 e–2t cost] = (–1)2 ds2  s2 + 4s + 5 
 
2
d  – (s + 4s + 3) 
=
ds  (s2 + 4s + 5)2 
–[(s2 + 4s + 5) (2s + 4) – 2 (2s + 4) (s2 + 4s + 3)
=
(s2 + 4s + 5)3
2(s3 + 6s2 + 9s + 2)
=
(s2 + 4s + 5)3
Gigatech Publishing House
Igniting Minds
Engineering Mathematics – III 2.27 Laplace Transformation (LT)

Example : 5

Find Laplace transform of :


(1 – et) e–2t – e–3t cos2t – cos3t
i) t ii) t iii) t
–2t 2
e sin t 1 – cost sin3t cost
iv) v) vi)
t t t
sin2t 1 – cost sin2t
vii) viii) ix)
t t2 t2
t t
etsint
x)  t dt ix) t  e–4t sin3t dt
0 0
Solution :
1 1
i) Since L[1 – et] = –
s s–1
 
1 – et  1 1 s
 L =   s – s – 1 ds =  log s –1 
 t  s s

 1 s 
=  Lt log – log s – 1
s 1 
1–
 s 
s
= – log
s–1
s–1
= log 
 s 
1 1
ii) Since L[e–2t – e–3t] = s+2 –s+3

e–2t – e–3t  1 1
 L =   s + 2 – s + 3  ds
 t  s

=  log s + 2
 s + 3 s
 1 + 2/s s+2
=  Lt log 1 + 3/s – log s + 3 
s 
(s + 2) (s + 3)
= – log = log
(s + 3) (s + 2)
s s
iii) Since L[cos 2t – cos3t] = s2 + 4 – s2 + 9

cos2t – cos3t  s s
 L =   s2 + 4 – s2 + 9  ds
 t  s

Gigatech Publishing House


Igniting Minds
Engineering Mathematics – III 2.28 Laplace Transformation (LT)

1 
= [log (s2 + 4) – log (s2 + 9)] s
2
1 (s2 + 4) 
= 2 log (s2 + 9) s

1 1 + 4/s2 s2 + 4 
= 
2
Lt log
1 + 9/s s + 9 
2 – log 2
s
1 (s2 + 9)
= log 2
2 (s + 4)
1 – cos2t  
iv) Since L [e–2t sin2t] = Le–2t 
  2 
1 –2t 1 –2t –2t
= 2 L[e (1 – cos2t)] = 2 L[e – e cos2t]
1  1 s+2 
= 2 s + 2 – (s + 2)2 + 4 

e–2t sin2t  1 1 s+2
 L = 2   s + 2 – (s + 2)2 + 4  ds
 t  s
 
1  1 
= log (s + 2) – log [(s + 2)2 + 4
2  2 s
1  (s + 2)2  
= 4  log (s + 2)2 + 4  s

1  1 (s + 2)2 
=  Lt log – log 
4  4 (s + 2)2 + 4 
s 1 + (s + 2)2
 
2
1 [(s + 2) + 4]
= log
4 (s + 2)2
1 s
v) Since L[1 – cost] = s – s2 + 1

1 – cost  1 s
 L =   s – s2 + 1 ds
 t  s
 s2  
=  logs – 1 log (s2 + 1) =
1 
log 2
 2  s 2  s +1 s

1  1 s2 
=  Lt log – log 2 
2  1 s +1
s 1 + s2
 
1 s2 + 1
= log  2 
2  s 

Gigatech Publishing House


Igniting Minds
Engineering Mathematics – III 2.29 Laplace Transformation (LT)

1
vi) Since L [sin3t cost t] = L[sin 4t + sin2t]
2
1  4 2 
= 2  s2 + 42 + s2 + 22 

sin3t cost  1  4 2 
L
2   s2 + 42 s2 + 22 
 = + ds
 t  s

1  1 –1  s  1 –1  s  
= 4
2 4 tan + 2 2 tan  2  s
4

1  –1  s  –1  s 
= 2  tan  4  + tan  2  s
1  –1  s  –1  s 
= 2  2 + 2 – tan  4  – tan  2 
1  s s
=  – tan–1   – tan–1  
2  4  2 
2
vii) Since L [sin 2t] = s2 + 2 2

sin2t 2
 L t  =  s2 + 22 ds
  s

=  2 1 tan–1  s 
 2  2  s
 s s
= – tan–1   = cot–1  
2 2 2
viii) We know by (v)
1 – cost  1 s2 + 1
L = log  2 
 t  2  s 
1 – cost  1 1 – cost  
 L = L 
 t2  t t 

1 (s2 + 1)
= 2 log s2 ds
s
Integrating by parts, we obtain
1 – cost  1 (s2 + 1) s2 (2s3 – 2s3 – 2s) 
L = log s –  s ds 
 t2  2 s 2 2
s +1 s 4
s
1 (s2 + 1) s2 s2  
= s log +2 2 ds
2 s 2
s + 1 s4  s
1 (s2 + 1) 
= s log + 2 tan –1
(s)
2 s 2
s
Gigatech Publishing House
Igniting Minds
Engineering Mathematics – III 2.30 Laplace Transformation (LT)
2
=
1   s +1 –1 
2 0 + 2  2  – s log s2 – 2tan s
.. 1 
 . Lt s log  1 + s2  = 0 
 s   
2
1  – s log s +2 1 – 2 tan–1s
=
2  s 
1
ix) Since L [sin2t] = 2 L [1 – cos2t]
1 1– 2s 
= 2  s s +4

sin2t 1 1 s
 L t  =   s – s2 + 4  ds
  2
s
1 (s2 + 4)
= 4 log s2 [Similar step as (v) above]

sin2t  1 (s2 + 4)
 L =  log ds
 t2  4
s
s2
Integrating by part, we obtain
1  (s2 + 4) s2  –8  
= s log –  s ds 
4  s 2
s +4 s 
2 3
s

1 s2 + 4 1 
= 4 s log s2 + 8  s2 +4 ds  s

1 s2 + 4 1 s 
= 4 s log s 2 + 4 tan
2 s
1 L+ 4 1  s2 + 4 1 s 
= s log (1 + 2) + 4 tan ()  s log
4 s  s  s2 + 4 tan 2
2
1 s +4   4 tan1 (s/2)
= 4 0 + 4(/2)  s log
 s2  
2
1 s +4
= 2  s log  2   4 tan1(s/2)
4  s  
1
(x) Since L [ et sin t] =
(s 1)2+ 1

et sin t 1
 L =  (s 1)2 + 1 ds
 t  s

= tan1 (s1)
 1  s

 1 1
= 2  tan (s 1) = cot (s1)

Gigatech Publishing House


Igniting Minds
Engineering Mathematics – III 2.31 Laplace Transformation (LT)

 t et sin t
 L  = 1 cot1 (s  1) [Using Theorem 2]
 t  s
o 
3
(xi) Since L [e4t sin 3t] = (s + 4)2 + 32
3
=
s2 + 8s + 25
 t  4t  1 3
 L   e sin 3t  = s s2 + 8s + 25 [using Theorem 2.]
0   
 t  d 3
 L  t  e 4t sin 3t  = ( 1) ds  s(s2 + 8s + 25) [using Theorem 3.]
 0   
d 3
= (3) ((s + 8s2 + 25s)1)
ds
= (3)(1) (s3 + 8s2 + 25s)2 (3s2 + 16s + 25)
3(3s2 + 16s + 25)
= (s3+8s2 + 25s)2
3(3s2 + 16s + 25)
=
s2(s2+8s + 25s)2

eat 1
Note : In general L  t  does not exist since  s + a ds does not exist.
 
s

Self-Assessment Exercise 2.3

Ex.1 Find the Laplace transform of


d5x
i) = 0, [where x(0) = x (0) = x(0) = 0 and x(0) = 1 = xiv (0)
dt5
s+1
Ans.: x(s) =
s5
d2y dy 2s10
ii) dt2 – 3 dt + 5y = 0, given that y(0) = 2, y(0) = 4. Ans.: y(s) = 2
s 3s+5
Ex.2 Find the Laplace transform of
t
s+1
i)  et cost dt Ans. : s(s2 + 2s + 2)
0
t
2
ii)  sin 2x dx Ans. : s(s2 + 4)
0
Ex.3 If L [F(t) ] = f(s) , then show that L [F(t)] = s f (s) – F(0). Use it to evaluate L[F (t) ],
s
when F(t) = e5t sint. Ans. : s2 + 10s + 26

Gigatech Publishing House


Igniting Minds
Engineering Mathematics – III 2.32 Laplace Transformation (LT)

Ex.4 If L [ F(t)] = f(s), then show that L[F(t) ] = s2f(s)  s F(0)  F(0). Use it to evaluate L
 16(s + 5)
[F(t)], when F(t) = e2t sin 4t. Ans. : 2
s + 4s + 20
Ex.5 Find the Laplace transform of
2s(s2  12)
i) t2 cos 2t Ans. :
(s2 + 4)3
2
ii) t2 e2t Ans. : (s +2)3
2a(s  a)
iii) teat sinat Ans. : 2
(s  2as + 2a2)2
16
iv) sin 2t  2t cost Ans. : 2
(s + 4)2
4(s + 1)
v) tet sin 2t Ans. : 2
(s + 2s + 5)2
3s 1 1
vi) t sin3 t Ans. : 2 (s2 + 1)2 – (s2 + 9)2
 
2(s + 4)
vii) t e4t sin 3t Ans. : (s2 + 8s + 25)2
2as
viii) t sin hat Ans. : 2 2 2
(s  a )
8(3s2  6s  13)
ix) t2 et sin 4t Ans. : 2
(s  2s + 17)3
1 3 6 6
x) (1 + tet)3 Ans. : + + +
s (s + 1)2 (s + 2)3 (s + 3)4
Ex.6 Find the Laplace transform of :
eat  ebt s+b
i) Ans. : log 
t s + a
cos at  cosbt 1 s2 + b2
ii) Ans. : log 2
t 2 s + a2
e 4t sin 3t (s + 4)
iii) t Ans. : cot1 3
1  cos2t 1 s2 + 4
iv) t Ans. : 2 log s2

sin2t 1 s2 + 4
v) Ans. : log 2
t 4 s
sinht 1 s+1
vi) Ans. : 2 log
t s1
eat  cos bt 2 2
 s +b 
vii) Ans. : log  
t  sa 
Gigatech Publishing House
Igniting Minds
Engineering Mathematics – III 2.33 Laplace Transformation (LT)

sin t sin 5t 1 s2 + 36
viii) Ans. : log 2
t 2 s + 16
1  cos3t 1 s2 + 9
ix) t Ans. : 2 log s2
cos 2t  cos 3t  et + 1 1 s2 + 9 1
x) Ans. : 2 log s2 + 4 + log 1  s 
t  
Ex.7 Evaluate :
 t t sin t  1
i) L  e  dt  Ans. : cos 1(s + 1)
t s+1
 0 
t t
 e sin t  1 s
ii) L  t  dt  Ans. : + cot1(s + 1)
t s2 s2 + 2s + 2 
 0 
t
  8(s + 1)
iii) L   tet sin4t dt  Ans. :
s(s2 + 2s + 17)2
0 
t
  6
iv) L   t sin 3t dt  Ans. :
(s2 + 9)2
0 
Ex.8 Obtain Laplace transform of following functions :
t
1 ex 1 (s + 1)
(i)  x
dx Ans. : log
s s
0
t
1 6(s + 4)
(ii)  t e4t sin 3 t dt Ans. :
s (s2 + 8s + 25)2
0
t
6
(iii) e4t  t sin 3t dt Ans. : (s2 + 8s + 25)2
0
Ex.9 Show that :
 t et  cos 2t  1 s2 + 4
(i) L  dt = log
0
t

2s (s 1)2
t 3t
 e sin 2t  1 1 s+3
(ii) L  dt = cot
0
t

s  2 
 t 3t  6s2 + 24s + 26
(iii) L t  e sin 2t dt =
s2 (s2 + 6s + 13)2
 0 
1  cos at 1
Ex.10 If L  = , then show that
 a2  s(s2 + a2)
t(1  cos at) 3s2 + a2
L =
 a 2
 s (s2 + a2)2
2

Gigatech Publishing House


Igniting Minds
Engineering Mathematics – III 2.34 Laplace Transformation (LT)

The convolution of two functions F(t) and G(t) is denoted by F(t)*G(t) and is defined as
t
F(t) * G(t) =  F(u) G (t  u) du
0
 Properties of Convolution :
(i) F * G = G * F ( Connmutativity)
(ii) (F * G) * H = F * (G * H) (Associativity)
(iii) F * (G1 + G2) = F * G1 + F * G2 (Distributivity)
(iv) F*0=0*F=0

If L [F(t)] = f (s) & L [G(t)] = g(s), then


t 
L [ F(t) * G(t)] = L   F(u) G (t  u) du  = F(s) g (s)
0 
Note : 1 Since convolution of F(t) & G(t) is commutative, then we have
t 
L [F(t) * G(t)] = L   F (t  u) G (u) du  = f(s) g (s)
0 
Note : 2 The convolution theorem is useful to find inverse transform.

Theorem : 5

If L [F(t) = f(s), then Lt F(t) = Lt s F(s) provided these limit exist.


t0 s

Laplace transforms are used to find solution of particular definite integral of type

 est F(t)dt, provided L (F(t)) exist.
0
 Working Procedure:
(i) Compare the given integral with standard form.
(ii) Find value of ‘s’ and F(t).
(iii) Evaluate L [F(t)] & find F(s).
(iv) Substitute value of ‘s’ to find the value of required Integral.
Gigatech Publishing House
Igniting Minds
Engineering Mathematics – III 2.35 Laplace Transformation (LT)

Illustrative Examples

Example : 1

Verify Convolution theorem for following pair of functions :


(i) F(t) = eat & G(t) = 1
(ii) F(t) = 1 & G(t) = sin wt
(iii) F(t) = cos wt & G(t) = cos wt
Solution :
(i) Here F(t) = eat
1
 f(s) = L [F(t) ] =
sa
And G(t) = 1
1
 g(s) = L [G(t) ] = s

Then,
1
R.H.S = F(s) g(s) =
s(s  a)
t 
L.H.S = L [F(t) * G(t) ] = L   F(u) G (t  u) du 
0 
t
 1 
= L   eau 1 du] = L [ (eat 1) 
a
0 
1 1 1 1
= a  =
s  a s  s(s  a)
 L.H.S = R.H.S Hence verified.
(ii) Here F(t) = 1
1
 L[F(t) ] = = F(s)
s
and G(t) = sin wt
w
 L[G(t)] = s2 + w2 = g(s)
Then,
w
R.H.S = F(s) g(s) = s(s2 + w2)

t 
L.H.S = L [F(t) * G(t)] = L   1 sin w (t u) du 
0 

Gigatech Publishing House


Igniting Minds
Engineering Mathematics – III 2.36 Laplace Transformation (LT)

t 
= L   sin (wt  wu) du 
0 
1 1 1 s
= L  w (1 cos wt)  = w  s  s2 + w2
   
w
= s(s2 + w2)
 L.H.S = R.H.S Hence verified.
(iii) Here F(t) = G(t) = cos wt
s
 L [ F(t) ] = L [G(t)] = s2 + w2

Then
s2
R.H.S = F(s) g(s) =
(s + w2)2
2

t 
L.H.S = L [F(t) * G(t) ] = L   F(u) G (t u) du 
0 
t
 
= L   cos wu. cos w (t u) du 
0 
t
 1 
= L   2 [ cos wt + cos (2 wu  wt)] du 
0 
1  sin wt 
= 2 L  t cos wt + w 
1 d s  1 w 
= (1)  2 +
2 ds s + w2 w s2 + w2 
1 s2 w2 + s2 + w2 s2
= 2  (s + w ) =
2 2 2
 (s + w2)2
2

 L.H.S = R.H.S Hence verified.

Example : 2

Verify Initial Value theorem :


(i) t + sin 2t (ii) (2t + 4)2 (iii) sin t + cos t
Solution :
(i) Here F(t) = t + sin 2t
1 2
 f(s) = s2 + s2 + 4

Now, L.H.S. = Lt F (t) = Lt (t + sin 2t) = 0


t0 t0

Gigatech Publishing House


Igniting Minds
Engineering Mathematics – III 2.37 Laplace Transformation (LT)

1 2
R.H.S. = Lt s f(s) = Lt s s2 + s2 + 4 = 0
s s
 
 L.H.S. = R.H.S. Hence verified.
(ii) Here F(t) = (2t + 4)2 = 4t2 + 16t + 16
4.2! 16 16
 L [F(t)] = f(s) = 3 + 2 +
s s s

Now L.H.S = Lt F(t) = 16


t0

R.H.S = Lt f(s) = Lt  82 + 16 + 16


s s
s s 
L.H.S = R.H.S Hence verified.
(iii) Hence F(t) = sin t + cos t
1 s
 L[F(t) = F(s) = s2 + 1 + s2 + 1

Now, L.H.S. = Lt F(t) = Lt (sin t + cost) = 1


t0 t0

1 s
R.H.S. = Lt s F(s) = Lt s s2+1  s2 + 1
s
 s

= Lt  Ys 2 + 1 2 = 1
s
1+Ys 1 + Ys 
L.H.S = R.H.S Hence verified.

Example : 3

Evaluate following integrals :


  
sin at cos sin bt  cos 4t
(i)  t dt (ii) e 2t
t sin t dt (iii)  et t dt
0 0 0
 2t 2  
e sin t
(iv)  t
dt (v)  te3t sint dt (vi)  t3 et sin t dt
0 0 0
  
et e 3t
(vii)  t dt (viii)  e2t sin3 t dt (ix)  t2 e3t sin 2t dt
0 0 0

(x)  te 4t cost dt.
0

Gigatech Publishing House


Igniting Minds
Engineering Mathematics – III 2.38 Laplace Transformation (LT)

Solution :
(i) We know by definition

L[F(t)] =  est F(t) dt = F(s)
0
 
sin at sin at
Now  t dt =  eot t dt
0 0
sinat
= L , where s = 0 [ by above definition]
 t 
a
Since L [sin at] = s2 +a2

sin at a
 L t  =  s2 +a2 ds [  by Theorem 4]
 
0
1 s   s
= a  a tan1 a = 2  tan1 a
   s  

sinat  1  s
  t dt = 2  tan a , where s = 0
s

Since Lt tan1 (s/a) =  0  if a >0


s0
  if a < 0

sinat  /2 ; if a >0
  t
dt = 
 /2 ; if a < 0
s

 
(ii) e 2t
t sint dt =  e2t (t sin t) dt
0 0

F (t) = t sint
=  est F(t) dt, where 
 s=2 

0
F (t)= t sint
= L (F(t) , Where  s=2 


d 1
Now L [ t sint ] = (1)  2  [  by Theorem 3]
ds s +1
2s
=
(s + 1)2
2


2s
  e2t t sin t dt = (s2 +1)2 Where s = 2
0

Gigatech Publishing House


Igniting Minds
Engineering Mathematics – III 2.39 Laplace Transformation (LT)

2×2 4
= 2 2=
(2 +1) 25

4
i.e.  e  2t t sin t dt = 25
0
 
cos 6t  cos4t
(iii)  e t
t =  est F(t) dt = L [F(t)]
0 0
cos 6t  cos4t
Where, F(t) = t
and s = 1

cos 6t  cos 4t s s
Now L =  s2 + 36  s2 + 16 ds [  using theorem 4.]
 t 
s

1
= 2 [log (s 2
+ 36)  log (s2
+ 16)]
s
2 
1  s + 36 
= 2 log s2 + 16s
1 s2 + 16
= 2 log s2 + 36

(cos 6t  cos 4t) 1 s2 +16
  et t dt = 2 log s2 + 36 , where s = 1
0
1 17
= log
2 37
Note : If we put s = 0 , we get the value of Integral.

cos 6t  cos 4t 1 16 1 4 2
 t dt = 2 log 36 = 2 log 9 = log 3
0
 
sin2t
(iv) e 2t
t
dt =  est F(t) dt = L [F(t)]
0 0
sin2t
where F(t) = t
and s = 2
2
sin t 1 1  cos 2t
Now, L t  = 2L
  t 

1  1 s  
= 2    s s + 4 
 2 ds [  by using Theorem 4)
s 

Gigatech Publishing House


Igniting Minds
Engineering Mathematics – III 2.40 Laplace Transformation (LT)

1 1 
= log s  log (s2
+ 4) 
2 2 s
2  2
1  2s  = 1 log s +24
= log
4 s +4 s 4  s 

e2t sin2t 1 2
s +2 4, where s = 2
  t dt = 4 log
 s 
0
1
= log 2.
4
 
(v) e 3t
(t sint) dt =  est F(t) dt = L[F(t)]
0 0
where F(t) = t sin t and s = 3
2s
Following (ii) , we obtain L [ t sint] = (s2+1)2

2s
  e3t sint dt = (s2 +1)2 , where s = 3
0
3
=
50
 
(vi) t 3 t
e sin t dt =  e st F(t) dt = L [F(t)]
0 0
where F(t) = t3 sin t and s =1
d3  1 
Now L [t3 sin t] = ( 1)3 [  using theorem 3]
ds3 s2 + 1
d2 2s d 2(s2 +1) 8s2
=  ds2 (s2 + 1)2 = ds  (s2 + 1)3 
   
2
d 2(1  3s )
= ds  (s2 + 1)3 
6s(s2 +1)  6s(1 3s2) 24s(s2 1)
= 2 =
 (s2 + 1)4  (s2 +1 )4

24 s (s2 1)
  t3 et sin t dt = (s2 +1)4 , where s = 1
0
= 0
 
et  e3t
(vii)  t
dt =  est F(t) dt = L [F(t)]
0 0
et  e3t
Where F(t) = and s = 0
t
Gigatech Publishing House
Igniting Minds
Engineering Mathematics – III 2.41 Laplace Transformation (LT)


et e3t
Now L =  L [ et  e3t ]ds { By theorem 4]
 t 
0

 1 1   s+1
=  s+1 s+3
 ds =

log s+3s
s
s+1 s+3
= 0  log s + 3 = log s + 1

et e3t s +3
  t dt = log s+1 , where s = 0
0
= log 3
 
(viii) e 2t 3
sin t dt =  est F(t) dt = L [F(t)]
0 0
where F(t) = sin3 t and s = 2
1
Now L [sin3 t] = L [  sin 3 t + 3 sin t]
4
1  3 3  6
= + =
4 s2 + 9 s2 +1 (s2 + 9) (s2 + 1)

6
  e2t sin3 tdt =
(s2 + 9) (s2 + 1)
, where s = 2
0
6 6
= =
13 × 5 65
 
(ix) t 2
e 3t
sin 2t dt =  est F(t) dt = L [F(t)]
0 0
Where F(t) = t2 sin 2t and s = 3
d2 2 d 4s
L[ t2 sin 2t] = (1)2 ds2 s2 +4 = ds (s2 +4)2
 
s2 +4  4s2 4(3s2  4)
= (4)  (s2 + 4)3  = (s2 + 4)3
 

4(3s2  4) 92
  t2e3t sin2tdt = (s2 + 4)3 = 2197 as s  3.
0
 
(x)  t e4t cost dt =  est F(t) dt = L [F(t), where F(t) = t cost and s = 4
0 0
d  s  s2 1
Now L [t cost] = (1) = 2 [  By Theorem 3]
ds s + 1 (s +1)2
2

Gigatech Publishing House


Igniting Minds
Engineering Mathematics – III 2.42 Laplace Transformation (LT)


s2 1
  t e 4t cost dt =
(s2 + 1)2
, where s = 4
0
15
=
289

Example : 4

Using power series expansion, find Laplace transform of :


cos t
(i) sin t (ii)
t
Solution :
(i) We know by power series expansion.
t3 t5 t7
sin sin t = t +  +…
3 5 7
1/2
t t3/2 t5/2 t7/2
Then sin t =  +  +…
1 3 5 7
Taking Laplace on both side, we set
t1/2 t3/2 t5/2 t7/2
L [sin t ] = L  +  + …
 1 3 5 7 
3/2 1 5/2 1 7/2 1 9/2
= s 3/2  3! s5/2 + 5! s7/2  7! s9/2 + …

n +1
[⸪ tn = sn+1 and also n +1 = n. n and 1/2 =  ]

1/2  1 3/2  1/2 


5/2 3/2 1/2  7/2 5/2 3/2 1/2 
= s 3/2  3! s5/2 +
s9/2  s9/2 …

  12 2  12 3 
   1  2 s 2 s 
= 3/2 1  2 +  + …
2s  2 s 2! 3! 
1
  t2 t3
= 2s3/2 e 4s [... eet = 1  t + 2!  3! + ...]

 e1/4s
i.e. L [sin t ] =
2 s3/2
t2 t4 t6
(ii) We know, cost = 1  +  +…
2! 4! 6!
Then
( t)2 ( t)4 ( t)6
cos t = 1  2! + 4!  6! + …

Gigatech Publishing House


Igniting Minds
Engineering Mathematics – III 2.43 Laplace Transformation (LT)
2 3
t t t
= 1  2! + 4!  6! + …
1 3 5
1
cos t  t 2 t 2 t2
 = t 2 +  +…
t 2! 4! 6!
Taking Laplace on both side, we obtain
 1 12 3 5

cos t   2 t t
2
t
2 
L  = L t  2! + 4!  6! + …
 t 
1/2 1 3/2 1 5/2 1 7/2
= s 1/2  2! s3/2 + 4! s5/2  6! s7/2 + …

 1 1/2  1 3/2 1/2  1 5/2 3/2 1/2 


= s1/2 2! s3/2 + 4! s5/2 6! s7/2 +…

  1 1 1 2 1 1 3


= 1 + – + …
s1/2
 4s 2! 4s 3! 4s 
1
 4s . . t t2 t3
= 1/2 e . e = 1  t +  + …
s  2! 3! 

(I) Periodic Functions :


(a) Definition :
A function F(t) is said to be periodic function with period T ( 0) if
F (t + T) = F(t) for all t > 0.
Example of periodic function :
Consider F(t) = sin t = 0 then F (t + 2) = Sin (t + 2) = Sint . i.e. F(t) in periodic
with period 2. The periodic wave form F(t) is shown graphically below ( Fig. No.
2.3)

Fig.: 2.3 : Graphical representation of sin t.

(b) Laplace transform of periodic function :


If F(t) is a periodic function with period T. then.

Gigatech Publishing House


Igniting Minds
Engineering Mathematics – III 2.44 Laplace Transformation (LT)

T
1
L [F(t)] =
1  esT
 est F(t) dt.
0

(II) Bessel functions Jo (x) and J1 (x)


We know that
x2 x4 x6
J0 (x) = 1 2+ 2 2 2 2 2+ …
2 2 .4 2 .4 .6

And J1 (x) =  J0 (x)
Then Laplace transform is defined as.
1
L[J0(x)] =
s2 +1
s
[... L [J0 (x)] = s J0 (s)  J0(0)

L [J1 (x)] = 1 2
s +1
i.e.
1
L [ J0 (x)] = 2
s +1
s
L [J1 (x)] = 1  2
s +1
(III) Unit Step functions (or Hear side function)
(a) Definition (Unit step function ) :
The unit step function U(t) is defined as follows :
0 ;t<0
U(t) =  1 ;t>0

(b) Definition (Displaced unit step function) :


The displaced unit step function U(t a) is defined as
0 ;t<a
U(t  a) =  1 ;t>a

Graphical representation of both functions are shown below in Fig. 2.4 (a) &
Fig, 2.4(b).

Fig.: 2.4

Gigatech Publishing House


Igniting Minds
Engineering Mathematics – III 2.45 Laplace Transformation (LT)

Clearly, we can observe that (from Fig. 2.4) displaced unit step function U(t a)
represents curve U(t) which is shifted ‘a’ units to the right along t–axis.
Note : 1
It is observed that at t = 0 & t = a, a step of unit height is formed, so it justify the name
unit step function.
Note : 2
From fig 2.4 , it is also observed that unit step functions U(t) & U (t a) are extensively
used to represent a part of the curve F(t).
Some other forms of unit step functions and displaced unit step functions are discussed
below :
c) Suppose that the function F(t) is multiplied with unit step function U(t) i.e. F(t)
U(t). Then F(t) U(t) is defined as :
0 ; t<0
F(t) U(t) = 
 F(t) ; t  0

F(t) U(t) only represents the portion of function F(t) on the right of the origin [See Fig.
2.5 (c)]

Fig.: 2.5

d) Suppose that the function F(t) multiplied with displaced unit step function U(t – a)
i.e. F(t) U(t – a), then
0 ; t<a
F(t) U(t – a) =  .... (7)
 F(t) ; ta

e) Suppose function F(t) is displaced by (t – a) i.e. F(t – a) and then multiplied with
displaced unit step function U(t – a) i.e. F(t – a) U (t – a). Then
 0 ; t<a
F(t – a) U(t – a) =  .... (8)
 F(t – a) ; t  a

Graphical representation of such type of functions [equation (7) and equation (8)] are
shown below with particular example.
Gigatech Publishing House
Igniting Minds
Engineering Mathematics – III 2.46 Laplace Transformation (LT)

 Example : 1
Consider F(t) = 5 sin t. We want displaced unit step function by t = 2 sec. i.e. U(t – 2)
Then
 0 ; t<2
F(t) U(t – 2) = 
 F(t) = 5 sint ; t  2
Also, if we shift function F(t) towards right by 2 unit, say 2 sec, then
0 ; t<2
F(t – 2) U(t – 2) = 
 5 sin(t – 2) ; t  2

Fig.: 2.6

Fig. 2.6 (a) represents the given function F(t) = 5 sint graphically in Fig. 2.6 (b), it is
switched off between t = 0, t = 2.
(Because u(t – 2) = 0, when t < 2) and is switched on start at t = 2.
In Fig. 2.6 (c), it is shifted to right side by 2 sec so that it begins 2 sec later in the same
fashion as before.
f) Suppose that the portion of the curve F(t) lying between a  t  b. [as shown in Fig.
2.7 (b)]. In such case we take difference of two displaced unit step functions which
generate a rectangular pulse. [as shown in Fig. 2.7 (a)]. i.e. we take
H(t) = [U (t – a) – U (t – b)]

Fig.: 2.7

If any other function F(t) is multiplied by H(t), then F(t) become zero outside the
interval a  t  b. It reproduces without any changes for all values of t within the interval
t  [a, b]

Gigatech Publishing House


Igniting Minds
Engineering Mathematics – III 2.47 Laplace Transformation (LT)

This H(t) is also known as “Filter function”


Hence the portion of the function F(t) laying between a  t  b is defined as :
0 ; t<a
F(t)  H(t) =  F(t) ; a  t  b .... (9)
0 ; t>b
Now we find the Laplace transform of all such kind of unit step functions.
g) Transforms of various kind of unit step function.
 
1
i) L[U(t)] = e –st
U(t) dt = e –st
1dt = s , s > 0
0 0

1
i.e. L[U(t)] =
s

ii) L [U(t – a)] = e –st


U(t – a) dt
0
a 
e–as
= e –st
(0) dt + e –st
1dt = s , s>0
0 0

e–as
i.e. L[U(t – a)] =
s

iii) L [F(t) U(t)] = e –st


F(t) U(t) dt
0

= e –st
F(t) dt = f(s)
0

i.e. L[F(t) U(t)] = f(s)



iv) L [F(t) U (t – a)] =  e–st F(t) U(t – a) dt
0

= e –st
F(t) dt [... by equation (7)
a

ta t=
Put t – a = z  dt = dz As
z0 z=

 L[F(t) U(t – a)] =  e–st (a + z) F(a + z) dz
0

Gigatech Publishing House


Igniting Minds
Engineering Mathematics – III 2.48 Laplace Transformation (LT)

= e –as
 e–sz F(a + z) dz = e–as L[F(a + t)]
0
–as
i.e. L[F (t) U(t – a)] = e L[F(t + a)]

v) L [F(t – a) U (t – a)] =  e–st F(t – a) U(t – a) dt
0

= e –st
F(t – a) dt [... by equation (8)
a
t=a t=
Put t – a = z  dt = dz Also as
z=0 z=
 

 L[F(t – a) U(t – a)] = e –s (a + z)


F(z) dz =  e–as e–az F(z) dz
0 0

= e–as  e–az F(z) dz
0
–as
i.e. L[F(t – a) U(t – a) = e f (s)
Note 1 : The above result is also equivalent to second shifting property.
Note 2 : If we put a = 0 in above identity, we obtain,
L[F(t) U(t)] = eos F(s) = f(s)
A similar result as in case (iii)
Note 3 : Unit step function are also denoted by H(t), H(t – a)
IV) Unit Impulse Function (Or Dirac–delta function)
a) Definition : Consider a function F(t) defined as :
0 ; t<a
1 
F(t) =   ; ata+ .... (10)
0  ; t>a+
Then the limiting form of the function F(t) with the property that its integral is
equal to 1 is called unit impulse function or Dirac delta function. It is denoted by
(t – a) and defined as :

(t – a) = Lt F(t)
0

In particular, if we put a = 0, we obtain


0 ; t<a
1
(t) = Lt F(t) where F(t) =   ; 0t
0
0 ; t>

Gigatech Publishing House


Igniting Minds
Engineering Mathematics – III 2.49 Laplace Transformation (LT)

b) Relation between unit step function & unit impulse function :


Above function F(t) in equation (10) can be expressed in terms of unit step function
as :
1
F(t) = [U(t – a) – U (t – a – )]

 U(t – a) – U(t – a – ) 
Lt 
then, Lt F(t) = 
0 0
  

= U (t – a)  ... d U(t – a) = U (t – a)
 dt
i.e. (t – a) = U(t – a) .... (11)
c) Transform of unit impulse function :
From equation (10)

L[F(t)] = e –st
F(t) dt
0
a a+ 
1
= e –st
(0) dt +  e –st

dt +  e–st (0) dt
0 a a+
a+
1 e–as  1 – e–s 
=
  e–st dt =
s   
a

 L[(t – a)] = Lt L[F(t)]


0

e–as  1 – e–s  e–as


= Lt = (s) = e–as
s    s
0

Hence L[(t –a)] = e–as

In particular, if a = 0, then
L [(t)] = 1

d) Shifting property of (t – a) :


It is given by  F(t) (t – a) dt = F(a) .... (12)


–

Its Laplace transform is defined as :


L [F(t) (t – a) = eas F(a)

Gigatech Publishing House


Igniting Minds
Engineering Mathematics – III 2.50 Laplace Transformation (LT)

Illustrative Examples

Example : 1

Find the Laplace transform of following function :


1 ; 0<t<a
i) F(t) =  and F(t + 2a) = F(t) (Square wave)
 –1 ; a < t < 2a
 E sint  0 < t < /
and F  t +
2  Half wave rectified
ii) F(t) =   = F(t)  
0  / < t < 2/    sin wave

t
a  0<t<a
iii) F(t) =  2a – t and F(t + 2a) = F(t) (Triangular wave)
 a  a < t < 2a

K
iv) F(t) =  T t  0 < t < T and F(t + T) = F(t) (Saw tooth wave)

Also sketch the graph of the function.


Solution :
i) Since the function is periodic with period T = 2a, and is given as ;
1 ; 0<t<a
F(t) = 
 –1 ; a < t < 2a
The graph of the function is shown below (Fig. 2.8)

Fig.: 2.8 – An alternating square wave


T
1
Now, L[F(t)] =  –st
1 – esT e F(t) dt
0
2a
1
=
1 – e–sT e –st
F(t) dt { ... T = 2a
0
a 2a
1  –st 
=  e –st
(–1) dt 
1 – e–sT  e (1) dt + 
0 0 

Gigatech Publishing House


Igniting Minds
Engineering Mathematics – III 2.51 Laplace Transformation (LT)
a 2a
1   e–st   e–st   1 –as –2as
 1 – 2e + c 
= –sT  +  =
1 – e   –s   –s   1 – e –2as
s 
0 0
–as 2
1 (1 – e ) 
= s  (1 – e–as) (1 + e–as) 
1  1 – e–as  1 as x
 ... tanhx = ex – e –x
–x
= s  1 + e–as  = s tan h 2  e +e
 E sint  0 < t < /
and F  t +
2 
ii) Given : F(t) =   = F(t)
0  / < t < 2/   
2
Clearly F(t) is periodic with period T =

Graph of F(t) is shown below (Fig. 2.9)

Fig.: 2.9 – Hall wave rectification of E sinwt.


T 2/
1 1
Now, L[F(t)] = 
–st
1 – esT e F(t) dt = –2s  e–st F(t) dt
0  0
1–e
/ 2/
1  
 e–st E sint dt + e–st (0) dt 
= –2s    
1–e  0 / 
/
1
= –2s  E e–st  sint dt
 0
1–e
E –st /
=  2 e 2 [–s sint –  cost] 
–2s s +  0

1–e
ax
 ...  eas sinbx dx = 2e 2 [asinbx – bcosbx] 
 a +b 
–s
E   
= –2s   1 + e  
 
1 – e   (s2 + 2)
E
= –s
 

(s2 + 2) 1 – e 
Gigatech Publishing House
Igniting Minds
Engineering Mathematics – III 2.52 Laplace Transformation (LT)

t
a  0<t<a
iii) Given : F(t) =  2a – t and F(t + 2a) = F(t)
 a  a < t < 2a

Clearly, F(t) is periodic with period T = 2a.


Graph of F(t) is shown in Fig. 2.10
Now,
T
1
L[F(t)] = 1 – e–sT e –st
F(t) dt Fig.: 2.10
0
2a
1
=
1 – e–2as e –st
F(t) dt
0
a 2a
1  (2a – t) 
 e–st t dt +
=
1–e –2as
 
a e –st
a
dt

0 0 
–st –st –st –st
=
1   t e – e  +  (2a – t)  e  – e  2a 
a

1–e –2as
  –s s  0 
2
 –s  –s  a 
–as –2as
1  1 – 2e 2 + e 
= a(1 – e ) 
–2as
s 
–as 2
(1 – e ) 1 as
= as2 (1 – e–as) (1 + e–as) = as2 tanh 2
 Kt
iv) Given : F(t) =   0 < t < T and F(t + T) = F(t)
 T
Clearly, F(t) is periodic with period T.
Graph of F(t) is shown below in Fig. 2.11
Now,
T
1
L[F(t)] = 1 – e–sT e –st
F(t) dt
Fig.: 2.11 – Saw tooth wave
0
T T
1 Kt K
=
1 – e–sT e –st
T
dt =
T(1 – e–sT)  te –st
dt
0 0
–st –st T
K  t  e  – e2 
=
T(1 – e )   –s  s  0
–sT

–sT
K 1  12 (1 – e–sT) – T e 
=
T (1 – e )  s
–sT
s 
K K e–sT
= –
Ts2 s (1 – e–sT)

Gigatech Publishing House


Igniting Minds
Engineering Mathematics – III 2.53 Laplace Transformation (LT)

Example : 2

1
If L[J0(t)] = 2 , show that
s +1
1 s
i) L[J0 (at)] = 2 ii) L[t J0(at)] = (s2 + a2)3/2
s + a2
 
2 3
iii)  –t
e J0(t) dt =
2
iv)  te –3t
J0 (4t) dt =
125
0 0
Solution :
1
i) Given : L[J0 (t)] = 2 = J(s)
s +1
then
1 s
L[J0 (at)] = aJa [... By change of scale property]
1 1 1
= a = 2
s 2
  +1 s + a2
a 
1
 L[J0 (at)] =
s + a2
2

1
ii) Since L[J0(t)] =
s + a2
2

1
and from (i) L[J0(at)] =
s + a2
2

d 1
then L[t J0(at)] = (–1) ds
s2 + a2
s
=
(s2 + a2)3/2
 

iii) e –t
J0(t) dt = e –st
F(t) dt = L[F(t)]
0 0
where F(t) = J0(t) and s = 1
1
 L[F(t)] = L[J0(t)] = 2
s +1

1
 e –t
J0(t) dt =
s2 + 1
, where s = 1
0
1 2
= =
2 2
Gigatech Publishing House
Igniting Minds
Engineering Mathematics – III 2.54 Laplace Transformation (LT)

 

iv)  te –3t
J0 (4t) dt = e –st
F(t) dt = L[F(t)]
0 0
where, F(t) = t J0 (4t) and s=3
Now,
d 1
L[F(t)] = L [t J0 (4t)] = (–1) ds (s2 + 42)1/2
s
= (s2 + 16)3/2

s 3 3
  te –3t
J0 (4t) dt = (s2 + 16)3/2 = (25)3/2 = 125
0

Example : 3

Find the Laplace transforms of :


i) t3 U(t – 2) ii) sint U(t – 4) iii) (t – ) U(t – )
iv) e–t sint U(t – ) v) e–t [1 – U(t – 2)]
Solution :
i) L [t3 U(t – 2)] = L[F(t) U (t – a)] where F(t) = t3 and a = 2
= e–as L[F(t + a)] = e–2s L[F(t + 2)]
= e–2s L [(t + 2)3] = e–2s L[t3 + 8 + 6t2 + 12t]
3! 8 6.2! 12
= e–2s  s4 + s + s3 + s2 
 
6 12 12 8 
= e–2s  + + +
 s4 s3 s2 s 
ii) L [sint U(t – 4)] = L[F (t) U(t – a)] where F(t) = sint and a = 4
–as
= e L [F(t + a)]
–4s
= e L [sin (t + 4)]
–4s
= e L[sint cos4 + cost sin4]
cos4 s sin4 
= e–4s  2 +
 s + 1 s2 + 1 
(cos4 + s sin4)
= e–4s
(s2 + 1)
iii) L[(t – ) U(t – )] = L[F(t – a) U(t – a)] where F(t) = t and a = 
–as –s
= e  F(s) = e L[F(t)]
e–s
= e–s L[t] = 2
s

Gigatech Publishing House


Igniting Minds
Engineering Mathematics – III 2.55 Laplace Transformation (LT)

iv) L[e–t sint U(t – )] = L[F(t) U(t – )] where F(t) = e–t sint
= e–s L[F(t + )]
= e–s L[e–(t + ) sin (t + )]
= –e–(s + 1) L [e–tsint]
1
= –e–(s + 1) 2
(s + 2s + 2)
v) L[e–t [1 – U(t – 2)] = L[e–t] – L[e–t U(t – 2)]
1 –t
= s + 1 – L[F(t) U (t – 2)] , where F(t) = e
1 –2s 1 –2s –(t + 2)
= s + 1 – e L[F(t + 2)] = s + 1 – e L[e ]
1 1
= – e–2s  e–2
s+1 s+1
[1 – e–(2s + 2)]
=
(s + 1)

Example : 4

Using Laplace transform, evaluate :


 
i)  –t
e cost H (t – 1) dt ii)  e–2t sint U(t – ) dt
0 0
Solution :
 
i) e –t
cost H (t – 1) dt =  e–st F(t) H (t – 1) dt
0 0

= L[F(t) H (t – 1)] [... By definition]


where F(t) H (t – 1) = cost H(t – 1)
and s = 1
Now, L[F(t) H(t – 1)] = e–s L[F(t + 1)] = e–s L[cos (t + 1)]
= e–s L[cost cos1 – sint sin1]
s cos1 sin1 s cos1 – sin1 
= e–s  s2 + 1 – s2 + 1  = e–s 
   s2 + 1 

s cos1 – sin1 
  e–t cost H (t – 1) dt = e–s  , where s = 1
 s2 + 1 
0
cos1 – sin1 
= e–1 
 2 
1  cos1 – sin1 
=
e  2 
Gigatech Publishing House
Igniting Minds
Engineering Mathematics – III 2.56 Laplace Transformation (LT)

 
ii)  e –2t
sint U(t – ) dt =  e–st F(t) U(t – ) dt where F(t) = sint and s = 2
0 0
= L[F(t) U(t – )] = e–s L[F(t + )]
= e–s L[sin (t + )] = e–s L[– sint]
–e–s
= s2 + 1

–e–s
  e–2t sint U(t – ) dt = s2 + 1 , where s = 2
0
–2 –2
=  –e  = e [... Area cannot be –ve]
 5  5
Example : 5

Express the following functions in Heaviside unit step functions and hence find their
Laplace transforms.
 sint  0t<
 t2  0<1<1
i) F(t) =  sin2t    < 2 ii) F(t) = 
 sin3t  t  2  4t  t>1
 sin2t  0<t<  cost  0<t<
iii) F(t) =  iv) F(t) = 
0  t>  sint  t>
Solution :
i) We can express function F(t) into Heaviside unit step function as :
F(t) = sint [U (t–0) – U(t – )] + sin2t [U(t – ) – U(t – 2)] + sin3t U(t – 2)
F(t) = sint U(t) + (sin2t – sint) U(t – ) + (sin3t – sin2t) U(t – 2)
Taking Laplace on both sides, we get
L[F(t)] = L[sint U(t)] + L[(sin2t – sint) U(t – )] + L[(sin3t – sin2t) U(t – 2)]
1 2 1  3 2 
= 2 + e–s  2 – + e–2s  2 –
s +1  s + 4 s2 + 1  s + 9 s2 + 4 
ii) We can write F(t) in terms of unit step function as
F(t) = t2 [U(t) – U(t – 1)] + 4t [U(t – 1)]
= t2 U(t) + (4t – 1) U(t – 1)
taking Laplace on both sides, we obtain
L[F(t)] = L[t2U(t)] + L[(4t – t2) U(t – 1)]
2
= s3 + e–s L[4(t + 1) – (t + 1)2]
2
= 3 + e–s L[3 + 2t – t2]
s
2 3 2 2
= 3 + e–s  + 2 – 3 
s s s s 
Gigatech Publishing House
Igniting Minds
Engineering Mathematics – III 2.57 Laplace Transformation (LT)

iii) We can express F(t) in terms of unit step function as :


F(t) = sin2t [U(t) – U(t – )]
= sin2t U(t) – sin2t U (t – )
taking Laplace on both sides, we obtain
L[F(t)] = L[sin2t U(t)] – L[sin2t U(t – )]
2
= s2 + 4 – e–s L[sin2 (t – )]
2 2 2
= s2 + 4 – e–s L[sin2t] = s2 + 4 – e–s s2 + 4
2
= (1 – e–s) 2
s +4
iv) We can express F(t) in terms of unit step functions as :
F(t) = cost [U(t) – U(t – )] + sint [U(t – )]
F(t) = cost U(t) + (sint – cost) U(t – )
Taking Laplace on both sides, we obtain
L[F(t)] = L[cost U(t)] + L[(sint – cost) U(t – )]
s
= 2 + e–s L[sin(t + ) – cos (t + )]
s +1
s s –1 s 
= 2 + e–s L[–sint + cost] = 2 + e–s  2 +
s +1 s +1  s + 1 s2 + 1
1
= 2 [s + (s – 1)e–s]
s +1

Example : 6

Find the Laplace transforms of :


i) sin3t (t – 2) ii) t U(t – 4) – t3 (t – 2)
iii) t2 U(t – 2) – cosht  (t – 4) iv) te–2t (t – 2)
Solution :
i) L[sin3t (t – 2)] = L[F(t) (t – a), where F(t) = sin3t and a = 2
–as
= e F(a)
= e–2s sin3(2) = e–2s sin6
ii) L[t U(t – 4) – t3 (t – 2)] = L[t U(t – 4)] – L[t3 (t – 2)]
= e–4s L [t + 4] – e–2s F(2) , where F(t) = t3
1 4
= e–4s  s2 + s  – e–2s (2)3
 
1 + 4s
= e–4s   – 8e–2s
 s2 
iii) L[t2 U(t – 2) – cosht  (t – 4)] = L[t2 U (t – 2)] – L[cosht  (t – 4)]
= e–2s L[t2 + 4t + 4] – e–4s cosh (4)
Gigatech Publishing House
Igniting Minds
Engineering Mathematics – III 2.58 Laplace Transformation (LT)

2 4 4
= e–2s  + + – e–4s cosh(4)
 s3 s2 s 
2 + 4s + 4s2 
= e–2s  – e–4s cosh(4)
 s3 
iv) L[te–2t  (t – 2)] = e–2s F(2), where F(t) = te–2t
= e–2s (2e–4) = 2e– (2s + 4)
= 2e–2(s + 2)

Example : 7

Show that :
 
1
i) e –3t
 (t – 4) dt = e12 ii)  F(t)  (t – a) dt = –F (a)
– –
Solution :

i)  e–3t  (t – 4) dt = F(4) where F(t) = e–3t


–
1
= e–12 =
e12
 

ii)  F(t)  (t – a) dt = [F(t) (t –



a)]– –  F(t)  (t – a) dt
– –

= 0–  F (t)  (t – a) dt [ By equation (12)


–
= –F (a)

Self-Assessment Exercise 2.4

Ex.1 Verify convolution theorem for following pair of functions :


i) F(t) = 1, G(t) = –1 ii) F(t) = e–t, G(t) = et
iii) F(t) = coswt, G(t) = 1 iv) F(t) = et , G(t) = t
Ex.2 Verify Initial value theorem, if limit exist.
i) sin3t ii) sin2t cost iii) etsint
Ex.3 Evaluate following integrals.

1
i) e –3t
sint dt Ans. : 10
0
Gigatech Publishing House
Igniting Minds
Engineering Mathematics – III 2.59 Laplace Transformation (LT)


sint 
ii)  t dt Ans. : 2
0

cosat – cosbt  1 1 +b2
iii)  e–t  dt Ans. : 2 log  1 + a2 
 t   
0

12
iv)  te –2t
sin3t dt Ans. : 169
0
Ex.4 Using Laplace transform, show that :
 
e–at – e–bt b e–tsint 
i)  t dt = log a ii)  t dt = 4
0 0
 
e–2tsinht 1 e–tsin2t 1
iii)  t dt = 2 log3 iv)  t dt = 4 log 5
0 0
 
e–tsin 3t  4
v)  t dt = 3 vi) e 3t
cos3t dt = 15
0 0
Ex.5 Using power series expansion, show that :

(–1)n – 1 (4n – 2)!
2
L[sint ] =  (2n – 1)! s4n – 1
n=1
Ex.6 Find the Laplace transform of full wave rectifier
sin t a sa
F(t) = such that F(t + a) = F(t) Ans. : cot h
a (s2a2 + 2) 2
Ex.7 Find the Laplace transform of meander function of period defined as :
 E  0 < t < a/2 a as
F(t) =  Ans. : 2s tanh 4
 –E  a/2 < t < a
Ex.8 Find Laplace transform of :
i) e–at J0 (bt) Ans. : (s2 – 2as + a2 + b2)–1/2
1
ii) J0 (ax) Ans. : 2
s + a2

Ex.9 Show that  J0 (t) dt = 1, where J0 (t) is a Bessel function of zero order.
0
Ex.10 Find the Laplace transform of :
s e–s
i) cos (t – 1) U(t – 1) Ans. :
s2 + 1
Gigatech Publishing House
Igniting Minds
Engineering Mathematics – III 2.60 Laplace Transformation (LT)

1 + 2s + 2s2 
ii) t2U (t – 2) Ans. : 2e–2s 
 s3 
e–2(s – 4)
iii) e4t U(t – 2) Ans. : s – 4
2e–s
iv) sin2t U(t – ) Ans. : 2
s +4
e–s
v) sin (t – 1) U (t – 1) Ans. : 2
s + 2
e–s (s cos1 – sin1)
vi) cost U(t – 1) Ans. : s2 + 1
Ex.11 Express the function in term of Heaviside unit steps functions and hence find
their Laplace transforms :
 et cost  0<t< (s – 1) + e–(s – 1) (s – 2)
i) F(t) =  Ans. :
t
 e sint  t> (s – 1)2 + 1
 cost  0<t<
ii) F(t) =  cos2t   < t < 2
 cos3t  t > 2
s s s s s
Ans. : s2 + 1 + e–s  s2 + 4 + s2 + 1  + e–2s  s2 + 9  s2 + 4 
   
–s
 sint  0<t< 1+e (s + 1)
iii) F(t) =  Ans. : s2 + 1 + e–s s2
 t  t>
e–t  0<t<3 1 – e–3(s + 1)
iv) F(t) =  Ans. :
0  t>3 s+1
 4  0 < t < 1 4 – 6e + 7e–3s
v) F(t) =  –2  1 < t < 3 Ans. :
s
 5  t > 3
Ex.12 Find the Laplace transform of :
1
i) e–4t (t – 3) Ans. :
e3(s + 4)
ii) (e–4t + logt) (t – 2) Ans. : e–2s (e–8 + log2)
(t – 3) 1
iii) t2 Ans. : 9e3s
–s

iv) sin2t  t – 4  Ans. : e 4
 
Ex.13 Evaluate :

16
i) e –t
(1 + 2t – t2 + t3) U(t – 1) dt Ans. : e
0
Gigatech Publishing House
Igniting Minds
Engineering Mathematics – III 2.61 Laplace Transformation (LT)



ii)  sin2t   t – 4  dt Ans. : 1
0

20
iii) e –t
(1 + 3t +t2) U(t – 2) dt Ans. : e2
0

1
iv)  e–4t  (t – 2) dt Ans. : e8
–
Ex.14 Prove that : (t) * (t) = (t)

(a) Definition : If L [ F(t) = f(s), then F(t) is said to be inverse Laplace transform of f(s)
and we write it as L1 [f(s)] = F(t)
Here L1 denotes the inverse Laplace transform.
1
For Example : Since L[e2t] = ;s>2
s2
Then
1 
L1  = e2t
s2
(b) Linearity Property :
If c1 and c2 are any arbitrary constant and f1(s) and f2 (s) are the Laplace transforms of
F1 (t) & F2 (t) respectively then,
L1 [c1f1(s) + c2f2 (s)] = c1 L1 [f1(s)] + c2 L1 [f2(s)]
= c1 F1 (t) + c2 F2 (t)

The inverse Laplace transform given below follow at once from the results of Laplace
transform given earlier :
1 1 
1. L1  s  = 1 2. L1  = eat
  s a
tn – 1
1

(n – 1)! 
L1  n =
if n is +ve integer
3.
s  
t(n – 1)  otherwise
 n

1  eat tn1 1 1
4. L1  = 5. L1(s2 + a2) = a sin at
(sa)n (n 1) !  

Gigatech Publishing House


Igniting Minds
Engineering Mathematics – III 2.62 Laplace Transformation (LT)

s  1  1
6. L1 = cos at 7. L1 = sinhat
(s + a2)
2
(s – a2) a
2

s 1  1 eat sin bt
8. L1 2 2  = cos hat 9. L1 2 =
(s – a )  (s  a) 2
+ b  b
(s  a) 
L1
s 1
10.  = eat cos bt 11. L1(s2 + a2)2 = 2a t sin at
(s  a)2 + b2  
s 1 1 1
12. L1 2 2 2 = 2a t sin hat 13. L1(s2 + a2)2 = 2a3 [ sin at  at cost at]
(s  a )   
Note : All the above result must be remembered.

h(s)
Suppose f(s) is a rational algebraic fraction. i.e. F(s) = g(s) , where h(s) and g(s) are
polynomials with degree of h(s) less than that of g(s). Then f(s) can be resolved into the sums
of rational functions called partial fraction. For doing partial fraction decomposition, the
following table is useful.
Factor in denominator Terms in partial fraction decomposition
(ax +b) (cx + d) A B
ax +b + cx + d
(ax + b)n A B C
(ax +b) + (ax +b)2 + (ax +b)3 + …. n.
times
(ax2 + bx +c) Ax + B
ax2 +bx + c

Table 2.4 : Rule for doing partial fraction, composition.

Example : 1

Find the inverse Laplace transforms of


s+1 s+4 (s2  1)2
i) ii) 2 iii)
s2 + 2 2 s +9 s5
s+1 s2  3s + 4 s+2
iv) 2 v) vi) 2
s +s+1 s3 s  4s + 13
Solution :
s +1 1 1  s + 1 
(i) Let f(s) = s2 + 22 , then L [f(s)] = L s2 + 22
s  1 
= L1  2 2 +L
1
[  due to linearty property
 s + 2   s 2
+ 22
= cos 2t + sin 2t
Gigatech Publishing House
Igniting Minds
Engineering Mathematics – III 2.63 Laplace Transformation (LT)

s+4 s + 4
(ii) Let f(s) = , then L1 [f(s)] = L1  2
2
s +9 s + 9
s 4
= L1s2 + 9 + L1s2 + 9
   
= cos 3 t + 4 sin 3t
(s21)2 s4 + 1  2s2
(iii) Let f(s) = s5 = s5
1 1 2
= + 
s s5 s3
1 1 2
Then L1[f(s)] = L1  s + s5  s3
 
1 1 2
= L1  s  + L1s5  L1s3
     
t4 24  24t2 + t4
= 1 + 24  t2 =
24
s+1 s+½+½
(iv) Let f(s) = s2 + s + 1 = 2 1 1
s +s+44+1

(s + ½) +½
= (s + ½)2 + ¾
(s + ½) ½
= (s + ½)2 + ¾ + (s + ½)2 + ¾
(s + ½)  1 1 1
Then L1[f(s)] = L1  + L 
 (s + ½) + ¾ 2
2
 (s + ½)2 + ¾
 3  1 t/2  3 
= e t/2 cos  2 t + e sin  2 t
  3  
1  3t 3t 
= e t/2  3 cos + sin 
3  2 2 
s2  3s + 4 1 3 4
(v) Let f(s) = s3 = s  s2 + s2
1 1 1
Then L1[ f(s)] = L1  s   3 L1s2 + 4 L1s3
     
= 1  3t + 2t3
s+2 s+2
(vi) Let F(s) = =
s2 4s +13 (s 2)2 + 9
(s  2) + 4 
= L1 
s+2 
Then L1 [f(s)] = L1  
 (s 2)2
+ 9  (s 2)2 + 9
s2 
L1  1 1 
= 2 + 4 L
 (s 2) + 3 
2
 (s 2)2 + 9
Gigatech Publishing House
Igniting Minds
Engineering Mathematics – III 2.64 Laplace Transformation (LT)

4
= e2t (cos 3t + e2t sin 3t
3
1
= [ 3e2t cos 3t + 4e2t sin 3t]
3

Example : 2

2s2  4 2s2  1
i) ii) (s2 + 1) (s2 + 4)
(s + 1) (s  2) (s  3)
21s  33 5s + 3
iii) 3 iv)
(s + 1) (s  2) (s  1) (s2 + 2s + 5)
2
s + 16 s2 + s
v) 2 2 vi)
(s + 1) (s + 4) (s + 1) (s2 + 2s + 2)
2

Solution :
2s2  4 A B C
i) Let = s+1+ +
(s + 1) (s  2) (s  3) s2 s3
 2s2  4 = A(s  2) (s  3) + B (s + 1) (s  3) + C (s + 1) (s  2)
1
Put s = 1 , we get A = 
6
4
Put s = 2 , we get B =  3
7
Put s = 3, we get C = 2
Thus,
2s2  4 1 4 1 7 1
L1 =   +
(s + 1) (s  2) (s  3) 6 (s + 1) 3 s  2 2 s  3
2s2  4
 L1   =  1 L1 1   4 L1  1  + 7 L1 1 

 (s + 1) (s  2) (s  3) 6 s + 1 3 s  2 2 s  3
1 4 7
=  et  e2t + e3t
6 3 2
2
2s  1 As + B Cs + D
ii) Let = + 2
(s2 + 1) (s2 + 4) s2 + 1 s +4
 2s2  1 = (As + B) (s2 + 4) + (Cs + D) (s2 + 1)
= (A + C) s3 + (B + D) s2 + (4A + C) s + (4B + D)
On comparing, we get
A+C = 0 C=A …. (i)
B+D = 2 ….. (ii)
4A + C = 0 (iii)  3A = 0  A = 0 [  from (i)]
4B + D = 1 (iv)  C = 0 [ from (i)]
Also from (iii) & (iv) , we get
Gigatech Publishing House
Igniting Minds
Engineering Mathematics – III 2.65 Laplace Transformation (LT)

B = 1 & D = 3
2s2  1 1 3
 (s2 + 1) (s2 + 4) = s2 + 1 + s2 + 4
2s2  1 1 1
 L1  (s2 + 1) (s2 + 4) = L1s2 + 1 + 3 L1 s2 + 4
     
3
=  sin t + sin 2t
2
21s  33 A B C D
iii) Let = + + +
(s +1) (s  2)2 s + 1 s  2 (s  2)2 (s  2)3
 21s  33 = A (s  2)3 + B (s + 1) (s  2)2 + C (s + 1) (s  2) + D (s +1)
Put s =  1, we get ; A = 2

s = 2, we get; D=3
Equating Co-efficient of s3, we get
A+B = 0  B=A  B = 2
Putting s = 0, we get  33 =  8A + 4B  2C + D
 C = 6
Thus,
21s  33 2 2 6 3
(s + 1) (s  2)2
= s + 1  s  2 + (s  2)2 + (s  2)3
21s  33 
L1 
2 2  1  1 
  L1 s + 1  L1 
= +6 L1  2 + 3 L
1
 (s +1) (s  2)2   s  2 (s  2)  (s  2)3
6t e2t 3t2 2t
= 2et  2e2t + + e
1! 2!
3
= 2et  2e2t + 6t e2t + t2 e2t
2
5s + 3 A Bs + c
iv) Let = + 2
(s 1) (s2 + 2s + 5) s  1 (s + 2s + 5)
 5s + 3 = A (s2 +2s +5) + (Bs + c) (s  1)
Putting s = 1, we get 8A = 8  A = 1 …. (i)
Putting s = 0, we get 5A  C = 3…. (ii)
Equating co-efficient of s2, we obtain
A + B = 0  B = A
Hence from (i), B = 1

Also from (ii), C = 2

Gigatech Publishing House


Igniting Minds
Engineering Mathematics – III 2.66 Laplace Transformation (LT)

Thus
5s + 3 1 s + 2
= + 2
(s  1) (s2 + 2s + 5) s1 s + 2s + 5
5s + 3 1  s + 2
 L1  = L1  + L1 s2 + 2s + 5
 (s  1) (s2 + 2s + 5) s  1  
(s + 1)  3
= et  L1  (s + 1)2 + 4
 
s + 1 3
= et  L1  + L1  
 (s + 1)2 + 4  (s + 1)2 + 4
3
= et  et cos 2t + et sin 2t.
2
2
s +6 p+6 2
v) Let (s2 + 1) (s2 + 4) = (p + 1) (p + 4) [  Pul s = p
A B
= (p + 1) + (p + 4)
 P+6 = A (p + 4) + B (p + 1)
 s2 + 6 = (A + B) s2 + (4A + B)
On comparing, we get
A+B = 1 ….(i)
4A + B = 6 ….(ii)
5 2
On solving, we get A = & B =  
3  3
Thus,
s2 + 6 5 1  2  1 
(s + 1) (s2 + 4)
2 = 3 (s2 + 1) + 3 (s2 + 4)
s2 + 6 5 1  1  2 1  1 
 L1 (s2 + 1) (s2 + 4) = 3 L (s2 + 1)  3 L (s2 + 4)
 
5 2 1
= sin t  sin 2t
3 3 2
1
= [ 5 sin t  sin 2t].
3
s2 + s As + B Cs + D
vi) Let (s2 + 1) (s2 + 2s + 2) = (s2 + 1) + (s2 + 2s + 2)
 s2 + s = (As +B) (s2 + 2s + 2) + (Cs + D) (s2 + 1)
On Equating co-efficients of each term, we get
A+C = 0 ….(i)
2A + B + D = 1 ….(ii)
2A + 2B + C = 1 ….(iii)

Gigatech Publishing House


Igniting Minds
Engineering Mathematics – III 2.67 Laplace Transformation (LT)

2B + D = 0 ….(iv)
From (1) C = A and from (4) D = 2B
 Equation (ii) & (iii) reduces to
2A  B = 1
& A + 2B = 1
3 1
On solving, we obtain A = 5 and B = 5
3 2
C= and D = 
5 5
s2 + s (3s + 1) (3s + 2)
 = 
(s2 + 1) (s2 + 2s + 2) 5 (s2 + 1) 5(s2 + 2s + 2)
 2 s2 + s  = 3 1  s  1 1  1 
L1 5 L s2 + 1 + 5 L s2 + 1
(s + 1) (s2 + 2s + 2)
3 s 2 1
=  5 L1s2 + 2s +2  5 L1 s2 + 2s +2
   
3 1 3 1 s + 1  1 1  1 
= 5 cost+ 5 sin t  5 L (s 11)2 + 1 + 5 L (s + 1)2 + 1
3 1 3 t 1 t
= 5 cost + 5 sin t  5 e cost + 5 e sin t
3 1
= (1  et) cos t + ( 1 + et) sin t.
5 5

(I) If L1 [ f(s)] = F(t) , then.


L1 [f(s  a)] = eat F(t)

(II) If L1 [f(s)] = F(t), then


d
L1 [sf(s)] = dt F(t) ; provided f(0) = 0

dn
In general, L1 [sn f(s)] = dtn [F (t)]

Provided, f(0) = f (0) = …  f n1 (0) = 0


(III) If L1 [f(s)] = F(t), then
t
f(s)
L  s  =  F(t) dt
1
 
0

In general;

Gigatech Publishing House


Igniting Minds
Engineering Mathematics – III 2.68 Laplace Transformation (LT)

(IV) If L1 [f(s)] = F(t) , then


d d
[t F(t)] =  L1 ds f(s)  L1ds f(s) =  [t F(t)]
   
(V) If L1 [f(s)] = F(t), then
  F(t)
L   f(s) ds =
1
,t>0
  t
s 

Illustrative Examples

Example : 1

Find inverse Laplace transforms of


(s +1) s + a s2 +1
i) log ii) cot 1  iii) log
(s1)  b  s(s+1)
2 s s2 +1
iv) tan 1 s2 v) cot 1 a vi) log
   (s 1)2
Solution :
(s +1)
(i) Let f(s) = log = log (s + 1)  log (s  1)
(s1)
Differentiate both side, we set
1 1
f (s) = s+1s1
1 1 
 L1 [f (s)] = L1 s+1  L1 
  s  1
d
 L1 ds f(s) = et  et
 
 [  t F(t) ] = et  et, (By Property – IV)
et  et
 F(t) =
t
s+a
(ii) Let f(s) = cot1  b 
 
1 (1) 1 b2
Then f (s) = b (s + a) 2 =
b (s + a)2 + b2
1+ 2
b

Gigatech Publishing House


Igniting Minds
Engineering Mathematics – III 2.69 Laplace Transformation (LT)

b
= (s + a)2 + b2
b
 L1[ f (s)] = L1  (s + a)2 + b2 = eat sin bt
 
 [ t F(t)] =  eat sin bt
eat sin bt
 F(t) =
t
2
s +1 
iii) Let f(s) = log  = log (s2 +1)  log s  log (s + 1)
s(s + 1)
on differentiating we get
2s 1 1
 f (s) = s2 + 1  s  s + 1
s 1 1
 L1 [f (s)] = 2 L1 s2 + 1  L1  s   L1 s + 1
     
[ t F(t)] = 2 cost 1  et
et + 1  2 cost
 F(t) =
t
2
(iv) Let F(s) = tan1  2
s 
On differentiating, we obtain.
1  43
F (s) = 4 s
1+
s4
4s
= 4
s +4
4s 4s
 L1 [f (s)] = L1 s4 + 4 = L1  

(s + 2)2  (2s)2
2

4s
=  L1 2 
 (s + 2s + 2) (s2  2s + 2)
1 1
=  L1 2  2 
(s  2s + 2) s + 2s + 2
1 1
=  L1 s2 + 2s + 2 + L1s2 + 2s + 2
   
1  + L1 1 
=  L1
(s  1)2 + 1 (s + 1)2 + 1
[  t F(t) =  et sin t + et sin t = (et  e  t) sin t
et  et
 F(t) =  sin t
 t 
2 sin ht sin t . . et  et
= [ . sin ht = ]
t 2

Gigatech Publishing House


Igniting Minds
Engineering Mathematics – III 2.70 Laplace Transformation (LT)

s
(v) Let f(s) = cot 1  
a
1 1 1  a2 
Then f  (s) = =
s2  a  a s2 + a2
1+ a2
a
=  s2 + a2
1
 L1 [f (s)] =  a a sin at =  sin at

 [ t F(t) ] =  sin at
sin at
 F(t) =
t
(s2 + 1)
(vi) Let f(s) = log = log (s2 + 1)  2 log (s  1)
(s  1)2
2s 2
 f (s) = s2 + 1 
s1
s  1 
 L1 [f (s) ] = 2 L1  2  2 L1 
s + 1 s  1
= 2 cos t  2et
[ t F(t)] = 2 (cost  et)
(et  cost)
F(t) = 2
t

Example : 2

Find the inverse Laplace transform of


1 1 1
i) ii) iii)
s(s + 1) s(s2 + 4) s2(s2 + 1)
Solution :
1 1/s + 1 g(s)
i) Let f(s) = s(s + 1) = s = s
g(s) 1
 L1[f(s)] = L1  s  where g(s) = s + 1
 
t
=  g(t) dt ….(i)
0
1
Now g(t) = L1 [g(s)] = L1 s + 1 = et
 
t t
et
Hence from (i) L1[ f (s)] =  et dt =  
0 10
F(t) = 1  et
Gigatech Publishing House
Igniting Minds
Engineering Mathematics – III 2.71 Laplace Transformation (LT)
2
g(s) 1/s + 4
ii) Let f(s) = =
s  s 
t
 L1 [f(s)] =  L1 [g(s)] dt
0
t
1
 L1 [f(s)] =  sin 2 t dt
2
0
t
1  cos 2t
= 2  2 0
1
= [ 1  cos 2t]
4
1
iii) Since L1 s2 + 1 = sin t
 
2 t t t t
11/s + 1
 L =   sin t dt =  [  cos t] dt
 s2  0
00 0
t
=  [ 1  cos t] dt
0
t
1 
 L1  = [ t  sin t]
 s2
(s +1)2
0
= t  sin t.

Theorem : If L1 [f(s)] = F(t) and L1 [g(s)] = G(t)


t
Then, L1 [ f (s) g(s)] =  F(u) G (t u) du
0
or
t
=  F(t u) G(u) du.
0
Note 2: If L1 [f (s)] = F(t) and L1 [g(s)] = G (t)
1
Suppose g(s) = s , then L1[ g(s)] = 1

t
f(s)
 L1 [f (s) g (s)] = L1  s  =  F(u) du [ same as property (III)]
 
0

Gigatech Publishing House


Igniting Minds
Engineering Mathematics – III 2.72 Laplace Transformation (LT)

Illustrative Examples

Example : 1

Use convolution theorem to find inverse Laplace transform of


1 1 s2
i) 2 ii) 2 3 iii)
(s + 1) (s + 1) (s + 1) (s + 1)2
2

1 s 1
iv) v) vi)
(s2 + 1) (s2 + 9) (s2 + 1) (s2 + 4) s3 (s2 + 1)
Solution :
1 1 1
i) Here (s + 1) (s2 + 1) = s + 1 s2 + 1 = f(s) g(s)
1
where, f(s) = s+1
1
and g(s) =
s2 + 1
So that
1
F(t) = L1 [f (s)] = L1 s + 1 = e t
 
1
and G (t) = L1 [g(s)] = L1 s2 + 1 = sin t
 
Then,
t
1
L 
1  = L1 [f (s) g(s)] =  e (tu) sin u du.
(s + 1) (s2 + 1)
0
t t
=  et eu sin u du = et  eu sin u du
0 0
u t
e
= et  [sin u  cos u]
2 0
et 1
et  [sin t  cos t] (0  1)
=
2 2 
1
= [sin t  cos t + et ]
2
1 1 1
ii) Consider (s2 + 1)3 = (s2 + 1)2 s2 + 1 = f(s) g(s)
1 1
where f (s) = (s2 + 1)2 F(t) = L1 (s2 + 1)2
 
So that
1 1 
g(s) = G(t) = L1  = sin t.
s2 + 1 s2 + 1
Gigatech Publishing House
Igniting Minds
Engineering Mathematics – III 2.73 Laplace Transformation (LT)

1  1 1 
Now F(t) = L1  = L1 
(s + 1) 2
2
s +1 s +1
2 2

F(t) = L1 [g(s) g (s)] = [ sin t sin t]


 by Convolution theorem.
t t
1
F(t) =  sin u sin (t  u) du  2  [ cos (2u  t)  cos t] du
0 0
t
1 sin (2u  t)
= 2  4 cos t
2  0
1
= ( sin t  t cos t) …. (i)
2
Therefore
1 1 (sin t  t cost ) sin t
L1(s2 + 1) 3 =
  2 
Again by convolution, we get
t
1
= 2  (sin u  u cos u) sin (t u) du
0
t t
1 1
= 2  sin u sin (t  u ) du  2  u cos u sin (t  u ) dy
0 0
t
1 1 1 
=
2 2 (sin t  t cost )  2  u [ sin t  sin (2u t) du] 
 0 
t t
1 1 1
= 4 (sin t  t cos t)  4  u sin t du + 4  u sin (2u t) du
0 0
2 t
1 1t u cos (2u t) sin 2u t
= (sin t  t cos t)  sin t +  +
4 42  2 t 0
1 t2 t cost sin t 
= sin t  t cos t  sin t  +
4 2 2 2 
1
= [(3  t2) sin t 3t cost]
8
s2 s s
iii) We can write (s2 + 1)2 = (s2 + 1) (s2 + 1) = f(s) g(s)

s
where f (s) = so that F(t) = cos t
s2 + 1
s
g (s) = s2 + 1 G(t) = cos t

Gigatech Publishing House


Igniting Minds
Engineering Mathematics – III 2.74 Laplace Transformation (LT)

Therefore,
t
L1 [ f (s) g (s)] =  F (u) G (t  u) du
0
t
s2 
Implies L1  =  cos u cos (t  u) du
(s +1)2
2
0
t
1
= 2  [ cos t + cos (2u  t) du ]
0
t
1 sin(2u  t)
= 2 4 cost + 2 0
1 sin t sin t
= t cost + +
2 2 2 
1
= [ t cost t + sin t]
2
1 1 1
iv) Here, we have (s2 +1)(s2 + 9) = s2 +1 s2 + 9 = f(s) g(s)
1 1
where f(s) = s2 +1 and g(s) = s2 + 9

So that
1 
F(t) = L1 [f(s)] = L1  = sin t
s2 +1
1 1
and G(t) = L1 [g(s)] = L1s2 + 9 = 3 sin 3t
 
Therefore,
1
L1  (s2 + 1) (s2 + 9) = L1 [ f(s) g (s)]
 
t
1
3
= sin u sin 3 (t  u) du.
0
t
1
= 3  sin u sin (3t  3u) du
0
t
1 1
32
= [ cos (4u  3t)  cos (3t  2u)]du
0
t
1 sin (4u 3t) sin (3t  2u)
= 6  4 
2

0
1
= ( 3 sin t  sin 3t)
24
Gigatech Publishing House
Igniting Minds
Engineering Mathematics – III 2.75 Laplace Transformation (LT)

s s 1 
v) L1   = L1 
(s + 1) (s2 + 4)
2
(s + 1) (s + 4)
2 2

L1 [ f(s) g(s)]


=
s 1
Where f(s) = s2 +1 and g (s) = s2 + 4

So that
1
F(t) = cos t and G (t) = sin 2t
2
Therefore,
t
s 1
L (s2 +1) (s2 +4) = 2  cos u sin 2 (t  u) du.
1
 
0
t
1
=
2  cos u sin (2t  2u) du.
0
t
1
= 4  [ sin (2t  u) + sin (2t  3u)] du.
0
1  cos (2t  u)  cos (2t 3u) t
4  
= +
1 3 0
t
1 1
= 4 [ cos (2t  u) + 3 cos (2t 3u)]0
1 1
4 =
[ (cos t  cos 2t) + 3 (cos t  cos 2t)]
1 4
= 4  3 (cos t  cos 2t)
1
= (cos t  cos 2t)
3
1 1 1
vi) L1  s3 (s2 + 1) = L1 s3 s2 + 1 = L1 [ f(s) g (s)]
   
1 1 t2
where, f(s) = 3 F (t) = L1  3 =
s s 2
1 1
and g(s) = 2 so that G(t) = L1  2  = sin t
s +1  s + 1
therefore
t
1 1
L  s3 (s2 + 1) =
1
 2 ( t  u)2 sin u du
 
0
t
1
2
= (t2 + u2  2tu) sin u du
0

Gigatech Publishing House


Igniting Minds
Engineering Mathematics – III 2.76 Laplace Transformation (LT)
t t t
1 2 
=
2   t sin u du +  u2 sin udu  2  tu sin u du 
0 0 0 
t t t
1  2  cos ut 
= 2 t 1 0 + [u ( cos u)] +  2u cos u du 2t  u sin u du
2

  0 
0 0
t t
1 1 1
= [ 1 + t2 cost] + ( t2cost) + 2 [4sinu + cosu]  t [ 4cosu + sinu]
2 2 2 0 0
1 1
= [1 + t2cost]  t2cost + [tsint + cost – 1] – t [tcost + sint]
2 2

=  1 + tsint + cost  1 + t2 cost  tsint 


2 
1 2
= t + cost – 1
2

Self Assessment Exercise 2.5

Ex.1. Find the inverse Laplace transforms of


2 1 4s
(i) Ans.: sin 4 t ii) Ans. 4 cash 4t
s2 + 16 2 s2 16
1 t3 3s  12
(iii) s4 Ans.: 6 iv) Ans.: 3t cos (2 ) 3 2t sin (2 2 )
s2 8
12 2s + 6
v) Ans.:  4e 4 t/3 vi) s2 + 4 Ans. : (2 cos 2t + 3 sin 2t)
4  3s
Ex.2. Determine Each of followings :
s2 + s +1 t 3/2 35 + 7t + t2
i) L1  s9/2  Ans. :
  9/2 4 2 
2 2
7(s + 1)  7 t4
ii) L1  Ans. : (1 + t2 + )
 5s5  5 24
3 2s 3
iii) L1  s + 2  s2 + 25 + s2 + 9 Ans. : 3e2t  2 cos 5t + sin 3t
 
Ex.3. Find the inverse Laplace transforms of
1 e4t t5
i) (s + 4)6 Ans.: 120
s 3
ii) Ans.: e3t ( cos 4t  sin 4t )
s2 + 6s + 25 4
6s  4
iii) Ans.: 2e2t ( 3 cos 4t + sin ut)
s2  4s + 20

Gigatech Publishing House


Igniting Minds
Engineering Mathematics – III 2.77 Laplace Transformation (LT)

Ex.4. Find inverse Laplace transform using partial fraction ;


3s + 7
i) Ans. : 4e3t  e t
s2  2s  3
3s +1
ii) Ans.: 2et  2 cos t + sin t
(s 1) (s2 + 1)
11s2  2s + 5 3
iii) Ans.: 5e2t  et/2 + 2et
(s 2) (2s  1) (s +1) 2
2s2  6s + 5 1 t 2t 5 3t
iv) Ans. e  e + e
s3 6s 2 + 11s  6 2 2
1 1 1
v) (s + 2) (s2 + 2s + 2) Ans.: 2 e2t 2 et ( cost  sin t)
2t
14s +10 2 7 3 3
vi) Ans.: e (cos t + sin t)
49s2 + 28s + 13 7 7 7
4s + 5 1 t 1
vii) Ans.: 3 e + 3 tet  3 e2t
(s  1)2 (s + 2)
s
viii) Ans.: (1  at) e at)
(s + a)2
s 1
ix) (s + 1)2 (s2 + 1) Ans.: 2 (sin t  tet)

Ex. 5. Find the inverse Laplace transforms of


1 t2
i) s(s + 1)3 Ans.: 1 et ( 1 + t + 2 )
1 1
ii) Ans.: + (1  cosat)
s(s2 + a2) a2
2
1 t
iii) 3 2
s (s + 1) Ans.: 2 + cost 1
1 e2t  1  2t  2t2
iv) Ans.:
s3(s  2) 8
Ex.6. Find the inverse Laplace transforms of
s+b eat  e bt
i) log s + a Ans.:
  t
sin t
ii) cot 1 (s) Ans : t
a2 2(1 cos at )
iii) log 1 + s2 Ans. :
  t
2 2
s + a 2(cosbt  cos at )
iv) log  2  Ans. :
s + b2 t
t
e sin t
v) tan1 (s + 1) Ans. : t
1 s2 a2 et 1
vi) log  2  Ans. :
2  s  t
Gigatech Publishing House
Igniting Minds
Engineering Mathematics – III 2.78 Laplace Transformation (LT)

Ex.7. Use Convolution theorem to find inverse Laplace transform of


s 1
i) 2 2 2
(s +a ) Ans. 2a t sin t
s2 a sin at  b sin bt
ii) (s +a ) (s2 + b2)
2 2 Ans.:
a 2  b2
1 1 e
iii) s(s + 1) (s + 2) Ans.: 2  e t + 2
1 1
iv) s(s2 +4) Ans.: 4 ( 1  cos 2t)
s2 1
v) Ans.: [ sin hat + sin at
s4  a2 2a
1
vi) s2(s + 1)2 Ans.: t (et + 1) + 2 (et 1)
1
vii) 2 2 Ans.: (t  sin t)
s (s + 1)
s 1
viii) Ans.: ( t sin 2t  2t2 cos 2t)
(s2 + 4)3 64
1 1 sin 3t
ix) (s2 + 9)2 Ans.: 18 ( 3  t cos 3t)
s2 +1 1
Ex.8. Find the inverse Laplace transform of log  s2 +s  Ans.: t ( 1 + et  2 cos t)
 
Ex. 9. Find inverse Laplace transform of
s2 + 2s  3 t 4e3t 3e2t
i) Ans.: 2 + 5  10 
s(s  3) (s + 2)  
s sin t t et
ii) Ans.:  2  2 
(s + 1) (s2 + 2)
2
 
Ex. 10. Find inverse Laplace transform of
2s +1 2t t/2  3 
i) Ans.: e sin  t ]
(s2 + s +1)2 3 2 
1 1
ii) (s + 1) (s2 + 1) Ans.: 2 [ sin t  cos t + et ]
2s +5 sin 3t
iii) Ans.: e2t 2 cos 3t + 3 
s2 + 4s + 13  
s+1 t2 et
iv) (s + 2s + 1)2
2 Ans.: 2

s+2 2t 1 e3t
v) 2 Ans.: + 
s (s + 3) 3 9 9

Gigatech Publishing House


Igniting Minds
Engineering Mathematics – III 2.79 Laplace Transformation (LT)

Laplace transforms are invaluable for any engineers as they make use of it in solving
ordinary differential equations and related initial value problems. Laplace & inverse Laplace
transforms are also used to solve system of linear ordinary differential equations.
The application of Laplace transform abound :
i) Electrical Circuit & Networks.
ii) Spring & Mixing problems.
iii) Signal processing, vibration of plates etc.
The process of solving an ordinary differential equation using the Laplace transform
method consists of following three steps :
Step 1 : Transformed the given ordinary differential equation into an algebraic equation
(known as Subsidiary Equation)
Step 2: Solve the subsidiary equation by purely algebraic manipulations.
Step 3: Transformed back (by taking inverse Laplace transform) solution of step – 2, to find
resultant (required) solution of the given problem.

Illustrative Examples

Example : 1

Find the solution of each of the following differential equations:


i) y + 2y + 5y = et sin t; y(0) = , y (0) = 1.
ii) y + 9y = cos 2t ; y (0) = 1, y (o) = 2
iii) y  3y + 2y = 12e 2t ; y (0) = 2, y (0) = 6
iv) y + y = t, y(0) = 1, y (0) = 2
v) y + 2y + y = tet , y(0) = 1, y (0) = 2
vi) y  y = et, y(0) = y (0) = y (0) = 0
vii) y + 4y + 8y = 1, y(0) = 0, y (0) = 1
viii) y  2y + y = e2t, y(0) = y (0) = 0
Solution :
(i) Given differential equation is
y + 2y + 5y = et sin t …..(i)
where y(0) = 0, y(0) = 1

Gigatech Publishing House


Igniting Minds
Engineering Mathematics – III 2.80 Laplace Transformation (LT)

Taking Laplace transform on both sides of (i) we get


L[y(t) + 2 y(t) + 5 y(t)] = L [et sin t]
1
L [y (t) ] + 2 L [y (t) ] + 5 L [y (t)] = (s+1)2 +1 [s2 y(s)  sy(0)  y(0)] + 2 [s y(s)  y (0) ]+ 5y (s)

On using the given conditions, above equation reduces to


1
(s2 + 2s + 5) y(s)  1 =
s2 + 2s + 2
1 1
or y(s) = (s2 + 2s + 5) (s2 + 2s + 2) + (s2 + 2s + 5)
1 1 1 1
= 3 s2 + 2s + 5  s2 + 2s + 2 + (s2 + 2s + 5)
 
1 1 2
= 3 s2 + 2s + 2  s2 + 2s + 5
 
1 1 2
= 3 (s + 1)2 + 1  (s + 1)2 + 22
 
Taking inverse Laplace transform both sides, we get
1 1  + 2 L1  1
y(t) = L1[y(s)] = L1  
3 (s + 1)2 + 1 3 (s + 1)2 + 22
1 2 1
or y(t) = 3 et sin t + 3 . 2 e t sin 2t

1 t
or y(t) = e (sin t + sin 2t) This is required solution.
3

ii) Given DEs is y + 9y = cos 2 t.


Taking Laplace on Both sides, we obtain.
s
[s2 y(s)  s y(0)  y (0) ] + 9y (s) =
s2 +4
On using given condition, we get
s
(s2 + 9) y(s) + s  2 = s2 + 4
s s2
Or y(s) = 
(s2 + 4) (s2 + 9) s2 + 9
Therefore,
y(t) = L1 [y(s)]
s s 1
= L1 (s2 + 4) (s2 + 9)  L1 s2 + 9 + 2 L1 s2 + 9
     
t
1 2
3
= cos 2u sin (3t  3u) du  cos 3t + sin 3t [ by Convolution Theorem]
3
0

Gigatech Publishing House


Igniting Minds
Engineering Mathematics – III 2.81 Laplace Transformation (LT)
t
1 1 2
=  [ sin (3t  u)  sin (54  3t)]  cos 3t + sin 3t
3 2 3
0
t t
1 1 2
= 6  sin (3t  u) du  6  sin (5u  3t) du  cos 3t + 3 sin 3t
0 0
t t
1  cos (3t  u) 1  cos (5u  3t)
= 
2
6  6 0
 cos 3t + sin 3t
1 0 5 3
1 1 2
= 6 ( cos 2t  cos 3t) + 30 (cos 2t  cos 3t)  cos 3t + 3 sin 3t
1 6 2
= cos 2t  cos 3t + sin 3t.
5 5 3
(iii) Given DEs is y  3y + 2y = 12 e2t with y(0) = 2 & y (0) = 6
On taking Laplace and using condition, we obtain.
12
[ s2 y(s)  2s  6]  3 [s y(s)  2] + 2 y(s) = s + 2
12
Or (s2  3s + 2) y(s)  2s = s + 2
12 2s
Or y(s) = +
(s + 2) (s2  3s + 2) (s2  3s + 2)
2s2 + 4s + 12
y(s) =
(s + 2) (s2  3s + 2)
2s2 + 4s + 12
=
(s  1) (s  2) (s + 2)
Taking inverse on both side , we obtain
2s2 + 4s + 12
L1 [y(s)] = L1  
(s  1) (s  2) (s + 2)
6
y(t) = L1 
7 1 
Or + +  [ on using partial fraction, we get]
s  1 s  2 s + 2
Or y(t) = 6et + 7e2t + et
Which is the required solution.
(iv) Given DEs y + y =t with y(0) =1, y(0) = 2
Taking Laplace on both side and using condition, we obtain
1
(s2 + 1) y (s)  s + 2 = 2
s
s2 1
Or y(s) = 2 +
s + 1 s2(s2 + 1)
On Taking inverse Laplace transform , we obtain
s2 1
y(t) = L1  2 + L1  2 2 
 s + 1 s (s + 1)
Gigatech Publishing House
Igniting Minds
Engineering Mathematics – III 2.82 Laplace Transformation (LT)

s  1  1 1 
= L1   2 L1  2 + L1  2  L1  2
(s + 1)
2
(s + 1) s  s + 1
= cost  2 sin t + t  sin t
y(t) = t + cos t  3 sin t
This is required solution.
v) Given DEs is y + 2 y + y = te t y(0) = 1, y(0) = 2
Taking Laplace on both sides, we obtain
1
[s2 y(s)  s + 2] + 2 [ sy(s) 1] + y(s) = (s + 1)2
1
Or (s2 + 2s + 1) y(s)  s =
(s + 1)2
1 1
Or y(s) = +
(s + 1)4 (s + 1)2
1 s+11
= +
(s + 1)4 (s + 1)2
1 1 1
y(s) = (s + 1)4 + (s + 1)  (s + 1)2

Taking inverse Laplace transform, we obtain


t3 et t3
y(t) = et  t e t + 3! = et 1  t + 
 3! 
which is required solution.
(vi) Given DEs y  y = et with y(0) = y(0) = y (0) = 0.
On taking Laplace transform and using given condition we obtain.
1
(s3 1) y(s) =
s1
1
Or y(s) =
(s3  1) (s  1)
1
=
(s  1)2 (s2 + s + 1)
On using method of partial fraction , we get
1 1 1 1 1 (s + 1)
y(s) =  3 +3 2+ 2
s1 (s  1) 3 (s + s + 1)
on taking Laplace transform , we obtain
1
1 1 1  s2 
y(t) =  3 et + 3 tet + 3 e t/2 L1   3 
2

s + 2 
2
 
et tet et/2  3 1 3 
3 
= + + cos t+ sin t
3 3 2 3 2 
Gigatech Publishing House
Igniting Minds
Engineering Mathematics – III 2.83 Laplace Transformation (LT)

vii) Given DEs is y + 4y + 8y = 1 with y(0) = 0, y(0) = 1


Taking Laplace on both sides, we get
1
[ s2 y(s)  sy(0)  y(0) ] + 4 [ s y(s)  y(0)] + 8 y(s) = s
1
Or (s2 + 4s +8) y(s)  1= s
1 1
Or y(s) = +
s(s2 + 4s + 8) s2 + 4s + 8
1 1
y(s) = s[(s + 2)2 + 22] + (s + 2)2 + 22

Taking inverse on both sides, we obtain.


1 1
Y(t) = L1  (s + 2)2 + 22 + 2 e2t sin 2t
 
 s 
t
1 1
=  e2t sin 2t dt + e2t sin 2t
2 2
0
t
1 e2t
= 2  8 [ 2 sin 2t + 2 cos 2t] + 2 e2t sin 2t
1
 0
1 e2t 1 1
= 2  4 [ sin 2t + cos 2t] + 8 + 2 e2t sin 2t
 
1 1 2t
=  e (cos2 t  3 sin 2t)
8 8
This is required solution.
viii) Given DEs y  2y + y = e2t with y(0) = y (0) = 0
Taking Laplace on both sides, we get
1
s2y(s)  2y(s) + y(s) =
s+2
1
y(s) = 2
(s + 2) (s  2s + 1)
1
=
(s + 2) (s  1)2
On using partial fraction we get
1 1 1 1 1 1
y(s) =  +
9 (s + 2) 9 (s  1) 3 (s  1)2
On taking inverse Laplace transform, we obtain.
1 2t 1 t 1 t
y(t) = e  e + te
9 9 3
1 2t
= [ e  et + 3tet ]
9
Gigatech Publishing House
Igniting Minds
Engineering Mathematics – III 2.84 Laplace Transformation (LT)

Example : 1

Solve the differential equation


y + 9y = 18t , y(0) = 0, y (/2) = 0
by using Laplace transform.
Solution : Given differential equation is
y + 9y = 18t
Taking Laplace on both sides, we obtain
18
[s2 y(s)  s y(0)  y (0)] + 9 y(s) = s2

Since y (0) is not known , so let y(0) = k.


Therefore
18
s2y(s)  k + 9y(s) = s2
k 18
Or y(s) = s2 + 9 + s2(s2 + 9)
k 2 2
= s2 + 9 + s2  s2 + 9
Taking inverse Laplace transform, we obtain.
k  2 2 
y(t) = L1  2 + L1  2  L1  2
s + 9 s  s + 9

y(t) = k  2 sin 3t + 2t ….(i)


 3 
Determine k : Put t = /2 in above in equation (i) we obtain.

y (/2) = k 2 sin 3 + 2


 3  2 2
k 2
0 = (1) + 
3
k 2
Or = 
3
Hence from (i)
y(t) =  sin 3t + 2t

which is required solution.


Gigatech Publishing House
Igniting Minds
Engineering Mathematics – III 2.85 Laplace Transformation (LT)

Example : 1

Solve following differential equation


t
dy
(i) + 3y(t) + 2  y(t) dt = t, given y(0) = 0
dt
0
t
dy
(ii) dt + 2y(t) +  y(t) dt = sin t, given y(0) =1
0
Solution :
t
Given y (t) + 3y (t) + 2  y (t) dt = t
0
Taking Laplace, we obtain
2 1
[ s y (s)  y (0) ] + 3 y (s) + s y(s) = s2

Or s + 3 + 2  y(s) = 1
 s s2
s 1
Or y(s) = s2(s2 + 3s + 2) = s(s + 1) (s + 2)
On using partial fraction and taking inverse Laplace, we obtain
11 1 1
L–1[y(s)] = L–1 2 s  s + 1 + 2
s+2
1 t e2t
Therefore, y(t) = e +
2 2
Which is required solution .
t
ii) Given y(t) + 2y(t) +  y(t) dt = sin t
0
Taking Laplace transform and using given condition, we obtain
s + 2 + 1  y(s) = 1 + 2 1
 s s +1
s s
Or y(s) = 2 +
s + 2s + 1 (s2 + 2s + 1) (s2 + 1)
1 1 1 1 1 1
= s + 1  (s + 1)2  2 (s + 1)2 + 2 (s2 + 1) [ on using partial fraction]

On taking inverse Laplace transform, we obtain


3 1
y (t) = et  t et + sin t which is required solution.
2 2
Gigatech Publishing House
Igniting Minds
Engineering Mathematics – III 2.86 Laplace Transformation (LT)

Self Assessment Exercise 2.6

Ex.1. By using Laplace transform, solve following differential Equations :


i) y + y = 0 , y (0) = 1, y(0) = 2 Ans.: y(t) = cost + 2 sin t
ii) y   3y = 9, y(0) = y(0) = 0 Ans. y(t) = e3t  3t  1
iii) y   2y + 2y = 0, y(0) = y (0) = 1 Ans. : y(t) = et cost
7 3 1
iv) y  + 4y + 3y = et, y(0) = y(0) = 1. Ans. : 4 et  4 e 3t + 2 tet
et
dx w  sin wt  w cos wt
v) + x(t) = sin wt, x(0) = 2. Ans.: 2 + 2 +
dt  w +1  (w2 +1)
Ex.2 Solve following initial value problem :
t2
i) y  2y + y = et, y(0) = 2, y(0) = 1 Ans. et (2  3t + 2 )
5 16 2t 15 3t
ii) (D2 + 50 + 6) y = 5et, y(0) = 2, y (0) = 1, Ans.: et + e  e
12 3 4
iii) y  + 2y + y = 6tet ; y(0) = 2, y (0) = 5 Ans. y(t) = et ( t3 +7t +2)
iv) y  3y + 2y = 4e2t ; y(0) = 3, y = 3, y (0) = 5
Ans.: y(t) =  7et + 4e2t + 4te2t
v) y + y = 2, y(0) =3, y (0) =1, y (0) = 2 Ans.: y(t) =5 +2t  sin t  2 cost
Ex.3 Solve BVP by using Laplace transform.
4 4 1
y + 9y = cos 2 t, y(0) = 1 , y (/2) = 1. Ans.: y(t) = 5 cos 3t + 5 sin 3t + 5 cos 2t

Ex.4 Solve following differential Equation :


i) y  y2y = 20 sin 2t; y(0) = 1, y (0) =2.
Ans. : y(t) = 2e2t  4et + cos 2t  3 sin 2t
ii) y + 2y  3y = sin t, y(0)= y(0) = 0
1 3
Ans.: 10 [ et cos h2t + 2 e t sinh2t  cos t  2 sin t]

Ex.5 Solve by Laplace transformations, following linear differential equations with


constant co-efficients.
i) (D3 + D) y = 2, where y = 3, Dy = 1, D2y = 2 at t = 0
Ans.: y(t) = 1 + 2t + 2 cos t  sin t
ii) (D  D) y = 2 cost, where y = 3, Dy = 2, D2y = 1 at t = 0
3

Ans.: y(t) = 3 sin ht + cos ht  sin t + 2


iii) (D  1)3 y = t2et, where y = 1, Dy = 0, D2y = 2 at t = 0
t2 t5
Ans.: y(t) = et 1  t  + 
 2 60 

Gigatech Publishing House


Igniting Minds
Engineering Mathematics – III 2.87 Laplace Transformation (LT)

Ex.6 Solve following Integro differential equation.


t
i) y(t) +  y(t) dt = 1  et Ans. y(t) = tet
0
t
ii) y(t) + 4y(t) + 5  y (t) dt = et, y(0) = 0
0
1 1
Ans. y(t) =  2 et + 2 e2t (cost + 3 sin t)
t
t2
iii) y (t) + y(t)  2  y(t) dt = 2 , y(0) =1, y (0) =  2
0
et 11 2t t 1
Ans. y(t) = + e  
3 12 2 4

Distractive Questions

Q. 1 Find the Laplace transforms of following functions.


i) (2t + 3)3
ii) cosh at – cos at
iii) e3t + 5t4 – 2cost + 3sint
Q.2 Obtain the Laplace transform of following functions :
i) 5 sin (2t + 3)
ii) 4cos2t – 5t2 + 2e3t
Q.3 Obtain the Laplace transforms of following functions :
i) e3t t
ii) e–3t t3
t
Q.4 If L [F(t)] = f(s), show that L  F  a   = a f(as)
  
–1/s
e
Q.5 If L[F(t)] = , find L[e–t F(3t)]
s
Q.5 Verify change of scale property for L[e3t  cos3t]
Q.7 Find the Laplace transform of
d2y dy
i) dt2 –3
dt + 5y = 0, given that y(0) = 2, y(0) = 4.
Q. 8 Find the Laplace transform of
i) t2 e2t
ii) t sin3 t
Gigatech Publishing House
Igniting Minds
Engineering Mathematics – III 2.88 Laplace Transformation (LT)

Q. 9 Evaluate :
t
 sin t 
i) L  et  t dt 
 0 
Q. 10 Show that :
 t et  cos 2t  1 s2 + 4
i) L  dt = log
0
t

2s (s 1)2
Q. 11 Verify convolution theorem for following pair of functions :
i) F(t) = 1, G(t) = –1 ii) F(t) = e–t, G(t) = et
Q. 12 Using Laplace transform, show that :
 
e–at – e–bt b e–tsint 
i)  t dt = log a ii)  t dt = 4
0 0
 
e–tsin 3t  4
iii)  t dt = 3 iv) e 3t
cos3t dt = 15
0 0
Q.13 Prove that : (t) * (t) = (t)
Q.14 Find the inverse Laplace transforms of
1
i) s(s + 1)3
1
ii)
s3(s  2)
iii) tan1 (s + 1)
Q. 15 Solve following Integro differential equation.
t
i) y(t) +  y(t) dt = 1  et
0



Gigatech Publishing House


Igniting Minds
Unit
Fourier and Z
3 Transforms

Syllabus :
Fourier Transform (FT) : Complex exponential form of Fourier series, Fourier integral
theorem, Fourier Sine & Cosine integrals, Fourier transform, Fourier Sine and Cosine
transforms and their inverses.
Z – Transform (ZT) : Introduction, Definition, Standard properties, ZT of standard sequences
and their inverses. Solution of difference equations.

3.1 Fourier Transform :

3.1.1 Introduction :
Fourier transform comes from the study of Fourier series. In the study of Fourier series,
complicated but periodic functions are written as the sum of simple waves mathematically
represented by sines and cosines. The Fourier transform is an extension of the Fourier series
that results when the period of the represented function is lengthened and allowed to approach
infinity.
A Fourier transform when applied to a partial differential equation reduces the number
of its independent variables by one. The theory of integral transforms afford mathematical
devices through which solutions of numerous boundary value problems of engineering can be
obtained e.g. conduction of heat, transverse vibrations of string, transverse oscillations of an
elastic beam free and forced vibrations of a membrane etc.
3.1.2 Integral Transform :
The integral transform of a function f(x) denoted by I [f(x)] and is defined by.
x2
I[f(x)] = ∫ f(x) K(λ,x) dx
x1

Gigatech Publishing House


Igniting Minds
Engineering Mathematics – III 3.2 Fourier and Z – Transforms

Where K (λ,x) is called the kernel of the transform. The function f (x) is called the
inverse transform of I [F(x)].
When K (λ,x) = e−iλx, we have the Fourier transform.

F (λ) = ∫ f (x) e−iλx dx
−∞
• Fourier Integral :
Consider a function f(x) which satisfies the Dirichlets conditions in every interval
(−∞,∞) then
∞ ∞
1
f (x) =

∫ ∫ f(u) e−iλ(u−x) dudλ
−∞ −∞
3.1.3 Fourier Transform and Inverse Fourier Transform :
I) Fourier Transforms :
Fourier Transform of the function f(x) defined in the interval of (−∞,∞) is defined as

F (λ) = ∫ f (u) e−iλu du and the inverse Fourier transform of F(λ) is defined as
−∞

1
f(x) =

∫ F (λ) eiλx dλ
−∞

II) Fourier cosine transform :


If f(x) is defined in the interval 0 < x < ∞ then Fourier cosine transform of f(x) is defined

as Fc(λ) = ∫ f(u) cos λu du and the inverse Fourier cosine transform of Fc (λ)is given
−∞

by.

2
f(x) =
π ∫ Fc (λ) cos λx dλ
0

III) Fourier sine transform :


If a function f(x) defined in the interval 0 < x <∞ then Fourier sine transform of f(x) is
defined as.

Fs (λ) = ∫ f (u) sin λu and the inverse Fourier sine transform of Fs (λ) is given by
o

2
f(x) =
π ∫ Fs (λ) sin λx dλ
0

Gigatech Publishing House


Igniting Minds
Engineering Mathematics – III 3.3 Fourier and Z – Transforms

Note :
1. Results (II) of Fourier cosine transform and inverse Fourier cosine transform can be
written in more symmetric form as.

2
Fc (λ) =
π ∫ f(u) cos λ u du
0


2
And f (x) =
π ∫ Fc (λ) cos λx dλ
0

2. Similarly result (III) of Fourier sine transform and inverse Fourier sine transform can
be written in more symmetric form as

2
Fs (λ) =
π ∫ f (u) sin λu du
0


2
and f (x) =
π ∫ Fs (λ) sin λx dλ
0

Table of Fourier Transforms and Inverse Transforms.


Sr. Name of the Interval Expression for the Inverse transform
No. transform transform
1. Fourier −∞ < x < ∞ ∞ ∞
1
F(λ) = ∫ f(u) e−iλu du f(x) =

∫ F(λ)e−iλx dλ
−∞ −∞

2. Fourier 0<x<∞ ∞ ∞
2
cosine Fc(λ) = ∫ f(u) cos λu du f(x) = ∫ Fc(λ) cos λx dλ
0
π 0

3. Fourier sine 0<x<∞ ∞ ∞


2
Fs(λ) = ∫ f(u) sin λu du f(x) = ∫ Fs (λ) sin λx dλ
0
π 0

Illustrations Examples

Type – I : Problems on Fourier Transform

Example : 1

−| x |
Find the Fourier transform of f(x) = e
−| x |
Solution : Here f(x) = e

Gigatech Publishing House


Igniting Minds
Engineering Mathematics – III 3.4 Fourier and Z – Transforms

Fourier transform of f(x) in the interval −∞ < x < ∞ is given by



F(λ) = ∫ f(u) e−iλu du
−∞

−| u |
= ∫ e ( cos λ u − i sin λ u) du
−∞
∞ ∞
−| u | −| u |
= ∫ e cos λu du − i ∫ e sin λu du
−∞ −∞

Since integrand in the first integral is even and second integral is odd

∴ F(λ) = 2 ∫ e u cos λu du + 0
0

 eu  2
= 2 2 (− cos λu + λ sin λu) =
1+λ 0 λ2
1+λ

Example : 2

1
Find Fourier sine transform of x

Solution :
1
Here f(x) = x

Fourier sine transform is since Fs(λ) = ∫ f (u) sin λ u du
0

1
= ∫ u sin λ u du
0
Put λu = t, u = t/λ
∴ λdu = dt
du = dt/λ
Also as limits u : 0 to ∞
∴ ⇒ t : 0 to ∞

λ dt
∴ Fs (λ) = ∫ t sin t λ
0

sin t π
= ∫ t dt =
2
0

Gigatech Publishing House


Igniting Minds
Engineering Mathematics – III 3.5 Fourier and Z – Transforms

Example : 3

Find the Fourier cosine transform of e−x + e−2x, x > 0 Nov. 2016
Solution :
f (x) = e−x + e−2x, o < x < ∞
The Fourier cosine transform of f(x) is given by

Fc(λ) = ∫ f(u) cos λ u du
0

= ∫ (e−u + e−2u) cos λu du
0
∞ ∞
= ∫e −u
cos λu du + ∫ e−2u cos λu du
0 0
−u
∞ −2u

=    e (−2 cosλu + λ sin λu)
e
2 (−cosλu + λ sin λu) + 
1 + λ 0 4 + λ2 0
1 1
= [0− (−1) ] + [ 0 − (−2)]
1 + λ2 4 + λ2
1 2 6 + 3λ λ2
= 2+ 2 =
1+λ 4+λ 4 + 5λλ + λ4
2

Example : 4

Find the Fourier cosine transform of the function. May 2016


cos x , 0 < x < a
f (x) = 
 0 , 0 x>a
Solution :
Fourier cosine transform of the function f(x) is given by

Fc (λ) = ∫ f(u) cos λ u du
0
a ∞
= ∫ cos u cosλu du + ∫ (o) cosλu du
0 a
a
1
= 2 ∫ [ cos (λ+1) u + cos (λ−1) u]du
0
a
1 sin(λ + 1)u sin (λ − 1) u
= 2  λ+1 +
λ−1 
 0
λ + 1)a sin (λ
1 sin(λ λ − 1) a
2  λ + 1 λ − 1 
= +

Gigatech Publishing House


Igniting Minds
Engineering Mathematics – III 3.6 Fourier and Z – Transforms

Example : 5

e−ax
Find the Fourier sine transform of x

Solution :
e−ax
Given : f(x) = x

Fourier sine transform is given by



Fs (x) = ∫ f(u) sin λu du
0


e−au
u
= ∫ e sin λu du = I(λ) say …(1)
0

∂ e−au
sin λu du
dI
By DUIS

= ∫ ∂λ  u 
0
∞ −au
e
= ∫ u u cos λu du
0

= ∫ e−au cos λu du
0

 e−au 
=  2 [ −a cos λu + λ sin λu]
a + λ
2

0
 1 
= 0 − 2 2 (− a)
 a +λ 
dI a
=
dλ a2 + λ2
Integrating we get

I(λ) = ∫a a
+ λ2
2 dλ + A

λ
I(λ) = tan−1 a  + A ….(2)
 
Put λ = 0 ⇒ I(λ) = 0 (from equation (1))
∴ from equation (2) , we have
0=0+A⇒A=0
λ
∴ λ) = tan −1  
λ) = Fs (λ
I(λ
a

Gigatech Publishing House


Igniting Minds
Engineering Mathematics – III 3.7 Fourier and Z – Transforms

Type 2 : Problems on Fourier Integral representation and


inverse Fourier transform.

Example : 1

Using Fourier cosine integral of e−mx (m > 0), prove that Nov. 2015

mcosλx dλ π −mx
∫ m2 + λ2
= 2 e , (m > 0, x > 0)
0
Solution :
Taking Fourier cosine transform of
f (x) = e−mx, we have
∞ ∞
Fc (λ) = ∫ f(u) cosλu du = ∫ e−mu cos λu du
0 0

 e−mu ∞
=  (− m cos λ u + λ sin λ u) 
m2 + λ2 0

= (0) − 2 1 2 (−m + 0)


 m +λ 
m
Fc (λ) = ….(1)
m + λ2
2

Now inverse cosine transform of


Fc (λ) is given by

2
f (λ) =
π ∫ Fc (λ) cos λx dλ
0

2 m
e−mx =
π ∫ m2 + λ2 cos λx dλ
0

mcos λx π −mx
∴ ∫ m2 + λ2
λ =

2
e
0

Example : 2

Find the Fourier transform of


1 for |x| ≤ 1
f (x) = 
 0 for |x| > 1
Hence evaluate

sinx
∫ x dx
0

Gigatech Publishing House


Igniting Minds
Engineering Mathematics – III 3.8 Fourier and Z – Transforms

Solution : The Fourier transform of f(x) is


∞ 1
F (λ) = ∫ f (u) e −iλu
du = ∫ (1) e−iλudu
−∞ −1
−iλu 1 −iλ
 e   e eiλ 
=   =  − 
 −iλ −1 − iλ − iλ
eiλ− eiλ 2 e−iλ−e−iλ
= =
iλ λ  2i 
2 2 sin λ
F (λ) = sin λ =
λ λ
Now by the inverse Fourier transform,

1
f (x) =

∫ F (λ) e−iλx dλ
−∞

1 2 sin λ −iλx
f (x) =

∫ λ
e dλ
−∞

1 sin λ −iλx
f (x) =
π
∫ λ
e dλ
−∞

sin λ −iλx 1 , |x| < 1
⇒ ∫ λ
e dλ = π f(x) =π
0 , |x| > 1
−∞
Putting x = 0 , we get

sin λ
∫ λ
dx = π
−∞

sin λ
∴ 2 ∫ λ
dλ = π ( ∴ since the integrand is even)
0

sin λ π
∴ ∫ λ
dλ = 2
0

sin x π
∴ ∫ x
dx =
2
( ∴ variable is immaterial in definite integral)
0

Example : 3

x sin mx π –m
−|x|
Find the Fourier sine transform of e . Hence show that ∫ 1 + x2 dx = 2 e , m > 0
0
Solution :
Fourier sine transform of f(x) is
Gigatech Publishing House
Igniting Minds
Engineering Mathematics – III 3.9 Fourier and Z – Transforms

Fs (λ) = ∫ f (u) sin λu du
0

= ∫ e−u sin λu du ( ∴ | x | = x, if x>o)
0

 e−u 
=  (− sin λ u −λ cos λ u)
 1+λ2 
0
λ
Fs (λ) =
1+λ2
Inverse Fourier sine transform is

2
f(x) =
π ∫ Fs (λ) sin λx dλ
0

2 λ
e−x =
π ∫ 1+λ2 sin λx dλ , x>0
0
put x by m, we get

2 λ
e –m
=
π ∫ 1+λ2 sin λm dλ m>0
0
replace λ by x , we get

2 x sin mx
e−m
=
π ∫ 1+x2 dx , m > 0
0

x sin mx π −m,
∴ ∫ 1+x2
dx =
2
e m>0
0

Example : 4

By considering Fourier cosine transform of e−mx, (m > 0) prove that. May 2015

cos λx π −mx
∫ x2+m2 dλ = 2m e m > 0, x > 0,
0
Solution : Fourier cosine transform of f(x) is
∞ ∞
Fc(λ) = ∫ f(u) cos λ u du = ∫ e −mu cos λ u du
0 0

 e−mu 
=  (− m cosλu + λ sin λu) 
 m2+λ2 
0
m
Fc(λ) =
m + λ2
2

Gigatech Publishing House


Igniting Minds
Engineering Mathematics – III 3.10 Fourier and Z – Transforms

Now Inverse Fourier cosine transform is



2
f(x) =
π ∫ Fc (λ) cos λx dλ
0

2 m
e−mx =
π ∫ m2+λ2 cos λx dλ
0

cos λ x π −mx
⇒ ∫ λ2 + m2 dλλ =
2m
e
0

Example : 5

Find the Fourier sine transform of the following function. May 2018
 x , 0 ≤ x ≤ 1
f(x) =  2 – x , 1< x ≤ 2
 0 , x > 2
Solution : Fourier sine transform of f(x) is

Fs(λ) = ∫ f (u) sin λ u du
0
1 2 ∞
= ∫ u sin λ u du + ∫ (2 − u) sin λu du + ∫ (0) sin λu du
0 1 2
1 2
 − cos λu −sin λu  −cos λu −sin λu
= (u) − 1   + (2−u)  λ  − (−1)  λ2 
 λ  λ 2
0      1
− cosλ sin λ − sin2 λ cos λ sin λ
=  + 2  +  + + 2 
 λ λ   λ
2
λ λ 
2 sin λ− sin 2 λ
=
λ2

Type 3 : Fourier Integral Representation

Example : 1

Using Fourier integral representation show that. Nov. 2017



λ3sin λx π
∫ λ4+4
dλ = 2 e−x cos x, x > 0
0
Solution : Consider
π
f(x) = 2 e−x cos x, x > 0 .…(1)

Since the integral contains the term sin λx, so use Fourier sine transform
Gigatech Publishing House
Igniting Minds
Engineering Mathematics – III 3.11 Fourier and Z – Transforms
∞ ∞
π
∴ Fs (λ) = ∫ f (u) sin λu du = ∫ 2 e−u cos u sin λu du
0 0

π
= 4 ∫ e −u [sin (λ +1) u + sin (λ−1) u] du
0
∞ ∞
π π
= 4 ∫e −u
sin ( λ +1) u du + 4 ∫ e−u sin (λ−u) u du
0 0
−u

π e 
=  2 [− sin (λ +1) u − (λ +1) cos (λ +1) u]
4  1+ (λ +1) 
0

π e−u 
=  2 [− sin (λ−1)u − (λ−1) cos (λ−1) u]
4  1+ (λ−1) 
0
π  λ + 1  π  λ− 1 
=    
4  1+ (λ +1)2 + 4  1+ (λ−1)2
π λ+1 λ−1 
=  2 + 
4  λ + 2 λ +2 λ − 2λ + 2
2

π 2λ3  π λ3
=  2 2 =
4  (λ + 2) − 4λ 
2
2 λ4+4
Now by inverse Fourier sine transform

2
f(x) =
π ∫ Fs (λ) sin λx dλ
0

2 π λ3
=
π ∫ 2 λ4 + 4 sin λx dλ
0

λ3 sin λx
f(x) = ∫ λ4 + 4
dλ by equation (1) , we get
0

λ3 sin λx π −x
⇒ ∫ λ4 + 4
λ =

2
e cos x
0

Example : 2

2 cos λx  0 x<0
Using Fourier integral representation show that ∫ 1+λ 2 dλ = 
 π e−x
,
x≥0
0
Solution :
 0 , x<0
Consider , f(x) =  ...(1)
 π e−x , x ≥ 0
Since integral contains the term cos λx, x > 0, so we can use Fourier cosine transform

Gigatech Publishing House


Igniting Minds
Engineering Mathematics – III 3.12 Fourier and Z – Transforms

∴ Fc (λ) = ∫ f(u) cos λu du
0
∞ ∞
= ∫ πe −u
cosλu du =π ∫ e−ucos λu du
0 0

 e−u 
= π [ − cos λu + λ sin λu
 1+ λ2 
0
 −1 
= π  2 (−1)
1+λ 
π
Fc (λ) =
1 + λ2
Now, Inverse Fourier cosine transform of Fc (λ) is

2
f (x) =
π ∫ Fc(λ) cos λx dλ
0

2 π
f(x) =
π ∫ 1+λ2
cos λx dλ
0

2cos λx
f(x) = ∫ 1 + λ2

0
By equation (1) we get

2cos λx  0 , x<0
∫ 1 + λ2
dλ = f(x) =  −x
π e , x≥0
0

Type IV : Solution of Integral Equations

Example : 1

Solve the following integral equations.



1 −λ, 0≤λ≤1
a) ∫ f (x) sin λx dx =  0 , λ≥1
0

b) ∫ f(x) cos λ x dx = e−λ, λ > 0
0
Solution :
a) Since the term sin λx present in the integral, we can use Fourier sine transform.

1 −λ, 0≤λ≤1 1 − λ , 0 ≤ λ ≤ 1
Fs (λ) = ∫ f(u) sin λu du =  0 , λ>1
⇒ Fs (λ) = 
 0 , λ>1
0
Now inverse Fourier sine transform of

Gigatech Publishing House


Igniting Minds
Engineering Mathematics – III 3.13 Fourier and Z – Transforms

2
Fs (λ) is f(x) =
π ∫ Fs (λ) sin λx dλ
0
1 ∞
2 2
=
π0
∫ (1 −λ) sin λx dλ + π ∫ (o) sin λx dλ
1
1
2  −cos λx −sin λx 
= (1 −λ)  − (−1)  x2 
π  x    0
2 −sinx 1  2 x − sin x
=  2 +   
π x x = π  x2 
b) Since the term cos λx present in the integral, we have to find inverse Fourier cosine
transform.

Consider , Fc (λ) = ∫ f (u) cos λu du = e−λ, λ > 0 ⇒ Fc (λ) = e −λ , λ > 0
0

Now inverse Fourier consine transform of Fc(λ) is



2
f(x) =
π ∫ Fc (λ) cos λx dλ
0

2
=
π ∫ e−λ cos λx dλ
0

2  e−λ (− cos λx + sin λx) 
=  2 
π 1+x 
0
2 1 
f(x) =
π1 + x 
2

Example : 2

1 −λ , 0 ≤ λ ≤ 1
Solve the integral equation ∫ f(x) cos λx dx =  0 , λ>1
and hence show that
0

sin2z π
∫ z2 dz = 2
0

Solution :
Since the term cos λx present in the integral we have to find inverse Fourier cosine transform.

1 −λ, o ≤ λ ≤ 1 1 −λ ,0≤λ≤1
Fc(λ) = ∫ f (u) cos λu du = 
 o, λ > 1 
 ⇒ Fc (λ) = 
 0 , λ>1
….(1)
0

Now inverse Fourier cosine transform of Fc(λ) is given by

Gigatech Publishing House


Igniting Minds
Engineering Mathematics – III 3.14 Fourier and Z – Transforms

2
f(x) =
π ∫ Fc (λ) cos λx dλ
0
1 ∞
2 2
=
π0
∫ (1 − λ) cos λx dλ + π ∫ (0) cos λx dλ
1
1
2  sin λx cosλx
= (1 −λ)  −(−1) − x2 
π  x  
0
1
2  sin λx cosλx
= (1 −λ)  − 
π  x   x2  0
2 −cosx 1 
=  2 + x2
π x 
2 1−cos x
f(x) =  2  ....(2)
π x 
from equation (1) we have

Fc (λ) = ∫ f(u) cos λu du
0

∫ π u2  cos λu du
2 1− cos u
=
0
u
∞ 2 sin2 2
2
=
π ∫ u2 cos λu du
0

Put λ = 0 , we get
u
∞ 2 sin2 2
2
[ Fc (λ) ] λ = 0 =
π ∫ u2 du
0
u
∞ 2 sin2 2
2
1 =
π ∫ u2 du
0
u
Putting 2 = z ⇒ u = 2z ⇒ du = 2 dz

2 2 sin2 z
∴ 1 =
π ∫ 4 z2 2 dz
0

π sin2 z
⇒ 2 = ∫ z2 dz
0

Gigatech Publishing House


Igniting Minds
Engineering Mathematics – III 3.15 Fourier and Z – Transforms

Self–Assessment Exercise 3.1


x2
−2 2
Ex. 1. Find the Fourier transform of f(x) = e Ans. : e−λ /2
−ax
e λ
Ex. 2. Find the Fourier sine transform of x Ans. : Fs (λ) = tan−1 a
Ex. 3. Find the Fourier sine and cosine transforms of f(x) = e−x and hence show that
∞ ∞
cos mx π −m x sin mx π −m
∫ 1 + x2 dx = 2 e and ∫ 1 + x2 dx = 2 e
0 0
Ex. 4. Find the Fourier cosine integral representation for the following functions.

1 , 0 ≤ λ ≤ 1 2 sinλ
(i) f(x) = 
0 , λ> 1 
 Ans. : f(x) =
π ∫ λ
cos λx dλ
0

6 (λ2+2)
(ii) f(x) = e−x + e −2x , x ≥ 0 Ans. :f(x)=
π ∫ λ4 + 5λ2+4 cosλxdλ
0
Ex.5. Find the Fourier sine and cosine transform of x e−ax
2aλ a2 – λ2
Ans. : Fs (λ) = Fc(λ) = 2
(a + λ )2 2 2
(a + λ2)2
Ex. 6. Find the Fourier sine and cosine transform of f(x) = e−x + e−2x
λ λ 1 2
Ans. : Fs (λ) = + Fc(λ) = +
1 + λ2 4 + λ2 1 + λ2 4 + λ2

1 , 0 ≤ λ < 1
Ex. 7. Solve the integral equation ∫ f(x) sin λx dx = 2 , 1 ≤ λ < 2
0 0 , λ ≥ 2 
(2 + 2 cos x − 4 cos 2x)
Ans.: f(x)
πx

2
Ex. 8. Solve the integral equation ∫ f(x) sin λu dx = e−λ Ans. :
π (1 + x2)
0
Ex. 9. Using the Fourier integral representation show that
∞ ∞
λ sin λx π cos λx π
(i) ∫ 1 + λ2
dλ = 2 e−x, x > 0 (ii) ∫ 1 + λ2
dλ = 2 e−x, x ≥ 0
0 0

sin λ cos λx π
(iii) ∫ λ
dλ = 2, 0 ≤ x ≤ 1
0
Ex. 10 Find the Fourier transform of
1 − x2 , |x|≤ 1
F(x) = 
 0 , |x| > 1

x cos x − sinx sin λ − λ cos λ
Ans.: 4 
x
and hence evaluate ∫ x3 cos 2 dx 
0  λ3 
Gigatech Publishing House
Igniting Minds
Engineering Mathematics – III 3.16 Fourier and Z – Transforms

Ex. 11 Find the Fourier transform of


0 0 ≤ x < a a cos λa − b cos λ b sin λ b − sin λ a
f(x) =  x , a ≤ x ≤ b Ans. :  + 
0 x>b  λ   λ2 
Ex. 12 Find the Fourier cosine transform of
Ans. : 10  2 + y2 + 4
1 1
f(x) = 2 e − 5x + 5e −2x
λ + 5 
Ex.13 Find the Fourier sine and cosine transform of
2
x 0≤x<1 1
f (x) =  0 , Ans. : 3 [ 2λ sin λ − λ2 cos λ + 2 (cos λ −1)]
 x>0 λ
1
[ 2λ cos λ + λ2 sin λ − 2 sin λ]
λ3
• Introduction :
Z transforms plays a key role in discrete analysis. The development of Communication
Engineering is based on discrete analysis. Z transforms are required to study discrete analysis,
whereas Laplace Transforms are required for continuous systems.
Z– transforms are useful in the study of control systems, communication systems, signal
& linear time invariant systems and also for solving difference equations.
3.2 Prerequisites :

3.2.1 Sequence :
An arrangement of numbers with certain definite rule is called as a sequence.
1. The sequence a, a + d, a + 2d, …, a + (n – 1) d , …. is arithmetic progression / sequence.
2. The sequence a, ar, ar2, ar3 …. arn, ……. is geometric progression / sequence.
3. The sequence u1, u2, u3, u4, …., un is called finite sequence
4. The sequence u1, u2, u3, u4, … un,…, ∞ is called infinite sequence
Ex. (i) 1, 5, 9, 13, 17, ..arithmetic sequence with first term a = 1, common difference d = 4
(ii) 2, 4, 8, 16, 32, …... geometric sequence with first term a = 2, common ratio r = 2
1 1 1 1
(iii) 1, 2 , 4 , 8 ….. is geometric sequence first term a = 1, common ratio r = 2

• The sequence are written in two ways :


(i) The sequence written as u0, u1, u2, u3 ….. un is called as listing method
(ii) This sequence is also written in terms of its kth term as {uk} , where k = 0, 1, 2, 3,
… ∞ and is called as kth term method. The kth term of the sequence is written as uk
or u (k) or f (k) or fk.
(iii) The infinite sequence {f (k)} for at all k’s is written as …. f (–2), f (–1), f (0), f (1),
f (2), f (3), …..
(iv) The term (0) is called as zeroth term or term at k = 0 of the sequence {f (k)}.

Gigatech Publishing House


Igniting Minds
Engineering Mathematics – III 3.17 Fourier and Z – Transforms

The notation ↑ is used to denote the term at k = 0 or zeroth term of the sequence. If
the zeroth term is not indicated in given sequence, we treat first term of the
sequence as zeroth term.
Ex. 1. In the sequence
1 1 1 1
…. 16 , 8 , 4 , 2 , 1 , 2, 4, 8, …

1 1 1
f (0) = 1, f (1) = 2, f (2) = 4, f (3) = 8, f (–1) = 2 , f (–2) = 4 , f (–3) = 4 ,
1
f (–4) = 16

2. In the sequence
1 1 1 1
… 16 , 8 , 4 , 2 , 1, 2 , 4, 8, …

1 1 1
F (0) = 2, f (1) = 4, f (2) = 8, f (–1) = 1, f (–2) = 2 , f (–3) = 4 , f (–4) = 8 ,
1
f (–5) = 16

Note : Even through, above two sequences in example (1) and (2) have same terms but
these sequences are not identical as their zeroth terms are different
3. In the sequence
3, 9, 27, 81
F (0) = 3, f (1) = 9, f (2) = 27, f (3) = 81….
3.2.2 Linearly Property :
If {f (k)}and {g (k)} be any two sequences having same number of terms, then
(i) {f (k) + g (k)} = {f (k)} + {g (k)}
(ii) {a f (k)} = a {f (k)}, where a is constant
(iii) {a f (k) + b g (k)} = a {f (k)} + b {g (k)}, where a,b are constants

3.3 Modulus of a Real Number :


If x is any real number
Then |x| = x, x ≥ 0.
= –x, x < 0.
(i) If |x| < a then x lies between –a and a, i.e – a < x < a.
(ii) If |x| ≤ a then – a ≤ x ≤ a
(iii) |x| > a means – ∞< x < –a and a < x <∞.
(iv) |r| < 1 means – 1< r < 1

Gigatech Publishing House


Igniting Minds
Engineering Mathematics – III 3.18 Fourier and Z – Transforms

3.4 Complex Number :


1. z = x+ iy , x, y ∈ R, i = –1 is called as a complex number

2. z = x – iy
3. |z| = + x2 + y2
4. |z1 . z2| = |z1| |z2|

5. z1 = |z1|
z2 |z2|
6. |z| = |a| means x2 + y2 = a2, a circle whose centre is (0, 0) and radius = a units.
7. |z| < |a| means x2 + y2< a2 which is interior of circle x2 + y2 = a2.
8. |z| > |a| means x2 + y2> a2 which is exterior of circle x2 + y2 = a2.
9. The complex number (x + iy) is represented by a point (x, y) on xy plane.
10. eiθ = cosθ + sin θ, e–iθ = cosθ – i sin θ
11. |eiθ| = 1, |e–iθ| = 1
eix + e–ix
12. cos x = 2
e – e–ix
ix
13. sin x = 2i
ex + e–x
14. cosh x = 2
ex – e–x
15. sinh x = 2
3.5 Series :
1. If u0, u1, u2, u3, ….un …. u∞ is a sequence then u0 + u1+ u2 +u3+++ un ++ u∞ is called as a
series and written as

u0 + u1+ u2 +u3+++ un ++ u∞ = ∑ uk
k=0

2. u0 + u1+ u2 +u3+++ un ++ u∞ = ∑ uk is called as a infinite series.
k=0
n
3. u0 + u1+ u2 +u3+++ un = ∑ uk is called as finite series.
k = 0

4. ∑ uk is also called as infinite series.
k=–∞

Gigatech Publishing House


Igniting Minds
Engineering Mathematics – III 3.19 Fourier and Z – Transforms

5. Convergence :

lim
Let ∑ uk be a infinite series, let Sn = u0 + u1+ u2 +u3+++ un and if S = L, where
n →∞ n
k=0

L is a unique finite number then ∑ uk is called as convergent series otherwise it is
k=0
called as a divergent series.
n
6. The finite series ∑ uk is always convergent.
k=0
7. If few terms are added or deleted form an infinite series, it does not change the
convergence or divergence of series.
3.6 Geometric Series :
1. The series a + ar + ar2 + ar3 + ar4 + + arn +++ ∞ is called as a geometric series.
ar ar2 ar3 arn
with first term = a and common ratio r = a = ar = ar2 = n−1
ar
a
2. a + ar + ar2 + ar3 +++ ∞ = 1 – r , |r| < 1

3. |r| < 1 means – 1 < r < 1, which is called as condition of convergence of geometric
series.
3.7 Z – Transform :

Let {f (k)} be a sequence and let z = x + iy, where x, y ∈ R , i = –1 then ∑ f (k) z–k
k=∞
is called as z Transform of sequence {f (k)} and denoted by z {f (k)}.

∴ Z {f (k)} = ∑ f (k) z–k
k=∞

1. Z is called as Z – Transform operator.


2. Z {f (k)} exists only if the series on r.h.s converges.
3. Evaluation of the series on r.h.s gives a fraction in terms of z say F (z), which is a
complex valued function.

∴ Z {f (k) } = ∑ f(k) z–k = F (z)
k=–∞

4. Z {f (k) + g (k)} = Z {f (k)} + Z {g (k)}


5. Z {a f (k)} = a Z {f (k)}, where a is a constant.
6. Z {f (k)} = – +++ f (–2) z2 + f (–1) z + f (0) z0 + f (0) z f(2) z −2 + f(3) z−3 ++++

Gigatech Publishing House


Igniting Minds
Engineering Mathematics – III 3.20 Fourier and Z – Transforms

Examples
1 1 1 1 
1. If {f (k)} =  , , , , 1 , 2,, 4,, 8
16 8 4 2
 ↑ 
1 1 1
Here f (0) = 1, f (1) = 2, f (2) = 4, f (3) = 8, f (–1) = 2 , f (–2) = 4 , f (–3) = 8 ,
1
f (–4) = 16
Then Z {f (k)} = f (–4) z4 + f (–3) z3 + f (–2) z2 + f (–1) z1 + f (0) z0 + f (1) z–1
+ f (2) z–2+ f (3) z–3
= 16 z4 + 8 z3 + 4 z2 + 2 z + 1 + z + z2 + z3
1 1 1 1 2 4 8
       
1 1 1 1 2 4 8
= 16 z4 + 8 z3 + 4 z2 + 2 z + 1 + z + z2 + z3
1 1 1 1
2. If {f (k)} = , , , , 1, 2 , 4, 8
16 8 4 2

1 1 1
Here f (0) = 2, f (1) = 4, f (2) = 8, f (–1) = 1, f (–2) = 2 , f (–3) = 4 , f (–4) = 8 ,
1
f (–5) = 16
Then, Z {f (k)} = f (–5) z5 + f (–4) z4 + f (–3) z3 + f (–2) z2 + f (–1) z + f (0) z0
+ f (1) z–1 + f (2) z–2
1 1 1 1 4 8
= 16 z5 + 8 z4 + 4 z3 + 2 z2 + z + 2 + z + z2
1 1 1 1 
3. If {f (k)} =  , , , , 1,, 2,, 4,, 8
16 8 4 2 
1
As zero the term is not indicated we select f (0) = 16
1 1 1 1
Here f (0) = 16 , f (1) = 8 , f (2) = 4 f (3) = 2 , f (4) = 1, f (5) = 2 , f (6) = 4, f (7) = 8
Then Z {f (k)} =f (0) z0 + f (1) z–1 + f (2) z–2 + f (3) z–3 + f (4) z–4 + f (5) z–5 + f (6) z–6+ f (7) = z–7
1 1 1 1
= 16 + 8 z–1 + 4 z–2 + 2 z–3+ z–4 + 2z–5 + 4z–6 + 8z–7
1 1 1 1 1 2 4 8
= 16 + 8 z + 4 z2 + 2 z3 +z4 + z5 + z6 +27
k
If f (k) =   , –2 ≤ k ≤ 2.
1
4.
2
–2 –1 0 1 2
 1 1   1 1 1
Then {f (k)} = 2 , 2 , 2 , 2 , 2  = 4, 2, , 2 , 4
1 1 1
           ↑ 
1 1
∴ Z {f (k)} = 4z2 + 2z + 1 + 2z + 4z2

Gigatech Publishing House


Igniting Minds
Engineering Mathematics – III 3.21 Fourier and Z – Transforms

5. If {f (k)} = {3k} , 0 ≤ k ≤ 3.
Then {f (k)}
(k) = {1, 31 , 32, 33} = 1↑ 3, 9, 27
∴ Z {f (k)}
(k) = f(0) z–0 + f(1) z–11 + f(2) z–2 + f(3) z–3
3 9 27
= 1 + z + z2 + z3

3.8 Z– Transforms of Standard


tandard Sequences
equences :
1. If f (k)) = ak , whose a is a constant and k ≥ 0
We know,

z {ak} = ∑ ak z–k
k=–∞
∞ k ∞ k
∑ z
a a
= ∑ zk = ∴K≥0
k=0 k=0
0 1 2 3 4 Fig.: 3.
3.1
= a + a  + a + a + a +++
z z  z z z
2 3 4
1 + z + z + z + z +++
a a a a
=
       
a
Which is a geometric series having first term = 1 and r = z .
1 |a|
= , |z| < 1
1 – z
a
 
1 |a|
= , |z| < 1
 
z−a
 z 
Fig.: 3.
3.2
k z
z {a } = z – a , |z| > |a|
z
∴ Z{ak} = z – a, |z| > |a|,
|a| k ≥ 0

Note that |z| > |a| (figure


figure 2)
2 is called as region
egion of absolute convergence (ROC). IIt means
the formula for z{ak} above is true only for those values of x, y which lies out side circle |z| =
|a| figure (3.1), i.e. a circle whose centre is (0, 0) and radius = a units.
2. If f (k) = ak , k < 0
∞ –1
We know, Z {f (k) = ∑ f(k) z ––k
∑ a k z –k
k=–∞ k=–∞
–1 k ∞ k
∑ z = ∑ z
a a
=
k=–∞ k=–1
Gigatech Publishing House
Igniting Minds
Engineering Mathematics – III 3.22 Fourier and Z – Transforms
–1 –2 –3 –3 –4
= a + z
a a a
+ z
+ z z +++
a
z        
2 3 4
= a + a + a + a +++ a
z z z z
       
z z
Which is a Geometric series with first term a and common ratio a

z
a z
= z , a < 1
1– a 
 
z
a
= (a – z)
a

– z – a |z| < |a|


z
=
 

Z {ak} = – z – a , |z| < |a| , k < 0


z

 
3. αk, where c and α are constants, k ≥ 0.
If f (k) = ckcosα

k
We know Z{c cosαk} = ∑ (ckcosα k) z–k
k=–∞
∞ iαk
+ e–iαk –k
= ∑ c
e k
z
k=0
 2 
∞ ∞
1 1
= 2 ∑ ce k iαk –k
z +2 ∑ ck e–iαk z–k ∴k≥0
k=0 k=0
∞ ∞
1 1
= 2 ∑ (c eiα z–1)k + 2 ∑ (c e–iα z–1)k
k=0 k=0
1 iα –1 2 iα –1 3
2 [1 + (c e z ) + (c e z ) + (c e z ) ++] +
iα –1
=
1 –iα –1 2 iα –1 3
2 [1 + (c e z ) + (c e z ) + (c e z ) +++] , Geometric series
–iα –1

= 2  |c eiα z–1 |< 1, |c e–iα z–1 |< 1


1 1 1
iα –1 +
1 – c e z 1 – c e–iα z–1
|c| |eiα| |c| |e–iα|
= 2 
1 z z
+ , < 1, |z| < 1
z – c eiα z – c e–iα |z|
z  z – c e–iα + z – c eiα 
= 2 (z – c eiα) (z – ce–iα) |c| < |z| , |c| < |z|

Gigatech Publishing House


Igniting Minds
Engineering Mathematics – III 3.23 Fourier and Z – Transforms

z (z – c cos α)
= , |z| > |c| , k ≥ 0
z2 – 2cz cos α + c2
z (z – c cos α)
∴ Z{ck cos αk} = , |z| > |c| , k ≥ 0
z – 2 c z cos α + c2
2

z az 1
4. z{a|k|} = z–a + 1–az , |a| < |z| < |a|
czsinα
5. z{cksinαk} = , |z| > |c|, k ≥ 0
z2 – 2czcosα + c2
z (z – coshα)
6. z{ckcoshαk} = 2 , |z| > max{|ceα| | ce–α|}, k ≥ 0
z – 2czcoshα + c2
czsinh α
7. z{cksinhαk} = 2 , |z| > max{|ceα| | ce–α|}, k ≥ 0
z – 2czcoshα + c2
z(z–cosα)
8. z{cosαk} = 2 , |z|>1, k ≥ 0
x –2xcosα+1
zsinα
9. z{sinαk} = 2 , |z|>1, k ≥ 0
z – 2zcosα + 1
10. Unit step function : A function UCK = 0, k < 0
= 1, k ≥ 0 is called as unit step function
z
z{U (k)} = z–1 , |z| >1

• Dirac Delta Function/ Unit impulse Function :


A function defined as, δ(k) = 1, k = 0
= 0, k ≠ 0 is called as a Dirac Delta Function.
11. z{δ (k)} = 1
z
12. z{1k} = z{1} = z–1 , |z| > 1, k ≥ 0
z
13. z{(–a)k} = z + a , |z| > |a|, k ≥ 0

Illustrative Examples

Example : 1

Obtain z transforms of (1) 23k , (2) 4–2k , (3) a5k for k ≥ 0


Solution :
1. Z{23k} = [(23)k]
z
= z [8k] = z–8 , |z| > 8 , k ≥ 0

Gigatech Publishing House


Igniting Minds
Engineering Mathematics – III 3.24 Fourier and Z – Transforms

2. Z{4–2k} = z{(4–2)k}
1 z 1
= z (16 )k = 1 , |z| > 16 , k ≥ 0
z – 16
z
3. Z{a5k} = z{(a5)k}= cz–a5 , |z| > a5 , k ≥ 0

Example : 2

Obtain Z- Transforms of following sequences for , k ≥ 0


kπ π
1. cos  2 + 4  2. sin(3k + 4) 3. 2kcos (2k + 1)
 
4. 4k sin(2k + 5) 5. 2k cosh6k 6. 4ksinh3k
Solution :
z (z – cosα)
1. We know z{cosαk} = 2
z – 2zcosα + 1
z sinα
z{sinαk} =
z2 – 2zcosα + 1
 kπ π   π kπ π
∴ Z cos  2 + 4

= Z cos 2 cos 4 – sin 2 sin 4
    
1  
Z cos 2  – Z sin 2 
kπ kπ
=
2     
π π
 z z – cos 2 z sin 2 
1  
=
2  π

π 
 z2 – 2zcos 2 + 1 z2 – 2zcos 2 + 1

1  z2
– z2 + 1  =
z z (z – 1)
= , |z| > 1
2  z 2
+ 1  2 (z2 + 1)
2. Z {sin(3k + 4)} = Z {sin3k cos4 + cos3k sin4}
= cos4 Z {sin3k} + sin4 Z {cos3k}

cos4 z2 – 2cos3 + 1 + sin4 z2 – 2cos3 + 1


z sin 3 z (z – cos3)
=
   
2
zcos4 sin3 + z sin4 – zsin4cos3
= (z2 – 2zcos3 + 1) , |z| > 1

3. Z {2k cos(2k + 1)} = Z {2k (cos2k cos1 – sin2k sin1)}


= cos (1) Z {2k cos2k} – sin (1) Z {2k sin2k}

cos (1) z2 – 4zcos2 + 4 – sin (1)  2 


z (z – 2cos2) 2z sin2
=
   z – 4zcos2 + 4 
zcos(1) (z – 2cos2) – 2z sin (1) sin2
= , |z| > 2
(z2 – 4zcos2 + 4)

Gigatech Publishing House


Igniting Minds
Engineering Mathematics – III 3.25 Fourier and Z – Transforms
k k
4. Z [4 sin(2k + 5)] = Z {4 [sin2k cos5 + cos2k sin5]}
= cos 5Z { 4k sin2k} + sin5Z{4kcos 2k}

=
cos5 [4z sin2]  z (z – 4cos2) 
z2 – 8zcos2 + 16 + sin5 z2 – 8z cos2 + 16
[4zcos5 sin2] + sin5 [z (z – 4cos2)]
= z2 – 8zcos2 + 16 , |z| > 4

Z – (z – 2cosh 6)
z {2k cosh 6k} = z2 – 4zcosh 6 + 4 , |z| > max {|2 e | |2 e |}, |z| > 2 e , k ≥ 0
6 –6 6
5.

2zsinh3
z{zk sinh3k} = z2–4zcosh3+4 , |z| > max{|2e | |2e |} , |z| > 2e , k ≥ 0
3 –3 3
6.

Example : 3

Obtain z Transforms of following sequences


1. 23k+1 2. emk + n 3. en – pk for k ≥ 0
Solution :
1. Z { 23k + 1} = Z{ 23k 21} = 2Z { 23k} = 2z{8k}

2 z–8 = z–8 ,|z| > 8


z 2z
=
 
2. Z {emk + n} = Z{emk · en}
= en z{emk} = en Z{(em)k}

en z–em ,|z| > em


z
=
 
n – pk n –pk
3. Z {e } = Z{e · e }
= en Z{e–pk} = en Z{(e–p)k}

en 
z 
= , |z| > e–p
z – e– p
Example : 4

Obtain z Transforms of following sequences


1. z {43k–1} , k < 0 2. z { (1/2)|k|
Solution :
1. z {43k–1} = z {43k · 4–1}
1 k 1   z 
= 4 z { (64) } = 4 –z–64
1 z 
= –  , |z| < 64, k < 0
4 z–64

Gigatech Publishing House


Igniting Minds
Engineering Mathematics – III 3.26 Fourier and Z – Transforms

1z
2
,   < |z| <
z 1 1
2. z { (1/2)|k| = 1+ 1
z–2 1–2z
2 1
|2|
2z z 1
= + , 2 < |z| < 2
2z – 1 2 – z

Example : 5

Obtain z Transforms of following


1. {4k + 6k} , k ≥ 0 , 2. {2k – 3k} , k ≥ 0
Solution :
1. Z{4k + 6k} = Z {2k} + Z {6k}
z z
= z–2+z–6 , |z| > 2 , |z| > 6
z z
= + , |z| > 6
z–2 z–6
2. {2k – 3k} = z {2k}–z {3k}
z z
= + , |z| > 3
z–2 z–3

Example : 6

Obtain z Transforms of following :


1. Z {2U (k)}, 2. Z {4δ (k)}
Solution :
1. Z{2U (k)} = 2 z{U (k)}

= 2 z – 1 =
z 2z
  z –1
2. Z{4δ (k)} = 4 Z {δ (k)} = 4

Example : 7

Show that :
 1 – cos kπ – sin kπ 
 2 2 z
Z = , k≥0
 2  (z – 1) (z2 + 1)
1  kπ kπ
L.H.S. = 
Solution : 2 z 1 – cos 2  – sin 2 
π π
 z z – cos2 z sin2 
1 z  
= 2  z–1 – 2 π

π 
 z – 2z cos2 + 1 z2 – 2z cos2 + 1

Gigatech Publishing House
Igniting Minds
Engineering Mathematics – III 3.27 Fourier and Z – Transforms
2
=  z – 2z – 2 z 
1
z – 1 z + 1 z + 1 
2

1 z (z2 + 1) – z2 (z – 1) – z (z – 1) 
=

2 (z – 1) (z2 + 1) 
3 3 2 2
= z + z – z + z2 – z + z = 1 
1 2z 
 (z – 1) (z + 1)  2 (z – 1) (z2 + 1)
2

=   = R.H.S., |z| > 1, k ≥ 0


z
(z – 1) (z2 + 1)
Example : 8

Obtain Z { e–k sin 2k}, k ≥ 0


k
 1 
Solution : = Z e sin 2k
  
1
= Z { ck sin 2k} , where c = e
1
cz sin2 e z sin2
= z2 – 2cz cos2 + c2 =
z2 – 2z ecos2 + e2
1 1
 
ez sin2 1
= , |z| > e
z2e2 – 2zecos2 + 1

Example : 9

Obtain Z{eik – e– ik} , k≥ 0


ik –ik
e – e  eix – e– ix
Solution : Z[eik - e-ik] = Z  2i  (2i) sinx = 2i
 

= 2i Z {sink}

2i z2 – 2z cos(1) + 1 
z sin(1)
=
 
2iz sin(1)
= , |z| > 1 , k≥0
z2 – 2zcos(1) + 1

Example : 10

2z [z – coshθ]
Show that Z{ekθ + e–kθ} = , k≥0
z2 – 2z coshθ + 1
 ekθ + e–kθ 
Solution : L.H.S. = Z [ekθ + e–kθ] = Z  2  2 
  
= Z {2 coshk θ} = Z {2cosh θk}
θ) 
2  2
z (z – coshθ
=  = R.H.S.
z – 2z cosh θ + 1
Gigatech Publishing House
Igniting Minds
Engineering Mathematics – III 3.28 Fourier and Z – Transforms

Example : 11
k
4 
Obtain z k!
 
k ∞ ∞
4  4k –k (4/z)k
Solution : = Z  
k!
= ∑ k! z = ∑ k!
k=0 k=0
(4/z)0 (4/z)1 (4/z)2 (4/z)3 (4/z)4
= 0! + 1! + 2! + 3! + 4! +++
1 4 (4/z)2 (4/z)3
= 1 + 1! z  + L2 + L3 +++
 
x x2 x3
= e4/z Qex = 1 + L + L +L +++
1 2 3

Example : 12
k
 1 
Obtain z 2  for k = 0 , 2 , 4 , 6 , ……
  
Solution :
k k
 1 
Z 2  = ∑ 1 z –k
   2
k = 0, 2, 4, 6,…
k
1
= ∑ 2z
k = 0, 2, 4, 6,…
0 2 4
=  1  +  1  +  1  +++
2z 2z 2z
1 2 1 4 1 2
= 1 + 2z + 2z , a geometric series with first term 1 & r =2z
 
   
1  1 2 > 1 = 1 4 z2 1
= , = , |z2| <
2 4 z  1 4z2
– 1 4
1 – 2z
1 1 – 4z2
 
Example : 13
k
 1 
Obtain Z 4  , k = 1 , 3 , 5 , 7 , ……
  
k k
 1 
Solution : Z 4  = ∑ – 1 z –k
    4
k = 1, 3, 5,…
k
1
= ∑ 4z
k = 1, 3, 5,…

Gigatech Publishing House


Igniting Minds
Engineering Mathematics – III 3.29 Fourier and Z – Transforms
3 5 2
= 4z + 4z + 4z +++ which is a geometric series with first term 4z, r = 4z
1 1 1 1 1
       
1 1
4z 4z 1
= = 16z2 – 1 , (16z2) > 1
 1 2
1 – 4z 16z2
 
4z 1
= , |z2| < 16
16z2–1

Example : 14

 1 
Obtain Z (k+1)! , for k ≥ 0
 
Solution

 1  1
Z  
(k+1)!
= ∑ (k + 1)! z
–k

k=0

... z–1 z–2 z–3


ex = 1 + 2! + 3! + 4! +++
Z  z–1 z–2 z–3 
= Z  1 + 2! + 3! + 4! +++ 
z–1 z–2 z–3 z–4
= Z  1! + 2! + 3! + 4! ++ 
 
–1  1

= Z [ez – 1] = Z ez - 1

Example : 15
1
1 1 1 –k
Obtain Z  
k!
, Z  
k!
= ∑ k! z
k=0
1 z–1 z–2
Solution : = 0! + 1! + 2! ++++
1 2 3
= ez
–1
= ez ... ex = 1 + x + x + x + + +
1! 2! 3!

Example : 16

1
Obtain Z k , k ≥ 0
 
= ∑ f(k) z–k
k =1
Solution :

z–k
z{f(k)} = ∑ k
k =1

Gigatech Publishing House


Igniting Minds
Engineering Mathematics – III 3.30 Fourier and Z – Transforms
–1 –2 –3 –4
z z z z
= 1 + 2 + 3 + 4 +++
2 3 4
= – log (1 – z–1) ... log (1 + x) = x - x - x - x ….
2 3 4

Example : 17
k
4 , k < 0
Find Z transform of { f (k) }, where f (k)=  2k
 , k≥0

Solution : Z { f (k) } = ∑ f(k) z–k
k=–∞
∞ ∞
= ∑ –k
f(k) z + ∑ f(k) z–k
k=–∞ k =0
–1 ∞
= ∑ 4k z–k+ ∑ 2k z–k
k=∞ k =0

– z – 4 + z – 2
z z
= , |z| < 4 , |z| > 2
   
z z
= – , 2 < |z| < 4
z–2 z–4

Example : 18

Find Z Transform of { f (k) }


k
 2 , k<0
where f (k)=  1k
  , k≥0
 2 
k
1  
Solution : Z { f (k) } = z{2 }k < 0 + z 2 
k

   k≥0 

– z – 2 +
z z 1
= , |z| < 2 , |z| > 2
  1
z–2
2z z 1
= – , 2 < |z| < 2
2z – 1 z – 2

Example : 19
k+1
 – 14 , k<0
Find Z transform of { f (k) }, where f (k)=   1k + 1 , k≥0
 – 5 
k+1 k+1
   
z – 4  + z – 5 

1 
1  
Solution : Z { f (k) } = , (k ≥ 0)
 
 k < 0    

Gigatech Publishing House


Igniting Minds
Engineering Mathematics – III 3.31 Fourier and Z – Transforms

 1 1 k  1 1 k
= z – 4 – 4  + z – 5 – 5 
   k < 0    
1  1 k 1  1 k
= – 4 Z – 4  – 5 Z – 5  , (k ≥ 0)
  k < 0   

– 4 – 
1 z  1  z  1 1
= 1 – 5  1  , |z| < 4 , |z| > 5
 z + 4 z + 5
    
z z 1 1
= 1 – , 5 < |z| < 4
4 z + 4 5 z + 5
1
   
z z 1 1
= – , 4 < |z| < 5
(4z + 1) (5z + 1)
Example : 20
|k|
 1 
Obtain Z Transform of  3  May 2014
  
1z
 1 
|k|
3
Z  3  = 1 < |z| < 1
z
Solution :
    1 + 1 ,
3 1
z–3 1–3z
3
=
3z
+
z
, 1 < |z| < 3
3z – 1 3 – z 3
Example : 21
|k|
 2 
Obtain Z Transform of  3  Dec. 2015
  
Solution :

|a| < |z| < a


|k| z az 1
We know Z{a } = z – a + 1 – az ,
 
2
 2 
|k|
3z
Z  3  = 2 < |z| < 1
z
    2 + 2 ,
3 2
z–3 1–3z
3
3z 2z 2 3
= + , 3 < |z| < 2
3z – 2 3–2z

Example : 22

Obtain Z Transform of Z { 4k e–6k } , k ≥ 0


Solution :
k
k – 6k  k 1 
Z { 4 e } = Z 4 e6 

  
Gigatech Publishing House
Igniting Minds
Engineering Mathematics – III 3.32 Fourier and Z – Transforms
k
 4 
= Z  e6 
  
z 4
= , |z| > e6
z –  6
4
e 
Example : 23

Obtain Z Transform of { 2k e2k } , k≥0


Solution :
Z { 2k e2k } = Z { (2e– 2)k}
z
= z – 2e– 2 , |z| > 2e– 2
ze2 2
= , |z| > e2
ze2 – 2

Self-Assessment Exercises 3.2

• Obtain the Z Transform of Following Sequences :


ez sin2
Ex.1 ek sin2k Ans. : z2 – 2ez cos2 + e2

kπ π z(z – 1)
Ex.2 cos  2 + 4  , k ≥ 0 Ans. :
  2(z2 + 1)
π
z2 coshα – z cosh2 – α
 
cosh  2 + α , k ≥ 0

Ex.3 Ans. :
  π
z2 – 2z cosh2 +1
 
z (z sin5 – sin2)
Ex.4 sin (3k + 6) , k≥0 Ans. : z2 – 2z cos3 + 1
z z
Ex.5 3– 2k + 1 , k≥0 Ans. : 1+z–1
z–9

Ans. : z – e2 + 4 z – 1
z z
Ex.6 e2k + 4 , k≥0
 
1 z
5 [ (– 3) – (– 4) ] , k ≥ 0
k k
Ex.7 Ans. : 5 (z2 + 7z + 12)

1 z
Ex.8 k k
5 [ (–3) – (– 8) ] , k≥0 Ans. : z2 + 11z + 24
1 11 k 2z2 + 3z
6 (– 2) + 6 4 , k ≥ 0
k
Ex.9 Ans. : z2 – 2z – 8

Gigatech Publishing House


Igniting Minds
Engineering Mathematics – III 3.33 Fourier and Z – Transforms

z ea sinθ
Ex.10 e – ak
sin(kθ) , k≥0 Ans. : 2 2a
z e – 2z ea cosθ + 1
1
1
Ex.11 , k≥0 Ans. : z (ez – 1)
(k + 1)!
1
1 z
Ex.12 k! , k ≥ 0 Ans. : e

π
z2 cosθ – z cos 8 –θ
 
Ex.13 cos  8 + θ , k ≥ 0

Ans. :
  π
z2 – 2z cos 8  + 1
 
|k|
Ex.14 1  – 3z
Ans. : (1 – 2z) (z – 2)
2
ak
Ex.15 k , k > 0 Ans. : log ( 1– az– 1)

Ex.16 Obtain Z Transform of following sequences for k ≥ 0


(i) e–ak sin 8k (ii) 22kcos 4k (iii) e– mk
πk π
(iv) 4k cosh 2k (v) emk + n (vi) 2k cos  2 + 4 
 
(vii) { –6 , –4 , –2 , 0 , 2 , 4 } (viii) { 2 , 4 , 6 , 8 }

k
πk
(ix) 3 (xi) cosh  2 
1
(x) ekα
   
(xii) e–k sink
–4
(–3)k z
Ex.17 k! , k ≥ 0 Ans. : e

πk z  ( 2 z – 1) cosθ – sinθ 
Ex.18 cos  4 + θ , k ≥ 0 Ans. :  
  2  z2 – z 2 + 1 
Ex.19 f (k) = 3 4 + 4 5
1 k 1 k 12z 20z 1
, k ≥ 0 Ans. : 4z – 1 + 5z – 1 , |z| > 4
   
ez e2z
–k
Ex.20 (e – e ) –2k
, k ≥ 0 Ans. : ez – 1 + e2z – 1

3.9 Theorems on Z – Transforms :

3.9.1 Theorem 1 : Change of Scale:


= F a
z
If Z{ f (k) } = F(z) then z { ak f (k) } = [ F(z)]
z→
z
a
 

Gigatech Publishing House


Igniting Minds
Engineering Mathematics – III 3.34 Fourier and Z – Transforms

3.9.2 Theorem 2 : Multiplication by k :


If Z{ f (k) } = F(z) then

Z { k f (k) } = – z dz  F (z)
d
(i)
 

Z { k2 f(k) } = – z dz  F(z)
d 2
(ii)
 

Z { k3 f (k) } = – z dz  F(z)
d 3
(iii)
 
2 2
(i) – z dz  ≠ (–1)2 z2 dz 
d d
Note :
   
2
(ii) – z dz  = – z  dz  – z dz 
d d d
    
3.9.3 Theorem 3 : Recurrence Formula :
p–1
p pd
If f(k) = k , where p is a positive integer then Z{ k } = –z dz [ Z (k) ]
3.9.4 Theorem 4 : Shifting Property :
(i) Shifting to the Right
Z { f (k) } = F (z) then Z { f (k–n) } = z– n F(z) , where k ≥ n , n is a positive integer
(ii) Shifting to the Left
Z { f (k+1) } = z F (z) – z f (0)
Z{ f (k + 2) } = z2 F (z) – z2 f (0) – z f (1)
Z{ f (k + 3) } = z3 F (z) – z3 f (0) – z2 f (1) – z f (2)

Illustrative Examples

Example : 1

Obtain Z { 4k U (k) }
Solution :
z
We know, Z {U (k) } = z – 1 , |z| > 1

By change of scale property,

z { 4k U(k) } = z–1
z z
= , |z| > 4
 z→z z–4
4

Gigatech Publishing House


Igniting Minds
Engineering Mathematics – III 3.35 Fourier and Z – Transforms

Example : 2

3z + 1
If Z { f (k) } = (z – 1) (z + 1) , Find Z { 2k f (k) }
3z + 1
Solution : Z { f (k) } = (z – 1) (z + 1) = F(z)

Z { 2k f(k) } =  3z + 1 
(z – 1) (z + 1)  z → z
2
3z
2 +1
= , by change of scale property
 z – 1  z + 1 
2  2 
2 (3z + 2)
=
(z – 2) (z + 2)

Example : 3

Obtain Z { k ak } , k≥0
Solution :
z
We know, Z { ak } = z – a , |z| > |a|

= F(z)
By "Multiplying by k" theorem,
d
Z { k ak } = – z dz [F (z)]

= – z dz z – a
d z
 
= –z
(z – a) (1) – z (1)
 (z – a)2 
– z (– a) az
= (z – a)2 = , |z| > |a|
(z – a)2

Example : 4

Obtain Z { k U(k) }
z
Solution : Z { U(k) } = z –1 , |z| > 1
By "Multiplying by k" theorem,

Z{ k U(k) } = – z dz z – 1
d z
 
z
= , |z| > 1
(z – 1)2

Gigatech Publishing House


Igniting Minds
Engineering Mathematics – III 3.36 Fourier and Z – Transforms

Example : 5

Obtain Z { k } , k≥0
Solution : Z { k } = z { k (1) }
= z { k 1k}
Let f (k) = 1k
z
∴ Z { f (k) } = z { 1k } = z – 1 = F(z)

– z dz z – 1 , by multiplication theorem
d z
∴ Z { k f (k) } =
 
z
∴ z{k} = , |z| > 1
(z – 1)2

Example : 6

Obtain Z { k2 } , k≥0
Solution :
z
We have Z{1} = z–1 , |z| > 1

∴ Z { k2 } = – z d 2  z 
 dz z – 1
– z dz  – z dz z – 1
d d z
=
  
z dz  z dz z – 1
d d z
=
  
z dz  z  (z – 1)2  = z dz – (z – 1)2
d z–1–z d z
=
    
z (z –1)3 =
z + 1 z (z + 1)
= , |z| > 1
  (z – 1)3
Example : 7

Obtain Z { 4k – 2 U (k – 2) }
Solution
Let f (k–2) = 4k – 2 U (k – 2)

∴ f (k) = 4k U (k)
∴ Z{ f (k) } = z { 4k U (k) }
=  z  by change of scaler rule
z – 2  z → z4

=  z  = F (z) , |z| > 4


z – 4
Gigatech Publishing House
Igniting Minds
Engineering Mathematics – III 3.37 Fourier and Z – Transforms

By shifting theorem,
Z [f (k–2)] = z– 2 F (z)

z– 2 z – 4 
z
=
 
1
Z { 4k – 2 U(k – 2) } = , |z| > 4
z (z – 4)

Example : 8

Obtain Z { 6k U (k – 1) }
Solution
Z { 6k U (k – 1) } = Z { 6k – 1 6 U(k – 1) }
= 6Z { 6k – 1 6 U(k – 1) }
= 6Z {f (k – 1)}
= 6z–1 Z {f (k)} , By shifting theorem
–1 k
= 6z [Z { 6 U(k) }]

6z–1 z – 1 
z
= , by change of scale
 z→z
6

6z–1 z – 6  =
z 6
= , |z| > 6
  z–6
Example : 9

Obtain Z { k2 } , k ≥ 0
Solution :
By Recurrence formula
d
Z{ kp } = – z dz [ Z (kp – 1 )]
d
Z{ k2 } = – z dz [ Z (k) ]

– z dz  (z – 1)2  =
d z z (z + 1)
=
  (z – 1)3

Example : 10

Obtain Z{ (k + 2) 2k } , k ≥ 0
Solution :
Z { (k + 2) 2k } = Z { k 2k + 2 2k }
= Z{ k 2k } + 2z {2k }

= –Z dz  z – 2  + 2 z – 2
d z z
   

Gigatech Publishing House


Igniting Minds
Engineering Mathematics – III 3.38 Fourier and Z – Transforms

– z  (z – 2)2  + z – 2
(z – 2) z 2z
=
 
2z 2z
= (z – 2)2 + z – 2
2z [ 1 + z – 2]
= (z – 2)2
2z (z – 1)
= , |z| > 2 , k ≥ 0
(z – 2)2

Example : 11

Obtain Z { k e– ak } ,k≥0
Solution :
We know,
z
Z { ak } = z–a
z z ea
Z {e– ak } = z – e– a = z ea – 1

By "Multiplying by k" theorem,


z ea
– z dz  z ea – 1
d
Z { k e– ak } =
 
(z e – 1) (1) – z (ea)
a
= – ea z 
 (z ea – 1)2 
z ea
= , |z| > e–a
(e z – 1)2
a

Example : 12

Obtain Z { k cos θk } ,k≥0


Solution :
z (z – cos θ)
We know, Z ( cos θk) =
z2 – 2z cos θ +1
By "Multiplying by k" theorem,
z2 – z cos θ 
–z dz 
d
Z { k cos (θk) } = 
 z – 2z cos θ + 1
2

(z2 – 2z cos θ + 1)(2z – cos θ) – (z2 – z cos θ)(2z – 2cos θ)


= – z  
 (z2 – 2z cos θ + 1)2 
–z2 cos θ+2z – cos θ
= –z  
 (z2– 2z cos θ+1)2 
z3 cos θ – 2z2 + z cos θ
=
(z2 – 2z cos θ + 1)2
Gigatech Publishing House
Igniting Minds
Engineering Mathematics – III 3.39 Fourier and Z – Transforms

Example : 13

Obtain Z{ (k + 1) 4k } , k ≥ 0
Solution :
We know,
Z { (k + 1) 4k } = Z { k 4k + 4k }

= – z dz  z – 4  + z – 4
d z z
   
= – z  (z – 4)2 + z – 4
z–4–z z
 
4z z
= (z – 4)2 + z – 4
z [4] + [z2 – 4z]
= (z – 4)2
z2
= , |z| > 4
(z – 4)2

Example : 14

Obtain Z { k2 4k–1 U(k–1) }


Solution :
We know,
Let f (k–1) = 4k–1 U (k)
F (k) = 4k U (k)
Z ( f (k) } = Z { 4k U (k) }
= z – 1 
z
, by change of scale
 z→z
4
z
= z–4
= F(z)
But Z { f (k – 1) } = z–1 z { f (k) } , by shifting theorem

z–1  z – 4
z
=
 
z
= z–4

∴ z { k2 f (k+1) } = – z d 2 z { f(k –1) } , by Multiplication by k


 dz

– z dz  – z dzz – 4
d d 1
=
  
z dz  z  (z – 4)2  
d (z – 4) – z
=
  
Gigatech Publishing House
Igniting Minds
Engineering Mathematics – III 3.40 Fourier and Z – Transforms

z dz  (z – 4)2 
d – 4z
=
 
2
z 
(z – 4) (– 4) – (– 4z)2(z – 4) 
=
 (z – 4)4 
– 4z (z – 4)  (z – 4)4 
(z – 4) – 2z
=
 
– 4z  (z – 4)3 
–z–4
=
 
4z (z + 4)
= , |z| > 4
(z – 4)3

Example : 15

Obtain Z { k e– k } ,k≥0
Solution :
z z
We know Z { e–k } = z – e– 1 = 1
z–e
ez 1
= ez – 1 = F (z) , |z| > e

∴ By “multiplication rule”
d
Z { k e–k } = – z dz [ F(z) ]

– z dz  ez – 1  = – z 
d ez (ez – 1) (e) – ez (e)
=
  
(ez – 1)2 
2 2
– z  (ez – 1)2  =
ez–e–ez ze 1
= , |z| >
  (ez – 1)2 e

Example : 16

Obtain Z { k2 U (k) }
Solution :
We know
z
Z { U (k) } = z–1 , |z| > 1

∴ By “multiplication by k2” rule

Z { k2 U(k) } = – z d 2 F(z)
 dz
2
=  –z d   z 
 dz  z – 1 
= – z dz  – z dz z – 1  = z dz  z  (z – 1)2 
d d z d (z – 1) – z
     

Gigatech Publishing House


Igniting Minds
Engineering Mathematics – III 3.41 Fourier and Z – Transforms

z dz  z (z – 1)2  = z dz  (z – 1)2 


d –1 d –z
=
    
2
–z
(z – 1) (1) – z 2 (z – 1) 
=
 (z – 1)4 
– z (z – 1)  (z – 1)4 
z – 1 – 2z
=
 
– z  (z – 1)3
–z–1 z (z + 1)
= = , |z| > 1
  (z – 1)3

Example : 17

Obtain Z { k δ (k) }
Solution :
We know
Z { δ (k) } = 1 = F(z)
d
∴ Z { k δ (k) } = – z dz [ F(z) ]
d
= – z dz (1)

= 0

Self-Assessment Exercise 3.3

Find the Z transforms of the following for k ≥ 0


z3 + 4z2 + 2
Ex.1 Z { k3 } Ans. : (z – 1)4 , |z| > 1
z4 + 11z3 + 11z2 + z
Ex.2 Z { k4 } Ans. : (z – 1)5 , |z| > 1

az (z + a)
Ex.3 Z { k2 ak } Ans. : (z – a)3 , |z| > |a|

2 z2 (z + 1)
Ex.4 Z { (k + 1) } Ans. : (z – 1)3 , |z| > 1

z ea
Ex.5 Z { k eak } Ans. : (z – ea)2 , |z| > |ea|

4z – 1 16 (z – 1)
Ex.6 If z { f(k) }= (z + 3) (z – 2) Ans. : (z + 12) (z – 8) , |z| > 12

then find z { 4k f(k) }


z z
Ex.7 If Z { f(k) }= z – 8 Ans. : z – 16 , |z| > 16

then find z { 2k f(k) }


Gigatech Publishing House
Igniting Minds
Engineering Mathematics – III 3.42 Fourier and Z – Transforms

z
Ex.8 Z { k U(k) } Ans. : (z – 1)2 , |z| > 1

4z
Ex.9 Z { k 4k } Ans. : (z – 4)2 , |z| > 4

z
Ex.10 Z { 6k U(k) } Ans. : z – 6 , |z| > 6

1
Ex.11 Z { 8k–2 U(k–2) } Ans. : z (z – 8) , |z| > 8

4
Ex.12 Z { 4k U(k–1) } Ans. : z –4 , |z| > 4

3z
Ex.13 Z { k 3k } Ans. : (z – 3)2 , |z| > 3

5z
Ex.14 Z { k 5k } Ans. : (z – 5)2 , |z| > 5

z2
Ex.15 Z { (1 + k) 8k } Ans. : (z – 8)2 , |z| > 8

6z (z + 6)
Ex.16 Z { k2 6k } Ans. : (z – 6)3 , |z| > 6

–3k z e3
Ex.17 Z { ke } Ans. : (ze3 – 1)2 , |z| > |e–3|

z e– a (z + e–a)
Ex.18 Z { k2e–ak } Ans. : –3
(z – e– a)3 |z| > |e | (May 2015)
5z (z + 5)
Ex.19 Z { k2 5k } Ans. : (z – 5)3 , |z| > 5

e4 z(1 + e4 z)
Ex.20 Z { k2 e–4k } Ans. : (e4 z – 1)3 , |z| > |e–4|

3.10 Inverse Z Transforms :


If Z { f (k) } = F (z)
then
{ f (k) } = Z–1 [ F (z) ]
∴ Z–1 [ F (z) ] = { f (k) }
1. Z–1 is called as a Inverse z Transform operator
2. Z–1 [ F (z) + G (z) ] = Z–1 [ F (z)] + Z–1 [ G (z) ]
3. Z–1 [ a F (z) ] = a Z–1 F (z) , where a is a constant

Z–1 z – a = { ak } , k ≥ 0 , |z| > |a|


z
4.
 

Gigatech Publishing House


Igniting Minds
Engineering Mathematics – III 3.43 Fourier and Z – Transforms

Z–1 z + a = { (– a)k } , k ≥ 0 , |z| > |a|


z
5.
 

Z–1 z – 1
z
6. = { 1k } = { 1 } , k ≥ 0 , |z| > 1
 

Z–1 z – 1
z
7. = { U(k) } , |z| > 1
 

Z–1 – z – a
z
8. = { ak } , k < 0 , |z| < |a|
  

Z–1 z – a
1
9. = { ak–1 } , k ≥ 0 , |z| > |a|
 

Z–1 (z – a)2


z
10. = { k ak } , k ≥ 0 , |z| > |a|
 

Z–1 (z – 1)2


z
11. = {k} , k ≥ 0 , |z| > 1
 

Z–1  (z – 1)3 
z (z + 1)
12. = { k2 } , k ≥ 0 , |z| > 1
 
Some useful Inverse Z – Transforms
Sr. No. F(z) { f(k) = z–1 [ F (z) ] }

1  z  ak , k≥0 |z|>|a|
z – a
2  z  1 , k≥0 |z|>1
z – 1
3 z (–a)k , k ≥ 0 |z|>|a|
(z + a)

ak
– z – a
4 z , k<0 |z|<|a|
 
5 1 ak–1 , k ≥ 0 |z|>1
(z – a)

6 1 –ak–1 , k < 0 |z|<|a|


(z – a)

7 z2 (k+1) ak, k≥0 |z|>|a|


(z − a)2

8 1 δ (k) ,∀k

9 z U(k) , ∀k |z|>1
z–1

Gigatech Publishing House


Igniting Minds
Engineering Mathematics – III 3.44 Fourier and Z – Transforms

3.10.1 Inverse Z–T by Using Standard Formulae :

Illustrative Examples

Example : 1

Obtain Z–1  4z + 1
z 1
|z| > 4 , k ≥ 0
 
Solution :

Z–1  4z + 1 Z–1  
z z
 
=
 1 
 4 z + 4
 
1 –1 z 
= 4 Z  1
 z + 4
 
k
1  –1  1
=   , |z| > 4 , k ≥ 0
4 4 

Example : 2

Obtain Z–1  2z – 1
2z
, k≥0
 
Solution :

Z–1  2z – 1 Z–1   = Z–1  z 


2z 2z
 
=   1
 2z – 2
1
 z – 2
   
k
1  1
=   , k ≥ 0 , |z| > 2
2 

Example : 3

Obtain Z–1  2z+1


2z+3
, k≥0
 
Solution :

Z–1  2z+1 Z–1 


2z+3 2z + 1 + 2 –1 2 
= = Z 1 +
   2z + 1   2z + 1
Z– 1 1 +
2 
=
 1 
 z + 2 
  
Z– 1 [ 1 ] + z–1  
1
=
 1 
 z + 2
 
δ (k) + – 
1 k–1
= , k≥0

2 
Gigatech Publishing House
Igniting Minds
Engineering Mathematics – III 3.45 Fourier and Z – Transforms

Example : 4

Obtain Z–1  z – 1 
z+2
, k≥0
 
= Z– 1 
z – 1 + 3
Solution :
 z–1 
= Z– 1  1 + z – 1
3
 
= Z–1 [ 1 ] + 3z– 1  z – 1
1
 
= δ k + 3 (1)k – 1
3.10.2 Method of Partial Fractions :
Proper Fraction: The fraction g (z) is called as proper fraction
f (z)
1.
 
if degree of f(z) < degree of g(z).

Improper Fraction : The fraction g (z) is called proper fraction


f (z)
2.
 
if degree of f(z) > degree of g(z).
3. The proper fraction can be resolved into partial fractions as below
f(z) A B
(z – a)(z + b) = (z – a) + (z + b)
where A & B are constants and can be found.

Illustrative Examples

Example : 1

z
Find Z–1 (z+2) (z–4) , |z| > 4 , k ≥ 0

Solution :
z
Let F(z) = (z+2) (z–4)
F(z) 1
z = (z+2) (z–4)
1
Resolve (z+2) (z–4) into partial fractions
1 A B
(z+2) (z–4) = (z+2) + (z–4)
A(z–4) + B(z+2)
= (z+2) (z–4)

Gigatech Publishing House


Igniting Minds
Engineering Mathematics – III 3.46 Fourier and Z – Transforms

Comparing Numerators, we get


1 = A(z – 4) + B(z + 2)
1
Put z – 4 = 0 ⇒ 1 = 0 + 6B ∴ B=6

1
Put z + 2 = 0 ⇒ 1 = A (– 2 – 4) ∴ A = – 6

1 1
–6 6
1
∴ (z + 2) (z – 4) = z+2+ z–4
1  1 1 
= 6  z – 4 – z + 2
F(z) 1  1 1 
∴ z = –
6  z – 4 z + 2
1  z z 
∴ F (z) = 6  z – 4 – z + 2 , |z| > 4 , |z| > 2
1
∴ Z–1 [ F(z) ] = { 4k – (–2)k } , |z| > 4 , k ≥ 0
6

Example : 2

3z2+2z
Obtain Z–1 z2–3z+2  , |z| > 2 , k ≥ 0 Dec. 2015
 
Solution :
3z2 + 2z
Let F (z) = (z – 2) (z – 1)
F (z) 3z + 2
∴ z = (z – 2) (z – 1)
3z+2 8 5
Resolve (z–2) (z–1) = z–2 – z–1
F(z) 3z+2
∴ z = (z–2) (z–1)
8 5
= z–2 – z–1

8 z–2 – 5 z–1
z z
∴ F(z) =
   

8 z–1 z – 2  – 5 z–1 z – 1 , |z| > 2 , k ≥ 0


z z
∴ Z–1 [F(z)] =
   
= k
8 {2 } – 5 {1 } k
, |z| > 2 , k ≥ 0

Gigatech Publishing House


Igniting Minds
Engineering Mathematics – III 3.47 Fourier and Z – Transforms

Example : 3

Obtain Z–1 20z2 + 9z + 1 


13z
 
Solution :
13z
Let F(z) = 20z2 + 9z + 1
13z
∴ F (z ) = (5z + 1) (4z + 1)
F(z) 13
∴ z = (5z + 1) (4z + 1)
13 A B
Resolve (4z + 1) (5z + 1) = 4z + 1 + 5z + 1
A(5z + 1) + B(4z + 1)
= (4z + 1) (5z + 1)
Comparing Numerators,
13 = A (5z + 1) + B(4z + 1)
Put
5z + 1 = 0 
A(0) + B 4– 5 + 1  = B 1 – 5 = B 5
1 4 1
1  ⇒ 13 =
z =
–5
        
∴ B = 65
Put
4z + 1 = 0 
A – 4 + 1  + 0
5
1  ⇒ 13 =
z =
–4
  

A
– 5 + 4 – 1
= = A
 4  4
∴ A = – 52
13 52 65
∴ (4z + 1) (5z + 1) = – 4z + 1 + 5z + 1
13 13
= –
z +  z + 1 
1
 5  4
Z– 1 
13z  –1  13z 
∴ Z–1 [ F (z) ] = 1–Z  1
z + 5  z + 4 
   
k k
13 –  – –  
1 1
=
  5  4 

Gigatech Publishing House


Igniting Minds
Engineering Mathematics – III 3.48 Fourier and Z – Transforms

3.10.3 Residue Method :


1. Let F (z) be some function of z
2. Simple pole : If F (z) = ∞ at z = a then z = a is called as a simple pole of F(z).
3. Multiple pole : If a pole is repeated for more than once then the pole is called as multiple pole.
4. Residue of F (z) at simple pole z = a = [ (z – a) F (z)]
z=a

5. If a pole z = b is repeated n times then


n–1
Residue of F (z) at  1  d n 
 = n – 1 (z – 1) F(z)
repeated pole z = b  n – 1  dz  z=b

3.10.4 Residue Theorem :


Let F (z) be a function of z, then Z–1 [f (z) ] = sum of all residues at all poles of [ z k–1 F
(z) ]
3.10.5 Working Procedure of Residue Theorem :
(i) Let F (z) be a given function of z
(ii) Find all simple and multiple poles of F (z)
(iii) Construct a new function [ z k – 1 F (z) ]
(iv) Find all Residues of [ z k – 1 F (z) ] say R1, R2, R3,…..
(v) Z– 1 [ F (z)] = [ R1 + R2 + R3 + + ]

Illustrative Examples

Example : 1

Obtain Z–1 (z – 1)(z – 2) 


2z
 
Solution :

Let F (z) =  2z 
(z – 1)(z – 2) 
∴ F(z) have simple poles at z = 1, 2
2zk
zk–1 (z – 1) (z – 2)  = (z – 1) (z – 2)
2z
∴ Let z k–1 F (z) =
 
k
R1 = R [ zk–1 F(z) ]z = 1 =  (z – 1)  2z 
 (z – 1)(z – 2)  z = 1
k
=  2z 
 z – 2 z = 1
2 (1)k k
= 1 – 2 = – 2(1)

Gigatech Publishing House


Igniting Minds
Engineering Mathematics – III 3.49 Fourier and Z – Transforms
k
R2 = R [ zk–1 F(z) ]z = 2 =  (z – 1)  2z 
 (z – 1)(z –2)  z = 2
k k
=  2z  2 (2)
= 2 – 1 = 2 (2)k
 z – 1 z = 2
Z–1 (z – 1)(z – 2)  =
2z
∴ R1 + R2 = – 2(1)k + 2 (2k)
 
= 2 [ 2 ]k + [ –1 ]k = 2[2k – 1]

Example : 2

Obtain Z–1 12z2 – 7z + 1


36z
 
36z 36z
Solution : Let F (z) = 12z2 –7z +1 = (4z – 1) (3z – 1)
3z
=
z –  z – 1
1
 4  3
1 1
∴ F (z) have two simple poles at z = 4 , z = 3
3zk
∴ z k–1 F (z) =
z – 1 z – 1
 4  3
3zk
R1 = R z = 4 =  z – 4 
1 1
∴ 1
      1 
 z–
 4 
z–3 z=1
 4

k
3 4
1
  k
= – 36  4
1
=
– 
1  
 12
k
3 3
1
  k
R2 = R z = 3 =  1
1
= 36
   1  3
 
12
k k
∴ Z 
–1 36z  = 36  1 – 1 
 12z2 – 7z + 1  3 4 
Example : 3

z2
Obtain Z–1 z2 + 1 May. 2010
 
Solution :
z2 z2
Let F(z) = z +12= (z + i) (z – i)
Gigatech Publishing House
Igniting Minds
Engineering Mathematics – III 3.50 Fourier and Z – Transforms

∴ F(z) have two simple poles at z= i , –i


zk–1 z2 zk+1
∴ z k–1 F(z) = =
(z2+1) (z + i) (z – i)
zk+1 (i) k + 1
R1 = [ R at z= i ] =  (z–i) (z+i)(z–i)  = i + i = 2! (i)k + 1
1

 z= i
k+1
∴ R2 = [ R at z= –i ] =  (z+i) z  = –
1 k+1
 (z+i)(z–i) z= –i 2i (– i)
z2
Z–1  z + 1
1 k+1 1
∴ = 2i (i) – 2i (– i)
k+1
 
1 k 1 1 k 1
= 2i (i) (i) – 2i (– i) (– i)
1
(i)k + (– i)k]
2[
=

Example : 4

Obtain Z–1 (z – 4) (z + 2)2 k ≥ 0 , |z| > 4


z
 
z
Solution : Let F(z) = (z – 4) (z + 2)2
∴ F (z) have a simple poles at z = 4 & multiple pole at z = –2
zk
∴ z k–1 F(z) = (z – 4) (z + 2)2
zk
∴ R1 = R ( z = 4 ) =  (z – 4) (z – 4)(z + 2)2 
 z= 4
k
=  (z + 2)2 
z
 z=4
4k
= 36
∴ R2 = R (z = –2 ) = [ R at a pole repeated 2 times ]
2–1
1  d  2 zk 
= 2–1 (z + 2)
2 – 1  dz  (z – 4)(z + 2)2  z = – 2
k
1  d  z 
=
1 dz z – 4 z = – 2
k–1 k
=  (z – 4) k z 2– z (1)
 (z – 4) z=–2
(– 2 – 4) k (– 2)k–1 – (– 2)k
= 36
k–1
– 6k (– 2) – (– 2)k
R2 = 36

Gigatech Publishing House


Igniting Minds
Engineering Mathematics – III 3.51 Fourier and Z – Transforms

Z–1 (z – 4) (z + 2)2 =


z
R1 + R2
 
4k – 6k (–2)k–1 – (– 2)k
= 36 + 36
1
4k – 6k (– 2)k–1 – (– 2)k]
36 [
=

Example : 5

Obtain Z–1 (z–6)(z–7) , |z| > 3


1
 
Solution :
1
Let F (z) = (z – 6) (z – 7)

∴ F (z) have two simple poles at z = 6, z = 7


zk – 1
∴ z k–1 F (z) = (z – 6) (z – 7)
k–1
∴ R1= R ( z = 6 ) =  (z – 6) z 
 (z – 6)(z – 7) z = 6
k–1
=  6 
 –1 
= – 6k – 1
k–1
∴ R2 = R (z = 7 ) =  (z – 7) z 
 (z – 6) (z – 7) z = 7
k–1
=  7 
 1 
= 7k – 1

Z–1 (z – 6) (z –7) =


1
∴ R1 + R2 = 7k – 1 – 6k – 1
 
Example : 6

Obtain Z–1 (z – 3) (z – 2) , |z| > 3


1
May 2007, Dec. 2017
 
Solution :
1
Let F (z) = (z – 3) (z – 2)
z k–1
∴ zk–1 F (z) = (z – 2) (z – 3)
∴ F(z) have two simple poles at z = 2, z = 3
zk – 1
∴ z k–1 F (z) = (z – 2) (z – 3)

Gigatech Publishing House


Igniting Minds
Engineering Mathematics – III 3.52 Fourier and Z – Transforms
k–1 k–1
∴ R1= R ( z = 2 ) = z  =
2 
= – 2k – 1
 z – 3z = 2  2 – 3
k–1
∴ R2 = R (z =3 ) = z 
 z – 2z = 3
k–1
=  3  = 3k – 1
 3–2
Z–1   =
1
∴ R1 + R2 = 3k – 1 – 2k – 1
(z – 3) (z – 2)
Example : 7
2
Obtain Z z2 + 4
–1z
 
Solution :
z2 + 4 = z2– (– 4) = z2 – i24 = z2 – (2i)2 = (z + 2i)(z – 2i)
z2 z2
∴ Let F (z) = z2 + 4 = (z + 2i) (z – 2i)

∴ F(z) have two simple poles at z = 2i, z = –2i


zk–1 z2 zk+1
∴ zk–1 F (z) = (z + 2i)(z – 2i) = (z + 2i) (z – 2i)
k+1
∴ R1 = R ( z = 2i ) =  (z – 2i) z 
 (z + 2i) (z – 2i)z = 2i
k+1
=  z 
 (z + 2i)z = 2i
k+1
=  (2i) 
 2i + 2i 
1 k+1
= 4i (2i)
k+1
R2 = R ( z = –2i ) =  (z + 2i) z 
 (z – 2i) (z + 2i) z = – 2i
k+ 1
=  z 
 (z – 2i) z = – 2i
(– 2i)k + 1
= – 2i – 2i
1
= – 4i(– 2i)k + 1

z2
Z–1 z2 + 4 = R1 + R2 =  4i (2i)k + 1 – 4i(– 2i)k + 1
1 1

   
1
(2i)k + 1– (– 2i)k + 1]
4i [
=

Gigatech Publishing House


Igniting Minds
Engineering Mathematics – III 3.53 Fourier and Z – Transforms

Example : 8

Obtain Z–1 z3– z2+z–1


z
 
Solution :

Z–1 z2 (z – 1) + 1 (z – 1)


z
=
 
= Z–1 (z–1)(z2+1)
z
 
= Z–1 (z – 1)(z + i)(z – i)
z
 
z
Let F(z) = (z – 1) (z + i) (z – i)
∴ F(z) have simple poles at z = 1, i, – i
zk
∴ zk – 1 F (z) = (z – 1) (z + i) (z – i)
(z – 1) zk 
∴ R1 = R ( z = 1 ) =
 (z – 1) (z + i) (z – i) z = 1
(1)k (1)k (1)k
= =
(1 + i) (1 – i) 1 – i2 = 2
k
∴ R2 = R ( z = i ) = (z – i) z 
 (z – 1) (z + i) (z – i) z = i
k k
=  z  =
(i)
 (z – 1)(z + i) z = i (i – 1) (i + i)
(i)k (i)k ik
= 2i (i – 1) = 2
2i – 2i = – 2i – 2
k
i
= – 2(i + 1)
 (z + i) zk 
∴ R3 = R ( z = – i ) =
 (z – 1) (z + i) (z – i) z = – i
(– i)k (– i)k
= (– i – 1)(– i – i) = – 2i (– i – 1)
k k
(– i) (– i) (– i)k
= 2
2i + 2i = – 2 + 2i = 2(i – 1)

z–1 (z3 – z2 + z – 1)


1
∴ = R1 + R2 + R3
 
(1k) ik (– i)k
= 2 + +
– 2 (i + 1) 2 (1 – i)
1 ik (– i)k
= + +
2 –2(i + 1) 2(1 – i)

Gigatech Publishing House


Igniting Minds
Engineering Mathematics – III 3.54 Fourier and Z – Transforms

Example : 9
3
Obtain Z  z– 1 
z
–1
  
Solution :
3 3
Let, F(z) =  z  = z 3
z – 1 (z – 1)
∴ F (z) have a triple pole at z = 1
z3 zk + 2
∴ zk – 1 F(z) = zk – 1 (z – 1)3 = (z – 1)3
 
∴ R = Residue at z = 1 (multiple pole repeated 3 times)
1  d3 – 1  z k + 2 
= (z – 1)3
3 – 1 dz
3 –1
 (z – 1)3 z = 1

1  d2 (k + 2) 
= (
2! dz2 z
)
z=1
1 d k + 1
= 2 dz (k + 2) z z = 1
1 k
= 2 [(k + 2) (k + 1) z ]z = 1
1
= 2 (k + 2) (k + 1)
3
∴ Z–1 
z   (k + 2) (k + 1)
=
 z– 1  2

Self-Assessment Exercises 3.3


2
Z z2 + 1 Ans. : cos  2  , k ≥ 0
–1 z kπ
Ex.1.
   

Z–1 (z – 1) (z – 2) 
10 z
Ex.2. Ans. : 10 (2k – 1) , k ≥ 0
 
4 z3
Z–1 (z – 1) (2z – 1)2  Ans. : 4 – (k + 3) 2 , k ≥ 0
1 k
Ex.3.
   

Z–1 (z – 2) (z – 3) 
1
Ex.4. Ans. : 3k – 1 – 2k – 1 , k ≥ 1, |z| > 3
 

z3
Z–1   Ans. : 9 – 9 4 – 3 (k + 1) 4 [May 2012]
16 4 1 k 1 1 k
Ex.5. 2     
 1
 z – 4 (z – 1)
  

Gigatech Publishing House


Igniting Minds
Engineering Mathematics – III 3.55 Fourier and Z – Transforms
k k
Z–1   Ans. : 20 4 – 5 
z 1 1
Ex.6.     
 z – 4 z – 5
1 1
 
k k
Z–1   Ans. : 2 – 2
z 1 1
Ex.7.
 1  1    
 z – 2 z + 2
 
k k
Z–1 (4z – 1) (3z – 1)  Ans. : 36 3 – 4 
36z 1 1
Ex.8.
     
k k
z3
Z–1   Ans. : 3 (2k) + 5 (– 2) – 15 2
2 2 1 1
Ex.9.   
 1
 z – 2 (z – 2) (z + 2)
 
k k k
z3
Ex.10. Z–1   Ans. : 4 3 – 3 2 + 3k 2
1 1 1
1  1        
2

 z–3 z–2 
  
k k
Ex.11. Z–1   Ans. : 18 2 – 3 
3z 1 1
 1  1     
 z – 3z – 2
 
k+1 k+1
z2
Ex.12. Z–1   Ans. : 6 2 – 3 
1 1 1
     
 z – 2 z – 3
1 1
 

Ex.13. Z–1  (z – 3) (z – 4)


1
Ans. : 4k–1 – 3k–1
 

Ex.14. Z–1  (z – 4)3


1 1
Ans. : 2 (k – 1) (k – 2) 4 k–3
 

Ex.15. Z–1  (z – 1)2


z+2
Ans. : (3k – 2)
 
 – 5k–1 , k ≤ 0
Ex.16. Z–1  (z – 4) (z – 5) , 4 < |z| < 5
1
Ans. :  – 4k–1
   , k≥0

Ex.17. Z–1  (z – 3) (z – 4) , |z| < 4


1
Ans. : 3k–1 – 4k–1 , k < 0
 
k k
Ex.18. Z–1   , |z| < 1 Ans. : 100 5 – 80 4
1 1 1
    
 z – 4 z – 5
1 1 5
 

Gigatech Publishing House


Igniting Minds
Engineering Mathematics – III 3.56 Fourier and Z – Transforms
k
 –20 41  (k < 0)
Ex.19. Z–1   , 1 < |z| < 1
z
1  5 Ans. :  1k  (k ≥ 0)
 z – 4 z – 5
1 4
   –20 5 
2z2 + 3z
Ex.20. Z–1 (z + 2) (z – 4) 
1 11
Ans. : 6 (–2)k + 6 4k
 

z3 – 20z
Ex.21. Z–1 ( z – 3)2 (z – 4) 
1
Ans. : 2 [ 2k + 2k2 2k ] – 4k
 

Ex.22. Z–1 z2 + 11z + 24 


z 1
Ans. : 5 [ (– 3)k (–8)k ]
 

Ex.23. Z–1 (z + 3)2 (z – 2) 


Z 1 k 1
Ans. : – 25 (– 3)k – 5(– 3)k + 25 2k
 

z3 – 20z
Ex.24. Z–1 (z – 3)2(z – 4) 
1
Ans. : 2 2k + k2 2k – 4k
 

(i)k (–i)k
Ex.25. Z–1 (z–1)(z2+1) 
2z
Ans. : 1 – i+1– 1–i
 

z2
Ex.26. Z–1 (z – 1) (z – 2) 
1
Ans. : 2 [ 3k + 1 – 1 ]
 
k
Ex.27. Z–1 (2 – z) (3z – 1)  Ans. : 3 – 2k
5z 1
   

3z2 – 18z + 26
Ex.28. Z–1 (z – 2) (z – 3) (z – 4)  Ans. : 2k–1 + 3k–1 +4k–1 , k > 0
 
k k
3z2 + 2
Ex.29. Z–1 (5z – 1) (5z + 2)  Ans. : 75 5 + 5 – 5
13 1 4 2
     

Ex.30. Z–1 (z – 1)2 


z
Ans. : k
 

z2 + z
Ex.31. Z–1 (z – 1) (z2 + 1) 
1
Ans. : 1 + 2 [(i)k – 2 + (– i)k – 2 ]
 

Ex.32. Z–1 (z + 1) (z – 2) 


z+3
Ans. : 2 (– 1)k – 1 – (– 2)k – 1
 

Gigatech Publishing House


Igniting Minds
Engineering Mathematics – III 3.57 Fourier and Z – Transforms

3.11 Difference Equations :


Let {f (k) } be a sequence. The relation in terms of f (k), f (k+1), f (k+2) .. is called as
difference equation.
The performance of discrete systems is expressed in terms of a difference equation. Z
Transforms and Inverse Z Transforms are useful for solving difference equations.
• Shifting theorem :
1. Z [ f (k – n) ] = z–n z [ f (k) ]
2. Z [ f (k + 1) ] = z F(z) – z f (0)
3. Z [ f (k + 2) ] = z2 F(z) – z2 f (0) – z f (1)
4. Z [ f (k + 3)] = z3 F(z) – z3 f (0) – z2 f (1) – z f (2)

Illustrative Examples

Example : 1

Solve the difference equation


6 f (k + 2) – f (k + 1) – f (k) = 0 …(1)
where, f (0) = 0, f (1) = 1 , k ≥ 0
Solution :
Take Z. T. of equation (1)
6Z { f (k + 2) – Z {f (k + 1) – Z { f (k) } = Z{0}
6 [z F(z) – z2 f (0) – z f (1)] – [z F(z) – z f(0)] – F(z) =
2
0
(6z2 – z – 1 ) F(z) = 6z
6z
F(z) = 6z2 – z – 1 …(2)

Use of Inverse z Transform :

Z–1 6z2 – z – 1
6z
Z–1 [ F(z) ] =
 
Z–1 (3z + 1) (2z – 1) = Z–1  
6z z
{ f (k) } =
   1   1
z + 3  z – 2
 
Z–1  z 
1 1 
= 1 – 1 Use method of partial fraction
 z – 2 z + 3  
  
6 –1  z z 
= 5Z  1 – 1
 z – 2 z + 3
 
k k
6  1
– –  
1
f(k) =
5  2   3 

Gigatech Publishing House


Igniting Minds
Engineering Mathematics – III 3.58 Fourier and Z – Transforms

Example : 2

Solve the difference equation Dec. 2010, May 2016


k
1
y (k + 1) + 2 y (k) = 2 1
…(1)
 

where, y (0) = 0, k ≥ 0
Solution :
Take Z.T. of both sides of equation (1)
k
 1 
Z { y (k + 1) } + 2 Z { y (k) } = Z  2 
1
 
1 z
zY (z) – zy (0) + 2 Y(z) = 1
z–2

∴ z + 1 Y (z) = z
…(2)
 2 z – 1 
 2
Take Z–1 on both sides of equation (2)
Z–1  
z
Z–1 [y(z)] = 
z + 2 z – 2
1 1
 
Z–1  z  
1
y (k) = 
  z + 2  z – 2
1 1

  
Z–1  z 
11  
= 1– 1  Use method of Partial fractions
 z – 2 z + 2 
   
= Z–1  
z z
 1 –  1
 z – 2 z + 2
 
k k
1   1 
y (k) =  –  –  
2   2 

Example : 3

Solve the difference equation


6x(k + 2) – 5x (k + 1) + x(k) = 0 … (1)
x(0) = 0 , x(1) = 2, k ≥ 0 Dec. 2006
Solution :
Take Z.T. of both sides of equation (1)
6Z[ x (k + 2)] – 5Z[x (k + 1)] + Z {x (k) = 0

Gigatech Publishing House


Igniting Minds
Engineering Mathematics – III 3.59 Fourier and Z – Transforms

∴ 6 [z2x (z) – z2x(0) – zx(1)] – 5[zx (z) – zx(0)] + x(z) = 0


(6z2 – 5z + 1) x(z) – 12z = 0
(6z2 – 5z + 1) x(z) = 12z
12z
∴ x(z) = 6z2 – 5z + 1 … (2)

Take Z–1 of both sides of equation (2)

Z–1 [x(z) ] = Z–1  6z2 – 5z + 1


12z
 
z(k) = Z–1 (3z – 1) (2z – 1)
12z
 
= Z–1   = 2 Z–1  
12z z
  1
 1   1
3z – 3 2z – 2  1  
 z – 3 z – 2
   
2 Z–1 z 
1  = 2 Z–1  z  6 – 6 
= 1    1 1 
  z – 3 z – 2
1
 z – 2 z – 3
     
= 2 Z–1 
6z 6z   –1  z  –1 z 
1– 1 = 12 Z  1 – Z  1
 z – 2 z – 3  z – 2  z – 3
      
k k
 1 1 
= 12    –   
 2  3 
Example : 4

Solve 12 f (k + 2) – 7 f (k + 1) + f (k) = 0 , where …(1)


f (0) = 0, f (1) = 3, k ≥ 0.
Solution :
Take Z.T. of equation (1)
12 Z [f(k+2) ] – 7 Z [f(k+1) ] + Z [ f(k) ] = 0
2 2
12 [ z F(z) – z f(0) – z f(1) ] – 7 [ z F(z) – z f(0) ] + F(z) = 0
[ 12z2 – 7z +1 ] F(z) = 36z
36z
∴ F(z) = 12z2 – 7z +1
36z 36z
= (4z – 1) (3z – 1) =
4  z – 4 3 z – 3
1 1
   
3z
= … (2)
 z –  z – 1 
1
 4  3 
Taking Z–1 of equation (2)

Gigatech Publishing House


Igniting Minds
Engineering Mathematics – III 3.60 Fourier and Z – Transforms

Z–1  
3z
Z–1 [F (z)] = 1  1  

 z – 4 z – 3  
 
3Z–1  z  
1
f(k) =
 1   1  
  z – 4 z – 3  
  
3Z–1  z 
12 12 
= – 1 
  z – 3  z – 4 
1
  
36 Z–1 
z z 
= 1 – 1
z–3 z–4
 
k k
= 36  Z–1  1  – z–1 
z z   1 1 
1  = 36  3 – 4 
 z – 3   z – 4 
    
Example : 5

Solve the difference equation


uk + 2 + 4 uk + 1 + 3uk = 3k … (1)
with u0 = 0 , u1 = 1 , k≥0
Solution :
Take Z.T. of equation (1)
Z{ uk + 2 } + 4Z{ uk + 1 } + 3Zuk = Z { 3k }
z
∴ [z2 U(z) – z2 u(0) – z u(1) ] + 4 [ z U(z) – z u(0)] +3U(z) = z–3
z
U(z) [ z2 + 4z + 3 ] – z = z–3
z
U(z) [(z + 1) (z + 3)] = z+z–3
z
U(z) = (z + 1) (z + 3)
z
+ (z + 1) (z + 3) (z –3)
z2 – 3z + z
= (z + 1) (z + 3) (z – 3)
z2 – 2z
U(z) = (z + 1) (z + 3) (z – 3) …(2)
Take Z–1 of both sides of equation (2)
2
= Z (z + 1) (z + 3) (z – 3)
–1 –1 z – 2z
Z U(z)
 

Gigatech Publishing House


Igniting Minds
Engineering Mathematics – III 3.61 Fourier and Z – Transforms

Z–1  z (z + 1) (z + 3) (z – 3)


z–2
U(k) =
  
= Z–1  z (z + 1) + (z + 3)+(z – 3)
A B C
  
  3 –5 1 
  8 12 24 
= Z–1  z z + 1 + z + 3 + z – 3
  
(use method of partial fraction)

Z–1 8 (z + 1) – 12 (z + 3) + 24 (z – 3) 


3 z 5 z 1 z
=
      
3 5 1
–1 k – –3 k + 3 k , |z|3
8 {( ) } 12{( ) } 24{( ) }
U(k) =

Example : 6

Solve the difference equation


yk + 2 + 6 yk + 1 + 9yk = 2k …. (1)
where y0 = 0 , y1 = 0 , k≥0
Solution :
Taking Z.T. of equation (1)
Z {yk+2 } + 6Z { yk+1 } + Z{9yk } = z { 2k }
z
[z2 Y (z) – z2 Y (0) – z Y (1)] + 6 [zY (z) – zY (0)] + 9y (z) = z – 2
z
∴ [z2 + 6z + 9] Y (z) = z – 2
z
Y (z) = 2
(z – 2) (z + 6z + 9)
z
Y (z) = (z – 2) (z + 3)2 …(2)
Take Z–1 on both sides of equation (2)

Z–1 (z – 2)(z + 3)2 = Z–1  z (z – 2) (z + 3)2


z 1
Z–1 [ y (z)] =
    
= Z–1  z z – 2 + z + 3 +(z + 3)2 
A B C
  
  1   1   1 
  25   25   5 
= Z–1  z z – 2 – z + 3 + (z + 3)2
      
1 –1  z z 5z 
= 25 Z z – 2 – z + 3 – (z + 3 )2
 
1
(2)k– (– 3)k + 5k (– 3)k]
25[
y(k) =

Gigatech Publishing House


Igniting Minds
Engineering Mathematics – III 3.62 Fourier and Z – Transforms

Self-Assessment Exercises 3.4

Solve the following difference equations :


k k k
yk + 1 + 4 = 4 Ans. : yk = 2  4 – 4 
yk 1 1 1
Ex.1 y0 = 0 , k ≥ 0
      
Ex.2 fk + 2 – fk + 1 + fk = 2k f0 = 2, f1 = 1, k ≥ 0 Ans. : fk = 1 – 2k + 2k

Ex.3 xk + 2 – 3xk + 1 + 2xk = 0 x0 = 0, x1 = 1, k ≥ 0 Ans. : xk = 1 – 2k


k k
y0 = 1, y1 = 1, k ≥ 0 Ans.: yk = 5 2 – 5 – 3
8 1 3 1
Ex.4 6yk + 2 – yk + 1 – yk = 0
   
Ex.5 fk + 3fk – 1 – 4fk – 2 = 0, k ≥ 2 f0 = 3, f1 = –2, Ans. : fk = 2 + (– 4)k

Ex.6 yk+2 – 4yk+1 + 3yk = 5k, k ≥ 0 y0 = 0, y1 = 1,k ≥ 0

Ex.7 f (k + 1) – f (k) = 1 f (0) = 0, k ≥ 0 Ans. : f (k) = k


k k k
f (k + 1) + 2 f (k) = 2 (May 2016) f (0) = 0, k ≥ 0 Ans. : 2 – – 2
1 1 1 1
Ex.8
     
Ex.9 f (k + 2) + 3f (k + 1) + 2f (k) = 0 f (0) = 0, f (1) = 1, k ≥ 0 Ans. : (– 1)k – (– 2)k

(May 2014)
Ex.10 f (k + 2) + 3f (k + 1) + 2f (k) = 0 f (0) = 0, f (1) = 2, k ≥ 0

Ans. : 2[(– 1)k – (– 2)k ]

Ex.11 12f (k+2) – 7 f (k + 1) + f (k) = 0 f (0 ) = 0, f (1) = 3, k ≥ 0 (Dec. 2016)


k k
Ans. :36  3 – 4 
1 1
    
Ex.12 6y (k + 2) – 5y (k + 1) + y (k) = 0 y (0) = 0, y (1) = 3, k ≥ 0
k k
Ans. : 18  2 – 3 
1 1
    
k k k
Ex.13 x (k) – 4x (k – 2) =  2 k ≥ 0 Ans. : – 15 2 + 3 (2)k + 5 (– 2)k
1 1 1 2 2
   
Ex.14 y (k+2) – 3y (k + 1) + 2y (k) U (k), f(0) = 0 Ans. : 2k – 1
–1
k k
Ex.15 4x (k) + x (k – 2) = 4 2 sin  2  , k ≥ 0 Ans. : (k + 1) 2 sin  2  , |z| > 2
1 kπ 1 kπ 1
       

Gigatech Publishing House


Igniting Minds
Engineering Mathematics – III 3.63 Fourier and Z – Transforms

Descriptive Questions

Q. 1 Find the Fourier sine and cosine transform of x e−ax



Q. 2 Solve the integral equation ∫ f(x) sin λu dx = e−λ
0
Q. 3 Find the Fourier transform of
0 0 ≤ x < a
F(x) =  x , a ≤ x ≤ b
0 x>b
Q. 4 Find the Fourier sine and cosine transform of
x2 0 ≤ x < 1
f (x) =  0 ,
 x>0

Obtain the Z Transform : cosh  2 + α , k ≥ 0



Q. 5
 
Q. 6 Find the Z transforms of the following for k ≥ 0
1. Z { k2 ak }
2. Z { (1 + k) 8k }
3. Z { k2e–ak }

4. Z–1 (z – 2) (z – 3) 
1
 
2
Z (5z – 1) (5z + 2) 
–1 3z + 2
5.
 
6. f (k + 2) + 3f (k + 1) + 2f (k) = 0 f (0) = 0, f (1) = 2, k ≥ 0
k
x (k) – 4x (k – 2) =  2 k ≥ 0
1
7.
 
Q. 7 Find the Fourier cosine transform of e−x + e−2x, x > 0
Q. 8 Using Fourier cosine integral of e−mx (m > 0) prove that

mcosλx dλ π −mx
∫ m2 + λ2
= 2 e , (m > 0, x > 0)
0
Q. 9 By considering Fourier cosine transform of e−mx, (m > 0) prove that.

cos λx π −mx
∫ x2+m2 dλ = 2m e m > 0, x > 0,
0
Q. 10 Find the Fourier sine transform of the following function.
 x , 0 ≤ x ≤ 1
f(x) =  2 – x , 1< x ≤ 2
 0 , x > 2

Gigatech Publishing House


Igniting Minds
Engineering Mathematics – III 3.64 Fourier and Z – Transforms

Q. 11 Using Fourier integral representation show that.



λ3sin λx π
∫ λ4+4
dλ = 2 e−x cos x, x > 0
0
|k|
 1 
Q. 12 Obtain Z Transform of  3 
  
|k|
 2 
Q. 13 Obtain Z Transform of  3 
  
2
Q. 14 Obtain z–1 z2–3z+2  , |z| > 2 , k ≥ 0
3z +2z
 
2
Q. 15 Obtain Z–1 z2 + 1
z
 
Q. 16 Obtain Z–1 (z – 3) (z – 2) , |z| > 3
1
 
Q. 17 Solve the difference equation
k
y (k + 1) + 2 y (k) = 2
1 1
…(1)
 
where, y (0) = 0, k ≥ 0
Q. 18 Solve the difference equation
6x(k + 2) – 5x (k + 1) + x(k) = 0 … (1)
x(0) = 0 , x(1) = 2, k ≥ 0

Gigatech Publishing House


Igniting Minds
Unit
Vector Differential
4 Calculus

Syllabus :
Physical interpretation of Vector differentiation, Vector differential operator, Gradient, Divergence and
Curl, Directional derivative, Solenoidal, Irrotational and Conservative fields, Scalar potential, Vector
identities.

 Scalar :
A quantity which has got magnitude alone but not direction is known as scalar quantity.
e.g. : Mass, Length, Time, Temperature, Work, Energy, Statistical data.
 Vector :
A quantity which has got both magnitude and direction is called vector quantity.
e.g. : Velocity, Acceleration, Force, Displacement, Momentum.
 Position Vector :
A point P in space can be associated with a vector by
jointing the point P with some point O in space called
 
origin of reference. Thus OP = a is associated with point P
and is called position vector of the point P. Fig.: 4.1

 Product of Vectors :
1. Scalar product (or) Dot product :
 
If a and b are two vectors inclined at an angle  with each
 
other, then the dot product of vectors a and b are defined
   
as a b = | a | | b | cos

= ab cos a = | a | Fig.: 4.2

Gigatech Publishing House


Igniting Minds
Engineering Mathematics – III 4.2 Vector Differential Calculus

b=|b|
Note :
 
i) If vectors a and b are in the same direction (i.e. parallel) then  = 0 cos = 1,
 
a  b = ab
   
ii) If vectors a and b are in the opposite directions  = , cos = 1 a  b = ab.
    
iii) If vectors a and b are perpendicular (or orthogonal)  = 2, cos = 0, a  b = 0
 
iv) If two vectors are equal a = b then
 
a  a = a2
   
v) a  b = b  a
2. Vector (or cross) product :
 
If a and b are two vectors inclined at an angle  with each
 
other then the cross product of vectors a and b is defined
 
^ where n^ is a unit vector normal to the
as a  b = ab sin n,
   
plane containing the vectors a and b such that a, b and n^
form a right handed system.
 
  a b
 |ab| = ab sin , n^ =   Fig.: 4.3
| a b |
Note :
   
i) If vectors a and b are in the same direction (i.e. parallel)  = 0, sin = 0, a  b = 0
   
ii) If vectors a and b are in the opposite direction  = , sin = 0, a  b = 0
    
iii) If vectors a and b are perpendicular (or orthogonal),  = 2 , sin = 1, a  b = abn^
   
iv) If two vectors are equal a = b a  a = 0
   
v) ( a  b) =  ( b  a)
 
vi) Let a = a1i + a2j + a3k, b = b1i + b2j + b3k
   i j k 
then a  b =  a1 a2 a3 
 b1 b2 b3 
 Unit vector :
A vector whose magnitude is unity (or one) is called unit vector.
 
If a is any vector and ^a a unit vector in the direction of a then we have

    a
a = |a|a a = 
|a|
Gigatech Publishing House
Igniting Minds
Engineering Mathematics – III 4.3 Vector Differential Calculus

Note : Let i, j, k are unit vectors along X–axis, Y–axis and Z–


axis. Then the dot product of unit vectors are ;
i  i = 1, j  j = 1, k  k = 1, i  j = 0, j  k = 0, k  i = 0
The cross product of unit vectors are
i  j = k, j  k = i, k  i = j, i  i = 0, j  j = 0, k  k = 0
 Scalar Triple Product :
   Fig.: 4.4
Let a, b and c are three vectors then their dot product is expressed as
          a1 a2 a3 
a ( b  c) = b  ( c  a) = c  ( a  b) =  b1 b2 b3 
 c1 c2 c3 

where, a = a1i + a2j + a3k

b = b1i + b2i + b3k

c = c1i + c2j + c3k
 2 2 2  2 2 2  2 2 2
| a |= a1 + a2 + a3 , | b | = b1 + b 2 + b 3 , | c | = c1 + c2 + c3
  
if any two vectors are equal then a  ( b  c) = 0
 Vector Triple Product :
  
Let a, b and c are there vectors then their vector (or cross) product is expressed as
        
a  ( b  c) = ( a  c) b ( a  b) c
 Angle between two vectors :
 
a b
The angle between two vectors is given by cos  =  
|a ||b |
 
where, a = a1i + a2j + a3k, b = b1i + b2j + b3k
 2 2 2  2 2 2
|a| = a 1 + a2 + a 3 | b |= b1 + b2 + b3
 Distance between two points :
The distance between two points is given by,
 Fig.: 4.5
PQ = (x2 – x1) i + (y2 – y1) j + (z2 – z1) k
 Position Vector :
The position vector

r = xi + yj + zk and

| r | = r = x2 + y2 + z2
x2 + y2 + z2 = r2 …. Equation of sphere
Coordinates of centre are C1 (0, 0, 0) and radius = r.
Fig.: 4.6

Gigatech Publishing House


Igniting Minds
Engineering Mathematics – III 4.4 Vector Differential Calculus

The dot product, cross product and unit vector are :



      r
rr = r , r  r = 0 , r = | r | ^r
2
 ^r = 
| r |

 r 
or r = r ^r i.e. ^r = r (i.e. r = | r |)

Perpendicular distance of the point (x1, y1, z1)


from the plane ax + by + cz + d = 0 is
ax1 + by1 + cz1 + d 
p =   
 a2 + b2 + c2 
Fig.: 4.7

 Direction Cosines :
Let any line OP make angle , ,  with x, y,
z – axes respectively, then cos, cos, cos are
called the direction cosines of this line which are
usually denoted by l, m, n.
l = cos, m = cos, n = cos and l2 + m2 + n2 = 1

Fig.: 4.8
 Direction Ratios :
If the direction cosines of a line be proportional to a, b, c then these are called
l m n
proportional direction cosines or direction ratios of the line, then a = b = c =
l2 + m2 + n2 1
= 2
a 2 + b 2 + c2 a + b2 + c2
a b c
 l= 2 , m= 2 , n= 2
a + b 2 + c2 a + b2 + c2 a + b 2 + c2
Note : Any unit vector can be expressed as,
^r = cos i + cosj + cosk = li + mj + nk

| ^r | = cos2 + cos2 + cos2 = l2 + m2 + n2 = 1


 Equation of a Straight Line :
i) Equations of the line through the point (x1, y1, z1) and having direction cosines l, m, n
are
x  x1 y  y1 z  z1
l = m = n
ii) The equations of the line joining the points (x1, y1, z1) and (x2, y2, z2) are
x  x1 y  y1 z  z1 x  x1 y  y1 z  z1
= =  a = b = c
x2 x1 y2 y1 z2 z1
where a, b, c are direction ratios of the line.

Gigatech Publishing House


Igniting Minds
Engineering Mathematics – III 4.5 Vector Differential Calculus

1. Differentiation of Vector :
 
If a vector r varies continuously as a scalar variable t changes, then r is said to be a

function of and is written as r = F (t). Here we define derivative of a vector function

r = F (t) as
   
dr dF lim F (t + t) - F (t)
= dt = t  0
dt t
2. General rules of differentiation :
  
If  u , v , w are scalar and vector functions of a scalar variable t, we have.
  
i)
dt
[ u + v + w] = ddtu + ddtv  ddtw
d   


d   d du
ii) ( u ) = u dt + dt 
dt
 
d
iii)
dt
(u  v ) = u  ddtv + ddtu 
v
 
d
iv)
dt
(u  v ) = u  ddtv + ddtu 
v
  
d    du  d v    dw
v)
dt [ ]
u  ( v  w ) = dt  ( v + w) + u   dt  w + u   v  dt 
 
   
  
 d v    dw
vi)
d
dt
[u  v  w] = ddtu  (v  w) + u   dt  w + u   v  dt 

 dr    
Obs. 1. If r (t) has a constant magnitude, then r = 0. For r (t)  r (t) = | r (t)|2 = constant.
dt
  
 dr dr dr 
⸫ r  dt = 0 i.e. either dt = 0 or dt  r (t).

  dr
Obs. 2. If r (t) has constant (fixed) direction then r  dt = 0, let ^r be a unit vector in the
  
direction of r (t) so that r (t) = r ^r r = | r |.
 
dr d^r dr ^  dr
= r + r and r 
dt dt dt dt
 d^r dr 
= r ^r  r dt + dt ^r
 

 dr d^r
r  dt = r2 ^r  dt = 0

 d^r 
since of ^r is constant  dt = 0
 

Gigatech Publishing House


Igniting Minds
Engineering Mathematics – III 4.6 Vector Differential Calculus

dr
 Geometrical Interpretation of .
dt
    
Let OP = r and OQ = r +  r then
     
OP + PQ = OQ or PQ = OQ  OP
dr

dr
dt as Q  P, t  0 the
Since = lim
dt
t0
 Fig.: 4.9
dr
direction of dt approaches the direction of the tangent
to the curve at P.

dr
i.e. : The vector is along the tangent to the space curve traced out by P.
dt
 
  d r d2 r
Let OP makes a position vector r and t is time the dt , dt2 are velocity and
 chord PQ 
acceleration.  lim = 1
 P ArcPQ 

Illustrative Examples

Example : 1
    
If F = a coswt + b sinwt, where a, b, w are constant show that,

d2 F 2

i) 2 + w F =0
dt

 dF  
ii) F  dt = w ( a  b) = constant.

Solution : Differentiating F with respect t we get

dF  
= a ( w sint wt) + b(w cos wt)
dt

d2 F  

dt2 = a( w2cos wt) + b( w2sin wt)



d2 F   
i) dt2 =  w2 ( acos wt + bsin wt) =  w2 F

d2 F 2

2 + w F = 0
dt

Gigatech Publishing House


Igniting Minds
Engineering Mathematics – III 4.7 Vector Differential Calculus

 dF    
ii) F  dt = ( acoswt + bsinwt)  ( a( wsinwt) + b(wcoswt))

 dF    
F  dt = ( a  a) ( w sinwt coswt) + ( a  b) (w cos2wt)
   
+ ( b  a) ( w sin2wt) + ( b  b) (w sinwt coswt)
   
= 0 + w ( a  b) cos2wt + w ( a  b) sin2wt + 0

 dF
F
dt
=

w ( a  b)

[... a  a = 0 b  b = 0 b  a = ( a  b)]
       

Example : 2
 
da   db   d     
If dt = c  a , dt = c  b then prove that dt ( a  b) = c  ( a  b)

Solution: By product formula of differentiation


 
d   d a   db
The derivative of dt ( a  b) = dt  b + a  dt
d        
 ( a  b) = ( c  a)  b + a  ( c  b)
dt
     
=  b  ( c  a) + a  ( c  b)
d              
( a  b) = [( b  a) c  ( b  c) a ] + [( a  b) c  ( a  c) b]
dt
           
= ( a  b) c + ( c  b) a + (( a  b) c  ( c  a) b
     
= ( c  b) a  ( c  a) b
  
= c  ( a  b)
Example : 3
   
d  d v d2 v   d v d3 v
Prove that  v   = v  3
dt  dt dt2  dt dt
Solution : The derivative of scalar triple product is
        
d  d v d2 v  d v d v d2 v  d2 v d2 v  d v d3 v
 
dt  v dt  dt2  = dt  dt  dt2 + v  dt2  dt2 + v  dt  dt3
 
 dv d3 v
= 0 + 0 + v  dt  dt3
 
 d v d3 v
= v 
dt dt3
  
(If any two vectors are equal then a  ( b  c) = 0]
Gigatech Publishing House
Igniting Minds
Engineering Mathematics – III 4.8 Vector Differential Calculus

Example : 4
  
If F = asinh t + bcosh t then prove that
 
 d F d2 F
i) F  =0
dt dt2
 
dF d2 F  
ii) dt  dt2 = a  b = constant

Solution : Differentiating F w.r.t. t we get,

dF  

dt = a (cosh t) + b (sinh t)

d2 F  
= a (sinh t) + b (cosh t)
dt2
  
 d F d2 F  d2 F
i) F  dt  dt2 = 0 ... F = dt2
  
[By scalar triple product if any two vectors are equal then a  ( b  c) = 0]
 
d F d2 F    
ii) dt  dt2 = [ a (cosh t) + b(sinh t)]  [ a(sinh t) + b(cosh t)]
 
d F d2 F    

dt  dt2 = ( a  a) (cosh t sinh t) + ( a  b) (cosh2 t)


   
+ ( b  a) (sinh2 t) + ( b  b) (sinh t cosh t)
   
= 0 + ( a  b) cosh2 t  ( a  b) sinh2 t + 0
 
dF d2F  
 = ( a  b) = constant.
dt dt2
       
[... ( a  a) = 0, b  b = 0, cosh2 t – sin2h t = 1 b  a =  ( a  b)]

Example : 5
 
     d r d2 r  
If r = t (cost a + sint b) where a, b are constant vectors. Prove that dt   dt2 + r  =
 
 
2 ( a  b)

Solution : Differentiating r w.r.t. t by using product formula we get,

dr    

dt = (cost a + sint b) + t(sint a + cost b)



d2 r      
= (sint a + cost b) + (sint a + cost b) + t(cost a sint b)
dt2

Gigatech Publishing House


Igniting Minds
Engineering Mathematics – III 4.9 Vector Differential Calculus

2
d r  

dt2 = (2sint – tcost) a + (2cost – tsint) b



 d2 r    
r + dt2 = t (cost a + sint b) + (2sint – tcost) a + (2cost – tsint) b

 d2 r  
r + = 2sint a + 2cost b
dt2
 
d r  d 2 r     
 
dt   r + dt2  = [(cost – tsint) a + (sint + tcost) b]  [(2sint) a + (2cost) b]
 
d r  d 2 r     
 
dt   r + dt2  = (cost – tsint) (2sint) ( a  a) + (cost – tsint) (2cost) ( a  b)
   
+ (sint + tcost) (2sint) ( b  a) + (sint + tcost) (2cost) ( b  b)
 
d r  d 2 r   
  [2cos2t – 2tsint cost + 2sin2t + 2tsint cost] ( a  b)
dt   r + dt2  =
 
= 2 (cos2t + sin2t) ( a  b)
 
= 2 ( a  b) = constant.
       
[... ( a  a) = 0, b  b = 0 , ( b  a) =  ( a  b), cos2t + sin2t = 1]

Example : 6
 
If a = sini + cosj + k, b= cosi sinj 3k

c = i + 2j + 3k.
d    
Find [ a  ( b c)] at  = 2
d
        
Solution : By vector triple product a  ( b  c) = ( a  c) b  ( a  b) c
  
a  ( b  c) =  1 sin2 + 2cos2 3 cos + 3 i + (sin2 – sin2
2 
+ 3 sin + 6)j + (3sin – 6cos) k.
d   
[ a  ( b  c)] = [cos2 – 4cos sin – 3cos + 3 sin + 3)i + [2sincos
d
2cos2 + 3sin + 3cos + 6)j + [3cos + 6sin]k.

At  = we get,
2
d   
[ a  ( b  c)] =  1  0  0 + 3 + 3  i + (0 + 2 + 3 + 0 + 6)j + (0 + 6)k
d  2 

=  2 + 3  i + 11j + 6 k
 2 

Gigatech Publishing House


Igniting Minds
Engineering Mathematics – III 4.10 Vector Differential Calculus

Example : 7

The position vector of a point at any time ‘t’ is given by r = et (cost i + sint j) show that
    
a = 2 v – 2 r where a and v are acceleration and velocity of a particle. Also find the angle
between the radius vector and the acceleration.

Solution : Differentiating r w.r.t. t we get,

 dr t
v = dt = e [(cost – sint) i + (sint + cost) j]

 d2 r t
a = dt2 = 2e ( sint i + cost j) …. (i)
 
2v – 2 r = 2et[ (cost – sint – cost) i + (cost + sint – sint) j]

a = 2et (sint i + cost j) …. (ii)
  
From (i) and (ii) we get a = 2 v – 2 r
The angle between the radius vector and acceleration is given by

a r 
cos =   =0  =
2
(.a  r = 0)
|a|r|

Example : 8

A particle is moving along the curve x = t3 + 1, y = t2, z = t. Find the velocity and
acceleration at t = 1

Solution : The position vector r = xi + yj + zk

r = (t3 + 1)i + t2j + tk

 dr 2
Velocity v = dt = 3t i + 2tj + k

At t = 1 v = 3i + 2j + k

 d2 r
Acceleration a = = 6ti + 2j
dt2

At t=1 a = 6i + 2j

Example : 9

Find the angle between the tangents to the curve r = (t3 + 2) i + (4t – 5) j + (2t2 – 6t) k at
t = 0 and t = 2.

dr

Solution : The tangent vector T = dt = 3t2 i + 4 j + (4t – 6) k

at t = 0 T1 = 4j – 6k, at t = 2.
  
T2 = 12i + 4j + 2k |T1 | = 52 , |T2| = 164
Gigatech Publishing House
Igniting Minds
Engineering Mathematics – III 4.11 Vector Differential Calculus

⸫ The angle between tangents is


 
T 1  T2 0 + 16 - 12 4
cos  =   = = .
|T1| |T2| 52 164 52 164


  
The vector differential operator  is defined as  = i + j + k it involves the
x y z
  
operators of partial differentiation such as , , and the component of unit vectors
x y z
along the axes i, j, k with the help of  we will define the gradient the divergence and
the curl.

 The gradient of a scalar function :


Let the scalar function  (x, y, z) be continuous and differentiable, then the gradient of
the function (x, y, z) is defined as
  
Grad  =  = i + j + k = vector quantity
x y z
 Geometrical Interpretation of Gradient :
  
If d r is differential of position vector r , then d r = dx i + dy j + dz k
  
 d r =  i + k  (i dx + j dy + k dz)

+ j
 x y z 
  
= dx + dy+ dz = d
x y z
 Since d is total differential of  :

If d r is on a level surface (x, y, z) = c

then d = 0 so that d r = 0 and therefore 
has a direction normal to the level surface.
Let  = |  | n^ where ^n is a unit vector normal Fig.: 4.10
to the surface (x, y, z) = c and Q is a point on the
surface  +  = c. Let PN = n be the perpendicular distance between the two surfaces

at P. Then the rate of change of  in the direction of normal to the surface = = lim
n n  0

  r 
= lim since  =  r
n n  0 n

 n^  d r
 = lim |  | = |  |
n n  0
n

Gigatech Publishing House


Igniting Minds
Engineering Mathematics – III 4.12 Vector Differential Calculus

Since n = PN = PQcos = r cos = n^  r
Hence the gradient of a scalar field  is a vector normal to the surface  (x, y, z) = c and
having a magnitude equal to the rate of change of  along this normal.
 Level surface :
If (x, y, z) be a single valued continuous scalar point function through any point p of
the region considered, we can draw a surface such that at each point on it the function
has the same value at P. Such a surface is called a level surface of the function. Thus the
surface (x, y, z) = constant, is a level surface. No two level surfaces (corresponding to
two different values of the constant) can intersect, otherwise the single valued function 
will have two different values at the point of intersection. If (x, y, z) represents
potential at the point (x, y, z) the equipotential surface (x, y, z) = c, is an example of a
level surface. Similarly, if  denotes temperature, the surface of constant temperature is
called isothermal surface.
 Some Properties of Gradient :
i) If  be constant, then grad  = 0
ii) If 1 and 2 be any two differentiable scalar functions, then.
(a) (1  2) = 1  2
(b) (c11 + c22) = c11 + c22
(c) (12 = 12 + 21
1 2112
(d)    = , 2  0
 2  
2
2
(e) if2 = f(1) then 2 = f(1) = f (1) 1
  f (r) 
(f) If r = xi + yj + zk and r = | r | then f(r) = f (r)r = r
r

 
Let |  r | = r = PQ and u^ be the unit vector in the direction of PQ. Then  r = r u^ . The
  
directional derivative of  at P in the direction of u^ is defined as = lim
r r r – ∞ r
where Q  P in direction of u. ^
n
Since u^  n^ = cos  and cos = , we have
r
n n
r = =
cos u^  n^
 lim n lim ^ ^ 
 = = un
r r  0 n/u^  n^ r  0 n
  ^ ^
= u^  n^ = u  n || = u^ 
r n

Gigatech Publishing House


Igniting Minds
Engineering Mathematics – III 4.13 Vector Differential Calculus

u
 D.D. =   u^ where u^ = 
|u|

Since || = and n^ || = 
n
 ^ Clearly
Hence the directional derivative is the resolved part of  in the direction u.
r
|| gives the magnitude of maximum rate of change of .

Illustrative Examples

Example : 1
  
If a = a1i + a2j + a3k and r = xi + yj + zk, r = | r | then prove that :
      f(r) 
i) ( a  r ) = a ii) ( a) r = a iii) f(r) = f(r) r = r r
  1 m 
iv)   rn dr = rn  1 r v) rm = mrm2 r vi) rm = rm+2 r
    
r    a r  a n( a  r )
vii) logr = viii)  | r |2 = 2 r ix)   =  n+2
r2  rn  rn r
     
  ba (a r ) (b  r )
x) b   [ a   logr] = r2  2 r4
Solution :
 
i) The dot product of a  r = a1x + a2y + a3z
  
( a  r ) =  i + j + k  (a1 x + a2 y + a3 z)
 

 x y z 
  
( a  r ) = i a1 + j a2 + k a3 = a
  
 ( a  r ) = a

ii) The dot product of a and  is
  
(a1i + a2j + a3k)   i + k 

a  = +j
 x y z 
  
= a1 +a + a3
x 2 y z
   a  + a  + a   (xi + yj + zk)
 ( a ) r =  1 x 2 y 3 z 
 

= a1i + a2j + a3k = a
  
( a ) r = a

Gigatech Publishing House


Igniting Minds
Engineering Mathematics – III 4.14 Vector Differential Calculus

 i  + j  + k   f(r)
iii) f(r) =  x y z 

r r r
= i f (r) + j f (r) + k f (r)
x y z
r r r
f(r) = f  (r)  i + j + k 
 x y z 
f  (r) 
= f (r) r = r
r
Since r2 = x 2 + y 2 + z2
r x r y r z
= , = , =
x r y r z r
r r r
 r = i +j +k
x y z
x y z 1
r = ir +j r +kr=r r

iv) Let f(r) = rn dr then f (r) = rn


f  (r)  rn  
 f(r) = r r  rn dr = r r = rn1 r

v) Let f(r) = rm and f (r) = mrm1


mrm1  m2
rm = r r = mr r
1 m m1
vi) let f(r) = rm = r and f(r) = mr
1 mrm1  m 
  rm =  r r =  rm+2 r
1
vii) Let f(r) = log r and f(r) = r
1/r  1 
 log r = r= 2r
r r

viii) We know that |r| = r
2
 |r| = r2 and f(r) = r2, f (r) = 2r
 2r  
 | r |2 = r r = 2r
 
a r  1  1
  rn  = rn ( a  r ) + ( a  r )   rn 
 
ix)
   
1     n 
= rn a + ( a  r )  rn+2 r
 
  
a r  a n  
  rn  = rn  rn+2 ( a  r ) r
 
Gigatech Publishing House
Igniting Minds
Engineering Mathematics – III 4.15 Vector Differential Calculus
  
  r a r
x) Consider a  log r = a  r2 = r2
 
 a r  a 2   
[ a  log r] =   r2  = r2  r4 ( a  r ) r (by previous example n = 2)
 
 
  ba 2    
 b  [ a  log r] =  4 ( a  r) ( b  r)
r2 r

Example : 2

If  is the acute angle between the surfaces xy2z = 3x + z2 and 3x2 – y2 + 2z = 1 at the
3
point (1, –2, 1), show that cos =
7 6
Solution :
Angle between two surfaces at a point is the angle between the normals to the surfaces
at that point. Now let
1 = xy2z  3x  z2 = 0 and 2 = 3x2  y2 + 2z  1 = 0

Then N1 = 1 = (y2z  3) i + (2xyz) j + (xy2 – 2z)k
= i – 4j + 2k at (1, –2, 1)

N2 = 2 = 6xi  2yj + 2k = 6i + 4j + 2k at (1, 2, 1)
 
N1N2 6 – 16 + 4   –6  3
 cos =   =
 = =
| N1 | |N2 |  21 56   14 6  7 6

Example : 3

Find the value of constants  and  so that the surfaces x2  yz = ( + 2) x and 4x2y +
3
z = 4 intersect orthogonally at the point (1, –1, 2).
Solution :
Let 1 = x2 – yz – ( + 2) x = 0 .... (i)
2 = 4x2y + z3 – 4 = 0 .... (ii)
The given point (1, –1, 2) must lie on both the surfaces.
Substituting in (i) we get  + 2 =  + 2   = 1
The surfaces (i) and (ii) will intersect orthogonally if the normals to them at (1, –1, 2)
are perpendicular to each other.
 at (1, –1, 2)
1  2 = 0 .... (iii)
Now 1 = {(2x – ( + 2)}i – zj – yk
= ( – 2) i – 2j + k at (1, –1, 2) as  = 1
and 2 = 8xyi + 4xj + 3z2k

Gigatech Publishing House


Igniting Minds
Engineering Mathematics – III 4.16 Vector Differential Calculus

= –8i + 4j + 12k at (1, –1, 2)


Substituting the values of 1 and 2 in (iii) we get
{( – 2)i – 2j + k}  (–8i + 4j + 12k) = 0
5
–8( – 2) – 8 + 12 = 0   = 2 = 2.5

  = 2.5 and  = 1

Example : 4

Find the directional derivative of the function  = x2 – y2 + 2z2 at the point P (1, 2, 3) in
the direction of the line PQ where Q has co–ordinates (5, 0, 4). In what direction it will be
maximum and what is its value?
Solution :

we have PQ = (5i + 4k) – (i + 2j + 3k) = 4i – 2j + k
 4i – 2j + k 1
Let u^ be a unit vector along PQ then u^ = = (4i – 2j + k)
16 + 4 + 1 21
Now  = 2xi – 2yj + 4zk
= 2i – 4j + 12k at the point (1, 2, 3)

 Directional derivative in the direction of PQ =   ^u
1
= (2i – 4j + 12k)  (4i – 2j + k)
21
1 28 4
= (8 + 8 + 12) = = 21
21 21 3
The directional derivative is maximum in the direction of the normal to the given
surface this is the direction of .  the maximum value of directional derivative at P(1, 2, 3)
= ||P = |2i – 4j + 12k|
= 4 + 16 + 144 = 2 41

Example : 5

Find the directional derivative of  = xy2 + yz2 at (2, –1, 1) in the direction normal to the
surface xy = z3 at the point (1, 4, 2)
Solution :
 = i(y2) + j (2xy + z2) + k (2yz)
 = i – 3j – 2k at the point (2, –1, 1)
3
let 1 = xy – z = 0
then 1 = yi + xj – 4z2k

u 4i + j – 12k
 u^ =  =
|u| 16 + 1 + 144

Gigatech Publishing House


Igniting Minds
Engineering Mathematics – III 4.17 Vector Differential Calculus

1
u^ = (4i + j – 12k)
161
 Directional derivative in the direction normal to the surface is =   u^
1
D.D. =   u^ = (i – 3j – 2k)  (4i + j – 12k)
161
1
= (4 – 3 + 24)
161
25
D.D. =
161

Example : 6

Find the directional derivative of  = x logz – y2 + 4 at (–1, 2, 1) in the direction of the


tangent to the curve x = e–t, y = 2sint + 1, z = t – cost at t = 0
Solution :
x
Now  = log z i – 2y j + k = (0) i – 4j – k at the point (–1, 2, 1)
z

let r = xi + yj + zk = e–ti + (2sint + 1)j + (t – cos t) k

 dr
T = dt = –e–ti + 2 costj + (1 + sint) k = –i + 2j + k at t = 0

^ = 1 1
 D.D. =   T [(0)i – 4j – k]  (–i + 2j + k) = (–8 – 1)
6 6
9
D.D. = –
6

Example : 7

If the directional derivations of  = a(x + y) + b(y + z) + c(x + z) has a maximum value


x–1 y–2 z–1
12 in the direction parallel to the line 1 = 2 = 3 Find the values of a, b, c.

Solution :

Here the direction vector u = i + 2j + 3k and
 = i(a + c) + j(a + b) + k(b + c) has maximum magnitude || =
12 in the direction parallel to the given line.

 u^ =   = || u^
||
 i(a + c) + j (a + b) + k(b + c) = 12 (i + 2j + 3k) equating
co–efficient of unit vectors we get,
a + c = 12 .... (i)
a + b = 24 .... (ii)

Gigatech Publishing House


Igniting Minds
Engineering Mathematics – III 4.18 Vector Differential Calculus

b + c = 36 .... (iii)
(m) – (i) we get,
b – a = 24 .... (iv)
From (ii) & (iv) we get b = 0, a = 24, c = 36

Example : 8

A scalar field f has at the point (1, 2) directional derivative +2 in the direction towards
(2, 2) and –2 in the direction towards (1, 1) determine gradf. at (1, 2) also find the direction
towards (4, 6)
Solution :
Given : f  ^a = + 2

f  ^a  i f + j f   ^a
  x y 
=
 
f f
+2 =  i + j   i
 x y 
f
 = 2
x

 i f + j f   b
ii) f  b =  x y 
 
f f
– 2 =  i + j   (–j) Fig.: 4.11
 x y 
f
 = 2
y
f f
 f = i +j = 2i + 2j
x y
iii) D.D. = f  ^i
1 1 14
= (2i + 2j)  5 (3i + 4j) = (6 + 8) =
5 5

When the vector operator  operates scalarly on vector point function the result is a

scalar quantity, this is known as the divergence of vector function. Thus if F = F1i + F2j + F3k
is a vector point function, then
  
div F = F =  i + j + k   (F1i + F2j + F3k)
 

 x y z 
 F1 F2 F3
F = + + = Scalar quantity
x y z
Gigatech Publishing House
Igniting Minds
Engineering Mathematics – III 4.19 Vector Differential Calculus

Consider a small parallelopiped of


dimensions x, y, z with its edges parallel to the
co–ordinate axes.
Let the co–ordinates of P be (x, y, z) the
mass of fluid flowing through the face PQ QP per
unit time is
(V)y xz = Vyxz = F(y) say Fig.: 4.12 .... (i)
and the mass of the fluid flowing out through the opposite face SRRS is
F Vy 
F(y + y) = F(y) + y = Vy + dy  xz .... (ii)
y  y 
by using Taylor’s series.
Taking the difference of (i) and (ii) we obtain the decrease of mass inside the
Vy
parallelopiped per unit time due to one pair of face as xyz.
y
Now taking into account the other two pairs of faces, we get the total decrease in the
Vx Vy Vz
mass of the fluid per unit time in the parallelopiped as  + +  xyz ...(iii)
 x y z 
Hence the rate of loss of fluid per unit volume is
Vx Vy Vz  
+ + =   v = div v
x y z
This is true exactly in the limit as the parallelopiped shrinks top as x, y, z approach
zero.

When the vector operator  operates vectorially on a vector point function the result is a

vector quantity, this is known as the curl of vector function. Thus if F = F1i + F2j + F3k is a
vector point function, then
   i j k 
curl F =   F =  /x /y /z 
 F1 F2 F3 
F F F3 F1  F2 F1 
= i 
2
– j  + k 
3
     
 y z   x z   x y 
= Vector quantity

Consider the rotation of a rigid body about a fixed axis. Referring to figure  is the
  
angular velocity of the body v is the linear velocity of the point P and OP = r taking ‘O’ as
origin of co–ordinates.

Gigatech Publishing House


Igniting Minds
Engineering Mathematics – III 4.20 Vector Differential Calculus
 
Let r = xi + yj + zk and  = 1i + 2j + 3k
  
We have v =  r
= (2z – 3y)i + (3x – 1z)j + (1y – 2x)k
 
 Curl v = v

= 2(1i + 2j + 3k) = 2
 1  1 
  = 2   v = 2 curl v

 Vector Identities :
i) ^r = li + mj + nk , |^r | = l2 + m2 + n2 = 1 = 1
  
ii) ( u  ) r = u
 
iii) ( u  ) = u  
Fig.: 4.13

    
1. For a constant vector A, div A = A = 0, curl A =   A = 0
 
2. If u and v are vector functions and  is scalar function then
   
i)   ( u  v) =   u    v
   
  ( u  v) =   u    v
  
ii)   ( u) =   u + (  u)
  
  ( u) =   u + (  u)
     
iii)   ( u  v) = v  (  u) – u  (  v)
         
  ( u  v) = (  v) u – (  u) v + ( v  ) u – ( u  ) v
         
iv)  ( u  v) = ( u  ) v + ( v  ) u + u  (  v) + v  (  u)
      
v) If u = constant, ( u  )  v =  ( u  v) – (  v) u
  
e.g. : Show that ( a  )  r = –2 a
        
( a  )  r =  ( a  r ) – (  r ) a = a – 3 a = –2 a

Illustrative Examples

Example : 1
 
If r = xi + yj + zk and r = | r | then show that

i)  r =3

ii)  r =0
1
iii) r = r r

Gigatech Publishing House


Igniting Minds
Engineering Mathematics – III 4.21 Vector Differential Calculus

Solution :
  i  + j  + k    (xi + yj + zk)
i)  r =  x y z 

   
 r = (x) + (y) + (z) = 1 + 1 + 1 = 3
x y z

  r = 3
  i j k 
ii)  r =  /x /y /z  = i (0 – 0) – j(0 – 0) + k(0 – 0)
 x y z 

  r = 0
iii) Let f(r) = r then f (r) = 1
f (r)  1
f(r) = r  r = r
r r

Example : 2

Find the divergence and curl of the vector F = (xyz) i + (3x2y)j + (xz2 – y2z)k at the
point (2, –1, 1).
Solution :
    
div F =   F = (xyz) + (3x2y) + (xz2 – y2z)
x y z

F = yz + 3x2 + 2xz – y2

F = 14 at the point (2, –1, 1)
   i j k 
curl F =   F =  /x /y /z 
 xyz 3x2y zx2 – y2z 

F = i(–y2 – 0) –j (z2 – xy) + k(6xy – xz)

F = –i – 3j – 14k = –(i + 3j + 14k)

Example : 3
  
Find div F and curl F, where F =  (x3 + y3 + z3 – 3xyz)
Solution :

Since F = (x3 + y3 + z3 – 3xyz)

F = i(3x2 – 3yz) + j(3y2 – 3xz) + k(3z2 – 3xy)
    
Now, div F =   F = (3x2 – 3yz) + (3y2 – 3xz) + (3z2 – 3xy)
x y z

F = 6x + 6y + 6z = 6(x + y + z)
Gigatech Publishing House
Igniting Minds
Engineering Mathematics – III 4.22 Vector Differential Calculus

 
i j k 
and   F =  /x /y /z 
 3x2 – 3yz 3y2– 3xz 3z2 – 3xy 
 
curl F =   F = i(–3x + 3x) – j(–3y + 3y) + k(–3z + 3z) = 0

 F = 0

Example : 4
 
If r 1 and r 2 are vectors joining the fixed points A(x1, y1, z1) and B(x2, y2, z2) to a
variable point P(x, y, z) using vector identities prove that :
   
i)  ( r 1 r 2) = r 1 + r 2
 
ii)   ( r 1  r 2) = 0
   
iii)   ( r 1  r 2) = 2 ( r 1 – r 2)
Solution :

Let r1 = (x – x1)i + (y – y1)j + (z – z1)k
 Fig.: 4.14
and r2 = (x – x2)i + (y – y2)j + (z – z2)k
   
  r 1 = 3,   r 1 = 0,   r 2 = 3,   r2 = 0
         
i) ( u  v) = ( u  ) v + ( v  ) u + u  (  v) + v  (  u)
         
( r 1  r 2) = ( r 1 ) r 2 + ( r 2 ) r 1 + r 1  (  r 2) + r 2  (  r 1)
     
( r 1  r 2) = r 1 + r 2 + 0 + 0 = r1 + r2
     
ii)   ( u  v) = v  (  u) – u  (  v)
     
  ( r 1  r 2) = r 2  (  r 1) – r 1  (  r 2) = 0
 
  ( r 1  r 2) = 0
         
iii)   ( u  v) = (  v) u – (  u) v + ( v  ) u – ( u  ) v
         
  ( r 1  r 2) = (  r 2) r 1 – (  r 1) r 2 + ( r 2  ) r 1 – ( r 1  ) r 2
     
  ( r 1  r 2) = 3r1 – 3r2 + r2 – r1
 
= 2r1 + 2r2

Example : 5
  
If F =  where ,  are scalar point functions then show that F  (  F) = 0
Solution :
 
We have curl F =   F =   ()

F =    +  (  )
 
 F  (  F) = ()  [   +  (  )]

Gigatech Publishing House


Igniting Minds
Engineering Mathematics – III 4.23 Vector Differential Calculus
 
F  (  F) = [  (  )] + ()2 [  (  )]
 
F  (  F) = 0
  
in scalar triple product a  ( b  c) = 0 if any two vectors are equal.

Example : 6
  
If r and r have their usual meaning and a and b are constant vectors, prove that :

i)   (rn r ) = (3 + n)rn

ii)   (rn r ) = 0
1 3
iii)    r   3   = 4
 r  r
    
iv) curl [( r  a)  b] = b  a
 
 a  r  2–n n   
v)    rn  = rn a + rn+2 ( a  r ) r
 
Solution :
  
i) Using vector identities we have   ( u) =   u +  (  u)
   nrn – 1  
   rn r = rn  r + rn   r = r r  r + rn (3)
 nrn 2   
  rn r = r + 3rn = (3 + n)rn (... r  r = r2 ;  r = 3)
r2
put n = –3 we get
1
  r3  = (3 – 3)r–3 = 0
 
  
ii)   ( u) =   u +  (  u)
  
  (rn r ) = rn  r + rn(  r )
nrn–1   n 
= r ( r  r ) + r (  r)
  
= 0 [... ( r  r = 0,   r = 0]
1 –3r–4  3
iii)   r3  = r–3 = r r = – r5 r
 
1 3 3
r  r3 = r  – r5 r  = – r 4 r
 
1 1
   r  r3  –3    r4 r  = –3[r–4  r + r–4 (  r )]
 
=
   
–5
–4r
–3  r r  r + r–4(3) 
 
=
 
1 4 3 –1 3
   r  r3  = (–3)  – r4 + r4  = (–3)  r4  = 4
      r
Gigatech Publishing House
Igniting Minds
Engineering Mathematics – III 4.24 Vector Differential Calculus
     
iv) curl [( r  a)  b] = –   [ b  ( r  a)]
     
= –   [( b  a) r – ( b  r ) a]
     
= – ( b  a) (  r ) +   [( b  r ) a]
     
= 0 +  ( b  r )  a + ( b  r ) (  a)
      
  [ b  ( r  a)] = ba (...   r = 0 ;   a = 0)
 
ar  
v)   =   [ a  r–n r ]
 r 
n

       
= (  r–n r ) a– (  a) rn r + (r–n r ) a – ( a)r–n r
    
= [r–n r + r–n ( r )] a – 0 + 0 –  (r–n r ) ( = a )
–n–1
=  –nr r r + r–n(3)  a – r–n  r – r r–n
 r 
n 2 3
– n+2  –n    
= r + a – r ( a ) r – r ( a )r–n
 r r 
n

= – nn + 3n  a – r–n a – r (a r–n)


 r r 
 –n–1
– n + 3  a   – nr
=  rn  a – rn – r  a 

r
   r 
 
–n+3–1  ar
=  a – r (– n)  n+2 
 r n
  r 
2–n n  
=
a + n+2 ( a r) r
rn r
 Repeated operations by  :
1. Div (grad ) =    = 2 (2 = 0, Laplace equation)
2. Curl (grad ) =    = 0 (Irrotational vector field)
 
3. Div (curl F) =   (  F) = 0 (Solenoidal vector field)
   
2
4. curl F =   (  F) = (  F)  –  F

Illustrative Examples

Example : 1

2
With usual notations prove that 2 f(r) = f(r) + r f(r) also determine f(r) such that
2f(r) = 0
Solution :
f(r) 
We know that f(r) = r r , then
Gigatech Publishing House
Igniting Minds
Engineering Mathematics – III 4.25 Vector Differential Calculus

f(r) 
2f(r) =   f(r) =    r r 
 
f(r)  f(r) 
=  r  r + r ( r )

2f(r) =  1  f(r) + f(r)  1   r + f(r) (3)


r r r
 1 f(r) 1   3
2f(r) =  r r r + f(r)  – r3 r    r + r f(r)
  
f(r) 2 f(r) 2 3
= r2 r – r3 r + r f (r)
2
2f(r) = f(r) + f (r), since 2 f(r) = 0 then
r
2 f(r) 2
f(r) + r f (r) = 0=1 =– r on integration
f(r)
we get, log f(r) = –2 log r + log c
C C
 f(r) =  f(r) = – + C1
r2 r
B
 f(r) = A+ (A = C1 , B = –C)
r

Example : 2

Prove the following :


i) 2rn = n(n + 1) rn–2
1 n(n – 1)
ii) 2 rn = rn+2

 r  2
iii) 2    =
 r2  r4
iv) 2 () = 2 + 2   + 2
6
v) 4(r2 log r) = r2

Solution :
i) f(r) = rn, f(r) = nr–n–1 f(r) = n(n – 1) rn–2
by formula
2
2f(r) = f(r) + r f (r)
2
2rn = n(n – 1) rn–2 + r nrn–1 = n(n + 1)rn–2
1
ii) f(r) = rn = r – n , f(r) = – nr – n –1, f(r) = – n(– n – 1) r– n – 2

Gigatech Publishing House


Igniting Minds
Engineering Mathematics – III 4.26 Vector Differential Calculus

by using formula
1 2
 2 rn = n (n + 1) r– n – 2 + r (– n r – n – 1)
1 n (n – 1)
 2 n =
r rn+2

r   
iii)  =  r– 2 r = r– 2 r + r– 2( r )
r2
r– 3   2 3 1
= –2 r r  r + 3r– 2 = – r2 + r2 = r2

 r  1 2.1 2
    r2  =  2  r2  = r4 = r4
2
   
put n = 2
iv) 2 () =    () =   [ + ]
=    + (  ) +      + (  )
2
 () = 2 + 2   + 2
2
v) 4(r2 log r) = 2 [2(r2 log r)] = 2F(r) = f(r) + f(r)
r
6 2 6 6
4(r2 log r) = – r2 + r  r = r2
2
F(r) = 4(r2 log r) = 2f(r) = f (r) + r f(r)
2
f(r) = r2 logr = 3 + 2log r + r r (1 + 2 log r)
1
f(r) = r2  + 2r  log r = 5 + 6 log r
r
1
f (r) = 1 + 2  log r + r  r  F(r) = 5 + 6 log r
 
6 6
f (r) = 3 + 2 log r F (r) = , F (r) = – 2
r r

Example : 3
  
If   F = 0 Show that curl curl curl curl F = 4 F.
Solution :
 
Consider curl curl F =   (  F)
    
curl curl F = (  F)  – 2 F = 0 – 2 F = F 1
  
curl curl curl curl F =         F =     F1
   
= (  F 1)  – 2 F 1 = 0 – 2 F 1 = – 2 (–2 F)
 
 curl curl curl curl F = 4 F

Gigatech Publishing House


Igniting Minds
Engineering Mathematics – III 4.27 Vector Differential Calculus

1. The divergence of a vector is zero everywhere in a field, that field is called solenoidal
   
i.e. div V = 0,   V = 0. Suppose   V = 0 then V determines a solenoidal field and we
 
know that from vector identities  · (  F) = 0 from the above equations we have V =

  F.
 
i.e. The solenoidal field V can be expressed as the curl of the another vector F.
2. The curl of a vector is zero everywhere in a filed, that field is called irrotational.
   
curl F = 0 i.e.   F = 0. Suppose   F = 0 then F determines an irrotational field and

we know that from vector identities   () = 0. From the above equations we have F
= .

i.e. The irrotational field F can be expressed as the gradient of scalar function

(i.e. F = )
To determine  :
i)  =  (F1dx + F2dy + F3dz) + C. Integrate F1 with respect to x keeping y, z as
constants.
Integrate F2 with respect to y keeping z as constant, excluding ‘x’ terms.
Integrate F3 with respect to z, excluding x, y, terms.
 
ii) If the vector function contains ‘r’ terms then use  F  d r + C.

Illustrative Examples

Example : 1

Verify whether the following vector field F = (y sinz – sinx) i + (x sinz + 2yz) j + (xy
cosz + y2) k is irrotational. If so find the corresponding scalar potential function  such that

F = 
Solution :

For irrotational vector field   F = 0
  i j k 
 F =  /x /y /x 
 ysinz – sinx xsinz + 2yz xycosz + y2

  F = i[(x cosz + 2y) – (x cosz + 2y)] – j[y cosz – y cosz] + k [sinz – sinz] = 0
 
   F 0 i.e. F is irrotational field.
To find scalar potential function .
 =  [(ysinz – sinx) dx + 2yzdy + 0] + C
Gigatech Publishing House
Igniting Minds
Engineering Mathematics – III 4.28 Vector Differential Calculus

 = xysinz + cosx + y2z + C


 = i(y sinz – sinx) + j (x sinz + 2yz) + k (xy cosz + y2)

 = F

Example : 2
 
If F 1 = (y + z)i + (z + x)j + (x + y)k and F 2 = (x2 – yz) i + (y2 – xz) j + (z2 – xy) k then
 
show that F 1  F 2 is solenoidal.
Solution :
 
For Solenoidal vector field div F =   F = 0
  
let F = F 1  F 2 then
      
  F =   ( F 1  F 2) = F 2  (  F 1) – F 1  (  F 2)
 
= F 2  (0) – F 1  (0) = 0
  i j k 
Now,   F1 =  /x /y /z
 y + z z + x x + y
= i(1 – 1) – j (1 – 1) – k (1 – 1) = 0
  i j k 
  F2 =  /x /y /z 
 x2 – yz y2 – xz z2 – xy
= i(–x + x) – j (–y + y) + k (–z + z)
= 0
 
 F1  F2 is solenoidal field.

Example : 3
 xi + yj
Prove that the vector field F = x2 + y2 is solenoidal as well as irrotational.

Solution :

For solenoidal field   F = 0

 i  + j    xi2 + yj2  =   2 x 2  +   2 y 2 
 F =  x y   x + y  x  x + y  y  x + y 
 
 1 – 2x  1 2y
F = (1) + x  2 2 2
+ 2 
2 (1) + –

2y
x 2 + y2  (x + y )  (x + y )  (x + y2) 
2

1 2 2 2 2
= (x2 + y2)2 [2x + 2y – 2x – 2y ] = 0

 F is solenoidal field.

For irrotational vector field   F = 0
Gigatech Publishing House
Igniting Minds
Engineering Mathematics – III 4.29 Vector Differential Calculus

i j k
  /x /y /z 
 F =  x y 
x +y 2 2
x2 + y2 0 
 2xy 2xy 
F = i(0 – 0) – j(0 – 0) + k  – 2 2+ 2
 (x + y ) (x + y2) 
2 2


F = 0

 F is irrotational field.

Example : 4

Show that the vector field f(r) r is irrotational. Determine f(r) such that the field is
solenoidal.
Solution :

For irrotational field   F = 0
  
   f (r) r =  f (r)  r + f (r) (  r )
f (r)   
= ( r  r ) + f (r) (  r )
r
   
  f (r) r = 0 (... r  r = 0,   r = 0)

For solenoidal field   F = 0
  
   f (r) r =  f (r)  r + f (r) (  r )
f (r)     
0 = r r  r + f (r) (3) (  r = 3 ; r  r = r2)

f(r) 3
 r f(r) + 3 f(r) = 0  =–
f(r) r
On integration we get
log f(r) = – 3 logr + log c
c
 f(r) =
r3

Example : 5

If ,  satisfy Laplace equation then prove that the vector ( – ) is solenoidal.

Solution : For solenoidal field   F = 0

 F =   ( – )
=   () –   ()
=    + 2 –    – 2
= 0 (... 2 = 0, 2 = 0 Laplace equation)
Gigatech Publishing House
Igniting Minds
Engineering Mathematics – III 4.30 Vector Differential Calculus

Example : 6
 1    
Show that F = r [r2 a + ( a  r ) r ] is irrotational. Hence find  such F = .

Solution :

For irrotational field   F = 0
 
   a  r 
   F =   (r a) +     r
 r  
   
   ar  ar 
  F = r  a + r(  a) +    r +   (  r )
 r   r 
1   1 1
= r ( r  a) + 0 +  r  ( a  r ) + ( a  r )  r   r + 0
    

 
...   
1   1   (a  r )  
 
 a =0 
 

= r ( r  a) + r ( a  r ) – r3 ( r  r )  r =0
  
r  r =0 
   
a r a r
= – r + r –0

= 0

 F is irrotational field.

 To determine scalar potential function  :


 
    (a  r )   
F  d r =  ra + r   dr
 r 
 
  (a  r )  
= r (a  d r ) + r r  dr
 
    ar
F  dr = r ( a  d r ) + r r dr
 
= d [r ( a  r )]
 
  =  F  dr + C
   
=  d[r ( a  r )] + C (... r  d r = r dr)
 
 = r( a  r ) + C
     
 =  [r ( a  r ) + C] = r  ( a  r ) + ( a  r ) r + 0
 
ar 

 = ra + r r = F

 F = 
Gigatech Publishing House
Igniting Minds
Engineering Mathematics – III 4.31 Vector Differential Calculus

Self–Assessment Exercise : 4.1

Ex.1: A particle moves on the curve x = 2t2, y2 = t2 – 4t, z = 3t – 5, where t is time. Find the
  
components of velocity and acceleration at time t = 1 in the direction i – 3 j + 2 k .
 8 14 –2 
Ans. 7 
 14 
  
Ex.2: A particle moves along the curve r̄ = (t3 – 4t) i + (t2 + 4t) j + (8t2 – 3t3) k where t
denotes time. Find the magnitudes of acceleration along the tangent and normal at time t = 2.
[ Ans. 16, 2 73 ]
Ex.3: Find the angle between tangent to the curve x = t, y = t2, z = t3 at t = 1.
Ans. cos–1 3
 7
 
Ex.4: A particle moves so that its position vector is given be r̄ = cos wt i + sin wt j . Show
 
that the velocity V of the particle is perpendicular to r̄ and r̄ v is a constant vector.
Ex.5: Find the unit tangent vector at any point on the curve x = 3 cost, y = 3sint, z = 4t.

  1    
Ans. T = 5 (–3sint i + 3cost j + 4 k ) 
 
  dr̄ 
Ex.6: If r̄ = cosnt i + sin(nt) j where n is a constant and t varies, show that r̄  =nk.
dt
Ex.7: Find radial and transverse component of velocity and acceleration of a particle
describing with constant angular velocity w, the curve r = a(1 + sin). If the particle moves
d
with constant speed V show that dt  r –1/2.

Ex.8: If a particle P moves along the curve r = ae with constant angular velocity w, then
show that the radial and transverse component of its velocity are equal and acceleration is
always perpendicular to radius vector and is equal to 2 rw2.
Ex.9: The position of a particle in polars at time t is given by r = 2 + 3cost,  = sint. Find the
radial and transverse components of acceleration and speed at t = . [Ans. : 4, 0, 1]
Ex.10: A particle describe the straight line r = asec with constant angular velocity w. Find
the radial and transverse components of velocity and acceleration.
[Vr = aw sec tan, Vs = aw sec,
ar = 2aw2sec tan2, as = 2aw2sec tan]
Ex.11: If r̄ = ā sinht + b̄ cosht then prove that
d2 r̄ dr̄ d2 r̄ dr̄ d2 r̄
(i) dt2 = r̄ , (ii) dt  dt2 = constant, (iii) r̄  dt  dt2 = 0.

Gigatech Publishing House


Igniting Minds
Engineering Mathematics – III 4.32 Vector Differential Calculus

Self–Assessment Exercise : 4.2

1. Find  for
i)  = 3x2y – y3z2 at the point (1, 2, –1) Ans. : 12^i – 9^j – 16k^
ii)  = x3 + y3 + 3xyz = 3 at the point (1, 2, –1) Ans. : –3^i + 9^j + 6k^
iii)  = x2y + 2xz – 4 = 0 at the point (2, –2, 3) Ans. : –^i + 2^j + 2k^
2. Find unit vector normal to the surface.
–1 ^
xy3z2 = 4 at the point (–1, –1, 2) Ans. : i + 3^j – k^
11
3. Find angle between normal to the surfaces xy2z = 3xz2 and 3x2 – y2 + xz = 1 at the point
3
(1, –2, 1) Ans. : cos =
7 6
(Hint : Angle between two surfaces is same as angle between their normal)
4. The temperature at a point (x, y, z) in space is given by T(x, y, z) = x2 + y2 – z. A mosquito
located at (1, 1, 2) desire to fly in such direction that it will get warm as soon as possible in
what direction should it fly. Ans. : 2^i + 2^j – k^
(Hint : Along normal to the surface)
5. In what direction from (3, 1, –2) is the directional derivative of  = x2y2z4 maximum and
what is its magnitude. Ans. : 96^i + 288^j – 288k^ 960T9
6. Find directional derivative of  = xy2 + yz3 at the point (21 – 11 1) in the direction of
15
normal to the surface x log z – y2 + 4 = 0 at (1, 2, 1) Ans. :
17
7. Find angle between the tangent planes to the surfaces x log z = y2 – 1 and x2y = 2 – z at
1 
the point (1, 1, 1) Ans. : cos1 
 30 
(Hint : Angle between two plane is same as angle between their normal)
8. Find direction derivative of x2y + y3z at (21 – 11 1) along the direction which make equal
2
angle with co–ordinate axes. Ans. :
3
9. If direction derivative of  = a(x + y) + b(y + z) + c(x + z) has maximum magnitude 12
in the direction.
x1 y2 z1 24 12
Parallel to the line 1 = 2 = 3 Find a, b, c. Ans. : 0,  ,
14 14
10. Find direction derivative of the function F = x2 – y2 + 2z2 at the point P(1, 2, 3) in the
28
direction of the line PQ where Q is the point (5, 0, 4). Ans. :
21
11. Find the directional derivative of  = x2 + 2y2 – 3z2 at (1, 2, 1) in the direction tangent to
x = t2 + t, y = 2t, z = 2 – t at t = 1. Ans. : 2 14
Gigatech Publishing House
Igniting Minds
Engineering Mathematics – III 4.33 Vector Differential Calculus


12. Show that the vector field F = (x2 + xy2) ^i + 1y2 + x2y) ^j is irrotational. Hence find the
 x3 y3 x2y2
scalar potential such that F = . Ans. : = 3 + 3 + 2 + C

13. Prove that the vector F(r) r is irrotational.


 ^
14. Show that the vector field F = (sin y + z) + ^i (x cosy – z) ^j + (x – y) k is

irrotational.Find scalar  such that F = grad . Ans. : = x siny + xz – yz + C
 ^
15. Show that the vector field F = 2xyz3^i + x2z3^j + 3x2yz2k is irrotational. Find the scalar

potential . Such that F = grad. Ans. : = x2yz3 + C
 r
16. Show that F = r2 is irrotational.
 ^  
17. If F = (y + z) ^i + (z + z) ^j + (x + y) k then show that curl curl curl cur F = 4F

(Hint : First show that F = 0)
 ^
18. Show that the vector fieldF = (y2 cosx + z2) ^i + 2ysinx ^j + 2xzk is irrotational. Hence

find  such that F = . Ans. : = y2 sinx + z2x + C
 ^
19. Show that the vector V = (x + 3y) ^i + (y – 3z) ^j + (x – 2z) k is solenoidal.
 C
20. Define F(r) such that F = F(r) r is solenoidal. Ans. : F(r) = r3

21. Show that the vector field F = (x2 – y2 + x) ^i + (x + y) ^j is irrotational. Find a scalar 
 1 1
such that F = grad . Ans. : = 3 x3 – xy2 + 2 (x2 – y2) + C
^
22. If r = x^i + y^j + zk Prove that :
i)  (rnr ) = (n + 3)yn ii)  (rnr ) = 0
 r   r   2
iii)  r3  = 0 iv) 2   r2   = r4
    
1  3
v)  r r3   = r4

vi) 2(rn) = n (n + 1) rn–2
  
a is constant vector and r = x^i + y ^j + zk Prove that curl ( a r ) = 2
^
23. If  a
   
24. If uF = v where u, v are scalar field and F is a vector field show that F . curl F = 0.
 1
25. Show that F = r [r2a + (
a r )r ] is irrotational. Hence find scalar potential .

[ = r (a r ) + c]
 ^   
26. If F1 = yz^i + xz^j + xyk , F2 = (
a r )
a then show that F1 F2 is solenoidal.

Gigatech Publishing House


Igniting Minds
Engineering Mathematics – III 4.34 Vector Differential Calculus

 1
27. Prove that F = (x^i + y^j ) is solenoidal.
x + y2
2

28. Find the function F(r) so that F(r)r is solenoidal.


29. If ,  satisfy Laplace equation then prove that () is solenoidal.
30. Evaluate : (r3r )

Descriptive Questions
Q.1 Find the directional derivative of  = x2yz3 at (2, 1, –1) along the vector –4^i – 4^j + 12k^
.
Q.2 Find the directional derivative at xy2 + yz3 at 2, –1, 1 along the line 2(x – 2) = y + 1 = z
–1
Q.3 Find the direactional derivative of  = xy2 + yz3 at (1, –1, 1) along the direction normal
to the surface x2 + y2 + z2 = 9 at (1, 2, 2)
Q.4 Show that vector field

F = (x2 – yz)^i + (y2 – xz)^j + (x2 – xy)k^ is irrotatioal hence find scalar potential  such

that F = 
Q.5 Show that the vector field

F = (x + 2y + 4z)^i + (2x – 3y – z)^j + (4x – y + 2z)k^ is irrotational and hence find

scalar potential  such that F = 
Q.6 Show that vector field

F = (y2cosx + z2)^i + (2ysinx)^j + 2xzk^ is irrotational and find scalar potential  such

that F = 
Q.7 Prove the following :
6
i) 4(r2 logr) = 2
r
 
   1  3 ( ar br ) (a b)
ii) a  b  r =  r3
   r5
2
iii) 2F(r) = F(r) + r F(r)
1 3
iv)  r   r3   = r4
  
 
 b a 2 (a r ) 
v) b  ( a  logr) = r2 – r4 (b r )

6
vi) 4 (r2 log r) = r2

Gigatech Publishing House


Igniting Minds
Engineering Mathematics – III 4.35 Vector Differential Calculus

1 
a 3 (a r )
vii)   a  r  = r3 – r3 r
 

 r 2
viii) 2 r2  = r4
 
Q.8 Find the contant a & b so that the surface ax2 – byz = (a + z) x will be orthogonal to
surface 4x2y + z3 = 4 at the point (1, –1, 2)
Q.9 Show that the vector field F(r) r is always irrotational and determine F(r) such that the
field is solenoidal also.
Q.10 Find directional derivative of  = xy + yz2 at (1, –1, 1) towards the point (2, 1, 2)

Q.11 Show thatF = (2xz3 + 6y)^i + (6x – 2yz)^j + (3x2z2 – y2)k^ is irrotational. Find the scalar

potential  sucha that F = 

Q.12 Show that the vector field given by A = (x2 + xy2)^i + (y2 + x2y)^j is irrotational f find

scalar potential such that F = .
Q.13 Find the directional derivative of  = x2 – y2 + 2z2 at the point P(1, 2, 3) in the direction
of line PQ, where Q is (5, 0, 4)



Gigatech Publishing House


Igniting Minds
Unit
5 Vector Integral Calculus
and Applications

Syllabus :
Line, Surface and Volume integrals, Work–done, Green’s Lemma, Gauss’s Divergence theorem,
Stoke’s theorem. Applications to problems in Electro–magnetic fields.


The line integral of F along path ‘C’ is defined as

(  
)
integral of tangential component of F i.e. F T along C.

Thus line integral =  FT ds
C

Where T is a unit vector tangent to the curve and ds
  dr 
is an element of arc of C. i.e. T = 
 ds  Fig.: 5.1

   dr  
  FT ds =  F ds ds =  F d r
C C C

If F = F1 i + F2 j + F3 k

And r = ix + jy + kz

 dr = i dx + j dy + k dz
 
Now, F d r = (F1 i + F2 j + F3 k)  (i dx + j dy + k dz)
 
F d r = F1 dx + F2dy + F3dz

  F d r 
=  [F1 dx + F2dy + F3dz]
C C
 
Let F represents a force of particle moving along the curve ‘C’ then work done =  
F d r
C

Gigatech Publishing House


Igniting Minds
Engineering Mathematics – III 5.2 Vector Integral Calculus and Applications

Illustrative Examples

Example : 1

x dx + y dy 
Evaluate  (x 2
+ y )
t
2 3/2 Where ‘C’ is the curve whose vector equation is r = e cos t i + e
t
C
sin t j along the arc of ‘C’ from the point (1, 0) to the point (e2, 0).
Solution :
Given : x = etcos t, y = et sin t x2 + y2 = e 2t
x dx + y dy = e2tdt
2 2
x dx + y dy e2t 2
  2 2 3/2 =  (e2t)3/2
dt =  e–tdt = [– e–t]
C (x + y ) t=0 t=0 0
–2
= 1–e

Example : 2

Evaluate  [yz i + zx j + xy k] d r where C is an arc of the helix x = a cos t, y = a sin t, z



C
= kt as t varies from 0 to 2.
Solution :
 
Since F = yz i + zx j + xy k and d r = i dx + j dy + k dz.
 
 F d r = yz dx + zxdy + xydz = d (xyz)
 2
  F dr =  d (xyz) = [xyz]C = [a cos t  a sin t  kt]
C C 0
= 0 (. sin 2 = 0, sin 0 = 0)

Example : 3
 
Evaluate  F dr where F = 3x i + (2xz – y) j + zk from A (0, 0, 0) to B (2, 1, 3) along
C
the line joining these points.
Solution :
 
We have F d r = 3x dx + (2xz – y) dy + z dz
The equation of line AB is Fig.: 5.2
x–0 y–0 z–0 x y z
2–0 = 1–0=3–0  2 =1 =3 =t
 x = 2t dx = 2dt
y = t dy = dt
z = 3t dz = 3dt
Limits

Gigatech Publishing House


Igniting Minds
Engineering Mathematics – III 5.3 Vector Integral Calculus and Applications

A (0, 0, 0)  t = 0
B (2, 1, 3)  t = 1
 
F d r = 3 (2t) (2dt) + (2 (2t) (3t) – t) dt + (3t) (3dt)
= (12t + 12t2 – t + 9t) dt = (12t2 + 20t) dt
1 3 2
1
t t
 (12t + 20t) dt = 12 3 + 20 2  = 4 + 10

  F d r
= 2
C t=0 0

 F dr = 14.

Example : 4
 
Evaluate  F dr where F= 3x2 i + (2xz – y) j + zk along the path x2 = 4y, 3x3 = 8z
C
from x = 0 to x = 2.
 
Solution : Here F d r = 3x2 dx + (2xz – y) dy + z dz
x2 3 3 1 9 2
And y = 4 , z = 8 x then dy = 2 x dx, dz = 8 x dx
3 x2 x 3 9
3x2 dx + 2x 8 x3 – 4  2 dx + 8 x38 x2 dx
 
F d r =
   
3
3 x 27
3x2 + x5 – + x5 dx
=
 8 8 64 
3
= 51 x5 – x + 3x2 dx
64 8 
2 3 6 4
2
51 x 51 x 1 x
 64 x5 – 8 + 3x2  dx = 64 6 – 8 4 + x3

  F d r
=
C x=0 0
51 (2)6 1 (2)4 3
= (2)6 6 – 8 4 + (2) – 0
51 (2)4 3
= 6 – (2)5 + (2)
51 1 96
= 6 – 2 + 8 = 6 = 16

Example : 5

A vector field is given F = sin y i + x (1 + cos y) j.
Evaluate the line integral over a circular path given by
x2 + y2 = a2 , z = 0.
Solution :
  Fig.: 5.3
Here F d r = sin y dx + x (1 + cos y) dy
 
F d r = sin y dx + x dy + x cos y dy = d (x sin y) + x dy
Gigatech Publishing House
Igniting Minds
Engineering Mathematics – III 5.4 Vector Integral Calculus and Applications

The parametric co–ordinates of circle are


x = a cos, y = a sin  and  0, to 2
and dx = – a sin  d. dy = a cos d

  F dr =  d (x sin y) +  x dy = (x sin y)C +  x dy
C C C C
2 2
= [a cos  sin (a sin )] +  a cos .a cos d
0 0
2 2 2
a2
0 + 2  (1 + cos 2) d = 2  + 2 
a sin 2
=
0  
0
2
a 2
= 2 [2 – 0] =  a

  F dr = a2.

Example : 6

Find the work done F in moving the particle along the curve joining the points A (0,1,1)
 1

to B (1, 1, 1) and a = i + j + k where F =
r

[
r2 a + ( a  r ) r
  
]
Solution :

Here

F d r = [r a + (a  r ) r ] dr
 
 a  r   
r ( a  d r ) +  r ( r  d r ) (.r  dr = r dr)
  
F d r =
 
 
a r
r (a  d r ) + 
 
=  r dr
 r 
= [
d r (a  r )
 
]

 F dr
C
= 
C
[  
]
d r ( a  r ) = r ( a  r ).
 

 (1 1 1)
 Work done =  F dr = [(x2 + y2 + z2)1/2(x + y + z)](0 1 1)
C
= [(1 + 1 + 1)1/2 (1 + 1 + 1) – (0 + 1 + 1)1/2 (0 + 1 + 1)]
Work done = 3 3 –2 2.
 
[Note : a  r = (i + j + k)  (xi + yj + zk) = x + y + z
 
r d r = (xi + yj + zk)  (dx i + dy j + dz k)
= x dx + y dy + z dz
r2 = x2 + y2 + z2  r dr = x dx + y dy + z dz
 
 r d r = x dx + y dy + z dz = r dr]

Gigatech Publishing House


Igniting Minds
Engineering Mathematics – III 5.5 Vector Integral Calculus and Applications

Example : 7
 
Evaluate  F dr where F = x2 i + y2 j + z2 k and C is the curve r = t i + t2 j + t3 k from

C
t = 0 to t = 1.
Solution : Given x = t, y = t2, z = t3
and dx = dt, dy = 2t dt, dz = 3t2 dt
 
 F d r = x2 dx + y2dy + z2dz = t2dt + t4 (2t dt) + t6 (3t2) dt
 
F d r = (t2 + 2t5) + 3t8) dt.
1 3
1
t 2 3
 (t + 2t + 3t ) dt = 3 + 6 t6 + 9 t9

  F d r 
= 2 5 8
C t=0 0
 1 1 1 3
 F dr = 3 +3 +3 –0 =3 =1
C

 F dr = 1
C

Example : 8

Show that the vector field given by F = (y2cos x + z3) i + (2y sin x – 4) j + (3x z2 + 2) k
is irrotational. Find the work done in moving the object in this field from (0, 1,  1) to
 –1 2.
2 

Solution : For irrotational field  × F = 0
i j k

    
⸫ × F =  x y z 
y 2
cos x + z3 2y sin x – 4 3x z2 + 2 

× F = i (0 – 0) – j (3z2 – 3z2) + k (2y cos x – 2y cos x)
 
  × F = 0, F is irrotational field.
The scalar potential function  is

 =  F dr =  [(y2 cos x + z3) dx + (0 – 4) dy + (0 + 2) dz]
C C

 =

 F d r  2
= [y sin x + z 3
x – 4y + 2z]
( 
2
 –1 2)

C
(0 0 –1)



 F dr = 1.1 + (2)3 .  – 4 (–1) + 2 (2) – (0 + 0 – 0 – 2)
C  2  

 Work done =  F dr = 4 + 11
C

Gigatech Publishing House


Igniting Minds
Engineering Mathematics – III 5.6 Vector Integral Calculus and Applications

Example : 9

Find the work done by the force F = (x2 – yz) i + (y2 – xz) j + (z2 – xy) k in taking a
particle from (1, 1, 1) to (2, 2, 0).
Solution :

For irrotational field  × F = 0
i j k

    
×F =  x y z 
x 2
– yz y2– xz z2 – xy 
= i (– x + x) – j (–y + y) + k (–z + z) = 0
 
 ×F = 0, F is irrotational field.

Now  =  F dr =  [(x2 – yz) dx + y2 dy + z2 dz]
C C
3 3 3
= x – xyz + y + z 
3 3 3
C
3 3 3
(2 2 0)
x y z
 F d r =  3 + 3 + 3 – xyz
 
 Work done =
C
(1 1 1)
8 8 1 1 1 16 10
= 3 +3 +0–0–3 –3 –3 –1= 3 –2= 3 .

Example : 10

Find the work done in moving a particle in the field F = (x + y) i + (y + z) j + (z + x) k
along the curve x = 2 (t + sin t), y = 2 (1 – cos t) in xy – plane from t = –  to t = .
Solution :
 
We have F d r = (x + y) dx + (y + z) dy + (z + x) dz
In xy plane z = 0, dz = 0 then
 
F d r = (x + y) dx + y dy
Given x = 2 (t + sin t) , dx = 2 (1 + cos t) dt
And y = 2 (1 – cos t), dy = 2 sin t dt
 
 F d r = [2 (t + sin t) + 2 (1 – cos t)] 2 (1 + cos t) dt
+ 2 (1 – cos t) 2 sin t dt
 
F d r = 4 [(t + t cos t + sin t + sin t cos t + 1 + cos t – cos t – cos2 t)
+ sin t – sin t cos t]
= 4 [t + t cos t + 2 sin t – cos2 t + 1]

Gigatech Publishing House


Igniting Minds
Engineering Mathematics – III 5.7 Vector Integral Calculus and Applications

1 + cos 2t
 t + t cos t + 2 sin t + 1 –  2 dt

 F d r 
= 4
C t=–
2

t 1 sin 2t
= 4  2 + t sin t – (1) (– cos t) – 2 cos t + t – 2 t – 4 
 
–
2 2
= 4  2 – 2 +  sin  – (–) sin (–) + cos  – cos (–)

  1
– (cos  – cos (–)) + 4 + 4 – 4 (sin 2 – sin 2 (–))

2 1
= 4 0 + 0 – 0 – 1 + 1 – (– 1 + 1) + – (0 – 0)
 4 4 
 
 F dr = 4+
2
C

Example : 11

x2 y2
Find the work done in moving a particle once round the ellipse a2 + b2 = 1, z = 0 under

the field of force given by F= (2xy + 3z2) i + ( x3 + 4yz) j + (2y2 + 6xz)k.
Solution :
 
We have Fd r = (2xy + 3z2)dx + (x3 + 4yz) dy + (2y2+ 6xz) dz
Since z = 0 ,dz = 0
 
Fd r = 2xy dx + x3dy
The parametric equations of ellipse are x = a cos, y = b sin  and dx =  a sin  d,
dy = b cos d and  0, 2.
 
 Fd r = 2 abcos sin  ( a sin  d) + a3 cos3 b cos d
 
Fd r = ( 2 a2b cos sin2 + a3b cos4) d

2
 2
 Fdr =  2a2b  sin2 cos d + 4 a3b 4
 cos  d 
C 0
0
3 2
sin  3 1 
 2a2 b  3
=
 3  0 + 4 a b 4 2 2
3 a3b
= 0+
4

 Work done =  Fdr
C
3
=  a3b.
4

Gigatech Publishing House


Igniting Minds
Engineering Mathematics – III 5.8 Vector Integral Calculus and Applications

Example : 12

Evaluate 
C
( r  d r ) along the circle represented by x
  2 
+ y2 = a2, z = 0 and r = xi + yj + zk.

Solution :
 
 i j k
We have r dr =  x y z
 dx dy dz
Since z = 0 , dz = 0

 
 i j k
 r  dr =  x y 0 = k (x dy – y dx)
 dx dy 0
The parametric co–ordinates of circle are x = a cos , y a sin 
And dx = – a sin  d, dy = a cos  d ,   0, 2.
  
r  d r = k [a cos  · a cos  d – a sin  (– a sin d)]
   
r  d r = k (a2 (cos2  + sin2 ) d] = k a2 d
2
  2 
 ( 
r  dr

) = k  a2 d = k a2 () = 2 a2 k
C 0
=0

Example : 13
  x2 y2
Evaluate  F dr where F = ey i + x (1 + ey) dy and c is the curve of a2 + b2 = 1, z =0
C
Solution :
 
We have F· d r = ey dx + x (1 + ey) dy
 
 F· d r = d (x ey) + x dy

 F dr =  d (x ey) +  x dy = (x ey)C +  x dy
C C C C
The parametric co–ordinates of ellipse are
x = a cos , y = b sin  and  varies from 0 to 2
dx = – a sin  d, dy = b cos  d.
2
 2
  F d r 
= [a cos  e b sin 
] +  a cos  · b cos  d/
C 0 0
2 2
= 0 + ab 
1 + cos 2 d = ab  + sin 2
0
 2  2  2 
0
 ab
 F d r 
=
2
[2 – 0] = ab.
C

Gigatech Publishing House


Igniting Minds
Engineering Mathematics – III 5.9 Vector Integral Calculus and Applications

Example : 14

Find the work done in moving a particle along the curve r = a cos  i + a sin  j + b k
 
from  = to  = under the field of force given by
4 2

F = (–3 a sin2  cos ) i + a (2 sin  – 3 sin3 ) j + b sin 2 k.

Solution : We know that work done =  
F d r
C
 
 F d r = [(– 3a sin2  cos ) i + a (2 sin  – 3 sin3 ) j + b sin 2 k]
· [– a sin  i + a cos  j + bk] d
 
F d r = (3a2 sin3  cos  + a2 2 sin  cos  – 3a2 sin3  cos  + b2 sin 2) d
 
F d r = (a2 sin2 + b2 sin2) d = (a2 + b2) sin2 d
 
2
– cos 2 2
F d r = (a2 + b2)  sin 2 d = (a2 + b2)  2 

 
 
C 
4 4

a 2 + b2 1 2 2
= 2 [1 + 0] = 2 (a + b )
 1
 work done =  F dr = 2 (a2 + b2)
C

Let C be a closed curve bounding the region R in the xy – plane. Let M (x, y) and
N M
N (x, y) be two continuous functions of x and y and let and are continuous in R, then
x y

 N – M dxdy.


(Mdx + Ndy) =   
C  x y 
R

Illustrative Examples

Example : 1
 
Using Green’s theorem evaluate  F dr where F = sinz i + cosx j + sin y k and C is
C
the boundary of the rectangle 0  x  , 0  y  1 and z = 3.
Solution :
 
Here F d r = sin z dx + cos x dy + sin y dz

Gigatech Publishing House


Igniting Minds
Engineering Mathematics – III 5.10 Vector Integral Calculus and Applications

Since z = 3 then dz = 0
 
 F d r = sin 3 dx + cos x dy
f1 f2
F1 = sin 3, F2 = cos x , =0, = – sin x.
y x
By Green’s theorem we have.

 F1 dx + F2 dy =  f2 – f1 dx dy
C R  x y 
1   1
 F1 dx + F2 dy =   (– sin x – 0) dx dy = [cos x] [y]
C y=0 x=0 0 0

= (– 1 – 1) (1 – 0) = –2

Example : 2

Evaluate  e–x (siny dx + cosy dy) by using


C
Green’s theorem. Where ‘C’ is the rectangle with vertices
 
(0, 0), (, 0),    , 0  .
 2  2 
Solution : Fig.: 5.4
–x –x
Here M = e sin y and N = e cos y
M N
= e–x cosy = – e–x cos y
y x

By Green’s theorem  M dx + N dy =  N – M dx dy.


C R  x y 


2
 e –x
(siny dx + cos y dy) =   (– e–x cos y – e–x cos y) dx dy
C x=0
y=0


2 –x  /2
= –2  –x e 
x=0
 e cos y dx dy = – 2  –10 [sin y]0
y=0

= 2 (e – 1) (1 – 0) = 2 (e– – 1).
–

Example : 3

Use Green’s theorem in plane to evaluate the integral


 [(2x2 – y2) dx + (x2 + y2) dy] where ‘C’ is the boundary in
C
the xy – plane of the area enclosed by the x –axis and semi
circle x2 + y2 = 1. In the upper half of xy – plane.
Fig.: 5.5

Gigatech Publishing House


Igniting Minds
Engineering Mathematics – III 5.11 Vector Integral Calculus and Applications

Solution :
By Green’s theorem, we have

N M
 M dx + N dy =   x –
y 
dx dy
C
R
Here M = 2x2 – y2 , N = x2 + y2
M N
= – 2y = 2x
y x
N M
 – = 2 (x + y)
x y
1 2
1–x
  2 2 2 2
[(2x – y ) dx + (x + y ) dy] =   2 (x + y) dx dy
C x = –1 y=0
2
1 2 1–x
y
= 2  xy + 2  dx
x = –1
 
0
1
 1 
= 2  x 1 – x2 + 2 (1 – x2) dx

x = –1 
1
1 1 x3
= 2 – (1 – x2)3/2 + x – 
 3 2 3 
–1
1 1 1 1 2 2 4
= 2 0 + 2 1 – 3 – – 1 + 3 = 2   +  =
     2 3 3 3
Example : 4

Apply Green’s theorem to evaluate  (3y dx + 2x dy)


C
where C is boundary 0  x  . 0  y  sin x.
Solution :
Fig.: 5.6
Here M = 3y, N = 2x,
M N
= 3, = 2
y x
N M
⸫ – = 2 – 3 = –1
x y
By Greens’s theorem we have
 sin x
 (3y dx + 2x dy) =   (–1) dx dy
C x=0 y=0
 sin x
= (–1)  (y)
0
dx
x=0
Gigatech Publishing House
Igniting Minds
Engineering Mathematics – III 5.12 Vector Integral Calculus and Applications
 
= (–1)  sinx dx = (–1) (– cos x)
0
x=0
= (–1) (1 + 1) = – 2

Example : 5

Using Green’s lemma evaluate the line integral  (cos x sin y – 4y) dx + sin x cos y dy
C
where C is the circle x2 + y2 = 1.
Solution : By Green’s theorem we know that
N M
 M dx + N dy =   x –
y 
dx dy
C R
Here M = cos x sin y – 4 y and N = sin x cos y
M N
= cos x cos y – 4 = cos x cos y
y x
N M
 – = cos x cos y – cos x cos y + 4 = 4
x y

 [(cos x sin y– 4y) dx + sin x cos y dy] =  4 dx dy.


C R
2
= 4 [Area of circle (r ) ] = 4 ( (1)2 ) = 4.

Example : 6

Using Green’s theorem evaluate  (x2 y dx + x2 dy) where C is the boundary described
C
counter clockwise of the triangle with vertices (0, 0), (1, 0) and (1, 1).
Solution : By Green’s theorem we have
N M
 M dx + N dy =   x –
y 
dx dy
C R
2
Here M = x y , N = x2
M N
And = x2 , = 2x
y x
N M
 – = 2x – x2
x y
1 x Fig.: 5.7
 (x2 y dx + x2) dy =   (2x – x2) dx dy
C x=0 y=0
1 x
=  (2x – x2) (y) dx
0
x=0

Gigatech Publishing House


Igniting Minds
Engineering Mathematics – III 5.13 Vector Integral Calculus and Applications
1 4
1
2 x
=  (2x – x ) dx = 3 x3 – 4 
2 3

x=0 0
2 1 5
= – –0=
3 4 12

Example : 7
 
Using Green’s theorem evaluate  F dr where F (2x2 y + 3z2) i + (x2 + 4yz) j + (2y2 + 6xz) k
C
and C is the curve enclosing a region y2 = 4 ax, x = a in the plane z = 0.
 
Solution : We have F d r = (2x2y + 3z2) dx + (x2 + 4yz) dy + (2y2 + 6xz) dz
Since z = 0, dz = 0
 
F d r = 2x2y dx + x2 dy
Here M = 2x2y , N = x2,
M N
= 2x2 , = 2x
y x
N M Fig.: 5.8
 – = 2x – 2x2
x y

N M
 By Green’s theorem  M dx + N dy =   x –y 
dx dy
C
R
a 2 ax a
ax
 2 2
(2x y dx + x dy) = 2   2
(2x – 2x ) dx dy = 4  (x – x2) (y) dx
C x=0 y=0 x=0 0

a
= 4  (x – x2) ( a x – 0) dx
x=0
a
= 4 a  (x3/2 – x5/2) dx
x=0
a
5 7
x2  x2
= 4 a 5 – 7
 2 2 0
 52 7
2 
a a
= 8 a 5 – 7 – 0

 
5
1 a 1 a 8 8
= 8a3 a2  5 – 7 = 8a3  5 – 7 = a3 – a4
    5 7

Gigatech Publishing House


Igniting Minds
Engineering Mathematics – III 5.14 Vector Integral Calculus and Applications

Example : 8
 
Using Green’s theorem evaluate  
F d r where F = cos y i + x (1 – sin y) j where C is
C
x2 y2
a2 + b2 = 1, z = 0.
f f
Solution : By Green’s theorem, we have  F1 dx + F2 dy =   x2 – y1 dx dy
C R
Here F1 = cos y , F2 = x (1 – sin y)
f1 f2
= – sin y = 1 – sin y
y x
f2 f1
 – = 1 – sin y + sin y = 1
x y

  cos y dx + x (1 – sin y) dy =  (1) dx dy = Area of Ellipse =  ab.


C
R


The surface integral of a vector point function F
over a surface S is defined as the integral of the normal

component of F throughout an element of area ds.

If n is a unit vector normal to ds the normal
  
component of F is given by F n and hence the Fig.: 5.9
surface integral can be expressed as
  
 F n ds =  F ds
s s
   
where ds = n ds and n =
||

Let n = cos  i + cos  j + cos  k
Now dx dy = projection of ds on the xy – plane is = ds cos  .

|
 
Also cos  = n  k |
dx dy dx dy
 ds = = Fig.: 5.10
cos   
kn | |
Similarly dy dz = projection of ds on the yz – plane is = ds cos 

Gigatech Publishing House


Igniting Minds
Engineering Mathematics – III 5.15 Vector Integral Calculus and Applications

dy dz dy dz
 ds = =
cos   
i n | |
dz dx = projection of ds on the zx – plane is = ds cos .
dz dx dz dx
 ds = =
cos   
j n | |
Illustrative Examples

Example : 1

  
Evaluate  F d s where F = (x + y2) i + 2x j + 2yz k and S is the surface of the plane
s
2x + y + 2z = 6 in the positive octant.
Solution : We have  = 2x + y + 2z – 6 = 0 then  = 2i + j + 2k.
  2i + j + 2k
 n = =
|| 3
dx dy dx dy
and ds = =
 
kn| | 2
3
 2i + j + 2k dx dy
(x + y2) i + 2x j + 2yz k). 

 F n ds =
 3  2
3
2 2
 (x + y2) + x + yz4 dx dy
=
3 3 3  2
3
2 dx dy
= [2x + y2 + 2yz] = [2x + y2 + y) (6 – 2x – y)] dx dy
3 2
3
 
F n ds = (2x + y2 + 6y – 2xy – y2) dx dy
= 2 (x + 3y – xy) dx dy
3 6 – 2x
 
  F n ds = 2   [x + (3 – x) y] dx dy
s x=0 y=0
Fig.: 5.11
3 6 – 2x
 y 2
 xy + (3 – x) 2 

 F n ds = 2 dx
s x=0 y=0

Gigatech Publishing House


Igniting Minds
Engineering Mathematics – III 5.16 Vector Integral Calculus and Applications
3 2
(6 – 2x)
= 2  x (6 – 2x) + (3 – x) 2 – 0 dx
x=0
3
= 4  [x (3 – x) + (3 – x)3] dx
x=0
2 3 4
3
x x (3 – x)
= 4 3 2 – 3 – 4 
 0
3 27 (3)4
= 4 2 9 – 3 – 0 – 0 – 4 
   
1 1 81
= 4 27 2 – 3 + 4 
   
27 81 9 81 1 9
= 4 +  =4 +  =49 + 
6 4 2 4  2 4
11
= 36  = 99
4

Example : 2

  
Evaluate  F d s where F = zi + xj – 3y2zk and S is the surface of the cylinder
s
2 2
x + y = 16 included in the first octant between z = 0 and z = 5.
Solution :
  2 (xi + yj)
Let  = x2 + y2 – 16 = 0 ,  = 2 (xi + yj), n = =
|| 2 x2 + y2
 xi + yj 1 dy dz dy dz
 n = = 4 (xi + yj) ; ds = = x
16
|
 
i n 4 |
  1 dy dz
 F n ds = (zi + xj – 3y2zk) · 4 (xi + yj) x
4
  xz + xy dy dz = (y+ z) dy dz
F n ds =
4 4 x
4
5 4 5 4 5
 y 16 2
  (y + z) dy dz =   2 + yz dz =   2 + 4z dz

 F n ds =
s z=0 y=0 z=0 0 z=0
5
  2
8z + 4 z  = [40 + 2  25 – 0] = 90
 F n ds =
 2
s 0

Gigatech Publishing House


Igniting Minds
Engineering Mathematics – III 5.17 Vector Integral Calculus and Applications

Example : 3

  
Evaluate  F n ds, where F = yzi + zxj + xyk and S is
s
that part of the surface of the sphere x2 + y2 + z2 = 1. Which
lie in the first octant.
Solution : Fig.: 5.12
Here  = x2 + y2 + z2 – 1 = 0,
 = 2 (xi + yj + zk)
  2 (xi + yj + zk)
n = = = xi + yj + zk.
|| 2 x2 + y2 + z2
dx dy dx dy
ds = = z
|
 
kn |
  dx dy
 F n ds = (yzi +zxj + xyk) · (xi + yj + zk) z
dx dy
= (xyz + xyz + xyz) z
dx dy
= 3xyz = 3xy dxdy
z
1 1 – x2
  

s
F n ds = 3  xy dx dy
x=0 y=0
2
1 2 1– x
y
= 3  x 2 dx
x=0
 
0 Fig.: 5.13
1
3
= 2  x (1 – x2) dx
x=0
1
3 x2 x4 3 1 1 3 1 3
= – = – = · =
2  2 4  2 2 4 2 4 8
0



Let S be an open surface bounded by a simple closed curve C and F = F1 i + F2 j + F3 k
be a vector point function having continuous first order partial derivatives then the vector form
of stokes theorem is
    
 F dr =   F n ds =    F d s
C
s s

Gigatech Publishing House


Igniting Minds
Engineering Mathematics – III 5.18 Vector Integral Calculus and Applications

The Cartesian form of Stokes theorem is

F F2 F1 F3 F2 F1


 [F1 dx + F2 dy + F3 dz] =   y3 – z 
dydz +  –
 z x 
dzdx +  –
 x y 
dxdy

C
s
Cor: Let F3 = 0 then we get special case of Stokes theorem.
i.e. Green’s theorem

f f
 F1 dx + F2 dy + =   x2 – y1 dx dy
C
R
Note : Fig.: 5.14

(i) If the surface is closed the bounding curve does not exist.

 
    F n ds = 0
s

     
(ii) If curl F =   F = 0 i.e. F is irrotational field then  F dr =    F n ds = 0
C
s
(iii) If two surfaces s1 and s2 have the same bounding curve C, then
     
   F n ds =    F n ds since each equals  F d r .
C
s1 s2

Illustrative Examples

Example : 1

State vector form the Stokes theorem and verify it for the vector function F = (x2 – y2) i
+ 2xyj in the rectangular region in the XOY plane bounded by the lines x = 0, x = a, y = 0, y =
b.
Solution : The vector form of Stokes theorem is
   
 F dr =   F n ds
C
s
i j k

    
Now,  F =  x y z 
x –y 2 2
2xy 0 

= i (0 – 0) – j (0 – 0) + k (2y + 2y) = 4y k

Gigatech Publishing House


Igniting Minds
Engineering Mathematics – III 5.19 Vector Integral Calculus and Applications

  dx dy
Here n = k and ds = = dx dy
 
kn | |
   
   F n ds = 4y k · k dx dy = 4y dxdy
a b b
  a 2
y
  4y dxdy = 4 [x]0  2  = 2ab2

  F n ds = …(1)
s x=0 y=0 0
 
Let F d r = (x2 – y2) dx + 2xy dy
(i) The line integral along the path C1 :
Here y = 0, dy = 0, x = 0, a then
 
F d r = x2 dx
a 3
a 3
x a
 x dx =  3  = 3 ,
 
 F d r = 2
C1 x=0 0 Fig.: 5.15

(ii) The line integral along the path C2 :


 
Here x = a, dx = 0, y = 0, b then F d r = 2xy dy = 2ay dy
b 2
b
y
 2ay dy = 2a  2  = ab2

  
F d r =
C2 y=0 0
(iii) The line integral along the path C3 :
 
Here y = b, dy = 0, x = a, 0 then F d r = (x2 – y2) dx = (x2 – b2) dx
0 0
x3
(x2 – b2) dx =  – b2 x

  
F d r =  3 
C3 a a
a3 a3
= 0 – 3 + ab2 = ab2 – 3 .

(iv) The line integral along the path C4 :


 
Here x = 0, dx = 0, y = b, 0 then F d r = 2xy dy = 0

  
F d r = 0
C4
    
 F dr =  F d r +

 F d r +

 F d r +

 
F d r
C C1 C2 C3 C4

 a3 a3
 F dr = 3 + ab2
+ ab2
– 3 + 0 = 2ab
2
…(2)
C

  
From (1) and (2) we get  F dr =    F n ds = 2ab2.
C
s
Hence the Stokes theorem is verified.

Gigatech Publishing House


Igniting Minds
Engineering Mathematics – III 5.20 Vector Integral Calculus and Applications

Example : 2

  
Evaluate   F n ds where F = xzi – yj + x2yk and S is the surface of the region
s
bounded by x = 0, y = 0, z = 0, 2x + y + 2z = 8 which is not included in the xz – plane.
i j k

    
Solution: we have   F =  x y z 
 xz –y x2y 

 F = x2 i + x (1 – 2y) j
Given  = 2x + y + 2z – 8 = 0
  2i + j + 2k
n = =
|| 4+1+4
1
= (2i + j + 2k) Fig.: 5.16
3
dx dz dx dz
ds = = 1 = 3 dxdz
|  
j n | 3
  1
   F n ds = [x2 i + x (1 – 2y)j] · (2i + j + 2k) 3 dx dz
3
= [2x2 – x (1 – 2y)] dx dz
4 4–x
 
   F n ds =   [2x2 + x (1 – 2y)] dxdz
s x=0 y=0
4 4–x
=   [2x2 + x [1 – 2 (8 – 2x – 2z)] dxdz
x=0 z=0
4 4–x
=   (6x2 – 15 x + 4xz) dxdz
x=0 z=0
4 2
4–x
z
=  6x2 z – 15xz + 4x 2  dx
x=0 z=0
4
=  (23x2 – 4x3 – 28x) dx
0
4
3 4 2
x x x
= 23 3 – 4 4 – 28 2 
 
0
(4)3 28 32
= (23) 3 – (–4)4 – 2 (4)2 – 0 = .
3
Gigatech Publishing House
Igniting Minds
Engineering Mathematics – III 5.21 Vector Integral Calculus and Applications

Example : 3


(  F ) n
 
Evaluate ds where F = (x3 – y3) i – xyzj + y3 k and S is the surface x2 +
s
4y2 + z2 – 2x = 4 above the plane x = 0.
i j k

    
Solution : we have   F = x y z 

x3 – y3 – xyz y3 
= i (3y2 + xy) – j (0 – 0) + k (–yz + 3y2)
  dx dy
And n = k , ds = = dzdy
 
kn | |
  
   F n ds = [i (3y2 + xy) + k (– yz + 3y2)] · k dxdy
 
  F n ds = (–yz + 3y2) dzdy
Since x = 0 then 4y2 + z2 = 4
y2 z2
1 +4 = 1 (Ellipse)

Put y = r cos  , r  0, 1
z = 2r sin  ,   0, 2
dydz = 2r drd
 
   F n ds =  (–yz + 3y2) dy dz
s s
2 1
=   (– r cos  · 2r sin  + 3r2 cos2 ) 2r dr d
=0 r=0
2 1
= 2   (– 2r3 cos  sin  + 3r3 cos2 ) dr d
=0 r=0
2 4
1
r
= 2  (– 2 cos  sin  + 3 cos )  4  d
2

=0
 0

 2 
1 sin2  2
= 2 · 4 – 2  2  + 3·4 cos2  d
    
 0
0 
1  1 
0 + 3·4 · ·  =
3
=
2  2 2 2

Gigatech Publishing House


Igniting Minds
Engineering Mathematics – III 5.22 Vector Integral Calculus and Applications

Example : 4
 
State Stokes theorem and use it to evaluate  = F d r where F = – 2y i+ xj + zk where

C
C is the loop of the curve x3 + y3 = 3axy.
  
Solution : The Stokes theorem in vector form is  F dr =    F n ds
C
s
ij k

    
We have  F =  x y z 
– 2y x z 
= i (0 – 0) – j (0 – 0) + k (1 + 2)
= 3k
  dx dy
And n = k , ds = = dxdy
 
kn | |
 
  F n ds = 3 dx dy
 
   F n ds =  3 dx dy
s s

= 3  3a2 du dv
s
 (x y)
 dxdy = du dv = 3a2 du dv
(u v)
1 1–u
 
   F n ds = 9 a2   du dv
s u=0 v=0
1
= 9a 2
 (1 – u) du
u=0
u2 9a2
= 9 a2  u – 2  = 2
 
  9 2
   F n ds = 2 a.
s
By using Jacobians we write
x2 y2
+ = 1u+v=1
3ay 3ax

Gigatech Publishing House


Igniting Minds
Engineering Mathematics – III 5.23 Vector Integral Calculus and Applications
2 2
x y
u = 3ay v = 3ax
2x –x2
 (u v)  3ay 3ay2  1
(x y)
=  y2 2y  = 3a 2

 –
3ax2 3ax 
 (x y)
= 3a2
(u v)

Example : 5

 
Use Stokes theorem to evaluate    F n ds over the unclosed surface of the cylinder
s
x2 y2 
2 + 2 = 1, bounded by the plane z = 5 and open at the end z = 0 for F = (x – y – z) i + (y–z–x)
a b
j + (z – x – y) k
i j k

    
Solution : We have   F =  x y z 

x – y– z y – z – x z – x – y 

 F = i (– 1 + 1) – j (–1 + 1) + k (–1 + 1) = 0
 
    F n ds = 0
Fig.: 5.17
s

Example : 6

Apply Stokes theorem to evaluate  (4ydx + 2z dy + 6x dz) where C is the curve of


C
intersection x2 + y2 + z2 = 6z , z = x + 5
Solution :

Given : F = 4y i + 2z j + 6x k then
i j k

    
 F =  x y z 
 4y 2z 6x 
= i (0 – 2) – j (6 – 0) + k (0 – 4)
= – (2i + 6j + 4k)
and  = x–z+5=0
 = i–k

Gigatech Publishing House


Igniting Minds
Engineering Mathematics – III 5.24 Vector Integral Calculus and Applications

  i–k
 n = =
|| 2
  1 –1 2
   F n = – (2i + 6j + 4k) · (i – k) = (2 – 4) = = 2
2 2 2

   
Using Strokes theorem we get  F d r =    F  n ds
C
s

 
   F n ds =  2 = 2 (Area of the circle (r2))
s s
The equation of sphere is x + y + z2 – 6z + 9 – 9 = 0
2 2

 (x – 0)2 + (y – 0)2 + (z – 3)2 = 9


C (0, 0, 3) and R = 3
0–3+5  2
Perpendicular distance p =   (1)2 + (–1)2  = 2 = 2
 
By Pythagoras theorem we have R2 = r2 + p2
Fig.: 5.18
 r2 = R2 – p2 = 9 – 2 = 7
Area of circle = r2 = 7
 
   F n ds = 2 (7) = 7 2
s

Example : 7

Apply Stokes theorem to prove that  (yi + zj + xk) · d r = – 2 2  a2 where C is the



C
curve x2 + y2 + z2 – 2ax – 2ay = 0, x + y = 2a.
Solution :

Given : F = yi + zj + xk then
i j k

    
 F =  x y z 
 y z x 
= i (0 – 1) – j (1 – 0) + k (0 – 1)
= – (i + j + k)
Here  = x + y – 2a = 0
Fig.: 5.19
  i+j
n = =
|| 2
  (i + j) 1+1
   F n = – (i + j + k) · =– =– 2.
2 2
Gigatech Publishing House
Igniting Minds
Engineering Mathematics – III 5.25 Vector Integral Calculus and Applications

  
By using stokes theorem we have  
F d r =    F n ds
C S

 
    F n ds =  – 2 dx = – 2 (Area of the circle r2)
S S

The equation of sphere is x2 + y2 + z2 – 2 ax – 2ay = c


 (x – a)2 + (y – a)2 + (z – 0)2 = 2a2
c (a, a, 0) r = 2 a.
In great circle the radius of the sphere is equal to the radius of the circle .
 Area of circle = r2 = 2 a2.
 
    F n ds = – 2 (2 a2)
S

= – 2 2  a 2.

Example : 8

  
Evaluate    F n ds where F = (x – y) i + (x2 + yz) j – 3xy2 k and S is surface of
S
cone z = 4 – x2 +y2 above XOY plane.
Solution :
i j k
     
We have  F =
x  y z 

x – y x2 + yz – 3xy2 
= – (6x + 1) yi + 3y2j + (2x + 1) k
 
Here n = k and ds = dx dy
 
    F n ds =  (2x + 1) dx dy
S S

In xy plane z = 0

2 4 2 2
4
2 r
  (2r cos  + 1) r dr d =  3 r3 cos + 2  d
=0 r=0 =0 r=0
2 3 2 (4)2 2
= 3 (4) [sin ] + 2 []0
0
(4)2
= 0 + 2 (2 – 0)

= 16

Gigatech Publishing House


Igniting Minds
Engineering Mathematics – III 5.26 Vector Integral Calculus and Applications

 
    F n ds = 16
S

 x2 + y2 = 16
Put x = r cos , y = r sin 
dxdy = r dr d
r = 0, 4 ,  = 0, 2

Example : 9

By using Stokes theorem show that  (y dx + z dy + x dz) = –  (cos  + cos  + cos


C S
) ds where s is the open surface bounded by the curve C and , ,  are the angle made by the
normal to the surface S with x, y, z –axes respectively.
 
Solution : Given F = yi + zj + xk and n = cos i + cos j + cos k
i j k
    
Then  F = 
x y z 
y 
z x 
= i (0 – 1) – j (1 – 0) + k (0 – 1)
= – (i + j + k)
 
   F n = – (i + j + k) · (cos i + cos j + cos k)
= – (cos  + cos  + cos )
By using Stokes theorem we get
  
 F d r

=    F n ds
C S

 (y dx + z dy + x dz) = –  (cos  + cos  + cos ) ds


C

Example : 10
 
State Stokes theorem and use to it evaluate  
F d r where F = zi + xj – 3y2 zk over
C
the surface of the cylinder x2 + y2 = 16 and the plane z = 5 and open at z = 0.
Solution : The statement of the Stokes theorem is
  
 
F d r =    F n ds
C
s
 
We have F d r = z dx + x dy – 3y2z dz
Since z = 0, dz = 0
Gigatech Publishing House
Igniting Minds
Engineering Mathematics – III 5.27 Vector Integral Calculus and Applications

x2 + y2 = 16
put x = 4 cos , y = 4 sin  , dy = 4 cos  d,   0, 2
 
F d r = x dy
2

  
F d r =  x dy =  4 cos  cos  d
C C =0


2
1 + cos2  1 2 1  sin 22
= 16  d = 16 2 () + 2  + 2
0
 2   0   
0
1
= 16 2 (2 – 0) + 0 = 16

Example : 11

Show that  [u  (r  v )] . dr = – (u  v


 
) ·  
ds
C S

Solution :

Given F = [u  (r  v )] = (u · v ) r – (u · r ) v

  [( u · v ) r ] –   [( u · r ) v ]
     
  F =
=  ( u · v )  r + ( u · v )( · r ) –  (u · r )  v – (u · r ) ( · v)
     

 
·.·  u · v = 0
= 0 + 0 – (u  v ) – 0
     r = 0 
 
   v = 0 
= – (u  v )
 

By Stokes theorem we have


 
 F dr =    F  ds

C S


  [
u  ( r  v )] · d r
  
= – ( u  v ) ·  d s
 
C
s

Example : 12

    
Prove that  ( a  r )  d r = 2 a   d s
C S
  
Solution : We have F = a  r then
  
 F =   [a  r ]

Gigatech Publishing House


Igniting Minds
Engineering Mathematics – III 5.28 Vector Integral Calculus and Applications
       
= (  r ) a – (  a ) r + ( r  ) a – ( a  ) r
  
= 3a – 0 + 0 – a = 2a
Using Stoke’s theorem we get,
   
 F  dr =    F  ds
C S

    
  (a  r )  dr = 2a   d s
C S

Example : 13


Evaluate    F  n^ ds where S is the curved surface of the paraboloid x2 + y2 = 2z
S

bounded by the plane z = 2 where F = 3(x – y) i + 2xz j + xy k.
Solution :

  i j

k

 = i (x – 2x) – j(y – 0) + k(2z + 3)
We have F =  x 
y z here n^ = k^  ds = dxdy
3(x – y) 2xz xy 

   F  n^ ds = (2z + 3) dxdy = 7dxdy since z = 2

    F  n^ ds =  7dxdy = 7(Area of circle r2)
S S

= 7 (4)
= 28.


If F is a vector point function finite and differentiable in a region R bounded by a

closed surface S, then the surface integral of the normal component of F taken over S is equal

to the integral of divergence of F taken over V the volume of the surface enclosed.
 
The vector form of Gauss divergence theorem is  F  n^ ds =    F dv.
S V

The cartesian form of Gauss divergence theorem is


F1 F2 F3 
 (F1dydz + F2dzdx + F3dxdy) =   + +
x y z 
 dxdydz.
S V

Gigatech Publishing House


Igniting Minds
Engineering Mathematics – III 5.29 Vector Integral Calculus and Applications

Illustrative Examples

Example : 1

Verify Gauss divergence theorem for F = (x2 – yz) i + (y2 – xz)i + (z2 – xy) k taken over
the rectangular parallelepiped 0  x  a, 0  y  b, 0  z  c.
Solution :
Consider it as rectangular parallelepiped whose edges are OB = a, OC = b, OF = c

Now, F = (x2 – yz) i + (y2 – xz) j + (z2 – xy) k

 F = 2x + 2y + 2z = 2(x + y + z)
c b a

    F dv = 2    (x + y + z) dxdydz
V z=0y=0 x=0
c b 2 c
z
= 2    xz + yz + 2  dxdy
z=0y=0 0
a b 2
c
= 2    cx + cy + 2  dxdy
x=0y=0
a b
2 2
y c
= 2   cxy + c + y  dx
x=0
 2 2 
0
a Fig.: 5.20
b2 bc2 
= 2   bcx + 2
c+ y dx
2 
x=0

x2 b2c bc2  a
= 2  bc + + x + x
 2 2 2 0
a2 ab2c abc2 
= 2 bc + +
2 2 2 
abc
= 2 (a + b + c) = abc (a + b + c) .... (1)
2

Now we shall calculate  F  n^ ds over six faces of the rectangular parallelepiped separately
S
and then add them.

i) For the face ABGD we have n^ = i, x = a ds = dydz, F  n^ ds = (x2 – yz) dydz
c b c b

 F  n^ ds =   (x2 – yz) dydz =   (a2 – yz) dydz
S1 z=0 y=0 z=0y=0
c 2 b c 2
y b
=   a2y – 2 z 0 dz =   a2b – 2 z  dz
z=0 z=0
Gigatech Publishing House
Igniting Minds
Engineering Mathematics – III 5.30 Vector Integral Calculus and Applications
2 2 c 2 2
=  a2bz – b  z  = a2bc – b c
 2 2 0 4
ii) For the face OCEF we have n^ = – ^i , x = 0, ds = dydz

F  n^ ds = (x2 – yz) dydz = + yz dy dz
c b 2
b 2
c 2 2
 y z bc
 F  n^ ds =   (+ yz) dydz =  2   2  = 4
S2 z=0y=0 0 0

iii) For the face ACED we have n^ = ^j ds = dx dz, y = b



F  n^ ds = (y2 – xz) dxdz = (b2 – xz) dxdz
a c a 2 c
 z
 F  n^ ds =   (b2 – xz) dxdz =   b2z – x 2 0 dx
S3 x=0 z=0 x=0
a 2
c
=   b2c – 2 x  dx
0
2 2 a 2 2
=  b2cx – c x  = ab2c – a c
 2 2 0 4
iv) For the face OBGF we have ^n = – ^j ds = dxdz, y = 0

 F  n^ ds = –(y2 – xz) dx dz = xz dx dz
a c
 x2 a  z2 c a2c2
 F  n^ ds =   xz dx dz = 
S4 x=0 z=0
 2 0  2 0 = 4

v) For the face DEFG we have n^ = k^ ds = dxdy z = c



F  n^ ds = (z2 – xy) dxdy = (c2 – xy) dxdy
a b a 2 b
 y
 F  n^ ds =   (c – xy) dxdy =  c2y – x  dx
2

S5 x=0 y=0 x=0


2  0
a
b 2
b 2
x2 a a2b2
=  c2b – 2 x  dx = c2bx – 2 2 0
= abc2 –
4
x=0

vi) For the face OCAB we have n^ = –k^ ds = dxdy, z = 0



 F  n^ ds = – (z2 – xy) dxdy = xy dx dy

a b 2 a 2 b 2 2
x y ab
 F  n^ ds =  xy dxdy =  2   2  = 4
S6 x=0 y=0 0 0

Adding (i) to (vi) we get for the whole surface S of the rectangular parallelopiped.
 b2c2 b2c2 a2c2 a2c2 a2b2 a2b2
 F  n^ ds = a2bc –
4
+
4
+ ab2c –
4
+
4
+ abc2 –
4
+
4
S

Gigatech Publishing House


Igniting Minds
Engineering Mathematics – III 5.31 Vector Integral Calculus and Applications

= a2bc + ab2c + abc2 = abc (a + b + c) .... (2)


 
n ds =    F dv = abc (a + b + c)
From (1) and (2) we get  F  ^
S V

Hence divergence theorem is verified.

Example : 2

 
Evaluate  F  n^ ds where F = axi + byj + czk and S is the surface of the sphere x2 + y2 + z2 = 4.
S
   
Solution : Here, F = (ax) + (by) + (cz) = a + b + c
x y z
 By Gauss Divergence theorem we have
 
 F  n^ ds =    F dv =  (a + b + c) dv = (a + b + c)  dv
S V V V
4
= (a + b + c)  Volume of sphere r3 
 3 
4 256
= (a + b + c)   (4)3 =  (a + b + c)
3  3

Example : 3


Evaluate  (4xzi – y2j + yzk)  d s over the cube bounded by the planes x = 0, x = 2, y = 0,
S
y = 2, z = 0, z = 2
Solution :
   
Here, F = (4xz) + (–y2) + (yz) = 4z – 2y + y = 4z – y
x y z
 
By Gauss divergence theorem we have  F  n^ ds =    F dv
S V
2 2 2 2 2 2 2
z
 =    (4z – y) dx dy dz =    4 2 – yz 
0
x=0 y=0 z=0 x=0 y=0
dxdy
2 2 2 2
4
=    4  2 – 2y  dxdy =   [8 – 2y] dx dy
x=0 y=0 x=0y=0
2 2
2 2
y
=   8y – 2 2  dx =  (16 – 4) dx
x=0 x=0 0
2
= 12 [ x ]0 = 24.

Gigatech Publishing House


Igniting Minds
Engineering Mathematics – III 5.32 Vector Integral Calculus and Applications

Example : 4

Evaluate [(xz2) dydz + (x2y – z2) dzdx + (2xy + y2z) dxdy] where S is the surface
S
enclosing a region bounded by hemisphere x2 + y2 + z2 = a2 above XY – plane.
Solution :

Here F = xz2i + (x2y – z2)j + (2xy + y2z) k
   
 F = (xz2) + (x2y – z2) + (2xy + y2z) = z2 + x2 + y2
x y z
By Gauss diverengence theorem we have,
 
 F  n^ ds =    F dv =  (x2 + y2 + z2) dx dy dz
S V V

Changing cartesian co–ordinates to spherical polar co–ordinates by taking x = rsin


cos, y = r sin sin, z = r cos
x2 + y2 + z2 = r2, dxdydz = r2 sin drdd

For hemisphere the limits are r = 0,a  = 0, ,  = 0,2
2
2 /2 a
 (x2 + y2 + z2) dxdydz =    r2r2 sin  dr d d
V =0 =0 r=0
5 a
=  r  [–cos ]/2 []2
 5 0 0 0

a5 4 5
= (1 + 1) (2 – 0) = a
5 5

Example : 5

Evaluate  [(z2 – x) dydz – xy dz dx + 3z dxdy] where S is closed surface of region


S
bounded by x = 0, x = 3, z = 0, z = 4 – y2.
Solution :

Here F = (z2 – x) i – xy j + 3zk then
   
F = (z2 – x) + (–xy) + (3z) = –1 – x + 3 = 2 – x
x y z
By using Gauss divergence theorem we have,
3 2 4–y2
 
 F  n^ ds =    F dv =    (2 – x) dxdydz
S V x = 0 y = –2 z = 0

Gigatech Publishing House


Igniting Minds
Engineering Mathematics – III 5.33 Vector Integral Calculus and Applications
3 2
4–y2
=   (2 – x) ( z ) 0 dxdy
x = 0 y = –2
3 2
x 2 3  y3 2
=  (2 – x) dx  (4 – y2) dy =  2x – 4y – 
x=0 y = –2
 2 0  3 –2

=  6 – 9   8 – 8 + 8 + 8  = 3  16 = 24
 2 3 3 2

Example : 6


Evaluate  (2yi + yzj + 2xzk)  d s over the surface of region bounded by y = 0, y = 3,
S
x = 0, z = 0, x + 2z = 6
Solution :

Here, F = 2yi + yzj + 2xzk then
   
F = (2y) + (yz) + (2xz) = 0 + z + 2x
x y z
By using Gauss divergence theorem we have
6 3 1/2(6–x)
 
 F  n^ ds =    F dv =    (2x + z) dxdydz
S V x=0 y=0 z=0
1
6 3 2 2 (6–x)
z
=    2xz + 2  dxdy
x=0 y=0
0
6 3
1
=
8   (–7x2 + 36x + 36) dxdy
x=0 y=0
1  x3 x2 6
–7 + 36 + 36x  [ y ]0
3
=
8  3 2 0
1 360
= [–504 + 648 + 216] [3] =  3 = 45  3 = 135
8 8
OR
3 3 6–2z
 
 F  n^ ds =    F dv =    (2x + z) dxdydz
S V z=0y=0 x=0
3 3 6–2z
=   [ x2 + zx] 0 dydz
z=0y=0
3 3
=   (2z2 – 18z + 36) dydz
z=0 y=0

Gigatech Publishing House


Igniting Minds
Engineering Mathematics – III 5.34 Vector Integral Calculus and Applications
3 2 3 3
=  2 z – 18 z + 36z [y]
 3 2 0 0
= (18 – 81 + 108) (3) = 135

Example : 7


Evaluate  (x2y3i + z2x3j + x2y3k)  d s where S is the curved surface of the sphere x2+y2 + z2 = a2
S

Solution : Here F = x2y3i + z2x3j + x2y3k then
   
F = (x2y3) + (z2x3) + (y2z3) = 2xy3 + 3y2z2
x y z
By using Gauss divergence theorem we have,
 
 F  n^ ds =    F dv = (2xy3 + 3y2z2) dxdydz
S V V

put x = rsin cos, y = rsin sin, z = rcos x + y2 + z2 = r2 2

dxdydz = r2sin drdd, limits r  0, a,   0, ,   0, 2


2  a
    [2rsin cos  r3sin3 sin3 + 3r2 sin2 sin2  r2cos2] r2sin drdd
=0 =0 r=0
2  a
=    [2r6sin5 sin3 cos + 3r6 sin3 cos2 sin2] drdd
=0 =0 r=0
2 
r a 7 . . 2 
= 0+3     7  0 sin3 cos2 sin2 d d  .  sin3 cos d = 0
=0 =0  0 
2 2
3a7 1 – cos2
=
7
2  sin3 cos2 d    2  d
0 0
7
3a 1 2 1 4a7
=  2    1   2 – 0 =
7 5 3 2  35

Example : 8

  
Evaluate  F  d s where F = (x + y2) i + yj – 2xzk and S is the surface bounded by the
S
planes x = 0, y = 0, z = 0 and x + y + z = 1
   
Solution : We have F = (x + y2) + (y) + (–2xz)
x y z

 F = 1 + 1 – 2x = 2(1 – x)
By using Gauss divergence theorem we get,

Gigatech Publishing House


Igniting Minds
Engineering Mathematics – III 5.35 Vector Integral Calculus and Applications
1 1–x 1–x–y
 
 F  n^ ds =    F dv =    2(1 – x) dxdydz
S V x=0 y=0 z=0
1 1–x 1–x–y
= 2   (1 – x) [ z ]
0
dxdy
x=0y=0
1 1–x
= 2   (1 – x) (1 – x – y) dxdy
x=0y=0
1
y2 1–x
= 2  (1 – x)  (1 – x) y – dx
x=0
 2 0
1
(1 – x)2 
= 2  (1 – x)  (1 – x)2 – dx
x=0
 2 
1 1
(1 – x)2 2
= 2 (1 – x) dx =  (1 – x)3dx
2 2
0 x=0
3 1
=  (1 – x)  = 0 + 1 (1)3 = 1
 –3 0 3 3

Example : 9


Evaluate (xi + yj + z2k)  d s where S is the curved surface of the cylinder x2 + y2 = 4
S
bounded by the planes z = 0 and z = 2
Solution :

Here F = (xi + yj + z2k) then
   
F = (x) + (y) + (z2)
x y z

   F = 1 + 1 + 2z = 2 (1 + z)
By Gauss divergence theorem we have Fig.: 5.21
2
 
 F  n^ ds =    F dv =   2(1 + z) dxdydz
S V z=0

2
2
z 4
=  2 2 + 2  dxdy = 2  2 + 2  dxdy
R 0 R

= 8  dxdy = 8 (Area of circle r2)


R

= 8 ( (2)2) = 32

Gigatech Publishing House


Igniting Minds
Engineering Mathematics – III 5.36 Vector Integral Calculus and Applications

Example : 10

Evaluate  (2x2y dydz – y2 dzdx + 4xz2 dxdy) over the curved surface of the cylinder
S
y2 + z2 = 9 bounded by x = 0 and x = 2.
Solution :

Here F = 2x2yi – y2j + 4xz2k then
   
F = (2x2y) + (–y2) + (4xz2)
x y z

F = 4xy – 2y + 8xz = 2(2xy – y + 4xz)
By using Gauss divergence theorem we have, Fig.: 5.22
2
 
 F  n^ ds =    F dv = 2  (2xy – y + 4xz) dxdydz
S V x=0
2
= 2  [ x2y – xy + 2x2z ] dydz
0
R

= 2  [4y – 2y + 8z] dydz = 4  (y + 4z) dydz


R R
2 2 2
Put y = r cos, z = r sin, y + z = r and dydz = rdrd
Limits r  0, 3,  = 0, 2
2 3

   F dv = 4   [r cos  + 4rsin ] rdrd
V =0 r=0
2 3
3
r
= 4  (cos + 4sin)   d
2
=0 0
3 2
(3)
= 4  [sin – 4cos]
3 0
= 4  9  [0 – 4 (1 – 1)] = 36  (0) = 0

Example : 11


Prove that  ( – )  d s =  (2 – 2) dv
S V

Solution :

Here F =  –  then

F =   () –   ()
=    + (  ) –    – ()
Gigatech Publishing House
Igniting Minds
Engineering Mathematics – III 5.37 Vector Integral Calculus and Applications

=    + 2 –    – 2
= 2 – 2
By using Gauss divergence theorem we have
 
 F  n^ ds =    F dv
S V


 ( – )  d s = (2 – 2)dv
S V

Example : 12

r
Prove that  r3  n^ ds = 0
S

Solution :

 r –3 
Here F = 3=r r then
r
   
F =   r–3 r = r3  r + r–3 (  r )
–3r –4   –3 3 3
= r  r + r (3) = – 5 r2 + 3
4 r r
3 3   
= – 3 + 3 =0 ( r  r = r2,   r = 3)
r r
By using Gauss divergence theorem we have
 
  r ^  r 
 F  n^ ds =    F dv   r3  n ds =     r3  dv = 0,
 
S V S V

Example : 13

dv r ^
Show that  r2
=  r2  n ds where S is the closed surface enclosing volume V. Hence
V S

xi + yj + zk 
evaluate   d s where S is the surface of the sphere x2 + y2 + z2 = a2
S
r2
Solution :

 r 
–2  
Here, F = r2=r r then   F =   r r–2
   –2r–3   –2
  r–2 r =   r–2  r + r–2 (  r ) = r  r + r (3)
r
2 2 3 1
= – 4  r2 + 3r–2 = – 4 + 2 = 2
r r r r
 By Gauss divergence theorem we have

Gigatech Publishing House


Igniting Minds
Engineering Mathematics – III 5.38 Vector Integral Calculus and Applications

 
 F  n^ ds =    F dv
S V

r 1
 r2  n^ ds =  r2 dv
S V

Transforming to spherical polar coordinates we get


put x = r sin cos, y = rsin sin z = rcos, x2 + y2 + z2 = r2, dxdydz = r2 sin drdd
Limits r  0, a,   0, ,   0, 2
2  a
1 1 2
 r2 dv =    r2
r sin drdd
V =0 =0 r=0
 a 2
= [ –cos ]0 [r]0 [] 0
= (1 + 1) (a – 0) (2 – 0) = 4a

Example : 14


Evaluate  r  n^ ds over the closed surface of the sphere of radius 2 with centre at origin.
S

Solution :
   
Here F = r then   F =   r = 3
By Gauss divergence theorem we have
 
 F  n^ ds =    F dv
S V

4
 3dv = 3  Volume of sphere 3r3 

 r  n^ ds =
S V
4
= 3   (2)3 = 32
3

Example : 15

1 
Show that the volume enclosed by a closed surface is 6  r2  d s
S

Solution : By using Gauss divergence theorem we have


 
 F  n^ ds =    F dv
S V
 2r    
2
Here, F = r = r = 2 r then   F = 2  r = 2  3 = 6
r

Gigatech Publishing House


Igniting Minds
Engineering Mathematics – III 5.39 Vector Integral Calculus and Applications


  r2  d s =  6dv = 6v
S V

1 
6 
 v = r2  d s
S

In study of electromagnetic field there are four vector partial differential equations
known as Maxwell’s equations whose solutions provide description of space time behavior of
the field vectors. Since these equations are representations of basic laws of faraday ampere
Gauass and coulomb of the electromagnetic fields. The Maxwell’s equations unify the study of
electromagnetic fields.

We define the solid angle dw at p as the surface intercepted on a sphere of unit radius by
a cone with vertex at the point p at which the solid angle is measured and generators passing

through contour of area ds which is subtending the angle. The area normal to vector r (axis of
the cone)

ds = dS  ^r = ds cos
ds dw
=
r2 1
ds
 dw =
r2
ds cos
=
r2
ds  ^r
= …. (i)
r2 Fig. 5.23
For electric flux due to charge q inside a volume V enclosed by surface S, we have, the flux
   q  q q q q
E across S =  E  d s =  40r2 ^r  d s =  40 dw = 40  dw =  4 = (Over a
S S S S
40 0
unit sphere for complete surface S)
  q
  E  d s =
0
…. (ii)
S

For volume distribution of charge


  1
  E  d s =
0 
 dv
S V
Fig. 5.24
 1  
i.e.    E dv =
0 
 dv E=
0
…. (1)
V V

Gigatech Publishing House


Igniting Minds
Engineering Mathematics – III 5.40 Vector Integral Calculus and Applications

which is known as Gauss’s Law.


for dielectric medium which has volume and surface charge distributions  and 

respectively we have to consider additional charge densities –   p (volume distribution) and pn
(surface distribution) due to the polarization.
Thus from equation (1) for volume distribution, we have
 1
E = (Total volume charge density distribution)
0
1 
= ( –   p )
0
 
i.e.   0 E = – p
 
i.e.   (0 E + p ) =  …. (iii)

The electric displacement D is defined by the relation
  
D = 0 E + p

We know that the total moment of dipoles per unit volume is known as polarization p which
  
is proportional to the electric intensity E at that point. i.e. p = K E
Where K is known as dielectric susceptibility.
    
We have D = 0 E + K E = (0 + K) E =  E
 
Thus D = E …. (2)
where  = 0 + K is known as dielectric constant and from (iii) the Gauss’s Law for
dielectric medium takes the form

 D =  …(3)

Since no free magnetic poles exist and as the effect of magnetization, we have to consider

volume distribution of magnetic pole density –  I , the Gauss’s Law i.e. equation (1) for
magnetic field take the form
 1
H = (Volume density of magnet poles)
0

 I  
= –    [0 H + I ] = 0 …. (1)
0

The magnetic induction vector B is defined as
  
B = 0 H + I …. (2)
  
But since the magnetization is proportional to the field intensity H, we have I = H …. (3)
where  is known as magnetic susceptibility.

Gigatech Publishing House


Igniting Minds
Engineering Mathematics – III 5.41 Vector Integral Calculus and Applications
   
 From (ii) and (iii) B = (0 + )H i.e. B = H …. (4)
Where  = 0 +  is known as permeability constant.
Thus the Gauss’s law for magnetic field takes the form

B = 0 …. (5)
 Steady current :
The flow of charge constitutes a
current, known as conduction
current. This is measured by the
total charge carried across an area.
Let N – number of charges / volume
e – charge on each particle Fig. 5.25
 
V – velocity of the charge at the point and d s be the area inclined at an angle  with
   
velocity V of the charges. The volume of charges crossing across d s per unit time = d s V
  
 Total number of charges crossing d s per unit time = N(d s V)
  
 The current crossing d s per unit time = e[N(d s  V )]
 
= (eNV)  d s …. (i)

The conduction current density J C, defined as the rate of charge carried across unit area

perpendicular to V is
 
J C = eNV …. (6)
  
 The flux of charge crossing d s = J C  d s

Hence the current I crossing area S is given by
 
I =  J C  ds …. (7)
S

 Equation of continuity :
Consider a closed surface S enclosing a volume V in the flow of charges having volume
density .
Now by continuity principle, Rate of decrease of charge in volume V = Rate of flux of
charge across S.
d  
dt 
i.e. – dv =  J C  d s
V S

d
  dt +   J C  dv = 0

i.e. (By Gauss’s theorem)
V

Since V is arbitrary, the equation of continuity is
Gigatech Publishing House
Igniting Minds
Engineering Mathematics – III 5.42 Vector Integral Calculus and Applications

d 
+   JC = 0 …. (8)
dt
d 
For steady current =0    JC= 0 …. (9)
dt

a) Force exerted on an element carrying a



current placed in magnetic induction B is

given by Fleming’s Left Hand rule. If I is
current in the direction t, then the force
exerted on element of unit length is given by
 Fig. 5.26
= (I^t)  B …. (i)
The current through the conductor of cross section unity is
I = NeV …. (ii) (V – velocity of charge)
 
 Force on the current carrying conductor of unit cross section = F = [NeV^t]  B
   
= NeV  B = N(eV  B)

Thus the force on charge e in field B is given by
  
F = eV  B
b) Faraday’s Law (Induced e.m.f.) :
When the magnetic flux linked with coil changes an induced e.m.f. is created and is given by
Faraday’s Law
 N = number of charges/volume 
…. (i)  e = charge on each particle 
dN
e.m.f. = –
dt  
 V = velocity of the charge at the point 

Let a coil placed in magnetic field B be

moved parallel to itself with velocity V1. In

time t it moves through a distance V1 t

and V is the velocity of the charge. The
 
resultant velocity of the charge is V + V1.
Hence the force exerted on charge is given
  
by = e [V + V1]  B. Thus the electric field

E due to motion of coil. Fig. 5.27
  
= [V + V1 ]  B …. (iii) (being force on unit charge)
 
let  r = AB = element of the coil = dr ^t then the potential drop over the length AB is
Gigatech Publishing House
Igniting Minds
Engineering Mathematics – III 5.43 Vector Integral Calculus and Applications

dVAB =

E r = { [V + V ]  B }  r

1

dVAB = ( V  B )  r + ( V  B )  r


1

Since the velocity V of charge is along the coil i.e. along ^t we have ( V  B )  r =
(r  V)  B = 0 (as r | | V)
Thus from (iii) we have,

dVAB = ( V 1 B )  r

= ( V 1 r) B
 

 
where V1   r 1 = area of swept out by AB in unit time
 
E  r = dVAB

= –
d  
dt
(
B ds ) = – dN
dt
 
Hence for complete coil, ∮ E   r = Induced e.m.f.
C

  d  
∮ E  r = –
dt 
B   s By Stoke’s theorem
C S

   B 
   E  d s =  t
 ds
S S

   B 

i.e.    E +   ds = 0
S
 t 
Since S is arbitrary, the Faraday’s law of electromagnetic induction is given by

  B
E = – …. (11)
t

a) The change in potential along a closed path


not threading the coil is zero as the change
in the solid angle is zero as P moves
completely along a closed path.
b) If the point P moves along a closed path
threading the circuit once, the change in the
solid angle is 4, hence change in potential
along a closed path threading the coil once
Fig. 5.28
is
Gigatech Publishing House
Igniting Minds
Engineering Mathematics – III 5.44 Vector Integral Calculus and Applications

i.e. The magnetic potential at P due to the


I


  
=–
4
 4 = – I  coil at P = – I4 where  p
p is the solid angle 
 subtended at P by the coil 

if H is the magnetic field at the point P, then
 
H=– (analogous to electrical field E)

For the small displacement  r for the point along such a path,

we have d =    r
   
= (–H)   r = – H   r
 The change in magnetic potential  for a closed path threading the coil = – I
 
= ∮ d = ∮ H  d r
C C

   
i.e. ∮Hdr = I =  J  d s
C S

   
By Stoke’s theorem    H  d s =  J  d s
S S
 
Hence Ampere’s Law gives   H = J …. (12) Fig. 5.28

It is to be noted that J is not the conduction current density alone.
 Displacement current :
 
From (12) we have   J =   (  H) = 0
But the equation of continuity of charge give
   
  JC = – = – (  D)
t t

Thus   J  0 in general, unless for steady current

  JC = 0  as  = 0 
 t 
Thus the equation (12) will not be consistent with equation of continuity. In order to make it
consistent with equation continuity, we assume

 D
J = JC + …. (13)
t

 D
   
Since  J =   JC + =   JC + (  D)
t t

Gigatech Publishing House


Igniting Minds
Engineering Mathematics – III 5.45 Vector Integral Calculus and Applications

  D  by equation of continuity 
 J =   JC + =0   
t  and   D =  from (3) 
and as well the equation (12) will be satisfied as
 
 J =   (  H) = 0

 D 
in result (13), J is sum of conduction current density J C and which is known as the
t
displacement current density. Hence Ampere’s Law (12) modifies to the form

 D 
  H = JC + …. (14)
t
 Maxwell’s Equations :
The Maxwell’s equations are :

i)   D = , Gauss’s law for dielectric medium.

ii)   B = 0, Gauss’s law for magnetic Induction.

 B
iii)   E = – , Faraday’s law
t

 D

iv)   H = JC+ , Ampere’s law.
t
   
With results D = e E and B = H.

Illustrative Examples

Example : 1

B   
Maxwell’s electromagnetic equations are   B = given B = curl A then deduce that
t

 A
E+ = – gradv where v is a scalar point function.
t
  
Solution : Since B = curl A =   A then
 
B   A
= (  A) =  
t t t
 
 B A
  E = – =–
t t

 A
 E +  = 0
t

Gigatech Publishing House


Igniting Minds
Engineering Mathematics – III 5.46 Vector Integral Calculus and Applications

   A 
 E + = 0
 t 
  
[We know that   F = 0, F is irrotational field then F = ]

 A
 E+ = –v = –gradv
t

Example : 2

Equations of Electromagnetic wave theory are given by :


 
i) D= ii)   H = 0
 
1  D  1 H

iii)   H =  + v  iv)   D = –
C  t  C t
where t is the time variable, C is constant and other symbols denote point functions prove
that

2
 1 2H 1 
a)  H – 2 2 = –   (V) and
C t C

2
 1 2D 1  
b)  D – 2 2 =  + 2 (V)
C t C t
  
Solution : We know that   (  F) = (  F)  – 2 F .
  
a)   (  H) = (  H)  – 2H
 1  
D  
  + V   = 0 – 2 H
 C  t 
1 
C t
(   D) +   (V)  = – 2H


1    1 H    
– +   (V)  + 2H = 0
C t  C t  

2 1 2H
 1 
  H– 2 = –   (V)
C t2 C
  
b)   (  D) = (  D)  – 2D
 1 
H  
– =  – 2D
 C t 
1   
– [  H] =  – 2D
C t

1   1  D  
– + V   =  – 2D
C t  C  t 
Gigatech Publishing House
Igniting Minds
Engineering Mathematics – III 5.47 Vector Integral Calculus and Applications

1 2D 1   
– 2 2 – 2 (V) =  – 2D
C t C t

21 2 D
 1  
  D– 2 2 =  + (V)
C t C2 t
Hence the result is proved.

Example : 3

If  is a scalar function of x, z and variable t (time) the electric and magnetic vectors E and
     1 
H are given by E = i– k H=– j where c is constant.
z x C t
Show that :

1 E  
i) H= ii)   H = 0,   E
C t

 1 H 2 2 1 2
iii)   E = – , if  satisfies the equation 2 + 2 = 2 2
C t x z C t
Solution :
j
 i 
k
 
i)

H =
 x y z 
 1  
0 –
C t 0 
 1    1   
= i0+  – j[0 – 0] + k   – –0
 z  C t   x  C t  
 1     1    
 H = i –k
C t  z  C t  x 

1      1   1  E
= i –k = ( E) =
C t  z x  C t C t

 1 E
 H =
C t

ii)

H =  i  + j  + k     – 1  j  = 0 – 1     + 0
 x y z   C t  C y  t 

H = 0 (...  is function of x, z, t)
 
the partial derivative w.r.t.y is zero ... = const. 
t  t 
      
 E = i +j +k  i– k
 x y z   z x 

Gigatech Publishing House


Igniting Minds
Engineering Mathematics – III 5.48 Vector Integral Calculus and Applications

       2 2
= +0–  = – =0
x  z  z  x  x z z x
2 2


E = 0 ...   =   
 x z z x 
i k
 
j
 
iii)

 E =
 x yz 
   
 z 0

x 
       
 E = i   –  – (0)  – j   –  –   
 y  x  z   x  x  z  z  
  
+ k  (0) – (0)   
 x y  z  
 2 2
 E = 0 2 + z j+0 (...  is function of x, z, t)
 x z 
 1 2 2 2 2
 Given :  2 +  2 = 12  2 
 E = 2 2 j
C t  x z C t 

 E =
1   1   1 
j =
C t  C t  C t
( – H )

 1 H
 E = –
C t
Hence the result is proved.

Example : 4
 
  H  E

Maxwell’s equations are given by   E = 0,   H = 0,   E = – ,H= then
t t
2
  u
show that E and H satisfy 2u = 2
t
Solution :
  
Here we write   (  E) = (  E)  – 2 E
 
H  
 – = 0 – 2 E
 t 

    2 E
(  H) = 2 E  2 E =
t t2
  
Now we take   (  H) = (  H)  – 2H
 
E  
  = 0 – 2H
 t 

Gigatech Publishing House


Igniting Minds
Engineering Mathematics – III 5.49 Vector Integral Calculus and Applications

  
2   H  
(  E) = – H  – = – 2H
t t  t 

2
 2 H
 H =
t2
  2u
 E and H satisfy 2u =
t2

Example : 5

 1 A  
Show that : E = – – , H =   A are solutions of the Maxwell’s equations :
C t
 
1 E 1 H

i) H= ii)   E = –
C t C t

 1  1 2 A

2
if 1)   A + =0 2)  A = 2 2
C t C t
 
Solution : Here we take   H =   (  A)
  
i)  H = (  A)  – 2 A
2

1   1  A
=  – 2
 C t  C t2

1   1 A 
= –  +
C t  C t 
1  
= – (– E)
C t

 1 E
 H =
C t

  1  A 
ii) Now we take  E =    –  –
 C t 
 1  
  E = –    – (  A)
C t

 1 H
 E = –0–
C t

 1 H
 E = –
C t

Gigatech Publishing House


Igniting Minds
Engineering Mathematics – III 5.50 Vector Integral Calculus and Applications

Descriptive Questions

Q. 1 Evaluate  F d‒r where.
C

 2  
F 3x i + (2xz – y) j + zk along the curve x = t, y = t2, z = t3 from (0, 0, 0) to (1, 1, 1).

 
Q. 2 Find the work done in moving a particle from the point 0 1 4 to  2 2 in the force
   
field.
   
F = (y sin z – sin x) i + (x sin z + 2yz) j + (xy cos z + y2) k.
   
Q. 3 Evaluate  F d‒r , using Green’s theorem where F – (2x2 – y2) i + (x2 + y2) j and C is the
C
circle x2 + y2 = 1 above the x–axis.
   
Q. 4 Evaluate  F d‒r , F = xy i + x2 j , where C is the curve y2 = x joining (0, 0) and (1, 1)
C

Q. 5 Apply Green’s theorem to evaluate  (3y dx + 2x dy) where C is boundary of 0  x  ,


C
0  y  sin x.
Q. 6 A vector field is given by
    x2 y2
F = cos y i + x [1 – sin y] j , evaluate  F d‒r where C is ellipse + = 1.
C
25 9
   
Q. 7 Find work done by the force F = (2xz3 + 6y) i (6x – 2yz) j + (3x2 z2 – y2) k in moving a
particle from (0, 0, 0) to (1, 1, 1).
Q. 8 Find the work done in moving a particle once around the circle x2 + y2 = 9 in the xy–
plane if the force field is given by
   
F = (2x – y – z) i + (x + y – z2) j (3x – 2y + 4z) k.
   
Q. 9 Evaluate  F d‒r where F = x2 i + xy j and C is the arc of the parabola joining (0, 0) and
C

(1,1) Equation of parabola is y = x2 .


Q.10 Use Green’s Lemma to evaluate the line integral.

 (cos x sin y – 4y) dx + sin x cos y dy where C is the circle x2 + y2 = 1.


C

   
Q.11 Prove that F = (4xy – 3x2 z2) i + 2x2 j – 2x3 z k is conservative field and also find the
work done in moving an object in this field from (0, 0, 0) to (1, 1, 1).

Gigatech Publishing House


Igniting Minds
Engineering Mathematics – III 5.51 Vector Integral Calculus and Applications

   
Q.12 Evaluate  (  F) ds, F = – 3 i + x3 j where the boundary of surface S is given by
S
x2 y2
+ = 1.
1 4
    
Q.13 Evaluate  (  F)  n^ ds , where F = xy2 i + y j + z2x k and S is the surface of a
S

rectangular lamina bounded by x = 0, y = 0, x = 1, y = 2, z = 0.

Q14. Apply Stoke’s theorem to calculate  4ydx + 2 zdy + 6ydz where C is the curve at
C

intersection of x2 + y2 + z2 = 6z and z = (x + 3).


    
Q.15 Using Stoke’s theorem evaluate  (  F)  ds for F = (x3 – y3) i – xyz j + y3 k where S
S

is the surface x2 + ay2 + z2 – 2x = 4 above the plane x = 0.



Q.16 Evaluate  (  F) ds where S is the surface x2 + 4y2 + z2 – 2x = 4 above the plane x = 0
S
   
and F = (x3 – y3) i – xyz j + y3 k .
    
Q.17 Evaluate  (  F)  ds where F (2y + x) i + (x – y) j + (z – x) k and S is surface
S

bounded by x = 0, y = 0, (x + y z) = 1, which if not included in XOY plane.

 (4y i )  ds where C is the curve of


  
Q.18 Use Stoke’s theorem to evaluate + 2z j + 6y k
C
2 2 2
intersection of x + y + z = 2z and x = z – 1.
   
Q.19 Verify Stoke’s theorem for F = y2 i + x j – (x + z) k over the area of the triangle with
vertices (0, 0, 0) , (1, 0, 0) (1, 1,0).
    
Q.20 Evaluate  (  F)  ds for F = y i + z j + x k over the surface x2 + y2 = 1 – z, z  0.
S

    
Q21. Evaluate  F d‒r by using Stoke’s theorem for F = 4y i – 4x j + 3 k where S is 0 disk of
C

radius 1 lying on the plane z = 1 and c is its boundary.



Q.22 Evaluate  (  F)  n^ ds, where S is the curved surface of paraboloid. x2 + y2 = 2z
S
   
bounded by the plane z = 2, where F = 3 (x – y) i – 2xz j + xy k.

Gigatech Publishing House


Igniting Minds
Engineering Mathematics – III 5.52 Vector Integral Calculus and Applications

    
Q.23 Using Stoke’s theorem evaluate  F d‒r where F = yz i + zx j + xy k and C is the curve
C

x2 + y2 = 1, z – y2.

(  
Q.24 Evaluate  yz i + zx j + xy k )  ds where S is the curved surface of cone x 2
+ y2 = z2
S

and z = 4.
   
( 
Q.25 Use Divergence theorem to evaluate  F  ds where F = 4x i – 2y2 j + z2 k ) and S is
S

surface bounding the region x + y = 4, z = 0 and z = 3.


1 1 ‒ 
Q.26 Prove that  r2 dV =  r2
 r d s where S is closed surface enclosing volume V.
V S
  
xi + y j + z k 
Hence evaluate  d s where S is surface of sphere x2 + y2 + z2 = a2.
r2
   
Q.27 Using Gauss Divergence theorem for the vector function F = (x3 – yz) i – 2x2 y j + zk .

Evaluate  F  ds where S is surface bounding the cube x = 0, y = 0, z = 0 and x = a, y =
S
a, z = a.

Q.28 Evaluate  F  ds where S is the curved surface of the sphere x2 + y2 + z2 = 4 above the
S
   
plane z = 0 and F = y2 z2 i + z2 x2 j + x2 y2k.

Q.29 Evaluate  (y 2   
z2 i + z2 x2 j + x2 y2 k )  ds where S is surface of the sphere x 2
+ y2 +
S
z2 = 4 in the positive octant.

Q.30 Evaluate  3x dy dz – 2y dz dx + 2z dx dy over surface of a sphere of radius a .


S



Gigatech Publishing House


Igniting Minds
Unit
6
Complex Variables

Syllabus :
Functions of complex variables, Analytic functions, Cauchy – Riemann equations, conformal
mapping Bilinear Transformation , Cauchys Integral theorem, Cauchy Integral formula, Laurent's
series and Residue theorem.

Theory of complex functions / variables are widely used in the study of fluid dynamics,
conduction of heat flow, electrostatics electromagnetic Engineering, two dimensional potential
problems. thermodynamics, electrical fields etc.

1. The complex numbers are denoted by z = x + iy where x, y ER and i = –1


2. |z| = x2 + y2

3. z =x – iy
4. The complex number x + iy is geometrically denoted by a point (x, y) on XY plane
5. The X axis is called real axis & Y axis is called as imaginary axis
6. |z| = |a| is x2 + y2 = a2, a circle with centre (0, 0) and radius a units in XOY plane or
z– plane.
7. z0 = x0+iy0  (x0, y0)
8. |z – z0|= |a| means | (x + iy) – (x0 + iy0)| = |a|
 | (x – x0) + i(y – y0) | = |a|
 (x – x0)2 + (y – y0)2 = |a|
 (x – x0)2 + ( y – y0)2| = a2 , which is a circle with centre (x0, y0) and radius a units
9. | z – z0 | < a, means ( x – x0 )2 + ( y – y0 )2 < a2, which is a interior part of circle with
centre (x0, y0) and radius a units.
Gigatech Publishing House
Igniting Minds
Engineering Mathematics – III 6.2 Complex Variable

10. | z – z0 | > a, means ( x – x0 )2 + ( y – y0 )2 > a2, which is a exterior part of circle whose
centre (x0, y0) and radius a units.
11. a < | z – z0 | < b, gives a region lies between two concentric circles having centre
(x0 , y0) and having radii a, b and that region is called as an open circular ring or open
annulus.
12. | z + z0 | = |a|, means ( x + x0 )2 + ( y + y0 )2 = a2, , a circle with centre (–x0, – y0) and
radius a units.
du
13. If f(x, y) = 0 then , gives the slope of tangent to the curve f(x, y) = 0
dx
14. If m1,m2 are slopes of tangents to the two curves and if m1  m2 = –1 , then two curves
will be orthogonal to each other.

Let u(x, y) and v(x, y) be real valued functions of real numbers x, y. Let z = x + iy,
where i = –1 then [u(x, y) + i v(x, y) ] is called as complex function and denoted by f (z)
 f (z) = u(x, y) + i v(x, y) = u + iv
 w = f (z) = u(x, y) + i v(x, y) = u + iv
Note that u(x, y) and v(x, y) are called real and imaginary parts of complex function w
or f(z).
Example :
If w = z3
then w = (x + iy)3 = x3 + 3x2(iy) + 3x (iy)2 + (iy)3
= (x3 – 3xy2) + i (3x2y – y3)
 u = x3 – 3xy2 , v = 3x2y – y3

If w = z
then w = x – iy = x + i(– y)
 u = x , v= – y

If w = f (x) and if for each value of z, there exists only one value of w then w is called as
a single valued function, otherwise it is called as a multivalued function.
Example : (1) If w = z2 and if z = 4 then w = 42 = 16
w = z2 is a single valued function
(2) If w = z and if z = 4 then w = 4 =  2
w = z is multivalued function

Gigatech Publishing House


Igniting Minds
Engineering Mathematics – III 6.3 Complex Variable

 Continuity :
A complex function f (z) is said to be continuous at point z = z0 if lim f (z) = f (z0)
zz0

(i) A function is said to be continuous in a domain R, if it is continuous at every point


in domain R.
(ii) A complex function w = u + iv is continuous if both u and v are continues
functions.
 Complex Differentiation :
Let w = f (z) be a single valued function of z
dw f(z + h) – f(z)
then dz = hlim
0  
h
dw |
(i) = f (z)
dz
dw
(ii) dz exists only if limit on r.h.s. exists.

(iii) The formula for differentiation of f (z) is as same to the formula for differentiation
in real calculus and hence all formulae of differentiation of real calculus are valid
in complex differentiation as below :
d d df dg
1) (constant) = 0 2) [fg] = 
dz dz dz dz
d d d dg df
3) dz [cf (z)] = c dz [f(z)] 4) dz [ f.g ] = f dz  g dz
d d n
5) dz [tanz] = sec2z 6) dz [z ] = n z
n–1

d d  f  gdf – fdg
7) dz [cosz ] = – sin z 8) dz g = g2
d d 1
9) [sec z] = secz tanz 10) (log z) = , etc.
dz dz z

A function f(z) , which is a single valued function and possesses a unique derivative at
all points of domain R, is called as an analytic function of z in region R.
An analytic function is also known as entire / regular / holomorphic function.
Example :
(1) If f (z) = z2 then f (z) = 2z , exists everywhere
(2) If f (z) = z3 then f (z) = 3z2 , exists everywhere
 Harmonic function :
A real valued function u(x, y) is called as a harmonic function if it satisfies a Laplace's
2u 2u
equation + =0
x2 y2

Gigatech Publishing House


Igniting Minds
Engineering Mathematics – III 6.4 Complex Variable

The complex function f(z) = u(x, y) + iv(x, y) will be an analytic function if it satisfies
the C – R equations,
u v u v
= and =–
x y y x
u u v v
Where the partial derivatives , , , are continuous functions of x, y in the
x y x y
region R.
Note : Consider an analytic function f (z) = (u + iv), then
(i) u and v are always harmonic functions .
(ii) u and v satisfies C–R equations.
(iii) real part u and imaginary part v are called as conjugate harmonic functions of each
other.
(iv) differentiation of an analytic function is also an analytic function
(v) polynomial function is always analytic function
(vi) u = c1, v = c2 forms mutually orthogonal trajectories
f
(vii) if f & g are analytic functions then ( f  g),   ,( f. g ) are also analytic functions if
g
g(z)  0
(viii) Milne Thompson method : analytic functions f (z) , f (z) can be expressed in terms of
only z by replacing x = z , y = 0
u v v u
(ix) f (z) = + i = – i
x x y y

The complex function f (z) = u(r, ) + iv(r, ) will be analytic if


u 1 v v 1 u
= and =–
r r  r r 
Note :
(i) The complex function f (z) = u(r, ) + iv(r, ), can be expressed in terms of only z by
replacing r = z,  = 0 if f (z) is an analytic function.

Illustrative Examples

Type : (A)

Example : 1

Find an analytic function f (z) = u + iv,


where u = x3  3xy2 + 3x2 – 3y2 + 6 …. (1)
and hence express f (z) in terms of only z
Gigatech Publishing House
Igniting Minds
Engineering Mathematics – III 6.5 Complex Variable

Solution :
We know, f(z) will be an analytic function
u v
if = …. (2)
x y
u v
=– …. (3)
y x
Diff (1) partially w.r.to x (y constant )
u
= 3x2 – 3y2 + 6x
x
v ... (2) above
 = 3x2 – 3y2 + 6x
y
Integrate partially w.r. to y (x constant )
y3
v = 3x2y – 3 3  + 6xy + A(x)
 
2 3
v = 3x y – y + 6xy + A(x) …. (4)
Differentiate (4) partially w.r.to x (y constant)
v
= 3y(2x) – 0 + 6y + A'(x)
x
v
= 6xy+6y+A'(x)
x
u ... (3) above
 – = 6xy+6y+A'(x)
y
u
 = –6xy–6y–A'(x) …. (5)
y
Diff (1) partially w.r. to y (x constant)
u
= 0 – 6xy + 0 – 6y
y
= – 6xy – 6y …. (6)
From (5) and (6) we get
– 6xy – 6y = – 6xy – 6y – A'(x)
 – A'(x) = 0
 A'(x) = 0
 A(x) = c
 From (4), v = 3x2y – y3 + 6xy+c …. (7)
 Required analytic function f(z) is
f(z) = u+iv
f(z) = (x3–3xy2+3x2–3y2+6) + i(3x2y – y3 + 6xy+c) …. (8)
Gigatech Publishing House
Igniting Minds
Engineering Mathematics – III 6.6 Complex Variable

Milne-Thompson Method :
Put x = z, y = 0 in (8)
 f(z) in terms of only z is
f(z) = (z3 + 3z2 + 6) + i (c)
= z3 + 3z2 + 6 + ic

Example : 2

Find a regular function f(z) = u+iv


Where v = –sinx sinhy …. (1)
Also express f(z) only in terms of z.
Solution :
Diff (1) partially w.r. to y (x constant)
v
= – sinx coshy …. (2)
y
u
= – sinx coshy …. (3)
x
... C–R equation u = v
x y
Integrate (3) partially w.r. to x (y constant )
 u = (–coshy) (–cosx) +A(y)
u = cos x coshy +A(y) …. (4)
Differentiate (4) partially w.r.to y ( x constant)
u
 = cosx sinhy + A' (y)
y
v ... by C – R equation u =  v
– = cosx sinhy + A' (y)
x x y
v
 = –cosx sinhy – A' (y) …. (5)
x
Diff (1) partially w.r. to x (y constant)
v
= –cosx sinhy …. (6)
x
From (5) and (6), we get
–cosx sinhy – A' (y) = –cosx sinhy
 – A'(y) = 0
 A'(y) = 0
 A(y) = c
 Required, u = cosx coshy + c
 Required analytic function f (z) is
Gigatech Publishing House
Igniting Minds
Engineering Mathematics – III 6.7 Complex Variable

f (z) = (cosx coshy + c ) + i (– sinx sinhy) …. (7)


Milnes Thompson Method :
Put x = z, y = 0 in (7)
f(z) = cosz + c

Example : 3

If v = y + ex cos y, show that


(i) v is a harmonic function
(ii) find u, harmonic conjugate of v, such that f(z) = u +iv be an analytic function
(iii) Express f(z) in terms of only z
(iv) show that u(x, y) = c1, v(x, y) = c2, where c1, c2 are constants, cuts each other
orthogonally.
Solution :
i) Let v = y + ex cos y …. (1)
v
 = 0 + ex cos y
x
2v
= ex cos y
x2
v
= 1 + ex (–sin y)
y
2v
= 0 + ex (–cos y) = – ex cos y
y2
2v 2v
 + = ex cos y – ex cos y = 0
x2 y2
 v is a harmonic function.
(ii) To find u, harmonic conjugate of v
Diff. (1) partially w.r. to x (y constant)
v
 = ex cos y
x
u v u
– = ex cos y  =–
y x y
u
 = – ex cosy …. (2)
y
Integrate (2) partially w.r. to y (x constant )
 u = – ex (siny)+ A' (x) …. (3)
Differentiate (3) partially w.r. to x (y constant )
u
= – ex siny + A'(x)
x

Gigatech Publishing House


Igniting Minds
Engineering Mathematics – III 6.8 Complex Variable

v u v
 = – ex siny + A'(x)  = …. (4)
y x y
Differentiate (1) partially w.r. to y (x constant )
v
 = 1 + ex (– siny)
y
= 1 – ex siny …. (5)
v
From (4) and (5), = – ex siny + A'(x) = 1 – ex siny
y
 – ex siny + A'(x) = 1 – ex siny
 A'(x) = 1
 A(x) = x
From (3) u = – ex siny + x
u = x – ex siny …. (6)
Required Analytic function f (z) is
f (z) = u + iv
= ( x – ex sin y) + i ( y + ex cosy) …. (7)
(iii) To express f (z) in terms of z by using Milne-Thompson method :
put x = z and y = 0 in (7)
f (z) = [ z – ez (0) ] + i (0 + ez (1))
= z + i ez
(iv) Orthogonal trajectories :
dy
We know that dx gives the slope of tangent to curve f(x,y) = 0

we have u = x – ex siny v = y + ex cosy


Let u = x – ex siny = c1 …. (8)
u
x
dy  
 = 
dx u
y
 
(1– ex siny )
= –  – ex cosy  = m1 …. (9)
 
Let v = y + ex cosy = c2 …. (10)
v
x
dy  
 = –
dx v
y
 
0 + ex cosy
= –  1 – ex siny 
 

Gigatech Publishing House


Igniting Minds
Engineering Mathematics – III 6.9 Complex Variable
x
e cosy 
= – = m2 …. (11)
1 – ex siny 
x x
 (m1) (m2) = 1– ex siny   – e xcosy 
 – e cosy  1 – e siny 
= –1
 Product of slopes of tangents to curves (8) & (10) is (–1)
 Curves (8) & (10) forms orthogonal system trajectory.

Type : B

Example : 1

Determine an analytic function f (z) = u + iv in terms of z if u = x2 – y2 – y


Solution :
Let u = u = x2 – y2 – y …. (1)
Differentiate (1) partially w.r. to x & y , we get
u
= 2x …. (2)
x
u
= –2y – 1 …. (3)
y
we know , if f (z) = u + iv, is an analytic function
u v
then, f ' (z) = +i
x x
u u
= + i –  (  C – R equations )
x  y
f '(z) = 2x + i (2y + 1) (2&3)
Put x = z , y = 0 Milne-Thompson method
= 2z + i (0 + 1)
f '(z) = 2z + i
Integrate w.r. to z ,
z2
f (z) = 2  2 + iz + c
 
f (z) = z2 + iz + c

Example : 2

If v = 4x3y – 4xy3, find harmonic conjugate of v such that f(z) = u +iv becomes an
analytic function.
Solution :
Let v = 4x3y – 4xy3 …. (1)
Gigatech Publishing House
Igniting Minds
Engineering Mathematics – III 6.10 Complex Variable

Differentiate (1) partially w.r. to x & y


v
 = 12x2y – 4y3 …. (2)
x
v
 = 4x3 – 12xy2 …. (3)
y
u v
We know, f ' (z) = +i
x x
v v   C–R equations u = v 
= +i
y x  x y 
f ' (z) = (4x3 – 12xy2 ) + i (12x2y – 4y3 )
= ( 4z3 – 0) + i (0 – 0)
( x = z, y = 0 Milnes-Thompson method)
f ' (z) = 4z3 + 0
Integrate w.r. to z we get
z4
f (z) = 4   + c
4
f (z) = z4 + c …. (4)
4
= (x + iy) + c
= x4 + 4(x3) (iy) + 6(x2)(iy)2 + 4(x)(iy)3 + (iy)4 + c
= x4 + 4iyx3 + 6x2(–y2) + 4x(–iy3) + i4y4 + c
= x4 + 4iyx3 – 6x2y2– 4ixy3 + y4 + c
= (x4 – 6x2y2 + y4) + i (4yx3 – 4xy3) + c
= (x4 – 6x2y2 + y4 + c) + i (4yx3 – 4xy3)
f (z) = u + iv
 u = x4 – 6x2y2 + y4 + c

Type : C

Example : 1

Find an analytic function f(z) = u + iv


if u + v = e–x (cosy – siny) …. (1)
Solution :
Differentiate (1) partially w.r.to x (y constant)
u v
+ = –e–x (cosy – siny) …. (2)
x x
Differentiate (1) partially w.r.to y (x constant)
u v
+ = e–x(–siny – cosy ) = – e–x(siny + cosy )
y y
Gigatech Publishing House
Igniting Minds
Engineering Mathematics – III 6.11 Complex Variable

v u
– + = – e–x(siny + cosy ) …. (3) (  C–R equations)
x x
Adding (2) & (3) we get
u
2 = –e–x(cosy – siny) – e–x(siny + cosy )
x
= –e–x(cosy – siny + siny + cosy )
u
2 = –2e–x cosy
x
u
 = –e–x cosy …. (4)
x
Subtract (3) from (2) , we get
v
2 = –e–x(cosy – siny) + e–x(siny + cosy )
x
= –e–x(cosy – siny – siny – cosy )
= 2e–x siny
v
 = e–x siny …. (5)
x
u v
We know, f '(z) = +i
x x
= (–e–x cosy) +i (e–x siny )
f '(z) = e–x [ – cosy +i siny ]
 f '(z) = e–z [ –1 – 0 ] ( x = z, y = 0)
f '(z) = –e–z = – e–z
Integrate w.r.to z,
f (z) = e–z + c
 f (z) = e–z + c

Example : 2

Find an analytic function f(z) = u +iv


if 2u + v = ex (cosy – siny) …. (1)
Solution : Differentiate (1) Partially w.r.to x ( y constant)
u v
2 + = ex(cosy – siny) …. (2)
x x
Differentiate (1) partially w.r.to y ( x constant)
u v
2 + = ex(–siny – cosy )
y y
v u
–2 + = e–x(–siny – cosy ) …. (3) (  C–R equations)
x x

Gigatech Publishing House


Igniting Minds
Engineering Mathematics – III 6.12 Complex Variable

Multiply equation (2) by 2 & add with (3),


u v u u
4 +2 – 2 + = 2(ex ) (cosy – siny) + ex(–siny – cosy )
x x x x
u
5 = ex [2cosy – 2siny – siny – cosy ]
x
u
5 = ex [cosy – 3siny ]
x
u ex
 = 5 [ cosy – 3siny ] …. (4)
x
Multiply equation (3) by 2 and subtract from (2),
u v v u
2 + +4 –2 = ex (cosy – siny) – 2ex(–siny – cosy )
x x x x
v
5 = ex [cosy – siny + 2siny + 2cosy]
x
v
5 = ex [3cosy + siny ]
x
v ex
 = 5 [ 3cosy + siny ] …. (5)
x
u v
We know, f '(z) = +i
x x
ex ex ..
= 5 [ cosy – 3siny ] + i 5 [ 3cosy+siny] . from (4) & (5)
ex
f '(z) = 5 [ cosy – 3siny + 3i cosy+ i siny]
ez
= [ 1 – 0 + 3i + 0 ]  x = z, y = 0
5
z
e
f '(z) = 5 (1+3i)
Integrate w.r.to z,
1+3i z
f (z) = e +c
5

Type : D

Example : 1

Determine the value of p such that


1 px
f (z) = 2 log (x2 + y2 ) + i tan1  y  …. (1)
 
becomes an analytic function

Gigatech Publishing House


Igniting Minds
Engineering Mathematics – III 6.13 Complex Variable

Solution :
We know f (z) = u +iv
1 px
u = log (x2 + y2 ) and v = i tan1 
2 y
u 1 1 
 = 2  x2 + y2  2x
x
x
= (x + y2 )
2 …. (2)
u 1 1 
= 2  x2 + y2  2y
y
y
= (x2 + y2 ) …. (3)
2
v 1 p = 2 y 2 2 p
=
x px y
2
(y + p x ) y
1+ y 
 
py
= y2 + p2x2 …. (4)
v 1  px y2  – px2 
= 2 – 2 =
y px  y  (y2
+ p 2 2
x )  y
1+ y 
 
px
= – 2 2 2 … (5)
(y + p x )
u v
We know , f (z) = u + iv , will be analytic if =
x y
y px
 y2 + x2 = – y2 + p2x2
Comparing, we get , –p = 1,p2 = 1
  p = –1, p2 = 1
  p = –1, p = 1
 p = –1
Secondly we know
u v
= –
y x
y py
 x2 + y2 –  y2 + p2x2
=
 
py
= – 2
y + p2x2
 p = –1, p2 = 1  p = –1, p = 1
 p = –1
Gigatech Publishing House
Igniting Minds
Engineering Mathematics – III 6.14 Complex Variable

Example : 2

Determine value of p for which the function


f(z) = rn cos n + i rn sin p .... (1)
Becomes an analytic function
Solution :
We know f (z) = u +iv
 u = rncos n , v = rn sin p
u
 = nrn–1 cos n
r
u
= – nrn sin n

v
= nrn–1 sin p
r
v
= prn cos p

We know, f (z) will be analytic function if
u 1 v
= r
r 
n–1 1
nr cos n = ( prn cos p)
r
 nrn–1 cos n = prn–1 cos p
 p=n
v 1 u
Also = –r
r 
1
 nrn–1 sin p = – r (– nrn–1 sin n)
nrn–1 sin p = nrn – 1 sinn 
 n=p
 p=n

Type : E

Example : 1

Determine the analytic function f(z) such that


R ( f '(z) ) = 3x2 – 4y – 3y2
Solution :
We know , if f(z) is an analytic function then f '(z) is also an analytic function

Gigatech Publishing House


Igniting Minds
Engineering Mathematics – III 6.15 Complex Variable

Let f '(z) = u + iv
u = 3x2 – 4y – 3y2 …. (1)
u
= 6x …. (2)
x
u
= –4 – 6y …. (3)
y
u v
But, = = 6x (By C – R equations)
x y
v
 = 6x
y
Integrate w.r.to y partially (x constant )
 v = 6xy + A(x) …. (4)
Differentiate w.r.to x partially ( y constant )
v
 = 6y + A'(x)
x
u
– = 6y + A'(x) ( C – R equations)
y
u
= – 6y – A'(x) …. (5)
y
From (3) & (5) , – 4 – 6y = – 6y – A'(x)
 –4 = – A'(x)
 A'(x) = 4
 A(x) = 4x + c
From (4), v = 6xy + 4x + c
 f '(z) = u + iv = (3x2 – 4y – 3y2 )+ i ( 6xy + 4x + c )
 f '(z) = 3z2 + 4 iz +ic ( x=z,y=0)
Integrate w.r. to z, we get
z3 z2
f (z) = 3  3  + 4i  2  + icz + c1
   
3 2
f (z) = z + 2iz + c1 z + c2

Example : 2

Prove that the analytic function with constant modulus is constant


Solution :
Let f (z) = u + iv …. (1)
u v u v
 = , = – …. (2)
x y y x
Given that f (z) has constant modulus
Gigatech Publishing House
Igniting Minds
Engineering Mathematics – III 6.16 Complex Variable

 | f (z) | = C
 | u + iv | = C
u2 + v2 = C
u2 + v2 = C2 …. (3)
Differentiate (3) w.r.to x partially
u v
2u + 2v = 0
x x
u v
 u +v = 0 …. (4)
x x
Differentiate (3) w.r.to y partially we get
u v
u +v = 0 …. (5)
y y
v u
–u +v = 0 …. (6)  (2)
x x
Multiply (4) by u and (6) by v and add
u v v u
u2 + uv – uv + v2 = 0
x x x x
u
 (u2 + v2 ) = 0
x
u
 = 0  u2 + v2  0
x
v
Similarly we can show = 0
y
u v
We know , f '(z) + i = 0 + i0 = 0
x x
 f (z) is a constant function

Example : 3

Find the condition under which the function


u = px3 + q x2 y + r xy2 +sy3 becomes a harmonic function
Solution :
u
 = 3px2 + 2qxy + ry2 +0
x
2u
= 6px + 2qy + 0
x2
u
= 0 + qx2 +2rxy + 3sy2
y
2u
= 0 + 2rx + 6sy
y2
Gigatech Publishing House
Igniting Minds
Engineering Mathematics – III 6.17 Complex Variable

We know that u will be harmonic if


2u 2u
+ = 0
x2 y2
 (6px + 2 qy) + (2rx + 6sy) = 0
(6px + 2 rx) + (2qy + 6sy) = 0
(3p + r) 2x + (q + 3s) 2y = 0
 3p + r = 0, q + 3s = 0
 r = –3p, q = –3s

Example : 4

If f(z) and f(z ) are analytic functions, then prove that f(z) is constant
Solution : Let f(z) = u + iv .... (1)
 f(z ) = u –iv .... (2)
As f(z) is an analytic function
u v
= …. (3)
x y
u v
= – …. (4)
y x
As f(z ) is an analytic function
u v
= – …. (5)
x y
u v
= …. (6)
y x
Adding (3) & (5) , we get
y
2 = 0
u
y
 = 0
u
Adding (4) & (6) , we get
v
2 = 0
x
v
 = 0
x
u v
But f ' (z) = +i
x x
f ' (z) = 0+i0=0
f ' (z) = 0
 f(z) is a constant function.
Gigatech Publishing House
Igniting Minds
Engineering Mathematics – III 6.18 Complex Variable

Example : 5

If f(z) is an analytic function of z, and f(z) = u +iv prove that


2 2
  +   | [ R f(z) ] | 2 = 2 |f '(z)|2
x2 y2
 
Solution :
f(z) = u + iv …. (1)
u v
 f '(z) = +i …. (2)
x x
 u  v2  u2  u2
2
|f '(z)| =   +   =   +  
2
…. (3)
 x  y  x  y
As f(z) is an analytic function, u & v must be harmonic functions
2u 2u 2 v 2 v
 + = 0, + = 0 …. (4)
x2 y2 x2 y2
2 2
 L.H.S. =  2 + 2 ( |R f(x) |2 )
x y 
2 2
=  2 + 2 u2
x y 
2 2 2 2
= 2 (u ) + (u )
x y2
  u    u 
= 2u  + 2u y 
x  x  y  
2
 u u u 2u u u
= 2u 2 + (2) + 2u 2 + (2)
x x x y y y
2 2 2 2
u u u u
= 2u  2 + 2  + 2   + 2  
x y  x y 
u 2 u 2
= 2    +   (  equation (4) )
 x  y 
= 2 |f '(z) |2 = R.H.S. (  equation (3) )

Example : 6

If f(z) is an analytic function show that


2 2
  +   | f (z) |2 = 4 | f '(z) |2
x2 y2
 
where f (z) = u+ iv
Solution :
We know | f (z) | = u2 + v2
| f (z) |2 = u2 + v2 …. (1)

Gigatech Publishing House


Igniting Minds
Engineering Mathematics – III 6.19 Complex Variable

u v v u
f ' (z) = +i = –i
x x y y
2 2 2 2
u + v v + u
|f ' (z) | = x x = y y
       
2 2
u v
 | f ' (z) |2 =   +   …. (2)
x x
u 2 v 2
=   +   …. (3)
y y
As u, v are harmonic functions, we know
2u 2u
+ = 0 …. (4)
x2 y2
2v 2v
+ = 0 …. (5)
x2 y2
2 2
  +   | f |2 2 2
 x2 y2 (z) =  2 + 2 (u2 + v2 )
  x y 
2 2 2 2 2 2
= 2 (u + v ) + (u + v )
x y2
  u v    u v 
=  2u
x  x
+ 2v  + 2u
x  y  y
+ 2v 
y 
2 2 2 2
2u u 2 v v 2u u 2v v
= 2u 2 + 2  + 2v 2 + 2   + 2u 2 + 2   + 2v 2 + 2  
x x x x y  y  y y 
2 2 2 2 2 2 2 2
    u u v v
= 2u  2+ 2 +2v 2+ 2 + 2   +2  +2  +2 
u u v v
x  y x  y x y x y
2 2 2 2
y v u v
= 2u(0) + 2v(0) + 2   +    + 2   +   
 x  x   y  y 
= 2 [ | f '(z) |2 + | f '(z)|2 ] ⸪ Refer (4) , (5)
= 4 | f '(z) |2 ⸪ Refer (2), (3)
= R.H.S.

Self-Assessment Exercise 6.1

Ex.1 Determine the harmonic conjugates of following functions u or v such that f(z) = u
+iv becomes an analytic function.
1. u = x2 – y2 – y Ans.: v = 2xy + x + c, f(z) = z2 + iz + c
2. v = y2 – x2 Ans. : u = 2xy + c, f(z)= –iz2 + c
2 3
3. u = 3xy – x Ans. : v = y3 3x2y + c, f(z) =–z3 + ic
x y i
4. v = x2 – y2 + 2 2 Ans. : u = –2xy + 2 2 + c, f(z) = iz2 + + c
x +y x +y z
5. u= e–x(x siny – y cosy) Ans. : v = e–x(ysiny + xcosy) + c, f(z) = iz e–z
Gigatech Publishing House
Igniting Minds
Engineering Mathematics – III 6.20 Complex Variable

y
6. u = log x2+y2 Ans. : v = tan–1  + c, f(z) = logz +c
x
2
7. u = e–2xy sin(x2 – y2) Ans. : v = –e–2xycos(x2 – y2) + c, f(z) = –i eiz + ci
8. u = x3y – xy3 Ans. : v = x4 + y4 – 6x2y2 + c , f(z) = z4 + e
y i
9. u= 2 2 Ans. : f(z) =
y +x z
10. u = ex[(x2–y2) cosy – 2xy siny] Ans. : f(z) = z2ez + ic
11. u = x sinx coshy – y cosx sinhy Ans. : f(z) = z sinz + c
3 2 2 2
12. u = x – 3xy + 3x – 3y + 1 Ans.: f(z) = z3 + 3z2 + 1 + ic
13. u = e2x(x cos2y – ysin2y) Ans. : f(z) = ze2z + ic
14. u = cosx coshy Ans. : f(z) = cosz + c
x
15. u = y + e cosy Ans. : f(z) = ez + ic – iz
16. v = ex siny Ans. : f(z) = ez + c
17. v = e–x(x cosy + y siny) Ans. : f(z) = 1 + ize–z
18. v = –sinx sinhy Ans. : f(z) = cosz +c
x–y 1 i
19. v = x2 + y2 Ans. : f(z) = + +ic
z z

Ans. : f(z) = z ez + c



20. v = e–x(x siny – y cosy)
21. u = r2 cos2 – r cos + 2 Ans. : u = –r2sin2 + rsin + c
22. u = ex(xcosy – ysiny) + 2sinx sinhy + x3 – 3xy2 + y Ans. : f(z) = zez+z3+2icosz–iz+e
23. u = – r3sin 3 Ans. : v = r3cos3 + c
24. u = r2 cos2 Ans. : v = r2sin2 + c
25. u = r3cos3 + rsin Ans. : v = r3sin3 – rcos + c
26. u = r3cos3 Ans. : v = r3sin3 + c
1 1 z2+1
27. v = r sin – sin Ans. : f(z) = z + =
r z z
28. v = rn sin n  Ans. : f(z) = rncosn = zn
29. u = x2 – y2 – 2xy – 2x + 3y Ans. : f(z) = z2 + iz2 – 2z – 3iz + 6i + c
1 1
30. u = r cos + r cos Ans. : f(z) = r sin – sin + c
r
31. u = e–xcosy + xy Ans. : v = y2 – x2 – 2e–x siny + c
Ex.2 Find the analytic function f(z) = u+iv if
x–y
32. u – v = 2 Ans. : f(z) = c – iz3
x + 4xy + y2
1 + 3i z
33. 2u + v = ex(cosy – siny) Ans. : f(z) =  e +c
 5 
Gigatech Publishing House
Igniting Minds
Engineering Mathematics – III 6.21 Complex Variable
–y
cosx+sinx–e 1 z
34. u–v= Ans. : f(z) = – cot   + c
2(cosx–coshy) 2 2
2sinx
35. u + v = e2y–e–2y–2cos2x

36. u – v = x3+ 3x2y – 3xy2 – y3 Ans. : f(z) = –iz3 + c


z2
37. u + v = r2(cos2 + sin2) Ans. : f(z) = + c
2
38. u – v = ex[(x – y)cosy – (x + y) siny] Ans. : f(z) = z ez +1
x–y 1
39. u + v = ex[cosy + siny] + x2 + y2 Ans. : f(z) = ez +
z+c
40. u + v = sinx coshy + cosx sinhy Ans. : f(z) = sinz + c
x–y 1
41. u + v = 3x + 3y + x2 + y2 Ans. : f(z) = 6z +
z +c
Ex.3 Find the values of a and b so that f(z)=u +iv is an analytic function
42. f(z) = (x + ay) + i (bx + cy) Ans. : a = –b, c = 1
43. f(z) = cos x (cosh y + a sinh y) + i sinx (cosh y + b sinh y) Ans. : a = b = –1
2 2
44. f(z) = (r cos 2) + i (r sin a) Ans. : a = 2
2 2 2 2
45. f(z) = (x + axy + by ) + i(px + qxy + y ) Ans. : a = 2, b = –1, p = –1, q = 2
ay
46. f(z) = log x2 + y2 + i tan 1  x  Ans. : a = 1
 
Ex.4 Various examples (miscellaneous examples)
47. Determine the analytic function f(z)= u + iv
if Im [f(z) ] = 12xy – 6y
48. Show that the analytic function with constant amplitude is constant
49. Find the orthogonal trajectories of the family of curves x3y –yx3 = c1
Ans.: x4 + y4–6x2y2 = c2
50. Determine the constant 'a' such that
u = eax cos 5y becomes harmonic function. Hence find harmonic conjugate of u
Ans. : a = 5, v = e5x sin 5y + c
51. Find the value of 'a' for which
f(z) = (ex cosy) + i (exsina y) is an analytic function
52. Find the orthogonal trajectories of the family of curves r2cos 2=c1
Ans. : r2sin2 = c2

Gigatech Publishing House


Igniting Minds
Engineering Mathematics – III 6.22 Complex Variable

The geometrical representation of a complex number (x + iy) by a point (x, y) on XOY


plane is called as Argands diagram.
Ex. : 1. The complex number (2 + 3i) is represented by (2, 3) in first quadrant.
2. The complex number 4 + 5i  (4, 5) is first quadrant
3. The complex number – 1 + i  (–1, 1) is second quadrant

We know w = f(z) = u + iv is called as a complex variable function


 f (z) = u + iv
 f (x + iy) = u + iv
This equation involves 4 variables x, y, u, v and hence f(x + iy) = u + iv cannot be
plotted on single frame. Hence the points (x, y) are plotted on XOY plane and points (u, v) are
plotted on another UOV plane with the help of w = f (z). This process is called as a
transformation of points (x, y) on XOY plane to points (u, v) on UOV plane.
Example :
Let w = z3 and z = x + iy
 w = (x + iy)3
= x3 + 3x2 (iy) + 3x (iy)2 + (iy)3
w = (x3 – 3xy2) + i (3x2y – y3)
 u = x3 – 3xy2
v = 3x2y – y3.
If x = 2, y = 3 then z = x + iy = 2 + 3i  (2, 3) lies in first quadrant of xy plane XOY or
Z plane.
and f(z) = w = u + iv
= [8 – 3(2) (9)] + i [3 (4) (3) – 27]
= [– 46] + i [9] ≡ (–46, 9) lies in second quadrant of UOV
Plane/W plane
 The point (2, 3) on XOY plane is transformed to the point (–46, 9) on UOV/W
plane by transformation w = z3.

Fig.: 6.1

Gigatech Publishing House


Igniting Minds
Engineering Mathematics – III 6.23 Complex Variable

Let c1, c2 be any two curves in XY Plane, which cuts each other at point P and makes an
angle of intersection as α.
Let the transformation w = f (z) transforms the curves c1, c2 in XY plane and suppose it
takes need shapes as curves c1ʹ , c2ʹ in UV Plane and let the angle between c1ʹ and c2ʹ be β.

Fig.: 6.2

If α = β, then the transformation w = f(z) is called as a conformal transformation /


mapping.

1. Translation : w = z + c, where c is complex constant.


2. Magnification : w = cz, where c is real constant.
1
3. Inversion and Reflection : w = .
z
4. Isogonal Transformation : w = z̄

az + b
(i) The transformation w = , where a, b, c, d are complex/real constants, is
cz + d
called as a bilinear Transformation.
(ii) The bilinear transformation can also be found by using the formula.
(w – w1)(w2 –w3) (z – z1)(z2 –z3)
(w – w3) (w2 – w1) = (z – z3) (z2 – z1)
– dw + b az + b
(iii) The mapping z = cw – a is inverse mapping of w = cz + d and also
called as bilinear transformation.
Gigatech Publishing House
Igniting Minds
Engineering Mathematics – III 6.24 Complex Variable

Illustrative Examples

Example : 1

Find a bilinear transformation w = f (z) which maps the points z = ∞, 1, i on Z plane to


the points w = 0, 1 , – i on W – plane respectively.
Solution :
Let the required bilinear is,
az + b
w = cz + d

 a z + 1
b
w = …(1)
c  z + d
b  b
 a + 1
b z
w = …(2)
c  + d 1
b b z
Put z = ∞, w = 0 in (2), we get,
 a + 0
b
0 =
c  + 0
b 
a
⸫ b = 0 …(3)

Put z = 1, w = 1 in (1), we get,


 a + 1
b 0+1 a
1 = = ⸪ =0
c
 +  d   + d
c b
b  b  b b
1
=
  + d
c
b b
c d
⸫ + = 1 …(4)
b b
Lastly put z = i, w = – i in (1), we get
 a (i) + 1
b
–i =
 c  i + d 
b b

Gigatech Publishing House


Igniting Minds
Engineering Mathematics – III 6.25 Complex Variable

 c  – i d = 1 ⸪
a
=0 …(5)
b  b b
d c
Solving (4), (5), we get   = 0, = 1
b b
⸫ Required bilinear transformation is
0+1 1
w = (1) z + 0 = z

1
⸫ w =
z

Example : 2

Find the bilinear transformation w = f(z) which maps the points z = – i, 0, 2 + i on


Z – plane to the points w = 0, – 2i, 4 on W – plane respectively.
Solution :
Let the required bilinear transformation be
(w – w1)(w2 –w3) (z – z1)(z2 –z3)
(w – w3) (w2 – w1) = (z – z3) (z2 – z1)
Given : w1 = 0, w2 = – 2i, w3 = 4
z1 = –i, z2 = 0, z3 = 2 + i
(w – 0)( – 2i –4) (z + i)(0 – 2 – i)
⸫ (w – 4) (– 2i – 0) = (z – 2 – i) (0 + i)
– 2w (i + 2) – (z + i) (2 + i)
– 2i (w – 4) = i(z – 2 –i)
w – (z + i)
=
w–4 (z – 2 – i)
wz – 2w – wi = – [wz + wi – 4z – 4i]
wz – 2w – wi = – wz – wi + 4z + 4i
wz – 2w + wz = 4z + 4i
2wz – 2w = 4 (z + i)
2w (z – 1) = 4 (z + i)
2 (z + i)
w =
(z – 1)

Example : 3

Find the bilinear transformation which maps z = 1, i, –1 on Z plane into the points w = i,
o, – i on W plane respectively.
Solution :
Let, z1 = 1, z2 = i, z3 = – 1

Gigatech Publishing House


Igniting Minds
Engineering Mathematics – III 6.26 Complex Variable

w1 = i, w2 = 0, w3 = – i
We know, the bilinear transformation is
(w – w1)(w2 –w3) (z – z1)(z2 –z3)
=
(w – w3) (w2 – w1) (z – z3) (z2 – z1)
(w – i)( 0 + i) (z – 1)(i + 1)
⸫ (w + i) (0 – i) = (z + 1) (i – 1)
w–i (z – 1) (i + 1) (i + 1)
⸫ w + i = – (z + 1) ⸱ (i – 1) (i + 1)
(z – 1) (1 + 2i + i2)
= – (z + 1) ⸱ (i2 – 1) 
 
(z – 1) 2i i (z – 1)
= – (z + 1) ⸱ (– 1 – 1) = (z + 1)
 
w–i i (z – 1)
⸫ =
w+i (z + 1)
N+D
By components and Dividends rule
ND
w–i+w+i iz  i + z + 1
=
wiw–i iz  i  z  1
2w iz  i + z + 1
=
2i iz  i  z  1
iz + i + i2z + 1 1 + iz
⸫ w = =
iz + i + z + 1 1 – iz

Example : 4

Find a bilinear transformation w = f (z) which maps z = 1, i, – 1 on z–plane to the points


w = 0, 1, ∞ on W plane respectively.
Solution :
Let the required bilinear transformation is
az + b
w = cz + d …(1)

 a z + 1
b
w = …(2)
c  z + d
b  b
We have, w = ∞, z = – 1
a (– 1) + 1
b
⸫ ∞ =
c (–1) + d
b b 

Gigatech Publishing House


Igniting Minds
Engineering Mathematics – III 6.27 Complex Variable

c d
– + 
1   b
b
⸫ ∞ =
– a  + 1
 b
c d
– + 
d b
0 =
– a  + 1
 b
c d
⸫ b = b …(3)

 a (1) + 1
b
Put z = 1, w = 0 in (2) 0 =
c  (1) + d
b  b
a
⸫ b = –1 …(4)

Lastly put z = i, w = 1 in (2)


 a i + 1
b –i+1 ... a = d
1 = c d = c
 i+    i +  c b b
b b b b
1–i
=
 c (1 +i)
b 
c 1–i (1 – i) (1 – i)
⸫ b = 1 + i = (1 + i) (1 – i)
1 – 2i + i2 – 2i
= 1 – i2 = 2 =–i
c d
 = –i =
b b
 Required Bilinear transformation is
–z+1 1–z 1 – z
w = – iz – i = – i (1 + z) = i 
1 + z
Example : 5

Find a bilinear transformation f(z) = w which transforms the points z = , i, 0 on z plane


to the points w = –1, – i, 1 on w plane respectively.
Solution :
Required Bilinear Transformation is
az + b
w = …(1)
cz + d
Gigatech Publishing House
Igniting Minds
Engineering Mathematics – III 6.28 Complex Variable

 a z + 1
b
w = …(2)
c  z + d
b  b
 a + 1
b z
w = …(3)
  + d 1
c
b b z
Put z = , w = –1 in (3),
 a + 0
b
–1 =
c  + 0
b 
 a  = –  c …(4)
b  b
Put z = 0, w = 1 in (2),
0+1
1 =
d
0 + b 
 
d
 b = 1 …(5)

Lastly put z = i, w = – i in (2)


 a i + 1
b
–i=
 c i + d
b b
 a i + 1
b ... d = 1, c =  a
i =
a b b b
– b i + 1
 
 a  i2 – i =  a  i + 1
b  b
a a
–b–bi = i+1

 a = – 1 …(6)
b 
 Required bilinear transformation is,
–z+1
w = z+1
1–z
=
1+z
Gigatech Publishing House
Igniting Minds
Engineering Mathematics – III 6.29 Complex Variable

Example : 6

Find a bilinear transformation w = f(z) which maps the points z = 0, – 1,  on z plane to


the points w = – 1, – (2 + i), i on w plane respectively.
Solution :
Required Bilinear Transformation is,
az + b
w = cz + d …(1)

 a z + 1
b
w = …(2)
c  z + d
b  b
a
 + 1
b z
w = …(3)
  + d 1
c
b b z
 a + 0
b
Put z = , w = i in (3), i =
c  + 0
b 
a  c
 b = i b …(4)

Put z = 0, w = – 1 in (2),
0+1
–1 = d
0+ 
b
d
 b = –1 …(5)

Lastly put, z = – 1, w = – (2 + i) in (2),


 a (– 1) + 1
b
– (2 + i) =
c (– 1) + d
b b
c
i   (– 1) + 1
b a c d
– (2 + i) = ⸪ b = i b & b = – 1
c  
– –1
b
c
i b – 1
 
– (2 + i) =
c + 1
b

Gigatech Publishing House


Igniting Minds
Engineering Mathematics – III 6.30 Complex Variable

c c
– (2 + i)  + 1 = i –1
b  b
c c
– (2 + i)   – i   = –1+2+i
b b
c
– b (2 + i + i) = (1 + i)
c
– b (2 + 2i) = (1 + i)
c
– 2 (1 + i) = (1 + i)
b

 c = –
1

a 1
= i   …(6)
b  2 b  2
 Required Bilinear Transformation is,
 a z + 1 1
i –2 + 1
b  
w = =
c  z + d 1
– 2 z + (– 1)
b  b  
iz + 2
= –z–2
iz – 2 
w = 
z+2
Example : 7

Find a bilinear transformation w = f(z) which maps points z = – 1, i , 1 on z plane to the


points w = 1, i , – 1 on w plane respectively.
Solution :
Let, z1 = – 1, z2 = i, z3 = 1
w1 = 1, w2 = i, z3 = – 1
We know bilinear transformation is,
(w – w1)(w2 –w3) (z – z1)(z2 –z3)
(w – w3) (w2 – w1) = (z1 – z3) (z2 – z1)
(w – 1)( i + 1) (z + 1)(i – 1)
⸫ (w + 1) (i – 1) = (z – 1) (i + 1)
w–1 (z + 1) (i – 1)2
⸫ = ⸱
w+1 (z – 1) (i + 1)2
z+1 i2 – 2i + 1
= z – 1 ⸱ i2 + 2i + 1
   
z + 1 z+1
= z – 1 (– 1) = –  z – 1
   
 (w – 1) (z – 1) = – (z + 1) (w + 1) = – [zw + z + w + 1]

Gigatech Publishing House


Igniting Minds
Engineering Mathematics – III 6.31 Complex Variable

Wz – w – z + 1 = – zw – z – w – 1
2zw = –2
1
w = –
z

Example : 8

Find a bilinear transformation which maps the points z = 0, 1,  on z plane to the points
w = –1, –i, 1 on w plane respectively.
Solution :
We know, the B.T. is
az + b
w = cz + d …(1)

a  z + 1
b
w = …(2)
 c z + d
b b
 +1
a
b z
w = …(3)
c  z + d 1
b  b z
Put z = , w = 1 in (3)
 a + 0
b
1 =
c  + 0
b 

  a = c  …(4)
b b
Put z = 0, w = – 1 in (2),
0+1
–1 = d
0 + b
 
d
 b = –1 …(5)

Put z = 1, w = – i in (2),
a + 1
b 
–i =
  (1) + d
a
b  b

Gigatech Publishing House


Igniting Minds
Engineering Mathematics – III 6.32 Complex Variable

a  + 1
b  a c d 
–i = ⸪ = b & b = – 1
a  – 1  b 
b 
a a
– b i + i – b = 1
 
a
– (i + 1) = 1–i
b
a 1–i
 –b = 1+i
a (i – 1) (i – 1)
b = (i + 1) × (i – 1)
i2 – 2i + 1 – 2i
= i2 –1 = –2 = i

a  = i
b 
 Required B.T. is
iz + 1
w =
iz – 1

Example : 9

Find a Bilinear Transformation w = f(z) which transforms the points z = – 2, 0, 2 on XY


plane to the points w = 0, i, – i on UV plane respectively.
Solution :
Let, z1 = – 2, z2 = 0, z3 = 2
w1 = 0, w2 = i, w3 = – i
Required B. T. is,
(w – w1)(w2 –w3) (z – z1)(z2 –z3)
(w – w3) (w2 – w1) = (z – z3) (z2 – z1)
(w – 0)( i + i) (z + 2)(0 – 2)
⸫ (w + i) (i – 0) = (z – 2) (0 + 2)
w (z + 2)
(2) = (–1)
w+i (z – 2)
2w (z – 2) = – (z + 2) (w + i)
2wz – 4w = – zw – zi – 2w – 2i
2zw – 4w + zw + 2w = – zi – 2i
3zw – 2w = – zi – 2i
w (3z – 2) = – i (z + 2)
i (z + 2)
 w =
(2 – 3z)
Gigatech Publishing House
Igniting Minds
Engineering Mathematics – III 6.33 Complex Variable

Example : 10

Show that the transformation W = coshz transforms the lines parallel to x–axis on XOY
plane to the hyperbola in UOV plane.
Solution :
Given transformation is
W = coshz
u + iv = cosh (x + iy) = cos [i (x + iy)]
= cos (ix – y)
= coshx sin y + i sinhx cosy
 u = coshx siny …(1)
v = sinhx cosy …(2)
Equation of a line parallel to x–axis is
y = b …(3)
Equations (1) & (2) changes to
u = coshx sinb
v = sinhx cosb
u
 coshx = sinb …(4)
v
sinhx = …(5)
cosb
Squaring and subtracting, we get
u2 v2
cosh2x – sinh2x = 2 –
sin b cos2b
u2 v2
 1 = 2 –
sin b cos2b
2 2
u v
 – = 1
sin2b cos2b
which is an equation of hyperbola in UOV plane.

Example : 11

Find the map of the circle |z| = 1 on XOY plane on W plane by the transformation
W = (1 + i) z.
Solution :
We know, the circle |z| = 1 is
x2 + y2 = 1 …(1)
The given transformation is
W = (1 + i) z
u + iv = (1 + i) (x + iy)
Gigatech Publishing House
Igniting Minds
Engineering Mathematics – III 6.34 Complex Variable

= x + iy + ix + i2y
= x + iy + ix – y
u + iv = (x – y) + i (x + y)
 u = x–y …(2)
v = x+y …(3)
Solving (2) and (3), we get
u+v v– u
x = 2 ,y = 2
 Equation (1) changes to,
2 2
u + v + v – u = 1
 2   2 
u2 + 2uv + v2 + v2 – 2uv + u2 = 4
2u2 + 2v2 = 4
 u2 + v2 = 2
which is a circle with centre (0, 0), rad = 2 units.

Example : 12

1
Show that the transformation w = z transform the circle |z – 2| = 2 in z–plane to the line
in w – plane.
Solution :
Given equation of circle on XOY plane is
|z – 2| = 2
|x + iy – 2| = 2
|(x – 2) + iy| = 2
 (x – 2)2 + y2 = 4
 x2 + y2 = 4x …(1)
1
Given transformation is w =
z
1
 z =
w
1 u – iv
x + iy = u + iv = (u + iv) (u – iv)
u – iv
= u2 +v2
u –v
= (u2 +v2)  + i u2 +v2 
   
u
 x =
u2 + v2
Gigatech Publishing House
Igniting Minds
Engineering Mathematics – III 6.35 Complex Variable

v
y = –
(u + v2)
2

 Equation (1) changes/transforms to


u2 v2  u 
(u + v ) (u + v2)2 = 4 u2 + v2
2 2 2+ 2

u2 + v2 4u
(u2 + v2)2 = (u2 + v2)
 4u = 1
1
 u = , a line on UV plane.
4

Example : 13

Find the image of |z| = 4 on XY plane to UV plane by the transformation. w = z + 1 – 4i


Solution :
The given curve on xy plane is
|z| = 4
 x + y2 = 16
2
…(1)
Given transformation is
w = z + 1 – 4i
u + iv = (x + iy) + 1 – 4i
= (x + 1) + i (y – 4)
 u = x + 1, v = y – 4
 x = u – 1, y = v + 4
 Equation (1) changes/transforms to
(u – 1)2 + (v + 4)2 = 16 …(2)
which is a circle on W plane with centre (1, –4) and radius 4 units.
 Transformation of circle on XY plane is a circle on UV plane.

Example : 14

Find the image of hyperbola 4x2 – 9y2 = 36 in XY plane on UV plane by using


transformation w = (1 – i) z.
Solution :
Given parabola is
4x2 – 9y2 = 36 …(1)
Given transform is
w = (1 – i) z
u + iv = (1 – i) (x + iy)
= x + iy – ix – i2y

Gigatech Publishing House


Igniting Minds
Engineering Mathematics – III 6.36 Complex Variable

u + iv = (x + y) + i (y – x)
 u = x+y& v = y–x
u+v u–v
y = 2 , x = 2
Equation (1) changes to
u–v 2 u+v 2
4 2  –9 2  = 36
   
4(u2 – 2uv + v2) – 9(u2 + 2uv + v2) = 144
4u2 – 8uv + 4v2 – 9u2 – 18uv – 9v2 = 144
5u2 – 5v2 – 26uv = 144
5u2 + 5v2 + 26uv + 144 = 0

Example : 15

z–i
Show that w = 1 – iz maps upper half of Z plane onto interior of unit circle in W plane.

Solution :
We know that upper half of Z plane is 1 st and 2nd quadrant.
 y > 0 …(1)
The given transformation is
z–i
w = 1 – iz
w – wiz – z + i = 0
– z (wi + 1) = – w – i = – (w + i)
w+i
 z = 1 + wi
u + iv + i
x + iy = 1 + i (u + iv)
u + i (v + 1)
= 1 + iu – v
[u + i(v + 1)] [(1 – v) – iu]
= ×
(1 – v) + iu [(1 – v) – iu]
u (1 – v) – iu2 + i(v + 1) (1 – v) – i2 u(v + 1)
= (1 – v)2 + u2
u (1 – v) – iu2 + i (1 – v2) + u(v + 1)
= (1 – v)2 + u2
(u – uv + uv + u) + i (1 – v2 – u2)
= (1 – v) 2 + u2
2 2
 2u   1 – v 2– u 2 
x + iy = 2 +i
(1 – v) + u  (1 – v) + u 
2

Gigatech Publishing House


Igniting Minds
Engineering Mathematics – III 6.37 Complex Variable
2 2
2u 1 – v –u
 x = 2 2 ,y = …(2)
(1 – u) + v (1 – u)2 + v2
But y > 0
1 – v2 – u2
 (1 – u)2 + v2 > 0

 1 – u2 – v2 > 0
 1 > u2 + v2
 u2 + v2 < 1
which is a interior of circle with centre (0, 0) and radius 01 unit.

Example : 16

i–z
Show that the transformation w = i + z transforms the area covered under |z| < 1 to the

area which is right half of UV plane W plane.


Solution :
The given curve is |z| < 1
 x2 + y2 < 1 …(1)
Given transformation is
i–z
w =
i+z
 w (i + z) = i–z
i (1 – w)
 z = (1 + w)
i (1– u – iv) v + i(1 – u)
 x + iy = 1 + u + iv = (1 + u) + iv
[v + i (1 – u)] [(1 + u) – iv]
=
[(1 + u) + iv] [(1 + u) – iv]
2v + i (1 – u2 – v2)
= (1 + u)2 + v2
2 2
 2v   1 – u 2– v 2 
= 2 + i
(1 + u) + v  (1 + u) + v 
2

2v 1 – u2 – v2
 x = 2
(1 + u) + v 2 ,y =
(1 + u)2 + v2 …(2)

 Equation (1) changes to


2
4v2 (1 – u2 – v2)
2 + 2 < 1
[(1 + u)2 + v2] [(1 + u) 2 + v2]
2 2
 4v2 + (1 – u2 – v2) < [(1 + u)2 + v2]

Gigatech Publishing House


Igniting Minds
Engineering Mathematics – III 6.38 Complex Variable

 4v2 + 1 + u4 + v4 – 2u2 + 2u2v2 – 2v2 < 1 + 4u + 6u2 + 4u3 + u4 + v4 + 2v2


+ 4v2u + 2u2v2
 0 < 4u + 8u2 + 4u3 + 4v2u
0 < 4u [1 + 2u + u2 +v2]
0 < (4u) [(1+ u)2+ v2]
 4u > 0 ⸪ (1 + u2) + v2 > 0
 u > 0
which is a right half of UV/ W plane.

Example : 17

Consider a region bounded by z = 1, y = 1 and y = 1 – z is XY plane. Determine the


transformation of this bounded region is UV plane by using the transformation w = z2
Solution :
The region bounded by x = 1, y = 1 and y = 1 – x is a triangle as shown in following
figure.
Given transformation is w = z2
 u + iv = (x + iy)2
= x2 + 2ixy + i2y2
= (x2 – y2) + i (2xy)
 u = x2 – y2, v = 2xy
(i) Boundary x = 1 :
we get, u = 1 – y2, v = 2y
v 2
 u = 1 – 2 Fig.: 6.3
 
v2
= 1– 4

v2 = – 4 (u – 1), a parabola symmetric on U axis.


which cuts Y axis at (0, 2), (0, – 2) and x axis at (1, 0).
(ii) Boundary y = 1 :
we get, u = x2 – 1, v = 2x
v 2
 u =   –1
2
v2
= 4 –1

 v2 = 4 (u + 1), a parabola symmetric on U axis.


which cuts Y axis at (0, 2), (0, –2) and x axis at (–1, 0).

Gigatech Publishing House


Igniting Minds
Engineering Mathematics – III 6.39 Complex Variable

(iii) Boundary y = 1 – x :
we get, u = x2 – (1 – x)2 = 2x – 1
v = 2xy
= 2x (1 – x )
u+1 (u + 1)
 v = 2  2  1 – 2 
  
1 – u
= (u + 1)  2 
 
1 – u2
=
2
 u2 = 1 – 2v, which is a parabola symmetric on U axis and
cuts V axis at (–1, 0) and (1, 0).
 New transformed region would be as shown below.

Fig.: 6.4

Self-Assessment Exercise 6.2

Ex. 1 Find the bilinear transformations w = f(z) which transforms/maps the following
points on z plane to w–plane respectively.
Sr. No. Points on z Points on w Answer
1
1. , i, 0 0, i,  w=–
z
i (1 – z)
2. 1, i, –1 0, 1,  w= 1+z

z + (i – 1)
3. 0, 1, 2 –1, i, 1 w = z – (i – 1)

(1 + iz)
4. 0, 1,  i, –1, – i w = – (z + i)

Gigatech Publishing House


Igniting Minds
Engineering Mathematics – III 6.40 Complex Variable

3z + 2i
5. 2, i, –2 1, i, –1 w=
6 + iz
z+1
6. 0, –1, i i, 0,  w= z–i

–i 5i – 3z
7. 0, –i, 2i 5i, , 3 w = 1 – iz

– 3i (z + 1)
8. – 1, 0, 1 0, i, 3i w=
(z – 3)
z + 1
9. 0, –i, –1 i, 1, 0 w = –i 
 z – 1
2i – 6z
10. 1, i, – 1 2, i, – 2 w = iz – 3

–1
11. , i, 0 0, i,  w= z

1 + (2i –1) z
12. 1, 0, – 1 i, 1,  w=
z+1
z–i
13. 0, 1,  –1, –i, 1 w=
z+i
1 –iz
14. 0, i,  0, 2 ,  w= 2

–2 i (z + 1)
15. –1, i, i +1 0, 2i, 1 – i w = 4z – 1 –5i

z–i
16. –1, 0, 1 1, i, –1 w=
iz – 1
1 + iz
17. 1, i, –1 i, 0, –i w = 1 – iz

iz + z + 1
18. –1, , i , i, 1 w= z+1
2i – 6z
19. 1, i, –1 2, i, –2 w = iz – 3

1
20. –1, i, 1 1, i, – 1 w=–
z
5+i 3z +2
21. 0, –1, i 2, ,  2  w= z+1
 
3i (1 + z)
22. –1, 0, 1 0, i, 3i w = (3 – z)

2 (z + i)
23. – i, 0, 2 + i 0, – 2i, 4 w= z–1

Gigatech Publishing House


Igniting Minds
Engineering Mathematics – III 6.41 Complex Variable

1+i 2 + zi
24. 1, 0, i , –2, –  2  w= z–1
 
i–z
25. 1, i, – 1 i, 0, – i w=
i+z
3z + 2i
26. 2, i, –2 1, i, – 1 w=
6 + iz
1 1 1
27. 0, 1, 2 1, 2 , 3 w=z+1

i (z + 2)
28. –2, 0, 2 0, i, – i w = 2 – 3z

29. Find the image of circle |z|= 2 on XY plane onto W plane by transformation w = z+3+4i.
Ans. : a circle (u – 3)2 + (v – 4)2 = 12 on UV plane.
30. Find the image of circle |z| = 3 on XY plane onto the plane W by transformation w = 4z.
Ans. : a circle with centre (0, 0) and rad = 12 units on UV plane.
31. Find the image of half plane x > 0 under the transformation w = i (z + 1).
Ans. : w = u + iv, u = –y, v = x + 1  x > 0  v > 1
32. Find the map of the square whose vertices are –1 – i, 1 – i, 1 + i, – 1 + i by the
transformation w = (1 + i) z.

Fig.: 6.5

Ans. : It is a square with vertices –2i, 2, –2, 2i


33. Find the image of vertices of a square whose vertices are 0, 1, 1 + i, i by using
w = (1 – i) z.
Ans. : Now vertices are 0, 1 – i, 2, 1 + i

Fig.: 6.6

Gigatech Publishing House


Igniting Minds
Engineering Mathematics – III 6.42 Complex Variable

34. Find the image of the region y > 1 under the transformation w = (1 – i) z
Ans. : u + v > 2
35. Show that the straight line y = x on XY plane maps to a circle on UV plane through the
z–1
transformation w = z + 1 . Ans. : u2 + v2 + 2v + 1 = 0  (u – 0)2 + (v + 1)2 = 2
2z + 3
36. Show that the transformation w = z – 4 transforms the circle x2 + y2 = 4x on XY plane
to the line 4u + 3 = 0 on UV plane.
37. Find the map of a circle |z –i| = 1 on XY plane onto the w–plane by using transformation
1 1
w=z. Ans. : v = –2 , a straight line in UV plane.

The Cauchys Integral theorem is one of the very important tool in evaluation of
complex integrals and plays very important role in complex function theory. The complicated
complex integrals can be easily evaluated with the help of Cauchys Integral formula and
theory of residues.
Simple Curve : A curve having no self–intersections is called as a simple curve.
Closed Curve : A curve whose end points coincides is called as a closed curve.
Simple Closed Curve : A curve having no self–intersections and whose end points coincides
is called as a simple closed curve and also called as contour.
Examples : Ellipse, circle, square, rectangle, triangle are examples of closed curves/contours.

Let f(z) be a continuous function of z defined at all points of a curve c from point A to
B, where z = x + iy and dz = dx + i dy.
B

The integral A  F(z) dz is called as a line integral or complex integral of f(z) along path
C
C from point A to point B.
Note : 1. We know, f(z) = u +iv, dz = dx + i dy.

2.  F(z) dz =  (u + iv) (dx + i dy) =  (udx – vdy) + i (vdx + udy)


C C C

3. The evaluation of complex integral depends upon the path of curve C


from point A to point B.
4. x + iy  (x, y)
5. The curve c is called as a path of integration.

6. If c is a closed curve then line integral = o F(z) dz


C

Gigatech Publishing House


Igniting Minds
Engineering Mathematics – III 6.43 Complex Variable

Illustrative Examples

Example : 1

Evaluate  F(z) dz, where f(z) = x + iy2 along (i) a line y = x from (0, 0) to (1, 1), (ii) a
C
parabola y = x2 from (0, 0) to (1, 1).
Solution : Given f(z) = x + iy2
We know, z = x + iy
 dz = dx + idy

Fig.: 6.7

  f(z) dz =  (x + iy2) (dx + idy)


C C

=  xdx + xi dy + i y2dx + i2y2dy


C

=  (xdx – y2dy) + i  (xdy +y2dx) …(1)


C C

(i) along a line y = x from (0, 0) to (1, 1) :


(1,1) (1,1)
 f(z) dz =  2
(xdx – y dy) + i  (xdy +y2dx) from (1)
C (0, 0) (0, 0)
(1,1) (1,1)
=  (xdx – y2dy) + i  (xdx +y2dy) (⸪ y = x, dy = dx)
(0, 0) (0, 0)
2 1 3 1 2 1 3 1
= x  – y  + i   + i y 
x
 2 0  3 0  2  0  3  0
1 1 i i
= 2 (1 – 0) – 3 (1 – 0) + 2 (1 – 0) + 3 (1 – 0)
1 1 i i 1 5
= 2 – 3 + 2 + 3 = 6 + i 6

Gigatech Publishing House


Igniting Minds
Engineering Mathematics – III 6.44 Complex Variable

(ii) along a parabola y = x2 from (0, 0) to (1, 1) :


y = x2
 dy = 2x dx

  f(z) dz =  (xdx – y2dy) + i  (xdy +y2dx), from (1)


C C C

=  (xdx – y2dy) + i  (x 2x dx + x4dx)


C C
2 1 3 1 3 1 5 1
= x  – y  + i 2x  + i x 
 2 0  3 0  3 0  5 0
1 1 2i i
= 2 (1 – 0) – 3 (1 – 0) + 3 (1 – 0) + 5 (1 – 0)
1 1 2i i 1 2 1
= 2 – 3 + 3 + 5 = 6 + i 3 + 5
1 13
= + i
6 15
Note : 1. In above example, though f(z) was same but the paths of integration in (i) and
(ii) were different and hence the evaluation of  f(z) dz are also different.
2. The evaluation of integral depends upon the path, if f(z) is not analytic
function of z.

Example : 2

Evaluate  (x2 – y2 + i2xy) dz along


C

(i) a line y = x from (0, 0) to (1, 1) (ii) a parabola y2 = x from (0, 0) to (1, 1).
Solution :

 f(z) dz =  (x2 – y2 + i2xy) (dx + i dy)


C C

=  [(x2 – y2) dx – 2xy dy] + i  [2xydx + (x2 – y2) dy] …(1)


C C

Fig.: 6.8

Gigatech Publishing House


Igniting Minds
Engineering Mathematics – III 6.45 Complex Variable

(i) along the line y = x from (0, 0) to (1, 1) :


y = x,  dy dx
 from (1),
(1,1) (1,1)

 f(z) dz =  2 2
[(x – x ) dx – 2xxdx] + i  [2yydy + (y2 – y2) dy]
C (0, 0) (0, 0)
x3 1  y 1
3

= 0–2  +i 2  +0
 3 0   3 0 
2 2
= – (1 – 0) + i (1 – 0)
3 3
2
= 3 (i – 1)

(ii) along the parabola y2 = x from (0, 0) to (1, 1) :


x = y2
 dx = 2y dy
From (1),

  f (z) dz =  [(x2 – x)dx – 2y2ydy)] + i  [(2y2y2ydy + (y4 – y2) dy]


C C C
3 2 4 5 5 3 (1,1)
x x y y y y
=  3 – 2 – 2  4  + i 4 5  + i  5  – i  3  
          (0,0)
1 1 1 4i
= 3 (1 – 0) – 2 (1 – 0) – 2 (1 – 0) + 5 (1 – 0)
i i
+ (1 – 0) – (1 – 0)
5 3
1 1 1 4i i i
= 3 – 2 –2+ 5 +5–3

= 2 – 3 – 3 + i 12 + 3 – 5
 6   15 
4 10
= – 6 + i 15
 
2 2i
= –3+ 3
2 2
= (– 1 + i) = (1 – i)
3 3
Note: 1. In above example no. (02)
(i) f(z) was same.
(ii) Paths (i) and (ii) were different.
(iii) Evaluations along both paths gives same evaluation.

Gigatech Publishing House


Igniting Minds
Engineering Mathematics – III 6.46 Complex Variable

2. Evaluations on both paths (i) and (ii) gave same answer because f(z) = (x2 – y2)
+ i2xy is an analytic function.

3. Evaluation of  F(z) dz is independent of paths if f (z) is an analytic function.


C

Example : 3

Evaluate  2
z̄ dz, where c is a curve along a line y = 2x from (1, 2) to (3, 6)
C

Solution :
We know, z̄ = x – iy
2
z̄ = (x – iy)2 = x2 + (iy)2 – 2ixy
= (x2 – y2) + i (–2xy)

  2
z̄ dz =  [(x2 – y2) + i (–2xy)] [dx + idy]
C
C
(3, 6)
=  [(x2 – 4x2) + i (–2x2x)] [dx + i2dx] ⸪ y = 2x,
(1, 2)
⸫ dy = 2dx
3
= [ –3x2 – 4ix2] [1 + 2i] dx
1
3 3 3
3
x x
= (1 + 2i)  (–3x – 4ix ) dx = – (1 + 2i) 3 3 + 4i  3 
2 2

1
  1
3
4i 4i
– (1 + 2i) x3 + 3 x3 = – (1 + 2i) 1 + 3  [x3 ]1
3
=
  1
 
4i
= – (1 + 2i) 1 + 3  (27 –1)
 
4i
= –26 (1 + 2i) 1 + 
 3

Example : 4
2+i

Evaluate  [2x + iy + 1] dz along the path x = t + 1, y = 2t2 – 1


1– i

Solution :
Given path is x = t + 1 and y = 2t2 – 1
 dx = dt and dy = 4tdt
Limits : Lower Limit :
x = t + 1 and x = 1  t = 0
Gigatech Publishing House
Igniting Minds
Engineering Mathematics – III 6.47 Complex Variable

also, y = 2t2 – 1 and y = –1  t = 0


Upper Limit :
x = t + 1 and x = 2
 t+1 = 2 t=1
Also, y = 2t2 – 1, y = 1
 2t2 – 1 = 1 t=1
2+i 2+i

  [2x + iy + 1] dz =  (2x + iy +1) (dx + idy)


1– i 1– i
1

=  [2 (t + 1) + i (2t2 – 1) + 1] [dt + i4  dt]


t=0
1

=  [2t + 2 + 2it2 – i + 1] [1 + 4it] dt


0
1

=  [2t + 8it2 + 3 + 12it + 2it2 – 8t3 – i + 4t] dt


0
1

=  [6t + 10it2 + 3 + 12it – 8t3 – i] dt


0
2 3 2 4 1
t t t t
= 6 + 10i   + 3t + 12i   – 8   – it
 2 3   2 4 0
1
10i
= 3t2 + 3 t3 + 3t + 6it2 – 2t4 – it
 0

= 3 + 10i + 3 + 6i – 2 – i
 3 
10
= 4 + i  3 + 5
 
25
= 4+ i
3
Example : 5
2 + 3i

Evaluate  (z2 + z) dz along a straight line.


1– i

Solution :
The integral path is a line from point (1, – 1) to (2, 3).
 Equation of a line through these points is
y2 – y1
y – y1 = x – x  (x – x1)
 2 1

Gigatech Publishing House


Igniting Minds
Engineering Mathematics – III 6.48 Complex Variable

y+1 = 3 + 1 (x – 1)
 2 – 1
y+1 = 4 (x – 1) = 4x – 4
y = 4x – 5
 dy = 4 dx
2 + 3i (2, 3)

  2
(z + z) dz =  [(x + iy)2 + (x + iy)] (dx + i dy)
1– i (1, – i)
(2, 3)

=  [(x2 – y2 + 2ixy) + (x + iy)] (dx + i dy)


(1, – i)
(2, 3)

=  (x2 – y2 + 2xiy + x + iy)] (dx + i dy)


(1, – i)
(2, 3)

=  [x2 – (4x – 5)2 + 2ix (4x – 5) + i (4x – 5) + x)] (dx + 4i dx)


(1, – i)
(2, 3)

= (1 + 4i)  [x2 – (4x – 5)2 + 8ix2 – 10ix + 4ix – 5i +x] dx


(1, – i)
2

= (1 + 4i)  [x2 (1 + 8i) – 6ix – 5i + x – (4x –5)2] dx


1
3 2
x x2 x2 (4x – 5)3
= (1 + 4i) (1 + 8i)  3  – 6i  2  – 5ix + 2 – 12 
     1
1 + 8i 1 1
= (1 + 4i)  (8 – 1) – 3i (4 – 1) – 5i (2 – 1) + (4 – 1) – (27 – (–1))
 3  2 12 
1 + 8i 3 28
= (1 + 4i)  3  (7) – 9i – 5i + 2 – 12 
  
7 3 28 56i 28 + 18 – 28 56i – 42i
= (1 + 4i) 3 + 2 – 12 + 3 – 14i = (1 + 4i)  +
   12   3 
3 14i 1
= (1 + 4i)  + = [– 103 + 64i]
2 3  6

Example : 6

Evaluate  z̄ dz, where c is a straight line from (0, 0) to (2, 0) to (3, 3)


C

Solution :
We know,
z̄ = x – iy
dz = dx + i dy

Gigatech Publishing House


Igniting Minds
Engineering Mathematics – III 6.49 Complex Variable

(i) Let O  (0, 0), A  (2, 0), B  (3, 3)


 Equation of line OA is y = 0,  dy = 0.
 Equation of line AB is y = 3 (x – 2) = 3x – 6,
 dy = 3 dx.

  z̄ dz =  z̄ dz
C O–A–B

Fig.: 6.9
=  z̄ dz +  z̄ dz
OA AB
(2, 0) (3, 3)

=  (x – iy) (dx + idy) +  (x – iy) (dx + idy)


(0, 0) (2, 0)
2 (3, 3)

=  (x – 0) (dx + 0) +  [x – 3xi – 6i] [dx + i 3dx]


0 (2, 0)
2 3

=  x dx +  (1 + 3i) [x – 3xi – 6i] dx


0 2
2 2 2 2 3
x x x
=   + (1 + 3i)  – 3i   + 6ix
 2 0 2 2 2
1 1 – 3i
= [4 – 0] + [1 + 3i]  (9 – 4) + 6i (3 – 2)
2  2  
1 5 15i
= (4) + (1 + 3i)  – + 6i
2 2 2 
5 3i 5  3i
= 2 + (1 + 3i) 2 – 2  = 2 + (1 + 3i)  2  = 9 + 6i
   
Example : 7

dz
Evaluate the integral  z – a , where c is the circle with centre at (a, 0) and radius r units.
C

Solution :
We know the circle with centre at (a, 0) and radius r units is,
(x – a)2 + (y – 0)2 = r2
 |z – a| = r
 (z – a) = rei 0    2
 z = a + rei, ⸪ a, r are constants
dz = riei d
2
dz ri ei d
  z–a =  r ei
=  i d
C C 0

= i (2 – 0) = 2i
Gigatech Publishing House
Igniting Minds
Engineering Mathematics – III 6.50 Complex Variable

Example : 8

Evaluate  (z – z2) dz, where c is the upper half of a circle |z – 2| = 3.


C

Solution :
|z – 2| = 3  modulus of (z – 2) is 3
Let  be amplitude of (z – 2), 0    

 z – 2 = 3ei, 0     (⸪ upper half of circle)


This is a circle having centre (2, 0), rad = 3
 z = 2 + 3ei , z2 = (2 + 3ei)2
⸫ z – z2 = (2 + 3ei) – (2 + 3ei)2
= – 2 – 9ei – 9e2i
dz = 3i ei d

  (z – z ) dz
2
= –  [2 + 9ei + 9e2i] d (3i ei)
Fig.: 6.10
C 0

= – 3i  (2ei + 9e2i + 9e3i) d


0

2ei 9e2i 9e3i 


= – 3i  2 + 2i + 3i 
 0
= 30

If f(z) is an analytic function and if f(z) is a continuous at each point on and within a
 f(z) dz = 0.
closed curve c, then O
C

Illustrative Examples

Example : 1

Verify Cauchys Integral theorem for f(z) = z2, where c is a closed curve along the sides
of an rectangle whose vertices are (0, 0), (2, 0), (2, 2), (0, 2).
Solution :
Given f(z) = z2, which is analytic function on and within a closed curve.
To prove that,  z2 dz =
O  f(z) dz = 0
O
C C

The closed curve C is along the sides OA – AB – BD – DO as shown in figure, whose


equations are respectively y = 0, x = 2, y = 2, x = 0.
Gigatech Publishing House
Igniting Minds
Engineering Mathematics – III 6.51 Complex Variable

  f(z) dz = O z2 dz
O
C C

 (x + iy)2 (dx + idy)


= O
C

 (x2 – y2 + 2ixy) (dx + idy)


= O
C

=  +  +  +  Fig.: 6.11
OA AB BD DO
2 2 0 0

=  x dx +  (4 – y
2 2
+ 4iy) idy +  (x – 4 + 4ix) dx +  – y2idy
2

0 0 2 2
3 2 3 0 0
1 2 y x y3
= (x3) + i 4y – + 2iy2 +  – 4x + 2ix2 – i  
3 0  3 0  3 2  3  2
1 8 8 i
= (8 – 0) + i 8 – + 8i + – + 8 – 8i – (– 8)
3  3   3  3
8 8i 8 8i
= + 8i – + 8i2 – + 8 – 8i +
3 3 3 3
f(z) dz = 0
 Cauchys Integral theorem is verified.

Example : 2

Verify Cauchys Integral theorem for f(z) = z, where closed curve is along the sides of a
triangle whose vertices are (0, 0), (4, 0), (4, 4).
Solution :
Given f(z) = z is an analytic function on and within a closed curve C, as drawn in
following figure as O – A – B – O.
The equations of sides of triangle OA,
OB, BO are respectively y = 0, x = 4 and x = y.
  f(z) dz =
O  z dz
O
C C

=  (x + iy) (dx + idy)


O
C

  
= OA + AB + BO Fig.: 6.12
( y=0
dy =0 ) ( x=4
dx = 0 ) ( x=y
dx = dy )
=  xdx +  (4 + iy) idy+  (y + iy) (dy + idy)
4 4 0

=  xdx + i  (4 + iy) dy + (1 + i)  (y + iy) dy


0 0 4

Gigatech Publishing House


Igniting Minds
Engineering Mathematics – III 6.52 Complex Variable
2 4 2 4 2 2 0
= x  + i 4y + i y  + (1 + i) y + i y 
 2 0  2 0 2 2 4
1
= 2 (16 – 0) + i[16 + 8i] + (1 + i) [(0) – (8 + 8i)]
= 8 + 16i – 8 – [8 + 8i + 8i – 8]
= 8 + 16i – 8 – 8 – 8i – 8i + 8
 f(z) dz =
O 0
C

Cauchy Integral theorem is verified.

Example : 3

Verify Cauchys Integral Theorem for f(z) = z – 1, over the closed curve c along |z – 2| = 4.
Solution : As |z – 2| = 4
 z – 2 = 4ei
 z = 2 + 4ei
dz = 4i ei d
2 2

  f(z) dz =
O  (z – 1) dz =  (2 + 4 ei – 1) 4iei d
C 0 0
2

= 4i  [ei + 4e2i] d
0
2
ei e2i
= 4i  +4
i 2i  0
e2i – 1 2 4i
4i  + e – 1]
 i  i [
=

= 4 [(cos2 + i sin2 –1) + 2 (cos4 + isin4 – 1)]
= 4 [(1 + 0 – 1) +2 (1 + 0 – 1)]
= 0
  f(z) dz =
O 0
C

 Cauchy Integral theorem is verified.



If f(x) is an analytic function inside and on a closed curve c enclosing a simply
connected region R and if point ‘a’ lies in the interior of R then,
f(z)
(i)  (z – a) dz = 2i f(a)
O
C

f(z) 2i
(ii) 
O
(z – a)2 dz = f '(a)
C 1
Gigatech Publishing House
Igniting Minds
Engineering Mathematics – III 6.53 Complex Variable

f(z) 2i
(iii) 
O
(z – a)3 dz = f (a)
C 2
f(z) 2i
(iv) 
O
(z – a)n dz = f(n –1)(a), where n  N, a natural number.
C n–1

where f (a) = [f (z)]z = a , – – – f  (a) = [f  (z)]z = a ,….

Illustrative Examples

Example : 1

z3 +1 

Evaluate the integral O
(z – 2)3
dz, where c is the closed curve |z| = 3.
C

Solution :
The function f(z) = z3 + 1 is an analytic on and within closed curve |z| = 3
The point a = 2 = (2, 0) lies inside the circle |z| = 3 ie x2 + y2 = 9, a circle whose centre
is (0, 0) and radius 3 units.
 By Cauchys Integral formula
f(z) 2i
O (z – a)3 dz = f (a)
C 2
z3 +1  2i
 
O
(z – 2)3
dz = [6z]z = 2
C 2
2i
= 2 (12) = 12  i

Example : 2

e4z 

Evaluate the integral O
(z + 1)4
dz, where c is the closed curve |z| = 2.
C

Solution :
Let f(z) = e4z, which is an analytic function on and within a closed curve |z| = 2
i.e. a circle x2 + y2 = 4, having centre at (0, 0) and radius 2 units.
 Here point a = –1  (–1, 0) lies within closed curve |z| = 2.
 By Cauchys Integral formula
f(z) 2i
O (z – a)4 dz = f (a) , where f(z) = e4z
C 3

f(z) = 4e4z

Gigatech Publishing House


Igniting Minds
Engineering Mathematics – III 6.54 Complex Variable
4z
e  dz = 2i (64 e4z)

O
(z + 1)4 z = –1 f(z) = 16e4z
C 3
2i –4
= 6 (64 e ) f(z) = 64e4z
64i –4
= e
3
64i
=
3e4

Example : 3

(z + 1)
 (z3 – 4z ) dz, where c is a closed curve |z + 2| = 1.5
Evaluate the integral O
C

Solution :
z+1 z+1 z+1
The function z3 – 4z = z (z2 – 4) = z (z – 2) (z + 2) has simple poles at z = 0, z = 2,

z = –2. Also we can say this function is not analytic at z = 0, z = 2, z = – 2


The given closed curve, is
|z + 2| = 1.5
|x + iy + 2| = 1.5
|(x + 2) + iy| = 1.5
2 2
 (x + 2) + y = 1.5
which is a circle whose centre (–2, 0) and radius 1.5 units.
 z = 0  (0, 0) lies outside circle
z = 2  (2, 0) lies outside circle
z = – 2  (–2, 0) lies inside circle
(z + 1) z+1
 O (z3 – 4z ) dz = O z (z – 2) (z + 2) dz
C C Fig.: 6.13
 z+1 
z (z – 2)
= O  (z + 2) dz , (Note this step carefully)
C
f (z) z+1
= O  (z + 2) dz, , where f(z) = z2 – 2z

= 2i f(a) = 2i f(– 2)


–2+1 1
= 2i  4 + 4  = 2i– 8
   
i
= –
4

Gigatech Publishing House


Igniting Minds
Engineering Mathematics – III 6.55 Complex Variable

Example : 4

z3
 (z2 – 9) dz, where c is a closed curve |z| = 2.
Evaluate the integral O
C

Solution :
z3 z3
The function z2 – 9 = (z + 3) (z – 3) , which has simple poles at z = 3, – 3.

The closed curve |z| = 2 is a circle with centre (0, 0) and radius 02 units.
The poles z = 3  (3, 0) and z = – 3  (–3, 0) lies outside the circle |z| = 2
 f (z) exists for all points on and inside |z| = 2
z3
where f(z) = (z – 3) (z + 3)
z3
O f(z) dz = O (z – 3) (z + 3)  dz = 0, by Cauchy Integral theorem.
C C

Example : 5

ez

Evaluate O  dz, where c is a closed curve |z + 1| = 1.
(z + 1) (z – 1)
C

Solution :
The closed curve is |z + 1| = 1 and (x + 1)2 + (y – 0)2 = 12, which is a closed curve
(circle) with centre at (–1,0) and radius 01 unit.
ez
The function (z + 1) (z – 1) is not analytic at z = –1, z = 1.

But z = – 1  (– 1, 0),
which lies inside |z + 1| = 1
ez (ez/z – 1)
 O  (z + 1) (z – 1) = O (z + 1) dz Fig.: 6.14
C C
f(z) ez
= O z + 1 dz, where f(z) = z – 1

e–1  i
= 2i [f(– 1)] = 2i  =–
 –1 –1 e

Example : 6
2
z
Evaluate  (z – 2) (z – 3) dz, where c is a circle |z| = 4.
C

Solution :
f(z) = z2 is an analytic function within and on a close curve |z| = 4.

Gigatech Publishing House


Igniting Minds
Engineering Mathematics – III 6.56 Complex Variable

 z = 2  (2, 0), z  (3, 0) both lies in side |z| = 4.


z2 1 1
  (z – 2) (z – 3) dz = O z2 z – 3 –z – 2 dz
O
C C
z2 z2
= O  z – 3 dz – O z – 2 dz
C C

2i 2 2i 2
= 1 (z )z = 3 – 1 (z )z = 2
= 18i – 8i
= 10i

Example : 7

logz 1

Evaluate O
(z – 2)3
dz , where c is |z – 2| = .
2
C

Solution :
1 1 2
The closed curve |z – 2| = is a circle (x – 2)2 + (y – 0)2 =   , whose centre is (2, 0)
2 2
1
and radius 2 units and a = 2  (2, 0) which lies inside closed above.
f(z) 2i
 
O
(z – a)3
dz = f (a)
2
logz
 
O
(z – 2)3 dz
 f(z) = logz 
=
2i  1   f(z) = 1/z 
2 –z2z = 2  
f(z) = –1/z2
1 i
= i –  = –
 4 4

Example : 8

cos2z

Evaluate O dz , where c is |z| =1.
C z – 3
 4
Solution :
The closed curve |z| =1 is a circle x2 + y2 = 1 with centre at (0, 0) and radius = 1 unit.
 3.14
and a = = = 0.78  (0.78,0), which lies in |z| = 1.
4 4
f(z) 2i
 O (z – a)3 dz = f (a)
C 2

Gigatech Publishing House


Igniting Minds
Engineering Mathematics – III 6.57 Complex Variable

f(z) = cos2z
2
cos z 2i  f(z) = 2cosz(–sinz) 
 
O
z – 3
dz = 2 [–2cos 2z]z =  = –sin2z 
 
C 4
 4 f(z) = –2cos2z

= i –2cos2 = 0
  
Example : 9

12z2 + 7z + 1 1

Evaluate O
 z+1 
dz, where c is |z| = 2
C

Solution :
1 1
The closed curve c is |z| = 2 , a circle whose centre is (0, 0) and radius 2 units.
 
1
The point z = –1 lies outside |z| =
2
12z2 + 7z + 1 1
  is analytic at all points on and inside |z| = .
 z+1  2
2
12z + 7z + 1
   z + 1  dz = 0 , by Cauchy’s Integral theorem
O
C 

Example : 10

z+4
Evaluate  (z2 + 2z + 5) dz, where c is a closed curve |z + 1  i| = 2.
C

Solution :
The closed curve is |z + 1 – i| = 2
|x + iy + 1 – i| = 2
|(x + 1) + i (y – 1)| = 2
 (x + 1)2 + (y – 1)2 = 22
This is a circle whose centre (–1, 1) and radius 2 units
z2 + 2z + 5 = z2 + 2z + 1 + 4
= (z + 1)2 + (– 4i2) Fig.: 6.15
= (z + 1)2 – (2i2)
= (z + 1 + 2i) (z +1 – 2i)
= [z – (–1 – 2i)] [z – (–1 + 2i)]
 z = – 1 – 2i  (– 1, –2) lies outside of |z + 1 – i| = 2
& z = – 1 + 2i  (–1, 2) lies on the closed curve |z + 1 – i| = 2
z+4 (z + 4)
  z2 + 2z + 5 dz = O (z + 1 + 2i) (z + 1 – 2i) dz
O
C C

Gigatech Publishing House


Igniting Minds
Engineering Mathematics – III 6.58 Complex Variable

(z + 4)/(2 + 1 + 2i)
= 
O dz
C  [z – (– 1 + 2i)] 
f(z) z+4
=  z – (– 1+ 2i) dz, where f(z) = z + 1 + 2i
O
z+4 
= 2i 
z + 1 + 2i  z  –1 + 2i
–1 + 2i + 4 
= 2i 
 + 2i + 1 + 2i
–1
3 + 2i
= 2i 
 4i 
 (3 + 2i)
=
2

Example : 11

2z2 + z
  z2 – 1  dz, where c is |z – 1| = 1.5,
Evaluate O
C

Solution :
The closed curve is |z –1| = 1.5
|(x – 1) + iy| = 1.5
(x – 1)2 + (y – 0)2 = (1.5)2, which is a circle with centre (1, 0) and rad 1.5
units.
z = 1  (1, 0), lies inside the closed curve.
z = – 1  (– 1, 0) , lies outside the closed curve.
2z2 + z 2z2 + z
O z2 – 1 dz = O (z + 1) (z – 1) dz
C C
(2z2 + z/z + 1)
= O  (z – 1)
dz
f(z) 2z2 + z
= O  z – 1 dz, where f(z) =  z + 1 
= 2i [f(1)] , by Cauchy Integral Formula
2+1
= 2i  1 + 1
 
= 3i

Example : 12

6z – 1

Evaluate O dz, where c is |z| = 3.
C (z + 2) (z – 4)

Solution :
The closed curve |z| = 3 is x2 + y2 = 9, a circle curve centre (0, 0) and radius 03 units.

Gigatech Publishing House


Igniting Minds
Engineering Mathematics – III 6.59 Complex Variable

z = – 2  (–2, 0), which lies inside the circle x2 + y2 = 9


z = 4  (4, 0), which lies outside the circle x2 + y2 = 9
6z – 1 [(6z – 1)/z – 4]
 O (z + 2) (z – 4) dz = O z+2 dz
C
f(z) 6z – 1
= O  z + 2 dz, where f(z) = z – 4
= 2i [f(– 2)], by Cauchy Integral formula
–12 – 1
= 2i  –2 – 2 
 
13 13i
= 2i  4  =
  2

Example : 13

4z + 1
 z (z + 2) (z – 3) dz, where c is a closed curve |z| = 1.
Evaluate the integral O
C

Solution :
The closed curve c is |z| = 1, a circle x2 + y2 = 1, with centre (0, 0) and radius 01 unit.
z = 0  (0, 0), lies inside closed curve c
z = –2  (–2, 0), lies outside closed curve c
z = 3  (3, 0), lies outside closed curve c

4z + 1  dz = O [4z + 1/(z + 2)(z –3)] dz


 
O
z (z + 2) (z – 3) z
C C
f(z) 4z + 1
= 
O
z dz, where f(z) = 2
z –z–6
= 2i [f(0)]
0+1 i
= 2i 0 – 0 – 6 = –
  3

Example : 14

e4z

Evaluate O dz, where c is |z – 2| + |z + 2| = 6.
C z – i
 4
Solution :
The closed curve c is
|z – 2| + |z + 2| = 6
|(x – 2)+ iy| + |(x + 2) + iy| = 6

Gigatech Publishing House


Igniting Minds
Engineering Mathematics – III 6.60 Complex Variable

(x – 2)2 + y2 + (x + 2)2 + y2 = 6
2 2
 (x – 2) + y = 6 – (x + 2)2 + y2
Squaring, (x – 2)2 + y2 = 36 + [(x + 2)2 + y2 ] – 12 (x + 2)2 + y2
x2 – 4x + 4 + y2 = 36 + x2 + 4x + 4 + y2 – 12 (x + 2)2 + y2
– 8x – 36 = – 12 (x + 2)2 + y2
2x + 9 = 3 (x + 2)2 + y2
Squaring, 4x2 + 36x + 81 = 9 [x2 + 4x + 4 + y2]
= 9x2 + 36x + 36 + 9y2
45 = 5x2 + 9y2
x y2
2
 + = 1, an ellipse which cuts x – axis at (3, 0),
9 5
(–3, 0) and y – axis at (0 – 5 ), (0 + 5 )

Fig.: 6.16

 3.14
z = 4 i = 4 i = 0.78 i  (0, 0.78), which lies inside ellipse.
e4z f(z)
 
O

dz = O  
dz, where f(z) = e4z
C  C 
z – 4 i z – 4i

= 2i f  4 , by Cauchy Integral formula
  

=

2i e
4 ( )

4
= 2i e

Example : 15

sin (z)2 + cos (z)2



Evaluate O (z + 2)2 (z – 1) dz, where c is |z – 1| = 1
C

Solution :
The closed curve is |z – 1| = 1
|(x – 1) + iy| = 1
2 2
(x – 1) + y = 1,
which is a circle having centre at (1, 0) and rad = 1 unit Fig.: 6.17

Gigatech Publishing House


Igniting Minds
Engineering Mathematics – III 6.61 Complex Variable

z = – 2  (–2, 0) lies outside c


z = 1  (1, 0) lies inside c
sin (z2) + cos (z2) [sinz2 + cosz2/(z + 2)2]
  (z + 2)2 (z – 1) dz =
O 
O
(z – 1)
dz
C C

f(z)
= 
O
(z – 1) dz,
sin (z2) + cos (z2)
Where, f(z) =
(z + 2)2
= 2i f(1), by cauchy integral formula
sin + cos
= 2i 
 (1 + 2)2 
–1
= 2i  9 
 
2i
= –
9

Example : 16

z2

Evaluate O
(z – 4)
dz, where c is a unit circle |z| = 1.
C

Solution :
z2
The function f (z) = z – 4 is an analytic function everywhere inside and on |z| = 1 but its

pole/singular point z  (4, 0) lies outside of |z| = 1.


  f(z) =
O 0, by Cauchy’s Integral Theorem
C
z2 
 
O
z – 4
dz = 0
C

Example : 17

5z – 2
 (z + 1) dz, where c is a closed curve |z – 1| = 1.
Evaluate O
C

Solution :
5z – 2
The function f (z)= is analytic everywhere within and closed curve (x – 1)2 +
z+1
(y – 0)2 = 12, a circle with centre (1, 0) and radius 01 unit but its singular point z = – 1  (–1,
0) lies outside the closed curve. By Cauchys Integral Theorem.
5z – 2
 (z + 1) dz = 0
O
C

Gigatech Publishing House


Igniting Minds
Engineering Mathematics – III 6.62 Complex Variable

Example : 18

z3 x2 y2

Evaluate O
(z + 1) (z – 4i) 2 dz, where c is an ellipse + =1
9 4
C

Solution :
x2 y2
z = – 1, z = 4i are singular points of given function and 9 + 4 = 1

is an ellipse as shown in figure.


The point z = –1  (–1, 0) lies inside c and
z = 4i  (0, 4) lies outside c
z3 [z3/(z – 4i)2]
  (z + 1) (z – 4i)2 dz
O 
= O (z + 1) dz Fig.: 6.18
C C

f(z) z3

= O
z+1
dz, where f(z) =
(z – 4i)2
= 2i (f(– 1))
(– 1)3 2i 2i
= 2i (1 – 4i)2 = =
  15  8i 15 + 8i
Example : 19

z4 x2 y2

Evaluate O
(z + 4) (z – i) 2 dz, where c is an ellipse + =1
9 16
C

Solution :
The closed curve is an ellipse as drawn in figure.
the points z = – 4  (–4, 0), lies outside c and
z = i  (0, 1), lies inside c
z4 (z2/z + 4)
 
O
(z + 4) (z – i)2 dz =  (z – i)2 dz
O
C C

f(z)
= 
O
(z – i)2 dz
Fig.: 6.19

2i
= f (i) by Cauchy Integral Formula
1

z4 (z + 4)4z3 – z4(1)
Where f(z) =   f(z) =
z+4 (z + 4)2
(i + 4) 4i3 – i4
= 2i  (i + 4)2 
 
4 – 16i – 1 3 – 16i
= 2i i2 + 8i + 16 = 2i 
  15 + 8i

Gigatech Publishing House


Igniting Minds
Engineering Mathematics – III 6.63 Complex Variable

Example : 20

z2 + cos2z
 
Evaluate O
 3
dz, where c is a circle |z| = 1.
C
 z – 4 
  
Solution :
 3.14
The curve |z| = 1 is a circle with centre (0, 0), radius 01 unit and z = = = 0.78
4 4
 (0.78, 0) lies within c.
z2 + cos2z f(z)
 
O
 3
dz = 
O 3 dz
C 
z – 4 z – 
   4
2i
= [f (z)] 
2 z=
4

2i
= [2 – 2cos2z] 
2 z=
4

= i [2 – 0] = 2i

Example : 21

sin2z

Evaluate O 4 dz, where c is |z| = 2.
z + 
 3
Solution :
The curve |z| = 2 is a circle with centre (0, 0), rad = 2 units.
 3.14
z = – 3 = – 3 =  1.04  (1.04, 0) lies within |z| = 2.
sin2z f(z) 2i
 
O
 4 dz =   4 dz = [f (z)] 
z +  z +  3 z = –3
 3  3
2i 4i
= [– 8 cos2z]  =
6 z=– 3
3

Example : 22

z+3
Evaluate O  (z – 2) (z + 1)2 dz, where c is boundary of a square whose vertices are (0,
1.5), (0, –1.5), (1.5, 0), (– 1.5, 0)
Solution :
The closed curve c is as drawn in figure.
z = 2  (2, 0) lies outside c and z = – 1  (– 1, 0) lies inside C.
Gigatech Publishing House
Igniting Minds
Engineering Mathematics – III 6.64 Complex Variable

z+3 (z + 3/ z – 2)
 
O
(z – 2) (z + 1)2 dz = 
O
(z + 1)2 dz
C

f(z)
= 
O
(z + 1)2 dz
C

2i
= f (–1)
1
Fig.: 6.20
(z – 2) (1) – (z + 3) (1) z+3
= 2i  , where f(z) =
 (z – 2)2  z = –1 z2
–5
= 2i  9 
 
10i
= –
9

Let f(z) be some function of z.


(i) Zero : If f(z) = 0 at z = a then z = a is called as zero of f(z).
(ii) Pole : If f(z) =  at z = b then z = b is called as pole of f(z).
(iii) Multiple/Repeated Pole : If a pole is repeated for more than once then it is called
as multiple pole.
Residue of f(z)
(iv) at simple pole  = [(z – b) F(z)]z = b
z=b 
Residue of f(z)
at multiple pole  1 n–1
 dn – 1 (z – c)n F(z)
(v) z = c repeated for  =
n – 1 dz z = C
n times 
 Residue Theorem :
If f(z) is an analytic function within and on a simple closed curve c, except at finite
number of isolated singular points within (inside) closed curve c, then,

 F(z) dz =
O 2i [sum of residues of f(z) at all singular points within c]

= 2i [R1 + R2 + R3 + + Rn]

Gigatech Publishing House


Igniting Minds
Engineering Mathematics – III 6.65 Complex Variable

Illustrative Examples

Example : 1

dz
 z2 (z + 8) dz, where c is a closed curve |z| = 3.
Evaluate O

Solution :
The closed curve |z| = 3 is a circle x2 + y2 = 9.
1
Let, f(z) = z2 (z + 8)

 Poles/singular points of f(z) are z = 0, z = – 8, out of which only z = 0  (0, 0) lies


inside/within c and z = – 8  (– 8, 0) lies outside c.
Hence we should find residue of f(z) only at z = 0 which is repeated 02 times.
Residue of f(z) 1  d2–1  2 1 
R= = 2–1 z 2
at z = 0  2–1  dz  z (z + 8)z = 0

1  d  1 
=
1 dz z + 8

= – 1 2 =–
1
 (z + 8)  z = 0 64
 By Residue theorem,
dz
 z2(z + 8) dz = 2i(R)
O
C
dz 1
 O z2(z + 8) dz = 2i – 64 
C

i
= –
32

Example : 2

sin z2 + cos z2
  (z + 2)2 (z – 1)  dz, where c is a closed curve |z – 1| = 1.
Evaluate O
C

Solution :
The closed curve is |z – 1| = 1, which is a circle whose centre is (1, 0) and radius 01 unit.
sin z2 + cos z2
Let, f(z) = (z + 2)2 (z – 1)
 z = – 2 and z = 1 are poles/singular points of f(z) out of which z = – 2  (– 2, 0) lies
outside c and z = 1  (1, 0) lies within c. Hence we should find Residue of f(z) only at
z = 1.

Gigatech Publishing House


Igniting Minds
Engineering Mathematics – III 6.66 Complex Variable
2 2
Residue of f(z) (z – 1) sin z +2 cos z  
 at z = 1  =

  (z + 2) (z – 1)  z = 1
sin + cos 1
= (1 + 2)2 = –9

 By Residue theorem,
sin z2 + cos z2 1
 O   (z + 2)2 (z – 1)  dz = 2i – 9 
 
C

2i
= –
9

Example : 3

1 – 8z
 z (z – 2) (z + 3) dz, where c is a closed curve |z| = 4.
Evaluate O
C

Solution :
1 – 8z
Let f(z) = z (z – 2) (z + 3)
 z = 0, z = 2, z = –3 are singular points/poles of f(z).
The closed curve is a circle |z| = 4, i.e. x2 + y2 = 16.
The poles z = 0  (0, 0), z = 2  (2, 0), z = – 3  (–3, 0) lies within/inside closed curve
c. Hence we should find residues at all three simple poles.
1 – 8z
R1 = {Residue at z = 0} = z z (z – 2) (z + 3)
  z=0
1 1
= (0 – 2) (0 + 3) = – 6
1 – 8z
R2 = {Residue at z = 2} = (z – 2) z (z – 2) (z + 3)
  z=2
1 – 16 15 3
= 2(5) = – 10 = – 2
1 – 8z
R3 = {Residue at z = –3} = (z + 3) z (z – 2) (z + 3)
  z = –3
1 + 24 25 5
= ( –3) (– 3 – 2) = 15 = 3
 By Residue theorem,
1 – 8z

O  dz = 2i [R1 + R2 + R3]
C z (z – 2) (z + 3)
1 3 5
= 2i – – + 
 6 2 3
– 1– 9 + 10
= 2i  =0
 6 

Gigatech Publishing House


Igniting Minds
Engineering Mathematics – III 6.67 Complex Variable

Example : 4

4z + 1

Evaluate the integral O
z (z + 2) (z – 3)
dz, where c is |z| = 1.

Solution :
The closed curve c is |z| = 1 which is x2 + y2 = 1, a circle with centre (0, 0) and radius 01
unit.
4z + 1
Let, f(z) =
z (z + 2) (z – 3)
 z = 0, z = –2, z = 3 are poles of f(z) out of which z = 0 lies inside the c and z = –2, z =
3 lies outside c.
 We should find Residue of f(z) only at z = 0.
1 + 4z 1
 { Residue at z = 0} = z z (z + 2) (z – 3)  = –6
 z = 0
 By Residue theorem,
4z + 1
 F(z) dz =
O 
O
z (z + 2) (z – 3) dz
C C

1 i
= 2i – 6  = –
  3

Example : 5

4z + 11
 z3 – 16z dz, where c is |z – 1| = 2.
Evaluate : O
C

Solution :
The closed curve c is (x – 1)2 + (y – 0)2 = 22, a circle whose centre is (1, 0) and radius 2
units.
4z + 11
Let, f(z) =
z (z + 4) (z – 4)
Z = 0, z = – 4, z = 4 are poles of f(z), out of
which only z = 0 lies inside/within given closed curve.
4z + 11
 Residue of f(z) at z = 0} = z z (z – 4) (z + 4) 
  z = 0
Fig.: 6.21
11 11
= ( –4) (4) = – 16

 By Residue theorem,
4z + 11 11 
 z3 – 16z dz =
O 2i –
 16 
C

–
11i
=
 6 
Gigatech Publishing House
Igniting Minds
Engineering Mathematics – III 6.68 Complex Variable

Example : 6

dz

Evaluate O
(z2 + 4)3
, where c is the circle |z – i| = 2.
C

Solution :
The closed curve c is, |z – i| = 2
 |x + iy – i | = 2
 |x + i(y – 1) | = 2
 (x – 0)2 + (y – 1)2 = 22,
which is a circle with centre (0, 1) and radius 02 units.
We know, z2 + 4 = 0 Fig.: 6.22
 z2 = – 4
 z = ± 2i
 (z + 4)3 = 0, gives us six poles z = –2i, –2i, –2i, 2i, 2i, 2i out
2

of which z = –2i  (0, –2) lies outside c and z = 2i  (0, 2) lies inside closed curve c.
Residue of f(z) at z = +2i  1  d2 3 1 
 which is repeated 03 times  = 2 (z – 2i)
 2  dz (z + 2i)3
(z – 2i)3
z = 2i
1  d2  1 
= 2 dz2 (z + 2i)3z = 2i
1 d –4 
= 2 dz [–3 (z + 2i) ]z = 2i
1 –5  1 
= 2 [12 (z + 2i) ]z = 2i = 6 (z + 2i)5 z = 2i
1 – 3i
= 6 (4i)5 = 512
 
 By Cauchys Residue theorem.
dz
 (z2 + 4)3 = 2i [sum of residues]
O

–3i 3
= 2i 512 =
  256
Example : 7

e2z

Evaluate O
(z + 2)
2 , where c is a closed curve |z| = 4.
C

Solution :
The closed curve c is |z| = 4, which is a circle with centre (0, 0), radius 04 units.
2 2
We know, [z2 + 2] = [(z + i)(z – i)]
= (z + i)2 (z – i)2

Gigatech Publishing House


Igniting Minds
Engineering Mathematics – III 6.69 Complex Variable

 z = – i  (0, – ) = (0, – 3.14), lies inside c


z = i  (0, ) = (0, 3.14), lies inside c
 R1 = R [z = i]
1 d 2 e2z 
= (z – i) 
1  dz (z + i) 2
(z – i) 2
z = i
d  e2z 
=
dz (z + i)2 z = i
2 2z 2z
(z + i) 2e – e (2) (z + i)
=  
 (z + i)4 z = i
2z 2z
2(z + i) e – 2 e 
=  
 (z + i)3 z = i
2z
2 e [z + i – 1]
=  
 (z + i)3 z = i
2e2i [i + i – 1] 2e2i [2i – 1]
= 3 =
( i + i) ( 2i)3
2e2i [2i – 1]
=
83i3
2 (cos2 + i sin2) (2i – 1)
=
83 (– i)
[cos2 = 1, sin2 = 0, ⸪ ei = cos + isin]
1 i2
[i3 = – i, – = = i]
i q
2 [1 + 0]
= [i(2i – 1)]
83
1 1
= [2i2 – i] = 3 (– 2 – i)
43 4
(2 + i)
R1 = –
43
R2 = R [z = – i]
1 d 2 e2z 
= (z + i)
1 dz (z + i)2 (z –i)2z = –i

d  e2z 
= dz (z – i)2z = –i
2 2z 2z
(z – i) 2e – e 2 (z – i) 
=  
 (z – i)4 z = –i
2z 2z
2e (z – i) – 2e 
=  
 (z – i)3
z = –i

Gigatech Publishing House


Igniting Minds
Engineering Mathematics – III 6.70 Complex Variable
2z
2e (z – i – 1)
=  (z – i)3 
 z = –i
2e–2i (– i – i – 1)
=
(– i – i)3
2 [cos2 – i sin2] [– 2i – 1]
=
(– 2i)3
2 [1 – 0]( – 2i – 1) (2i + 1)
= 3 3 =
– 8 i 43(– i)
i (2i + 1)
=
43
(– 2 + i) (i – 2)
R2 = 3 =
4 43
e2z

 O 2 dz = 2i [R1 + R2]
C (z2 + 2)

2i 
– (2 + i) i – 2  2i
= +  = [– 2 – i + i – 2]
 43 43  43
2i 2i
= (– 4) = –
43 

Self-Assessment Exercise 6.3

E.1 : Evaluate  z2 dz, where c is a curve on straight, line joining.


C
2 + 11i
the points 0 and (2 + i). Ans. :  3 .

E.2 : Evaluate  z2 dz, where c is a straight line joining the points (1, 1) and (2, 4).
C
1
Ans. :  [86 + 18i] .
3

E.3 : Evaluate  z2 dz, where z = 2x, 0  x  1, z = 2 + i (x + 1), 1  x  2.


C
1 2 1 2

[Hint : Ans. : z 2
dz =  z dz +
2
z 2
dz =  (4x ) 2dx +  [2 + i (x – 1)]2 idx
2

C 0 1 0 1
1
= (2 + 11i)
3
2
E.4 : Evaluate  z2 dz, where z = x + ix2, 0  x  1 Ans. : (– 1 + i) .
C
3

Gigatech Publishing House


Igniting Minds
Engineering Mathematics – III 6.71 Complex Variable

E.5 : Evaluate  (z – z2) dz, where c is the lower half of a circle having its centre at (2, 0) and
C
radius 03 units. Ans. : –30.
2 x 5
E.6 : Evaluate  z̄ dz, along a line y = 2 from (0, 0) to (2, 1). Ans. : 3 (2 – i) .
C

dz
E.7 : Evaluate  , m  – 1 along a circle |z – a| = r Ans. : 0.
C
(z – a)m

E.8 : Evaluate  (x2 – iy) (dx + idy) from point (0, 0) to (1, 1) along a line, (i) y = x and (ii)
C
1 1
along a parabola y = x2 . Ans. : 6 (5 – i), 6 (5 + i) .

E.9 : Evaluate  (x – y + ix2) (dx + idy) from point (0, 0) to (1, 0) to (1, 1) along a straight
C
1 i 1
line. Ans. : 2 + 3 and 2 (–2 + i)
   
E.10 : Verify Cauchys Integral Theorem for f(z) = z + 8 over the curve c along sides of a
rectangle whose vertices are (–1, 0), (1, 0), (1, 1), (– 1, 1).
E.11 : Verify Cauchys Integral Theorem for f(z) = z2 + 1 over the curve c along the sides of a
triangle whose equations are y = 0, x = 0, y = 1 – x.
E.13 : Verify Cauchys Integral Theorem for f(z) = z over the curve c along |z| = 6.

E.14 : Evaluate  (z2 + 3z) dz along a circle |z| = 2 from point (2, 0) to (0, 2).
C
1
Ans. : – (44 + 8i) .
3
e5z 

E.15 : Evaluate the integral, O
(z + i)4
dz, where c is the closed curve |z| = 3.
C

125ie–5i
Ans. : 3 .
z+6
 (z3 – 4z) dz, where c is the closed curve |z –2| = 1.5.
E.16 : Evaluate the integral, O
C

i
Ans. :  2 
 
z–1
 z(z2 – 4) dz, where c is the closed curve |z| = 1.5.
E.17 : Evaluate the integral, O
 
C
1
Ans. : 2i 4 
 
2 2 2
z +z+1 x x
 z2 – 7z + 12 dz, where c is a closed curve 25 + 9 = 1
E.18 : Evaluate O
 
C

Gigatech Publishing House


Igniting Minds
Engineering Mathematics – III 6.72 Complex Variable
2
z +1 – i
E.19 : Evaluate O dz, where c is |z| = 1. Ans. : 2
C z (2z + 1)
z  dz, where c is |z – 2| = 1 .
E.20 : Evaluate O2 Ans. : 4i
C z – 3z + 2 2
e2z
 (z – 1) (z – 2) dz, where c is |z| = 3.
E.21 : Evaluate O Ans. : 2i [e4 – e2]
C
sin2z

E.22 : Evaluate O
 2
dz, where c is the circle |z| = 1. Ans. : i
C 
z – 6
 
z + 2 1
 z2 + 1 dz, where c is |z–i| = 2 .
E.23 : Evaluate O Ans. :  (i + 2)
C
e–2z
 (z – i)2 dz, where c is the curve along the sides of vertices at (–2, 2),
E.24 : Evaluate O
C

(–2, –2), (2, –2), (2, 2) . Ans. : – 4i [cos2 – i sin2]


cosz2
E.25 : Evaluate O  (z – 1) (z – 2) dz, where c is |z| = 3. Ans. : 4i
C
z4
E.26 : Evaluate O (z – 3i) dz, where c is |z –2| < 5 Ans. : 162 i
C
z2 + 1
E.27 : Evaluate O  z2 – 1 dz, where c is |z–1| =1. Ans. : 2i
C
2z2 + z
E.28 : Evaluate O   z2 – 1  dz, where c is |z–1| =1. Ans. : 3i
C
z dz 1
E.29 : Evaluate O (z – 1) (z – 2)2 where c is |z–2| = 2 . Ans. :  2i
C

4z2 + z i
  z2 – 1  dz, where c is |z–1| = 3.
E.30 : Evaluate O Ans. : 3
C

sin2z 4i

E.31 : Evaluate O dz, where c is |z| = 2. Ans. : 3
C z + 4
 3
z+2 8i

E.32 : Evaluate O
(z + 1)2 (z – 2)
dz, where c is |z–i| = 2. Ans. : –
9
C

cosz

E.33 : Evaluate O dz, where c is |z + 3 – i| = 3. Ans. : – i
C z (z + 2)
z–3
 2
E.34 : Evaluate O dz, where c is |z + 1 + i| = 2. Ans. : (2 + i)
C z + 2z + 5
2z2 + z + 5 x2 y2

E.35 : Evaluate O dz, where c is 4 + 9 = 1. Ans. : 14i
C z – 32
 2
ez 1 2i
 (z + 1) (z + 2) dz, where c is |z + 1| = 2
E.36 : Evaluate O Ans. : e
C

Gigatech Publishing House


Igniting Minds
Engineering Mathematics – III 6.73 Complex Variable

1 – 2z 3

E.37 : Evaluate O
z (z – 1) (z – 2)
dz, where c is |z| =
2
Ans. : 3i
C

 cotz dz, where c is the circle |z| = 5.


E.38 : Evaluate O Ans. : 6i
C

 log z dz, where c is the circle |z| = 1.


E.39 : Evaluate O Ans. : 2i
dz
 z2 , where c is |z| = 1.
E.40 : Evaluate O Ans. : 0
C
z4 – 1
 z2 (2z + 1) (z + 2) dz, where c is |z| = 1.
E.41 : Evaluate O Ans. : 0

 z2 + z + 1 dz, where c is z + 2  =1.


2z + 3 3i 2
E.42 : Evaluate O Ans. : (i 3 – 2)
3
sinhz

E.43 : Evaluate O dz, where c is |z – 3| = 2. Ans. : –2
z –  i
 2 
z+2
 z2 + 4 dz, where c is |z – 1| = 1.
E.44 : Evaluate O Ans. : 0
e3iz
 (z + )3 dz, where c is |z–| = 3.
E.45 : Evaluate O Ans. : 0
e2z
 (z – i) dz, where c is |z–1| + |z+1| = 1.
E.46 : Evaluate O


   

Method : We consider, in these cases, the closed curve c is |z| = 1, a unit circle having
centre at (0, 0) and radius 01 unit.
(i) Let z = ei, where  varies from 0 to 2.  dz = ieid.
dz dz
(ii)  d = i =
ie iz
1
z+
ei + e–i 2 1 1 1
(iii) We know, cos = = 2 = 2 z + z = 2z (z2 + 1)
2  
ei – e–i 1  1 1 2
(iv) We know, sin = = z– = (z – 1)
2i 2i  z  2iz
2

Hence we get   f(z) dz and can be evaluated by using, Cauchys


f(sin,cos) d = O
0 C

Residue Theorem.

Gigatech Publishing House


Igniting Minds
Engineering Mathematics – III 6.74 Complex Variable

Illustrative Examples

Example : 1
2

 13 + 5 cos
d
Evaluate
0

Solution : Let the closed curve be |z| = 1, a unit circle.


dz 1 1
Put, z = ei,  d = iz , cos = 2 z + z 
 
dz
2
d iz
  13 + 5 cos = OC 5 1
0 13 + 2 z + z 
 
1 dz 1 dz
= iO  5 1 = i C
O
C  13z + 5 z2 + 5
z 13 + 2 z + z
    2 2
z 
 z 
1 dz 2 dz
= iO  5 26 = 5i O 26
C z2 + z + 1 C 2
z +  z + 1
2 5  5 
2 dz
= O
5i C 
 1
…(1)
z + 5 (z + 5)
 
1
The integrand in (2) has simple poles at z = –5, z = – 5
1
 z = (–5, 0) which lies outside |z| = 1 and z  (– 5 , 0) lies inside |z| = 1. Hence we
1
should find Residue of integrand only at z = –
5
1 1 1
 Residue at z = –  = z +   

 5   5  1
  z + 5 (z + 5)z = –1
    5
=  1  =
1
=
5
z + 5z = –15 1 24
–5 + 5

By Cauchys Residue theorem, from (1)


2

 13 + 5 cos
d 2 dz
5i C 
= O
1
0 z + 5 (z + 5)
 
= 2i [Summation of Residues]

Gigatech Publishing House


Igniting Minds
Engineering Mathematics – III 6.75 Complex Variable

2  5 
= 2i 
5i 24

=
6

Example : 2
2

 5 + 4 sin
d
Evaluate
0

Solution :
Let c be a closed curve |z| = 1
dz
Let z = ei,  dz = ie i d, d = iz
1  1
sin = z–
2i  z
dz
2
d iz 1 dz
 (5 + 4 sin) = OC 1 =iO  2 1
z 5 + i z – z 
1
0 5 + 4 2i z  z  C
    
1 dz dz
= 
O =O 
i C 5zi + 2z2 – 2 C 2z2 + 5iz  2
z
 zi 
dz
= O  5iz
C
2z2 + 2 z  1
 
1 dz
=
2C

O
i
…(1)
(z + 2i) z + 2
 
i i
The integrand in (2) has poles at z =  2 , z =  2i, out of which z =  2 lies within |z| =
i
1 and z = 2i lies outside of |z| = 1. Hence we should find Residue of integrand only at z =  2
.
i i 1
Residue at z = –  = z +   
 2  2  i  
 (z + 2i) z + 2 z = – i
     2

=  1 
z + 2iz = –2i
1 1 2
= i = 3i = 3i
–2 + 2i 2

Gigatech Publishing House


Igniting Minds
Engineering Mathematics – III 6.76 Complex Variable

⸫ By Cauchys Residue theorem, from (1)


2

 5 + 4 sin
d 1 dz
= 2O i
0 (z + 2i) z + 
 2
2i  2  2
= =
2 3i 3

Example : 3
2
d 2
Show that  5  4 cos
=
3
0

Solution :
dz 1 1
Let z = ei, d = iz , cos = 2 z + z 
 
dz
2 2 2
d iz dz
⸫  5  4 cos =  4 1 = 
 2z2  2
0 0 5 z +  0 iz 5z
2 z  z 
1 dz 1 dz
iO  2z2 + 5z  2 =  2i O
=
5
C  2
C
z  z + 1
 2 
1 dz
=  O
2i
 1
…(1)
(z  2) z  2
 
1 1
⸫ The integral of (1) has poles at z = 2, z = out of which z = 2 lies outside z = lies
2 2
inside |z| = 1
1 1 1
Residue at z = 2 = z – 2  
1
    
 (z  2) z  2 z = 1
    2

=  1  = 1
1 2
= –3
z  2z = 12
–2
2
By Cauchy Residue theorem from (1).
2
d 1 dz
 5  4 cos
=  
O
2i C 1
0 (z  2) z  
 2
1
=  [2i (sum of residues)]
2i
1 2 2
=  (2i)   =
2i  3 3

Gigatech Publishing House


Igniting Minds
Engineering Mathematics – III 6.77 Complex Variable

Example : 4
2
d
Evaluate  17  8 cos
…(1)
0

Solution :
dz 1 1
Let z = ei, d = , cos = z + 
iz 2 z
dz
2
iz
= 
8
0 17  z + 
1
2 z
dz
= O  z 2 + 1 
C
iz 17  4 
  z 
dz 1 dz

= O
17z – 4z2 – 4 
= O
i C – 4z2 + 17z – 4
C
iz 
 z 
1 dz
= 
O
–4i C 17
z2 – z + 1
4
1 dz
=
–4i

O
1
…(2)
(z – 4) z – 4
 
1 1
This integrand has poles at z = 4, z = 4 out of which z = 4 lies outside |z| = 1 and z = 4
1
lies inside |z| = 1 and hence we should find residue only at z = 4
1 1 1
Residue at z = 4 = z – 4  
1
    
 (z  4) z  4 z = 1
    4
1 4
= 1 = – 15
 – 4
4 
 From equation (2) and equation (2)
2
d 1 dz
  17  8 cos  = 
O
–4i C 1
0 (z – 4) z – 4
 
1  –4
= 2i  
–4i  15
2
=
15

Gigatech Publishing House


Igniting Minds
Engineering Mathematics – III 6.78 Complex Variable

Example : 5
2
d
Evaluate  (5  3 cos)2
…(1)
0

Solution :
Let c be a closed curve |z| = 1.
dz 1 1 1 2
 Let z = ei, d = , cos = z +  = (z + 1)
iz 2 z 2z
dz
2
d iz 1 dz
  (5  3 cos)2 = OC 2 =
i

O 2 2
0 5  3 1 (z2 + 1) C
z 10z – 3z – 3
 2z   2z 
1 4z dz 1 4z dz
= 
O = O
i C [3z2 – 10z + 3]2 i C [(3z – 1) (z – 3)]2
1 4z dz
= 
O
i C (3z – 1)2 (z – 3)2
…(1)
1
 Integral in (1) has double pole at z = 3 which lies inside |z| = 1 and double pole z =
3, which lies outside |z| = 1.
1  d z – 12 4z
Residue at z = 3 =  3 


  dz
   1 2
9 z  (z  3)  1
2

   3 z = 3
4  d  z 
=  
9 dz (z – 3)2z = 1
3
4 (z – 3)2 (1) – 2 (z – 3) (z)
=
9  (z – 3)4  z=1 3
4 (z – 3) – 2z 4 – 3 – z
= 9  (z – 3)3  z = 1 = 9 (z – 3)3 z = 1
3 3
1 –10
4  –3–3  4  3 
=
9  
1 – 33
=
9  –512 
 3    27 
5
=
64
 From (1) and Residue theorem
2
d 1 2i + 5
 (5  3 cos)2
=
i  64 
0
5
=
32

Gigatech Publishing House


Igniting Minds
Engineering Mathematics – III 6.79 Complex Variable

Example : 6
2
sin2
Evaluate  5 + 4 cos
d
0

Solution :
dz 1
Let z = ei, d = iz ,  z2 = e2i, z2 = e –2i
e2i – e –2i 1  2 1
 sin2 = = z – 2
2i 2i  z
1 1
= 2iz2 [z4 – 1] and cos = 2z (z2 + 1)
1 4
2 2 (z – 1)
2iz
    d = O
sin2 dz
 
 5 + 4 cos  C 1 iz
0 5 + 4 2z (z2 + 1)
1 z4 – 1
= 2i2 O 2 dz
C
z3 5 + z (z2 + 1)
 
4
1 z –1
= O
2i2 C 3 5z + 2z2 + 2
dz
z
 z 
4
1 z –1
= 2i2 O z2 [2z2 + 5z + 2] dz
C
–1 z4 – 1
= O
2 C z (2z + 1) (z + 2) dz
2 …(1)

1
 The integrand in (1) has poles at z = 0, z = –2, z = – out of which z = 0 is double
2
1
pole and z = –2 simple pole which lies inside |z| = 1 and z = 2 lies outside |z| = 1
d  z4 – 1 
R1 = Residue (at z = 0) = dz z2 z2(2z + 1) (z + 2)
 z = 0
d  z4 – 1 
= dz (2z + 5z + 2)z = 0
2

2 3 4
= (2z + 5z + 2) 24z – (z –2 1) (4z + 5)
 (2z + 5z + 2) z = 0
5
= 4
4
1 z + 1  z –1
R2 = Residue at z = –2 =  2  2


   1
 z 2 z + 2 (z + 2)z = –1
     2

Gigatech Publishing House


Igniting Minds
Engineering Mathematics – III 6.80 Complex Variable
4
=  2z – 1 
2z (z + 2)z = –1
2
1 15
–1 –
16 16 5
= 1 1 = 5 = –4
2 4 –2 + 2
   4
From (1) and by Cauchy’s Residue Theorem
2
sin2 1 z4 – 1
 5 + 4 cos  d =
O –2O 2
z (2z + 1) (z + 2)
dz
0
1 5 5
= – 2 [2i] 4 – 4
 
= 0

Example : 7
2
sin2
Evaluate  5 + 4 sin
d
0

Solution :
dz 1  1 1
Let z = ei, d = z– 2
iz
, sin =
2i  z = 2iz (z – 1)
2
–1  1
4 z – z 
2
sin2 dz
  5 + 4 sin
d = 
O
1 iz
C
0 5 + 4 2iz (z2 – 1)

1 (z2 – 1)2
= 
– 4i O 10iz + 4z2 – 4 dz
C
z3 
 2iz 
1 (z2 – 1)2
=  z2 [5iz + 2z2 – 2] dz
– 4i O
C
i
1 (z2 – 1)2
=  z2 [2z2 + 5iz – 2] dz
–4 O
C

1 (z2 – 1)2
= 
–8 O i
dz …(1)
C
z2 (z + 2i) z + 
 2
i
The integrand in (1) has poles at z = 0, z = – 2i, z = – 2 out of which z = 0 a double pole
i
and z = – 2 simple pole which lies inside |z| = 1 and z = –2i lies outside |z| = 1.

Gigatech Publishing House


Igniting Minds
Engineering Mathematics – III 6.81 Complex Variable
2 2
 R1 = Residue (at z = 0)  d z2  (z – 1) 
= dz  i 
 
z (z + 2i) z + z z = 0
2
   
2 2
d (z – 1)
= dz  
i 

 (z + 2i) z + z z = 0
   
2 2
 d  (z – 1) 
= dz  5 
 z2 + 2 iz – 1 z = 0
   
5iz 5i
z + 2 z – 1 2(z2 – 1) (2z) – (z2 – 1)2 2z + 2 
2
   
=  z2 + 5 zi – 1 2 
  2   z=0

5i
0 – (+1)  2 
  5i
= ( –1)2 = –2

2 2
i  z + i   (z – 1)
R2 = Residue at z = –   2  2

i 
=
 2 
 z (z + 2i) z + 2  z = –i
    2
2 2
=  (z – 1) 
z2 [z + 2i] z = –i
2
2
i – 12
4 
= 2
i  – i + 2i
4   2 
1
– – 1 2 25
 4  16 25  8  25
= 1 3i  = – 3i = 16 – 3i = – 6i
–4 2
  8
25
= 6 i
From (1) and by Cauchy’s Residue Theorem
2
sin2 1 5i 25i 1 –15i + 25i

O d = – 8 (2i) – 2 + 6  = – 8 (2i) 
0
5 + 4 cos     6 
i i
= – (25i – 15i) = – (10i)
24 24
5
=
12

Gigatech Publishing House


Igniting Minds
Engineering Mathematics – III 6.82 Complex Variable

Self-Assessment Exercises 6.4

Evaluate the following integrals :


2
d
E.1 :  2 + cos
Ans. : – 
0
2
d 2
E.2 :  5 – 4cos
Ans. : 3
0
2

 3 + 2cos
d
E.3 : Ans. :
0 5
2
d 5
E.4 :  (5 – 3cos)2
Ans. : 32
0

d 2
E.5 : Ans. :
(2 + cos)2 3 3
2
d 2
E.6 :  1 – 2acos + a2
, 0<a<1 Ans. :
1 – a2
0
2
d
E.7 :  1 + sin2
Ans. :  2
0
2
cos 
E.8 :  5 + 4cos
d Ans. : 6
0
2
2a2
 1 – 2acos + a2  d,
cos2
E.9 : 0<a<1 Ans. : (1 – a2)
0
2
d 2
E.10 :  a + bcos
, a>b>0 Ans. :
a – b2
2
0
2
d 2
E.11 :  97 – 72cos
Ans. :
65
0
2
cos
E.12 :  13 – 12cos
d, Ans. : 0
0
2
d 2
E.13 :  a + bsin
, a>b>0 Ans. :
a 2 – b2
0
2
cos 13
E.14 :  5 – 4cos
d Ans. : 8
0
2
sin2 2
E.15 :  a + bcos
d Ans. : b2 (a – a2 – b2 )
0

Gigatech Publishing House


Igniting Minds
Engineering Mathematics – III 6.83 Complex Variable
2
d 
E.16 :  13 + 5cos
Ans. :
6
0
2
cos3 
E.17 :  5 – 4cos
d Ans. : 12
0
2
cos
E.18 :  3 + sin
d Ans. : 0
0
2
d 
E.19 :   + cos
Ans. : 2
0  –1
2
d 
E.20 :  5 – 3cos
Ans. :
2
0
2
sin2 
E.21 :  5 – 4cos
d Ans. :
4
0
2
cos23 3
E.22 :  5 – 4cos2
d Ans. :
8
0
2 2
sin 
E.23 :  3 + 2cos  d Ans. : 0
0
2
d 2
E.24 :  [1 – 2asin + a2]
, 0<a<1 Ans. :
1 – a2
0
2
cos2 d 
E.25 :  16 – 8 cos + 1
Ans. :
120
0
2

 5 + 4cos  d


sin2
E.26 : Ans. : 0
0
2 2
sin 
 5 + 4sin  d
5
E.27 : Ans. :
0
12
2
d 5
E.28 :  (5 – 3sin)2
Ans. :
32
0
2

 5 + 4cos  d


1 + 2cos
E. 29 : Ans. : 0
0
2 2
sin  – 2cos
  2 + cos  d
2
E. 30 : Ans. :
0 3

Gigatech Publishing House


Igniting Minds
Engineering Mathematics – III 6.84 Complex Variable

Let f(z) be an analytic function of z on two concentric circles c1, c2 with radius r1, r2
respectively with centre at a and also analytic in the annular region R bounded by c1, c2 then at
each point z on R, f(z) can be expressed as a convergent series including positive and negative
powers of (z – a) as below and called as Laurents Series.
f(z) = a0 + a1(z – z0) + a2 (z – z0)2 + a3 (z – z0)3 + + +
+ b1 (z – z0) –1 + b2 (z – z0) –2 + b3 (z – z0) –3 + + +
 

=  n
an (z – z0) +  bn (z – z0)–n
n=0 n=1

= A+B
1
where, an =  f(z) dzn+1 , n = 0, 1, 2, 3, …….
2i C (z – z0)
1

1
bn =  f(z) dzn+1 , n = 1, 2, 3, 4 …….
2i C (z – z0)
2

Fig.: 6.23

Where,
1. A is called as analytic part and B is called Principal Part of Laurents series.
2. If Principal Part B contains terms b1 (z – z0)–1 then x = x0 is called simple pole of f(z) and
if B contains two terms b1 (z – z0)–1 + b2 (z – z0)–2 then z = z0
is called as double pole of f(z).

Illustrative Examples

Example : 1

1
Express f(z) = z (z – 1)2 as a Laurent’s series at the point z = 1.

Solution :
To express the given f(z) as Laurent’s series at z = 1,
We put, z1 = z – 1
z = z1 + 1

Gigatech Publishing House


Igniting Minds
Engineering Mathematics – III 6.85 Complex Variable

1 1 1 –1
 f(z) = 2 = 2 = 2 (1 + z1)
z (– 1) (z1 + 1) z 1 z1
1 2 3
= 2 [1 – z1 + z1 – z1 + ….+]
z1
1 1 2
= 2 –
z + 1 – z1 + z 1 ………
z1 1

1 1 ... z = z – 1
= – + 1 – (z – 1) + (z – 1)2 ….
(z – 1)2 (z – 1) 1

which is valid for |z–1| <1

Example : 2

1
Obtain the Laurent’s series for f(z) = for | < | z | < 2.
(1 – z) (z + 2)
Solution :
We can write ,
1 1 1
(1 – z) (z + 2) = 3 (1 – z) + 3 (z + 2) , by partial fractions …(1)
1 1
Firstly, = – for |z| > 1
1–z (z – 1)
1 1 1 –1
= – = – 1 – 
1 z z
z 1 – z 
 
1 1 1 1
= – z 1 + z + z2 + z3 +++ 
 
1 1 1 1
= – z – z2 – z3 – z4 …….

1
= –  zn …(2)
n=1

Secondly for |z| < 2,


1 1 1 z –1
z+2 = z = 2 1 + 2
2 1 + 2
 
1 z  z  2  z 3 
= 2  2 + 2 – 2 + ……
1 –
1 z z2 z3
= 2  22 + 23  24 +  + 

(–1)n zn
=  2n+1 …(3)
n=0

Gigatech Publishing House


Igniting Minds
Engineering Mathematics – III 6.86 Complex Variable

Put (2) and (3) in (1), we get


1 1 1
f(z) = (1 – z) (z + 2) = 3(1  z) + 3(z + 2)
 
1 1 1 (–1)n zn
= –
3  +
zn 3  2n+1
, which is valid for 1 < |z| < 2.
n=1 n=0

Example : 3

7z – 2
Find the Laurent’s expression of f(z) = (z +1) (z) (z – 2) , 1 < |z+1| < 3.

Solution :
Let z + 1 = t, z = t – 1, and z – 2 = t – 1 – 2 = t – 3
7z – 2 7(t – 1) – 2
 f(z) = =
(z +1) (z) (z – 2) t (t – 1) (z – 2)
7t – 9
= t (t – 1) (t – 3)
A B C 3 1 2
= t +t–1 +t–3 =– t + + , (by Partial Fraction)
t1 t3
3 1 2
= –t + 1 – t
t1 –  31 – 
 t   3 
3 1  1–1 2  t –1
= – t + t 1 – t – 3 1 – 3
   
3 1 1 1 1 2 t t2 t3
= – + 1 + + 2 + 3 ++ – 1 + + + ++
t t t t t  3  3 9 27 
3 1 1 1 2 t t2 t3
= – + + 2 + 3 ++ – 1 + + + ++
 t t t t  3  3 9 27 
2 1 1 2 t t2 t3
= – t + t2 + t3 + + – 3 1 + 3 + 9 + 27 ++
   
2 1 1 2 1 1
= – + + ++ – 1 + (z + 1) + (z + 1)2 ++
z + 1 (z + 1)2 (z + 1)3 3 3 9 
1 < |z + 1| < 3

Example : 4

z2 – 6z – 1
Express f(z) = (z – 1) (z – 3) (z + 2) as a Laurent’s series on the region 3 < (z + 2)| < 5.

Solution :
First resolve f(z) into partial fractions.
z2 – 6z – 1 A B C
f(z) = (z – 1) (z – 3) (z + 2) = z – 1 + z – 3 + z + 2

Gigatech Publishing House


Igniting Minds
Engineering Mathematics – III 6.87 Complex Variable

1 1 1
= + +
z–1 z–3 z+2
1 1 1
= t–3 +t–5 + t Put, z + 2 = t  z = t – 2}
1 1 1
= 3 + t +t
t 1 – t  – 5 1 – 5
   
1 1  3 1 t –1
– 1 –  ……
–1
= + 1–
t t t 5  5
1 1 3 9  1 t t2
= + 1 + + 2 ++ – 1 + + ++
t t t t  5  5 25 
 n  n
1 1 3 1 t
= t +t   t  –5  5
n=0 n=0
 
1 3n tn
= t  tn+1 –  5 n+1
n=0 n=0
 
1 3n (z + 2)n
=
z+2
+  (z + 2)n+1
–  5n+1
, 3 < |z + 2| < 5
n=0 n=0

Example : 5

1
Find Laurent’s series of f(z) = (3 – z) sin z +2 about z = – 2.

Solution :
Let z + 2 = x,  z = x – 2
1  1
f(z) = (3 – z) sin  = (3 – x + 2) sin  
z +2  x
1 (1/x)3 (1/x)5 
= (5 – x)  – + +
x 3! 5! 
1 1 1
= (5 – x) x – 6x3 + 120x5  +  +
 
5–x 5–x 5–x
= x – 6x3 + 120x5 ….
5 5 1 1 1
= x – 1 – 6x3 + 6x2 + 24x5 – 120x4 ….
5 1 5 1 1
= –1+ + 2 – 3 – + ….
x 6x 6x 120x4 24x5
5 1 5 1
= – (1) + z + 2 + 6 (z + 2)2 – 6 (z + 2)3 – 120 (z + 2)4 ….
1 5
= – 1 + 5 (z + 2) –1 + (z + 2) –2 – (z + 2) –3 …, a Laurent’s series.
6 6
Gigatech Publishing House
Igniting Minds
Engineering Mathematics – III 6.88 Complex Variable

Descriptive Questions

Q. 1 Determine the harmonic conjugates of following functions u or v such that f(z) =


u +iv becomes an analytic function.
1. u = log x2+y2
2. u = x sinx coshy – y cosx sinhy
3. v = e–x(x cosy + y siny)
4. u = r3cos3
Q. 2 Find the analytic function f(z) = u+iv if
cosx+sinx–e–y
1. u – v =
2(cosx–coshy)
x–y
2. u + v = ex[cosy + siny] + 2
x + y2
3. Find the image of circle |z| = 3 on XY plane onto the plane W by transformation
w = 4z.

Q. 3 Evaluate  z2 dz, where c is a straight line joining the points (1, 1) and (2, 4).
C

2 x
Q. 4 Evaluate  z̄ dz, along a line y = 2 from (0, 0) to (2, 1).
C

Q. 5 Verify Cauchys Integral Theorem for f(z) = z + 8 over the curve c along sides of a
rectangle whose vertices are (–1, 0), (1, 0), (1, 1), (– 1, 1).
z  dz, where c is |z – 2| = 1 .
Q. 6 Evaluate O2
C z – 3z + 2 2
sin2z
Q. 7 
Evaluate O dz, where c is |z| = 2.
C z + 4
 3
e2z
Q. 8 
Evaluate O
(z – i)
dz, where c is |z–1| + |z+1| = 1.
2

 3 + 2cos
d
Q. 9
0
2
sin2
Q. 10  a + bcos
d
0
2

 5 + 4cos  d


1 + 2cos
Q. 11
0



Gigatech Publishing House


Igniting Minds
Q. 1 (a) Solve any two : (8)
(i) (D2 – 4D + 3)y = x3e2x
Solution :
Auxiliary equation is m2 – 4m + 3 = 0  m = 1, 3
 C.F. = c1ex + c2e3x
1 1
P.I. = D2 - 4D + 3 e2x x4 = e2x (D + 2)2 - 4 (D + 2) + 3 x3
1
= e2x 2 x3 = -e2x [1 + D2]-1 x3 Dx3 = 3x2
D -1
D2x3 = 6x
2x 2 4
= -e [1 – D + D - + …..] x 3
D3x3 = 6
2x 3 2x 3
D4x3 = 0
= -e (x – 6x + 0) = -e (x – 6x)
The complete solution is y = C.F. + P.I.
y = c1 ex + c2 e3x – e2x (x3 – 6x)
d2y dy
(ii) (1 + x)2 + (1 + x) + y = 2sin [log (1 + x)]
dx2 dx
Solution :
Put 1+x = ez  z = log (1 + x) and x = ez – 1
dy d
 (1 + x) dx = Dy where D = dz
d2y
(1 + x)2 dx2 = D (D – 1) y

 D (D – 1)y + Dy + y = 2 sinz
(D2 – D + D + 1) y = 2 sinz
 (D2 + 1) y = 2 sinz
The auxiliary equation is m2 + 1 = 0 m=i
C.F. = c1 cosz + c2 sinz
1 1
P.I. = 2 sinz = 2  z sinz = z  sinz dz
D2 + 1 2D
= -z cosz

Gigatech Publication House


Igniting Minds
Engineering Mathematics - III Q.2 Solved University Question Paper (Dec. 2016)

The general solution is y = C.F. + P.I.


 y = c1 cosz + c2 sinz – z cosz
y = c1 cos [log (1 + x)] + c2 sin [log (1 + x)] - log (1 + x) cos [log (1 + x)]
2
(iii) (D2 – 1) y = 1 + ex using method of variation of parameters.

Solution :
The auxiliary equation is m2 – 1 = 0  m = –1, 1
 C.F. = c1e–x + c2ex and P.I. = uy1 + vy2
Here, y1 = e–x and y2 = ex
–x x
y1 y2  e –x ex = 1 + 2 = 2
 W =  y1 y2  = –e e
xy2 2 e–x 1
Now, u = –  W = –  1 + ex  2 dx = –  ex (1 + ex) dx
1 1 1 + ex – ex
u = –  ex – 1 + ex dx = –  – e–x – 1 + ex  dx
   
x
e
u = –  e–x – 1 + 1 + ex dx = –[– e–x – x + log (1 + ex)]
 
xy1 2 e–x
v =  W =  1 + ex  2 dx = log (1 + ex)

 P.I. = uy1 + vy2


P.I. = (e–x + x – log (1 + ex)) e–x + log (1 + ex) ex
The general solution is y = C.F. + P. I..
y = c1e–x + c2ex + (e–x + x – log (1 + ex)) e–x + log (1 + ex) ex.
(b) Solve using Laplace transforms : (4)
d2y
dx2 + y = t,
given y (0) = 1, y(0) = -2.
Solution : Taking Laplace transform of the given differential equation.
 L{y (t)} + L {y (t)} = L {t}
1
 s2y (s) – sy (0) - y (0) + y (s) = s2
1
(s2 + 1) y (s) = s2 + s – 2
1 s 2
 y (s) = 2 2 + -
s (s + 1) s2 + 1 s2 + 1
1 3 s
= 2- 2 +
s s + 1 s2 + 1
Taking inverse Laplace transform we get

Gigatech Publication House


Igniting Minds
Engineering Mathematics - III Q.3 Solved University Question Paper (Dec. 2016)

1 -1  1   s 
L-1 {y (s)} = L 2  - 3L  2  + L-1  2 
s  s +1 s +1
 y (t) = t – 3 sint + cost
Q. 2 (a) A circuit consists of an inductance L and condenser of capacity C in series. An
e.m.f. Esin nt is applied to it at time t = 0, the initial charge and initial current being
1
zero, find the current flowing in the circuit at any time t for  n. (4)
LC
Solution :
dI d2q
The potential drop across the inductance L is L dt or L dt2 and the potential drop across
q
the capacitance is .
c
d2q q
 By Kirchhoff’s Law, we get L dt2 + c = E sin nt
d2q 2 E 2 1
 dt2 + w q = L sin nt where w = Lc
The auxiliary equation is (D2 + w2) q = 0  D =  iw
C.F. = c1 cosw t + c2 sinw t
1 E E 1 1
P.I. = 2 2 sin nt = 2 2 sin nt where w = n
D +w L Lw -n LC
E
 q = c1 cos wt + c2 sin wt + sin nt …. (i)
L(w 2 - n2)
dq E n cosnt
I = dt = - wc1 sin wt + wc2 cos wt + L w 2 - n2 ….(ii)
Initially q = 0, I = 0, when t = 0 we get
from (i) 0 = c1 + 0  c1 = 0
E n E n
from (ii) 0 = 0 + wc2 + L w2 - n2  c2 = - Lw w2 - n2
E n E n
 I = - w Lw w2 - n2 cos wt + L w2 - n2 Cos nt
En
I = (cosnt - cos wt)
( w2 - n2) L
(b) Solve any one : (4)
e-at - e-bt 
(i) Find : L .
 t 
1 1
Solution : The Laplace transform of L {e-at} = s + a and L {e-bt} = s + b

f (t)
 L t 
 
=  f (s) ds
S

Gigatech Publication House


Igniting Minds
Engineering Mathematics - III Q.4 Solved University Question Paper (Dec. 2016)

e-at - e-bt 
 1 1
 s + a - s + b  ds = [log (s + a) – log (s + b)] s

L   =
 t 
S
 e-at - e-bt  s+b
L   = 0 – log (s + a) + log (s + b) = log
 t  s+a
s+7
(ii) Find : L-1  s2 + 2s + 2 
 
s+7 s+1+6 (s + 1) + 6
Solution : f (s) = s2 + 2s + 2 = s2 + 2s + 1 + 1 = (s + 1)2 + 1
 (s + 1) + 6   s + 6
L-1 {f (s)} = L-1  2  = e-t L-1  2 
 (s + 1) + 1   s + 1
 f (t) = e-t (cost + 6sint)
(c) Find Laplace transform of : (4)
L [sint U (t – 4)].
Solution : The Laplace transform of :
 L {f (t) U (t – a)} = e-as L {f (t + a)}
 L {sint U (t – 4)} = e-4s L {sin (t + 4)}
= e-4s L {sint cos4 + cost sin4}
1 s 
= e-4s cos 4 2 + sin4 2
 s +1 s + 1
Q. 3 (a) Solve the integral equation : (4)

 f(x) cosx dx = e-


0

where  > 0.
Solution : The inverse Fourier cosine transform is
 
2 2
f(x) =
  FC() cosx d =
  e- cosx d
0 0

f(x) =
2  e-
2 (-cosx + xsinx)

 1 + x 0
2 1  2 1 2
f(x) = 0- 2 (-1 + 0) =
 1+x   1+x
(b) Solve any one : (4)
2k
(i) Find z – transform of f(k) = k , k  1.
2
 
2 k
z 2
Solution : z {2k} =  2kz-k =   z  = 2 = z - 2 |z|>2
k=1 k=1 1-z

Gigatech Publication House


Igniting Minds
Engineering Mathematics - III Q.5 Solved University Question Paper (Dec. 2016)

k  
2  1 1 1
 z k 
 
= 2  z (z - 2) dz =  z – 2 – z  dz
Z Z

= [log (z – 2) – log z] z
z 
= -log (z – 2) + log z = log  | z | > 2.
z - 2 
(ii) Find inverse z-transform of :
1
F(z) = (z - a)2 , | z | < a.

Solution : F(z) has a pole of order 2 at z = a


 The residue of F(z) at z = a is
1 d2 - 1  2 1 k - 1
r = lim 2 - 1 (z - a)  2z
za (2 - 1)! dz  (z - a) 
d k–1
= lim [z ] = lim (k – 1) zk – 2
za dz za

= (k – 1) ak – 2
 1 
 z-1   = – (k – 1)ak – 2 k<0
 (z - a)2 

(c) If directional derivative of : (4)


2 2 2
 = ax y + by z + cz x
x-1 y-3 z
at (1, 1, 1) has maximum magnitude 15 in the direction parallel to 2 = -2 = 1 ,
hence find the values of a, b, c.
Solution :  = (2axy + cz2) i + (ax2 + 2byz) j + (by2 + 2czx) k
()(1,1,1) = (2a + c) i + (a + 2b) j + (b + 2c) k.

and u = 2i – 2j + k

 u^ = where |  | = 15
|  |
2i - 2j + k 1
u^ = = (2i – 2j + k)
4+4+1 3
  = |  | u^
2i - 2j + k 
(2a + c)i + (a + 2b) j + (b + 2c) k = 15 
 3 
 2a + c = 10 …. (i) (i) + 2 (ii)  c – 4b = 30 …. (iv)
a + 2b = -10 …. (ii) (iv) + 4 (iii)  8c + 4b = 20 … (v)
b + 2c = 5 …. (iii) from (iv) and (v)
Gigatech Publication House
Igniting Minds
Engineering Mathematics - III Q.6 Solved University Question Paper (Dec. 2016)

we get,
20 55 50
a = , b = - and c =
9 9 9
OR
Q. 4 (a) Attempt any one : (4)
1 n(n - 2)
(i)    r   rn   = rn + 1
  
1 -nr-n-1  n 
Solution :  n =  r-n = r = - n+2 r
r r r
1 n  –n 
   r   n   =   r – n + 2 r   =    n + 1 r 
 r     r  r 
1
  r  rn   = -n   [r-(n + 1) r ] = -n [ r–(n + 1)  r + r – (n + 1)   r ]
  

  
– (n + 1)r– (n + 1) – 1   – (n + 1) 
= -n  r  r +r (3)
 r 
(n + 1) 3 – (n + 1) 3
= – n – rn + 3 r2 + rn + 1 = – n  rn + 1 + rn + 1
   
–n
= rn + 1 (– n – 1 + 3)
n (n – 2)
=
rn + 1
2
(ii) 2 f(r) = f (r) + r f (r).
f (r) 
Solution : We know that  f (r) = r r
f (r) 
 2 f (r) =   ( f (r)) =    r r 
 
f (r)  f (r) 
 2 f (r) =   r + (  r )
r r
r f  (r) r – f  (r) 1  r r

 r r  f (r)
= r2 + r (3)
1 f  (r)    3
= 2  f  (r) - r  r + f (r)
r  r  r
1 f (r)  2 3
= 2  f  (r) - r + f (r)
r  r  r
f  (r) 3
= f (r) – r + r f  (r)
2
 2 f(r) = f  (r) + f  (r).
r

Gigatech Publication House


Igniting Minds
Engineering Mathematics - III Q.7 Solved University Question Paper (Dec. 2016)

(b) Find the values of the constant scalars a, b, c if the vector point function :

V = (x + 2y + az) i + (bx – 3y + z) j + (4x + cy + 2z) k
is irrotational. (4)

Solution : For irrotational field   V = 0
i j k
     
 V =  x y z 
 x + 2y + az bx - 3y + z 4x + cy + 2z 

V = i [c – 1] – j [4 – a] + k [b – 2] = 0
 c–1=0 , 4 – a = 0, b – 2 = 0
 a=4 , b = 2, c = 1.
(c) Obtain f (k), given that :
fk + 2 – 4fk = 0, k  0, f (0) = 0, f (1) = 2 (4)
Solution : Taking Z-Transform of the given equation we get
z {f (k + 2)} – 4z {f (k)} = 0
z f (z) – z2 f (0) – z f (1) – 4f (z)
2
=0
2
(z – 4) f (2) = 2z
2z 2z
 F (z) = z2 – 4 = (z – 2) (z + 2)
2  z z 
F (z) = – Taking inverse Z-transform
4 z – 2 z + 2
1  -1  z  -1  z 
z-1 {F (z)} = z  –z  
2  z -2 z +2
1 k
 f(k) = [2 – (– 2)k] k  0.
2
Q. 5 Attempt any two :
(a) Using Green’s theorem, show that the area bounded by a simple closed curve C is
given by :
1
2  (x dy – y dx)
Hence find the area of the ellipse x = a cos, y = bsin. (6)
Solution : By Green’s theorem we have
F2 F1 
 F1 dx + F2 dy =   –
x y 
 dx dy
C S

1 1
given 2 (x dy – y dx) = 2  (1 + 1) dx dy
S

here F1 = – y F2 = x
Gigatech Publication House
Igniting Minds
Engineering Mathematics - III Q.8 Solved University Question Paper (Dec. 2016)

F1 F2
= –1, = 1
y y
1
2 
=  2 dx dy = Area of Ellipse
S

1
2 C
(x dy – y dx) = ab

(b) Use the divergence theorem to evaluate : (6)



 (y2z2i + z2x2j + x2y2k)  d s
S

where S is the upper half of the sphere x2 + y2 + z2 = 9 above the xoy plane.
  
Solution : By divergence theorem  F  d s =    F dv
S V

Here F = y z i + z2x2j + x2y2k
2 2


    Fdv = 0
V

F = 0
 
  F  n^ ds +  F  n^ ds = 0
S S1

 
 F  n^ ds = –  F  n^ ds Fig. : 1
S S1

for surface s1 : n^ = – k^ ds = dx dy

^
F  nds = – x2y2 dx dy

put x = r cos y = r sin
^
 F  nds = -  x2y2 dx dy x2 + y2 = r2
S S1
dx dy = r dr d
2 3
Limits r  0 3
= –   r5 sin2 cos2 dr d
  0 2
=0 r=0
2 6 3
r
= –   6 0 sin2 cos2 d
=0
/2
(3)6
= – 6 4  sin2 cos2 d
0

Gigatech Publication House


Igniting Minds
Engineering Mathematics - III Q.9 Solved University Question Paper (Dec. 2016)

(3)6 1 1 
= – 6 4422
(3)6   (3)6
= –  =–
6 4 24
  (3)6
  F  ^n ds =
24
S

(c) Verify Stokes theorem for : (7)



F = (y – z + 2) i + (yz + 4) j + xz k
Over the surface x = 0, y = 0, z = 0, x = 2, y = 2
Solution : By Stokes theorem we have
  
 F d r =    F  n^ ds …. (i)
C S

  i 
j k
 
 F =  x y z 
y-z+2 yz + 4 xz 
= i (0 – y) – j (z + 1) + k (0 – 1)
= – y i – (z + 1) j – k
Here, n^ = k^ and ds = dx dy

   F n^ ds = – dx dy
2 2
 2 2
    F n^ ds = –   dx dy = - [x]0 [y]0 = – 4 …. (ii)
S x=0 y=0
 
Now, F d r = (y – z + 2) dx + (yz + 4) dy + xz dz
Since z = 0, dz = 0
 
 F d r = ydx + 4 dy
i) The line integral along the path c1 :
y = 0, dy = 0 x  0, 2
   
 F d r = 0  F d r = 0
C1

ii) The line integral along the path c2 : Fig. : 2


x = 2, dx = 0, y  0, 2
2
   
 F d r = 4 dy   F d r = 
2
4 dy = 4 [y]0 = 8
C2 y=0

Gigatech Publication House


Igniting Minds
Engineering Mathematics - III Q.10 Solved University Question Paper (Dec. 2016)

iii) The line integral along the path c3 :


y = 2, dy = 0, x  2, 0
0
   
F d r = y dx = 2 dx   F d r =  2 dx = 2 [x] 2 = – 4
0

C3 x=2

iv) The line integral along the path c4 :


x = 0, dx = 0 y  2, 0
0
   
F d r = 4 dy   F d r =  4 dy = 4 [y]2 = – 8
0

C4 2

         
  F d r =  F d r +  F d r +  F d r +  F d r
C C1 C2 C3 C4

= 0+8–4–8=–4 …. (iii)
From (ii) and (iii) we get
  
 F d r =    F n^ ds
C S

= -4
Hence the Stokes theorem is verified.
OR
Q. 6 Attempt any two :

(a) Evaluate  F dr where
C

F = (5xy – 6x2) i + (2y – 4x) j
and C is the arc of the curve in the xoy plane, y = x3 from (1, 1) to (2, 8) (6)
Solution :
 
Here F d r = (5xy – 6x2) dx + (2y – 4x) dy
Since y = x3 then dy = 3x2dx and x  1, 2
 
 F d r = (5x4 – 6x2) dx + (2x3 – 4x) 3x2 dx
2
 
 F d r =  [6x5 + 5x4 – 12x3 – 6x2] dx
C x=1
2
= [x6 + x5 – 3x4 – 2x3]1
 
 F d r = 64 + 32 – 48 – 16 – 1 – 1 – 3 + 2 = 35.
C

Gigatech Publication House


Igniting Minds
Engineering Mathematics - III Q.11 Solved University Question Paper (Dec. 2016)

 
(b) Evaluate  F d s where
S

F = yzi + zxj + xyk and S is the part of the surface of the sphere x2 + y2 + z2 = 1
which lies in the first octant. (6)
Solution : By Gauss’s divergence theorem we have
 
 F n^ ds =    F dv
S V
   
F = (yz) + (zx) + (xy)
x y z
= 0+0+0=0

  F n^ ds = 0
S

   
  F n^ ds +  F n^ ds +  F n^ ds +  F ^n ds = 0
S S1 S2 S3

Now consider for surface s1, n^ = - k^ ds = dx dy



F n^ ds = -xy dx dy put x = r cos , y = r sin
 x2 + y2 = 1 dx dy = r dr d
 F n^ ds = - xy dx dy
r  0 1   0 /2
S1 S1

/2 1

 F n^ ds = -   r3 cos sin dr d
S1 =0r=0
sin2 /2
4 1
- 4   2  =-42 =-8
r 1 1 1
=
 0  0   
Similarly we get for s2 and s3.
   
  F n^ ds = -  F n^ ds -  F n^ ds -  F n^ ds
S S1 S2 S3

 1 1 1 3
 F ^n ds = + + = .
8 8 8 8
S

(c) Use Stokes’ theorem to evaluate : (7)



 (4yi + 2zj + 6yk)  d r
C

where C is the curve of intersection of x2 + y2 + z2 = 2z and x = z – 1.

Gigatech Publication House


Igniting Minds
Engineering Mathematics - III Q.12 Solved University Question Paper (Dec. 2016)

Solution :
By Stokes theorem we have
  
 F d r =    F n^ ds
C S

  i j
 
k

Now  F =  x y z 
 4y 2z 6y  Fig. : 3

 F = i (6 – 2) – j (0 – 0) + k (0 – 4) = 4 (i – k)
 i-k
Since  = x – z + 1 = 0 then ^n = =
|  | 2
 (i - k) 4
   F  n^ ds = 4 (i – k)  = (1 + 1) = 4 2
2 2

   F  n^ ds =  4 2 ds = 4 2 (Area of circle r2)
S S

Given circle is a great circle then the radius of sphere and radius of circle is equal.
C (0, 0, 1) and r = 1

    F  n^ ds = 4 2 ( (1)2)
S

= 4 2.
Q. 7 (a) If
–y
v = ,
x2 + y2
find u such that, f(z) = u + iv is analytic function. (4)
Solution :
y v 2xy v y2 - x2
Given : v = – x2 + y2 then = (x2 + y2)2 and = (x2 + y2)2
x y
v u
 By C – R equation = –
x y
u 2xy
 – =
y (x + y2)2
2

2xy put x2 + y2 = 1
 u = –  (x2 + y2)2 dy + f (x) 2y dy = dt
dt x x
u = – x  2 + f (x) = + f (x) = 2 + f (x)
t t x + y2
u (x2 + y2)  1 – x  2x
= + f  (x)
x (x2 + y2)2
Gigatech Publication House
Igniting Minds
Engineering Mathematics - III Q.13 Solved University Question Paper (Dec. 2016)

y – x2
2
= + f  (x)
(x2 + y2)2
u v
By C-R equation =
x y
y2 – x2 y2 – x2
 (x2 + y2)2 = (x2 + y2)2 + f (x)  f (x) = 0  f(x) = C
x
 u = +C
x2 + y2
x y
and f(z) = u + iv = x2 + y2 + i x2 + y2 + C
1
put x = z, y = 0 f(z) = +C
z
(b) Evaluate :
z+4
∮ z2 + 2z + 5 dz,
C
where C is a circle |z – 2i| = 3 / 2. (5)
Solution :
Here z = – (1 + 2i) – (1 – 2i) are simple poles.
3
the point z = – (1 + 2i) lies outside the circle |z – 2i| =
2
its residue r1 = 0
3
and the point z = – (1 – 2i) lies inside the circle |z – 2i| = 2

The residue of f(z) at z = – 1 + 2i is

r2 = lim (z + 1 – 2i) F(z)


z  -1 + 2i

(z + 1 - 2i) (z + 4)
= lim
z  -1 + 2i (z + 1 - 2i) (z + 1 + 2i)
-1 + 2i + 4 3 + 2i
= -1 + 2i + 1 + 2i = 4i

By residue theorem we have  f(z) dz = 2i (r1 + r2)


C

z+4 3 + 2i  
  z2 + 2z + 5 dz = 2i  0 + 4i  = 2 (3 + 2i) .
C

(c) Find the bilinear transformation which maps points 0, –1,  of Z–plane onto – 1,
– (2 + i), i of W–plane. (4)
az + b
Solution : The bilinear transformation W = cz + d …. (i)

Gigatech Publication House


Igniting Minds
Engineering Mathematics - III Q.14 Solved University Question Paper (Dec. 2016)

b
at z = 0, w=–1 –1 =  b=–d
d
z = – 1, w = – (2 + i)
– a +b
– (2 + i) = – c + d
b
a+
z a+0 d
z = , w = i w = d  i = c + 0  a = ci = 2 i
c+
z
 – (2 + i) (– c + d) = – a + b = – a – d = – (ci + d)
d
c (2 + 2i) = (2 + i – 1) d  d = 2c or c=
2
d
2 iz – d iz – 2
From (i) we get W = d = .
z+2
2 z + d

OR
Q. 8 (a) Find the condition satisfied by a, b, c and d under which,
u = ax3 + bx2y + cxy2 + dy3
is harmonic function. (4)
2 2
u u
Solution : For harmonic function + =0 …. (i)
x2 y2
u
= 3ax2 + 2bxy + cy2 (6ax + 2by) + (2cx + 6dy) = 0
x
2u
= 6ax + 2by (6a + 2c) x + (2b + 6d) y = 0
x2
u
= bx2 + 2cxy + 3dy2  6a + 2c = 0 and 2b + 6d = 0
y
2 u
= 2cx + 6dy c = – a, b = – 3d
y2
(b) Evaluate :
2 d
0 5 – 3 cos
using Cauchy’s theorem. (5)
Solution :
1 dz
Here we put z = ei and = e-i d = |z|=1
z iz
1 i -i 1 1
 cos = (e +e ) = z+ 
2 2 z

Gigatech Publication House


Igniting Minds
Engineering Mathematics - III Q.15 Solved University Question Paper (Dec. 2016)

dz
2
d iz 2 dz
 =  3 1 = – 3i  1
z+  C z –  (z – 3)
0
5 – 3cos C 5–
2 z  3
1
the pole z = lies inside the circle | z | = 1.
3
and the pole z = 3 lies outside the circle | z | = 1
 its residuce r1 = 0
1
The residue of f (z) at z = 3 is

1 1 1
r2 = lim  z –  F(z) = lim  z – 
z  1/3  3  z  1/3  3 1
z – (z – 3)
 3
1 3
r1 = =–
1 8
3–3
By residue theorem we get
2
d 2 dz 2
 5 – 3cos
= –
3i C  1
= – 2i (r1 + r2)
3i
0 z – (z – 3)
 3
2 3 
= –  (2i)  –  = .
 3i   8 2
(c) Find the image of straigth line y = x under the transformation : (4)
z–1
W=
z+1
Solution : The given transformation is
z–1 1+W
W = z+1  W (z + 1) = z – 1 z=1–W
1 + u + iv (1 + u) + iv (1 - u) + iv
x + iy = 1 - u - iv = (1 - u) - iv  (1 - u) + iv
[1 – u2 – v2] + i 2v
x + iy = (1 – u)2 + v2
1 – u 2 – v2 2v
 x = y=
(1 – u)2 + v2 (1 – u)2 + v2
Since y = x  2v = 1 – u2 – v2
2 2
u + v + 2v – 1 = 0  (u – 0)2 + (v + 1)2 = 2
C (0, – 1), r = 2



Gigatech Publication House


Igniting Minds
Q. 1 (a) Solve any two : (8)
(i) (D2 + D + 1) y = x sin x
Solution : To find complimentary function C. F.
Auxiliary equation is D2 + D + 1 = 0
b  b2  4ac
 D = 2a
1  1  4 1  i 3
= 2(1) = 2

= 1  i  3
 2 2
x
   3  3 
 C.F. = yc = e 2 c1 cos  2  x + c2 sin  2  x
     
To find P.I. :
1
We know P.I. = y = f(D) x
1
=
D + D + 1x sin x
2

1 2D + 1
= x D2 + D +1 sin x   (D2 + D +1)2 sin x
   
⸪ D2 = –1
1 (2D +1)
= x sin x  sin x
1+ D +1  (1 + D + 1)2
1 2D + 1
= x D (sin x)   D2  sin x
 
2D + 1
= x  sin x dx    sin x ⸪ D2 = 1
 1 
= x ( cos x) + (2D + 1) sin x
y1 =  x cos x + 2 cos x + sin x
 General Solution = Complimentary Function + Particular Integral
G.S. = C.F. + P.I.
x
   3  3 
y=e 2
a cos  2  x + c2 sin  2  x  x cos x + 2 cos x + sin x
     

Gigatech Publication House


Igniting Minds
Engineering Mathematics – III Q.2 Solved University Question Paper (May 2017)
2
dy dy
(ii) Solve (2x + 3)2 2  2 (2x + 3)  12y = 6x …(1)
dx dx
Solution :
This is a Legende’s Linear differential equation.
Put. 2x + 3 = ez
 2x = ez 3
1
 x = 2 ( ez 3) & z = log (2x +3)
d2y
 (2x + 3)2 dx2 = 22 D (D  1) y = 4 (D2  D) y
dy
(2x + 3) dx = 2 Dy

 Equation (1) changes to


1
4 D (D 1) y  2(2Dy) 12y = 6  (ez 3)
2 
( 4D2  4D  4D 12) y = 3 (ez 3)
(4D2  8D 12) y = 3 (ez 3)
3
 (D2  2D 3) y = 4 (ez 3) ….. (2)

This is a L.D.E. with constant coefficients.


To find C.F. (Complimentary Function) :
Auxilary equation is
D2  2D  3 = 0
(D 3) (D + 1) = 0
 D = 3, 1 (Real & distinct roots)
 C.F. = y = c1 e3z + c2 ez , where z = log (2x + 3)
To find Particular Integral :
1
We know P.I. = F(D) F(z)
1 3 (ez  3)
=
D2 2D3 4 
3 1 1
= 4 2 ez  2 3eoz
D  2D  3 D  2D  3 
31 z 1 
= 4 e  3
4 3 
3 ez
= 4  4 + 1
 
z
3 e
= 4 1  4  , where z = log (2x + 3)
 

Gigatech Publication House


Igniting Minds
Engineering Mathematics – III Q.3 Solved University Question Paper (May 2017)

 General Solution = Complimentary Function + Particular Integral


3 ez
y = c1 e3z + c2 ez + 1   , where z = log (2x +3)
4 4
(iii) Solve (D2 + 3D + 2)y = sin ex using method of variation of parameters.
Solution :
To find C.F.
Auxiliary Equation is
D2 + 3D + 2 = 0
(D + 2) (D + 1) = 0
D = 1, 2
 C.F. = y = c1 ex + c2 e2x …(2)
To find P.I.
Treat the parameters c1, c2 as variables
 Let c1 = u, c2 = v
we have y1 = ex , y2 = e2x
 P.I. = y = uy1 + vy2 …. (3)
 y1 y2 
 W =  y y  = y1y2  y2 y1
 1 2
= e x (2 e2x)  (e2x) (  ex)
=  2 e3x + e3x
= e3x
We know,
 y2 x
u =  w dx
e2x sin (ex)
= –    dx
  e 3x 
=  ex sin (ex) dx put t = ex
=  sin t dt dt = ex dx
= ( cost ) =  cos (ex)
yx ex sin (ex)
Secondly v =  w1 dx =   e3x dx
=   e2x sin (ex) dx
=   ex sin (ex) ex dx , put t = ex
=   t sint dt
=  [(t)  sin t dt   (1) (cost) dt ]
( Integrate by parts)

Gigatech Publication House


Igniting Minds
Engineering Mathematics – III Q.4 Solved University Question Paper (May 2017)

=  [t ( cost ) + sin t] =  sin t + t cost


v =  sin (ex) + ex cos (ex)
 P.I. = uy1 + vy2 = [ cos (ex)] (ex) + [sin ex + ex cos ex] (e2x)
 G.S. = C.F. + P.I.
y = c1 e + c2 e  e cos e – e2x sin ex + ex cos ex
x 2x x x

y = c1 ex + c2 e2x – e2x sin (ex)


(b) Solve the following differential equation by using Laplace transform : (4)
d2y dy
+ 2 + y = tet where y(0) = 1, y(0) = 2. …(1)
dt2 dt
Solution :
Given y (0) = 1, y(0) = –2
Taking Laplace transform of both sides of equation (1), we get
 L[ y (t) ] + 2L [ y1 (t) ] + L {y (t)}= L [t et]
1
 [ s2 Y(s)  s y(o)  y1(o)] + 2 [ sY(s)  y(o) ] + Y(s) =  2
s s  s + 1
1
s2 Y(s)  s + 2 + 2 sY(s) 1] + Y(s) = (s +1)2
1
[s2 + 2s +1] Y(s)  s + 2 2 =
(s+1)2
1
(s2 + 2s +1) Y (s) = s + (s + 1)2
s 1
 Y(s) = (s+1)2 + (s+1)4 …. (2)

Taking Inverse Laplace Transform of equation (2), we get


s 1
L1 [Y(s)] = L1 (s+1)2  + L 1 (s+1)4
   
 (s + 1)1  1
y(t) = L1 + L1  
 (s + 1)2  (s + 1)4
1 1 1
= L1 s+1  (s + 1)2  + L1 (s + 1)4
   
1  1  1  1  1  1 
= L s+1  L (s + 1)2 + L (s + 1)4
     
1 1
= et  et L 1  2 + et L1  4
s  s 
3
t
= et  et [t] + et 3!
 
1
= et  tet+ 6 et t3
t3
y = e t 1  t + 
 6
Gigatech Publication House
Igniting Minds
Engineering Mathematics – III Q.5 Solved University Question Paper (May 2017)

Q. 2 (a) An electric circuit consists of an inductance 0.1 henry, a resistance R of 20 ohms


and a condenser of capacitance C of 25 × 106 farads. If the differential equation of
electric circuit is:
d2q dq q
L dt2 + dt + C = 0, …(1)

then find the charge q and current i at any time t given that at t = 0, q = 0.05
columbs, i = 0 (4)
Solution :
Given that : L = 0.1 , R = 20, c = 25 × 106
Equation (1) changes to
d2q dq q
(0.1) 2 + 20 + = 0
dt dt 25 × 106
2
dq dq 10q
+ 200 + = 0
dt2 dt 25 × 106
7
D2 + 200 D + 10  q = 0
 25 
D2 + 200 D + 10000000 
 25  q = 0
[D2 + 200 D + 400000] q = 0 ….. (2)
This is to L.d.e. with constant coefficients.
Auxilliary equation is
D2 + 200 D + 400000 = 0
 200  40000  (400000)4
 D = 2(1)
 200   1560000
= 2
 D =  100  i 100 39
100t
 C.F. = G.S. = q = e [c1 cos (100 39 ) t + c2 sin (100 39 ) t] …. (3)
To find values of c1 & c2 where q = 0.05 = i = 0 at t = 0
For equation (3) 0.05 = e[ c1 cos (o) + c2 (o)]
 c1 = 0.05
Diff. equations (3) with r. to t
dq
i. = dt =  100 e 100t [c1 cos (100 39) t + c2 sin (100 39) t]

+ e 100 t [  c1 sin ( 100 39 t) ( 100 39) + c2 cos (100 39) t (100 39)]
But i = 0 at t = 0
 0 =  100 e [ c1 + o] + e [0 + c2 (100 39)]
0 =  100 c1 + c2 (100 39)
Gigatech Publication House
Igniting Minds
Engineering Mathematics – III Q.6 Solved University Question Paper (May 2017)

c1 0.05
 c2 = =
39 39
0.05
= 6.255 = 0.008
 Required solution of differential equation (1) is
q = e 100t [ 0.05 cos (100 39) t + 0.008 sin (100 39) t]
(b) Solve any one : (4)
(i) Find :
 t sin t 
L dt
0 t 
Solution :
1 sin t
 s L  t  ( ⸪ L.T. of on integral)

1  1 
s   s2 +1 

= ds (  L.T. of division by t rule)
s
1 
= [ tan1s]s
s
1
= s [ tan1   tan1s]
1 
= [  tan1s]
s 2
(ii) Find
3s + 1 
L–1 
 + 1)4
(s
3s 1 
Solution : = L1  +
 (s+1)4 (s+1)4
3(s+11) 1 
= L1  +
 (s+1)4 (s+1)4
3 3 1 
= L1   +
 (s+1)3 (s + 1)4 (s+1)4
3 2
= L1  (s+1)3 – (s+1)4 ⸪ L [ eat f (t)] = [ L f(t)]
  ss+a
1 1 1 tn
= 3 et L1 s3  2 et L1 s4 ⸪L1 sn + 1 =
      n
t2  3
–t  t  3 et t2 2 et t3
= 3 et    2 e  3 = 2  6
 2  
3 1
= et  t2  t3
2 3 
Gigatech Publication House
Igniting Minds
Engineering Mathematics – III Q.7 Solved University Question Paper (May 2017)

(c) Evaluate the following integral using Laplace transform : (4)


 t e3t sin t
0

Solution : We know L [ f(t)] =  e st f (t) dt
0
 
  t e3t sin t at =  e 3t (t sin t) dt
0 0

 =  e st [t sin t] dt , where s = 3
0
= L [ t sin t]
d 1 d
=  ds s2 +1 ⸪ L [t f(t)] =  ds [L (f(t)]
 
1
=  (s2 +1)2 (2s)
 
2s 2(3) 6
= (s2 + 1)2 = (9+1)2 = 100 ( s = 3)
3
=
50
Q. 3 (a) Find inverse sine transform if : (4)
1
Fs () = ea

Solution : We know, by formula of Inverse sine transform,
 
2 2 1
f(x) =
  Fs() sin x d =
   easin x d
0 0
 a
 
2 e 
f (x) =

 sinx d
 
….. (1)
0
Diff both sides w r to x, by using Differentiation under integral sign rule.
 
d 2 e(a)  2  e(a)
f1(x) = dx   
sinx d =  x  sinx d

0   0

 a 
2 e 2
=
  
(cos x) () d =
  e–a cos x d
0 0
a 
2 e 
= 2 [ a cos x + x sin x]
a +x 
2
0

Gigatech Publication House


Igniting Minds
Engineering Mathematics – III Q.8 Solved University Question Paper (May 2017)

2 a 
f (x) =
 a + x 
2 2

On Integration, we set
2 a 
f(x) =  2 2 dx
 a +x 
2 x
f(x) = tan1 + c …. (2)
 a
2
At x = 0, f(o) = tan1 (o) + c


ea
 
2
 c = f(0) =
 
 (0) d = 0 from (i)
0
2 x
 f(x) = tan1  
 a
(b) Attempt any one : (4)
(i) Find z – transform of
2k
f(k) = k! , k  0.

Solution : We know,

Z {f(k)} =  f(k) zk
k=
k  k
2  2
Z  k! 
 
=   k!  z k k0
k=0

(2/z)k (2/z) (2/z)1 (2/z)2 (2/z)3
 =  k!
=
o!
+
1!
+
2!
+
3!
+++
k=0
(2/z) (2/z)2 (2/z)3
= 1+ + + +++
1 2 3

x x x
= e2/z ⸪ ex = 1 + + + ++
 1 2 3 
(ii) Find the inverse z–transform of :
z (z +1)
, | z | > 1.
z22z +1
z(z + 1) z(z + 1)
Solution : Let F(z) = =
z2  2z +1 (z  1)2
F(z) has a multiple pote at z = 1 of order 2
z(z+1) 
 Construct a new function zk1 F(z) = zk1 
 (z1)2
Gigatech Publication House
Igniting Minds
Engineering Mathematics – III Q.9 Solved University Question Paper (May 2017)
k k+1 k
 = z (z+1)2  = z + z2 
 (z1)   (z1) 
k–1
Residue of (z f(x)) 1 d
R =  at double pole z = 1  =
  1 dz
[(z 1)2 zk + 1 F(z)]
z=1

d  z k+1 + z k  
= 0 (z 1)2
dz  (z1)2  z = 1

=  d (zk+1 + zk)
dz z = 1
= [(k +1)zk + k z k 1] z = 1
= (k + 1) (1) k + k (1) k 1
z (z + 1)
 z1  = [ (k + 1) (1)k + k (1) k1]
 (z  1)2 
(c) Find directional derivative of (4)
 = xy2 + yz3 at (1, 1, 1) along the vector i + 2j + 2k
Solution : Let 
a = i + 2j +2k
 
a i + 2j + 2k i + 2j + 2k
 a = = =
| |

a 1+4+4 3
2 3
Given :  = xy + yz
  =  [xy2 + yz3]
i  + j  + k   (xy2 + yz3)
=  x y z

= i [y2 + 0] + j [2xy + z3] + k [ 0 + 3yz2]
= i  j  3k at (1,1,1)

 d.d [] =  . a
i +2j + 2k  1
= (i j3k).  = [(i j3k) . (i +2j + 2k)]
 3  3
(1) 2  6 7
= =
3 3
OR
Q. 4 (a) Attempt any one : (4)
(i) Prove that :
     
  b ×a 2( a  r ) (b × r )
b ×  ( a  log r) = r2  r4
Solution :
 
L.H.S = b ×  ( a   log r)

Gigatech Publication House


Igniting Minds
Engineering Mathematics – III Q.10 Solved University Question Paper (May 2017)

  1 r 
 = b × a   ⸪  f (r) = f  (r) r
 r r 
  r  
r
= b ×   a  r2  ⸪r= r
 
 a r 
= b ×   r2 
 
 r2 (a  r )(a  r ) r2 
= b × 
 r4 
 a r2  (a  r ) 2 r2–2r    
= b ×  ⸪  (a  r ) = a
 r4 
   
 a r2  2 (a  r ) r 
 
×  n2
= b ⸪ rn = n r r
 r4 
     
( b × a r2 )  2 ( a  r ) (b × r )
= r4
     
(b × a ) 2 ( a  r ) (b × r )
= r2  r4
= R.H.S.
     
a × r a 3 (a  r ) r
(ii)  × 3 = 3 +
r r r5
a × r 
Solution : L.H.S. =  ×  3 
 r 
 
=   [ (r 3) ( a × r )]
  –3
=  × [ a × ( r r )]
 –3    –3
= [  . ( r r )] a  ( a  ) r r
–3    –3 
= [ ( 3 + 3) r ] a  ( a ) r r ⸪  rn r = (3 + n) rn
  –3
=  ( a  ) ( r r )
–3     –3 
=  [r ( a  ) r + r ( a  ) r ] ⸪ ( a  ) works liked
–3    –3   
=  [r ( a ) + r ( a   r )] ⸪ ( a  ) r = a
 –3   –5 
=  [a r + r (a  3 r r ) ⸪ rn = n r n2 r

Gigatech Publication House


Igniting Minds
Engineering Mathematics – III Q.11 Solved University Question Paper (May 2017)

 –3    –5
=  [ a r + r ( a  r ) (3 r )]
 –3    
 (a  r ) r 
=  ar 3
 r5 
  
a 3( a  r ) r
=  3+ = R.H.S.
r r5
(b) Show that :
   
F = (6xy + z3) i + (3x2 – z) i + (3xz2  y) k is irrotational. Find Scalar  such that

F =  (4)
 
Solution : We know, that F is irrational if  × F = 0
i j k
 
   
  F = 
x y z 

6xy + z3 3x2z 3xz2  y 
= i[1 – (1)] – j[3z2  3z2] + k[6x – 6x] = 0

 F is important vector function/field.
To find scalar potential function  :

We know d =   d r
  
= F dr Given F = 
= [(6xy + z ) i + (3x2 2) j + ( 3xz2 y) k]  [ i dx + j dy + k dz]
3

d = (6xy + z3) dx + (3x2 z) dy + (3xz2  y) dz


 Integration, we get

 d =  (6xy + z3) dx +  (3x2  z) dy +  (3xz2  y) dz + c


z constant x=0
y z constant
x=0 y=0

x2
 = 6y  2  + z3 x + [ 0  zy] + [ 0  0] + c
 
 = 3yx2 + z3x  zy + c
(c) Obtain f(k), given that :
1 1 k
f(k + 1) + f(k) =   k  0, f(0) = 0
2 2
Solution : Taking Z–Transform of the given equation (1), we get
1   1 
Z{f(k + 1)} + Z2 f(k) = Z 2 k
    

Gigatech Publication House


Igniting Minds
Engineering Mathematics – III Q.12 Solved University Question Paper (May 2017)

1 z
 [z F(z) – z f(0)] + F(z) =
2 1
z2
1 z
(z + 2 ) F(z) = 1  f(o) = 0
z
2
z
F(z) = …..(2)
1 1
(z  2 ) (z + 2 )

Taking Z1 on both of (2), we get


z
Z1 [F(z)] = Z1  1

1
  
 z 2 z + 2
 
z
 f(k) = Z1  1 

1

 z  2 z + 2
 
1 1 
= Z1  1  1 By using Partial Fraction
z  2 z + 2 
 
z z  1 k 1 k
= Z1    f(k) =     
1 1 2  2
z  2 z+ 
 2
Q. 5 Attempt any two :
(a) Verify Green’s theorem in plane for

(xy + y2) dx + x2 dy
C
where C is the boundary of the closed region bounded by
y = x and y = x2. (6)
Solution :

 (xy + y2) dx + x2 dy =  u dx + v dy
C

 u = xy + y2, v = x2
u v
= x + 2y = 2x
y k
The equation y = x, a line through the origin and
y = x2, a parabola symmetric to y – axis
 The closed curve c is as shown in figure from O – M – P
 N – O and the enclosed area is A. To verify Greens Fig. : 1
theorem, means to prove that.

Gigatech Publication House


Igniting Minds
Engineering Mathematics – III Q.13 Solved University Question Paper (May 2017)

 udx + v dy =  v  u  dx dy
A 
C
x y 

 L.H.S. =  udx + v dy =  (xy + y2) dx + x2 dy


C O–M–P–N–O

 
= O–M–P [(xy + y2) dx + x2 dy] + P–N–O [( xy + y2) dx + x2 dy]
y = x2 y=x
dy = 2xdx dy = dx

=  [(x. x2 + x4) dx + x2 2x dx] +  [(x x + x2) dx + x2 dx]


O–M–P P–N–O

=  (x3 + x4+ 2x3) dx +  (2x2 + x2) dx


O–M–P P–N–O

=  (3x3 + x4) dx +  (3x2) dx


O–M–P P–N–O
1 0
x4 x5 1 x3 0
=  (3x3 + x4) dx +  (3x2) dx = 3  4  +  5  + 3 3 
0 1
    0  1

= 3 + 1  (0 + 0) + ( 0  1) = 15 + 4 1 = 19 1 =  1
4 5 20 20 20
1
  u dx + v dy =  20
C

v u 
Secondly, R.H.S. =  k  y  dx dy
A

=  [2x  (x + 2y)] dx dy =  (x  2y) dx dy


y=x
1
y=x  1
=   2 ( x  2y) dx dy =  dx   2 (x  2y) dy 
x=0 y=x x=0 y = x 
1 x 1
2y2 
dx  xy 
x
=   2  y = x2
=  dx [xy  y2]y = x2
x=0 x=0
1 1
=  2 2
dx [ (x x  x )  (x x  x )] = 4
 dx [0  (x3  x4)]
x=0 x=0
1 1
 x5 x4
=  ( x  x ) dx =
4 3

 5 4 0
0

Gigatech Publication House


Igniting Minds
Engineering Mathematics – III Q.14 Solved University Question Paper (May 2017)

v
 u  1 1
 x  y  dx dy = R.H.S. = 5  4
A
45 1
= = 
20 20
(b) Evaluate : (6)

 (xi + yi + z2 k)  d S
S

where S is the curved surface of the cylinder


x2 + y2 = 4
bounded by the planes z = 0 and z = 2.
Solution :
There are total 03 surfaces on the given cylinder
(i) s1 : Plane surface z = 0
(ii) s2 : Plane surface z = 2
(iii) s3 : curved surface of x2 + y2 = 4 , z = 0
We know, Gauss Divergence theorem is as

   
v
(. F ) dv =  ( F  n̄ ) ds
S

Where s = s1 + s2 + s3 Fig. : 2

      
v
(F ) dv =  (F n̄ ) ds +  (F  n̄ ) ds +  (F  n̄ ) ds
S1 S2 S3

       
  (F  n̄ ) ds =
v
( F )dv   (F  n̄ ) ds   (F  n̄ ) ds
S3 S1 S2

= I1 – I2 – I3 …. (*)
For I1 :

But, F =   (xi + yi + z2k) = 1 + 1 + 2z = 2 + 2z = 2 (1 + z)

 I1 =  (F ) dv =  2 ( 1 + z) dx dy dz


v v
2
=   2 (z + 1) dx dy dz
z=0
2
2
z
= 2   2 + z dx dy = 2  4 dx dy
 z = 0
= 8  dx dy = 8 (surface area of cylinder)
= 8 [ 2  r h] = 8 [ 2()(2)(2)] [  r = 2 & h = 2]
Gigatech Publication House
Igniting Minds
Engineering Mathematics – III Q.15 Solved University Question Paper (May 2017)

= 64 
 
For I2 : z = 0 , i.e. on plane XY, n̄ =  k̄ :
  
 F  n̄ = (xi + yj + z2k)  (  k̄ ) =  z2
 
  ( F  n̄ ) ds =   z2 dx dy = 0 (⸪ z = 0 on XY plane)
S3


For I3 : z = 2, i.e. plane ‖al z =0 : n̄ = k
  
 F  k̄ = ( xi + yj + z2k) k̄ = z2
 
  ( F  n̄ ) ds =  z2 dx dy =  4 dx dy = 4  dx dy
S2 S2 S2 S2
2
= 4 [ Area of S2] = 4 [  r ] = 4 ( (4)) = 16
  
 from, (*)  F  ds =  (F n̄ ) ds = I1 – I2 – I3 = 64   (0) – 16  = 48 
S S3


(c) Verify Stoke’s theorem for : F = (2x  y) i yz2j  y2 zk over the surface of
hemisphere x2 + y2 + z2 = 1 above the xoy plane. (7)
Solution :
To verify Strokes theorm, we have to show that –
  
∮ F  dr =  [ ( × F ) n̄ ] ds
C S

Where c is the area x2 + y2 = 1, z = 0


& S is the curved surface of x2 + y2 + z2 = 1 and
s1 is circular area on sphere on XY plane.
Fig. : 3

 L.H.S. = ∮ F dr = ∮ [(2x y) dx  (yz2) dy  y2 = dz]
C C

= ∮ (2x  y) dx  0  0, ⸪z=0
C
Put x = cos
 y = sin
 dxdy==–cos
sin d

 Put cos = t
d
= ∮ (2x  y) dx
C

 –sin d = dt

  = 2  t = 1
=0t=1

Gigatech Publication House


Igniting Minds
Engineering Mathematics – III Q.16 Solved University Question Paper (May 2017)

= ∮ (2 cos   sin ) ( sin  d)


C
2 2
=   2 cos  sin  d +  sin2  d
0 0
/2
= 0+4  sin2  d
0
1 
= 4 2  2 =  (by reduction formula)
 
 
R.H.S. =  [(  × F )  n ds
S
i j k

     
Where, ×F =  x y z  = k̄
 2xy yz2 y2 z 
  
⸫ RHS =   × F  n  ds =  ( k̄ k̄ ) dx dy
S


= dx dy . & n̄ = k
= Area of circle
=  r2 (⸪ r = 1), radius of x2 + y2 =1
= 
  
  ( × F )  n̄ ) ds = ∮ F  dr̄ =  Hence proved.
S C

OR
Q. 6 Attempt any two :
(a) Find the work done in moving a particle from (1, 2, 1) to (3, 1, 4) in a force field

F = (2xy + z3) i + x2 j + 3xz2 k (6)
Solution :
The path between two given points is not defined and hence firstly we should check the

conservative property of given vector field F
i j k
  
  
×F =
x 
y z 

2xy + z3 x2 3xz2 
= I [ 0  0]  j [3z2  3z2] + k [ 2x  2x] = 0
Gigatech Publication House
Igniting Minds
Engineering Mathematics – III Q.17 Solved University Question Paper (May 2017)


 F is irrotational conservative vector field and hence the evaluation of line integral / work
done is independent of path between two given points.
 
 work done =  F . dr
C


= c ( 2xy + z3) i + x2 j + 3xz2 k)  ( idx + jdy + kdz)
(3,1,4)

=  ( 2xy + z3) dx + x2 dy + 3x z2 dz
(121)

  
= (2xy + z3) dx + x2 dy + x=0 3xz2 dz
yz constants z constant x = 0
y=0
2
(3, 1, 4)

= 2y x  + z3 x +0+0
 2 (1, –2, 1)
(314)
= (x2y + z3x)
(121)

= [ (9 + 192)  [2 1] = 201 1 = 200


(b) Prove that :
 12 dV =  12 r̄ . ds̄
v r s r

where S is closed surface enclosing the volume V. Hence evaluate :


 xi + yj2 + zk . ds̄
s r
where S is surface of the sphere
x2 + y2 + z2 = a2 [6]
Solution :
r
Let, 
u = 2 
r2 = r r
1
⸫ 
u =   r–2 r = 2
r
We know, by Gauss Divergence theorem,
 u  n̄  ds =  (   u ) dv
s v

   r2 n̄  ds =  1
s r  v r2 dv

  r2  ds =  12 dv
s r v r
Gigatech Publication House
Igniting Minds
Engineering Mathematics – III Q.18 Solved University Question Paper (May 2017)

 Secondly =  xi +yj2 +zk  ds


s  r 
    
=   r2  ds̄  =   r2 n̄  ds
s r  s r 
=  12 dv by Gauss Divergence theorem
v r
=  12 dx dy dz
v r

=  12 r2 sin  d dr d ⸪ x = r sin  cos 


v r
⸪ y = r sin  sin 
⸪ z = r cos 
2
⸪ dx dy dz = r sin  dr d d
=  sin  d dr d
v
 a 2
=  sin  d  dr  d r = 0 to r = a
0 0 0
 = 0 to  = 
Q = 0 to Q = 2
 a 2
= ( cos ) (r) ()
0 0 0
=  [cos   cos 0] (a  0) (2  0)
= (2) (a) (2) = 4 a 
(c) Verify Stokes theorem for (7)

F = y2i + xyi  xzk
Where, S is the hemisphere :
x2 + y2 + z2 = a2, z  0.
Solution :
To verify Stokes theorem, means to show that
    
∮ F dr =
s
[( × F )  n̄ ] ds
C
Let c be the circle x2 +y2 = a2 , z = 0 and s be the
curved surface of hermisphere and s1 be circular
area on sphere on XOY plane.
 
 L.H.S. = ∮ F  dr Fig. : 4
C

= ∮ (y2 i + xy j  xz k), (i dx + j dy + k dz)


C

Gigatech Publication House


Igniting Minds
Engineering Mathematics – III Q.19 Solved University Question Paper (May 2017)

= ∮ [ y2 dx + xy dy  xz dz]
C

= ∮ y2 dx + xy dy . z = 0
C
(⸪ z = 0, Put x = a cos, y = a sin, dx = – a sin d, dy = a cos d
Put t = cos, dt = – sin d, – dt = sin d,  = 0, t = 1,  = 2, t = 1)
2
=  a2 sin2  (a sin  d) + a3 cos  sin  cos  d
0
 2 3 2

= a   sin  d +  cos2  sin  d
3

 0 0 
/2 1
 
= a3  4  sin3  d +  t2 ( dt)
 0 1 
3
2 8a
=  4a3 3 =  3
 
k

 i 
j

Secondary, ×F =  x y z 
y 2
xy xz 
= i [ 0  0 ]  j [ z o] + k [ y 2y] = zj – 4k
Q. 7 (a) If  + i is complex potential for an electric field (which is analytic) and
y
 =  2xy + 2 ,
x + y2
find a function . (4)
Solution :
y
Let,  =  2xy + …..(1)
x2 +y2
Diff. (1) partially w.r.. to x, (y constant)
 1
= 2y + y (  2 (2x)
x (x + y2)2
 2xy
= 2y  (x2 +y2)2 ….(2)
x
But as [ + i ] is an analytic function, we know, C.R.E. are
 
= …..(3)
x y
 
= …..(4)
y x

Gigatech Publication House


Igniting Minds
Engineering Mathematics – III Q.20 Solved University Question Paper (May 2017)

 From (2) & (3)


  2xy
= =  2y  2
x y (x + y2)2
 2xy
 =  2y  (x2 + y2)2
y
Partially Integrating w.r. to y (keeping x as constant)
2y2 2xy
 =   2    (x2 + y2)2 dy + A(x)
 
2y
=  y2  x  2 2 2 dy + A(x) [Put t = x2 +y2 dt = 2y dy]
(x y )
dt 1
=  y2  x  t2 + A(x) =  y2  x (  t ) + A(x)
1
=  y2 + x (x2+y2) + A(x)
x
  =  y2 + x2 +y2 + A(x) …..(5)

Diff (5) Partially w.r. to x (keeping y constant)


 (x2 + y2) (1)  x (2x)
= 0+ + A (x)
x  (x2 + y2)2 
 y2  x2
= (x2 +y2)2 + A (x) ….. (6)
x
 y2  x2
  = (x2 +y2)2 + A(x) ⸪ by (4)
y
 x 2  y2
 =  A(x) ….. (7)
y (x2 +y2)2
Finally differentiation of (1) w.r. to y partially gives
 (x2 + y2)(1)  y (2y) x2  y2
=  2x +  2  = 2x + (x2 +y2)2 …..(8)
y  (x2 + y2) 
Equating r.h.s. of equation (7) & (8) , we get
x 2  y2 x 2  y2
2 2 2  A (x) =  2x +
(x +y ) (x2 +y2)2
  A (x) = 2x
 A (x) = 2x
x2
A(x) = 2   = x2
2
x
 from (5)  = y2 + 2 + x2
(x + y2)
x
= (x2 y2) + 2 2
(x +y )

Gigatech Publication House


Igniting Minds
Engineering Mathematics – III Q.21 Solved University Question Paper (May 2017)

(b) Evaluate :
z+4
∮ (z + 1) 2 (z + 2)2 dz,
C
1
where ‘C’ is a circle |z + 1| = (5)
2
Solution :
z +4
Let F(z) = (z +1)2 (z +2)2 ….. (1)

 z = 1 is double pole of F(z)


and, z =  2 is also double pole of F(z)
1
The closed curve, c is |z + 1| =
2
1
 | x + iy +1| = 2
1 2
(x +1)2 + (y  0)2 = 2
 
1
which is a circle with centre (1,0) & radius 2 unit.
Fig. : 5
 z = 1 = (1,0) lies within C and z = 2 = (2,0) lies outside c.
Hence, we should find Residue of F(z) only at z = 1

 Residue of  d
R =  F(z) at double pole =  [(z + 1)2 F(z)]
  dz z = –1
z = 1

=  d (z + 1)2 (z + 4) 
dz (z + 1)2 (z + 2)2z = 1

=  d (z + 4)2
dz (z + 2)  z = 1
2 2
= (z + 2) d(z + 4)  (z4+ 4) d(z + 2) 
 (z + 2)  z = 1
2
 (z + 2) (1)  (z + 4) 2(z + 2) 
=
 (z + 2)4  z = 1
 (z + 2)  2 (z + 4)
=
 (z + 2)3  z = 1
(1 + 2)  2 (1 + 4) 16
= = = 5
(1 + 2)3 1

z+4
  (z + 1)2 (z + 2)2 dz = 2 i [R] = 2i(5) = 10i
c

Gigatech Publication House


Igniting Minds
Engineering Mathematics – III Q.22 Solved University Question Paper (May 2017)

(c) Find the bilinear transformation, which maps point 1, 0 , i of z – plane onto the
1
points , 2, 2 (1 + i) of w–plane (4)

Solution : Let the required linear Transformation is


az + b
w = cz +d ...(1)

a z + 1
b 
w = ...(2)
c  z + d
b  b
0+1
We have z = 0 , w = 2, from (ii) , 2 = d
0 + b
 
d 1
 b = 2 ...(3)

We have z =1, w = , from (iii)


c z + d
1 b b
w = a
z+1
b 
 c + d
1 b b
 =
 a + 1
b
  + d
c
b b
0 =
a  + 1
b 
c d
 0 = b + b
   
  c =  d
b b
 c  =   1  = 1 … (4)
b  2 2
1
Lastly z = i, w =  ( 1 + i) , from (ii)
2
a  i + 1
1 b
 2 (1 + i) =
c  i  d
b b

Gigatech Publication House


Igniting Minds
Engineering Mathematics – III Q.23 Solved University Question Paper (May 2017)

a  i + 1 a  i + 1
1 b b
 2 ( 1 + i) = =
1 i  1 1 ( i 1)
2 2 2
1 a  i + 1
 4 (i +1) (i 1) =
b
1 a  i + 1
 4 (i2 1) =
b
1 a  i + 1
 (  1 1) =
4 b
1 a  i + 1
=
2 b
a  i = 1  1 =  1
b 2 2
2
a
  =  1 i i
= 2i = 2
 
b 2i
 a = i ...(5)
b 2
Form (2) , (3), (4) and (5)
i z + 1
2 iz + 2
w = = 
1
 z 1 z  1
2 2
iz + 2
 w = 
z  1
OR
Q. 8 (a) Show the analytic function with constant amplitude is constant. (4)
Solution :
Let, F(z) = u + iv be an analytic function.
v
we know, amplitude [F(z)]= tan1  
u
 Given that amplitudes [F(z) ] = constant (c)
v
 tan1 u = c
 
v
 u = tan c
v
= k …(1)
u
Diff (1) partially w.r. to x (y constant)
v u
u v
x x
v 2 = 0

Gigatech Publication House


Igniting Minds
Engineering Mathematics – III Q.24 Solved University Question Paper (May 2017)

v u
u v = 0 …(2)
x x
Similarly diff (1) partially w. r. to y, (x constant)
We get,
v u
u v
y y
= 0
u2
v u
 u v = 0
y y
u v u v u v
u v = 0 ... (3) ⸪ = & = 
x x x y y x
Multiply (2) by v & (3) by u, subtracting
We get
u
(u2 + v2) = 0
x
u
= 0 ⸪ u2 + v2  0
x
v u v
 Similarly, we can show =0 ; =0 ; =0
x y y
 u = c, v = c
 f(z) = u + iv = constant
(b) Evaluate :
5  5i

 (z + 1) dz,
2 + 4i
along the line joining points (2 + 4i) and (5  5i). (5)
Solution :
Let z = x + iy
 dz = dx + i dy
5  5i (5  5)
  (z +1) dz =  ( x + iy +1) (dx + idy)
2 + 4i (2  4)
(5  5)
=  [ (x +1) + iy] dx + [ (x + 1) + iy] idy
(2  4)
we know, the line passing through (2, 4) & (5, 5) is
5  4
y  4 =   (x  2)
 5 2 
9
y  4 =   (x  2)
 3

Gigatech Publication House


Igniting Minds
Engineering Mathematics – III Q.25 Solved University Question Paper (May 2017)

y4 = 3 (x  2) =  3x + 6
y = 3x + 6 + 4 =  3x + 10
 y =  3x + 10
 dy =  3 dx
 from (*)
5  5i (5  5)

 (z +1) dz =  [ x + 1 + i (10 3x)] dx + [(x +1) + i (10  3x)] i (3dx)


2 + 4i (2  4)
5
=  [ x + 1 + 10i  3xi  3ix  3i + 30  9x] dx
2
5
=  [ 8x + 31 + 7i  6xi] dx
2
2 2
5

=  8 x  + 31x + 7 ix  6i x  
 2 2 2
5
= [ 4x2 + 31x + 7 ix  3i x2]
2
= ( 100 + 155 + 35 i  75i )  ( 16 + 62 + 14i 1 2i)
= 55  40i + 16 62 2i
= 9  42i
(c) Find the image of Hyperbola (4)
x2  y2 = 1,
1
under the transformation w =
z
Solution : Given transformation is
1
w = z
1
 z = w
1
x + iy = u + iv
u  iv
=
(u + iv) (u  iv)
u iv
=
u2 + v2
u  v 
x + iy =  2 +i
u + v2 u2 + v2
Equating R & I parts
u
x = u2 + v2
Gigatech Publication House
Igniting Minds
Engineering Mathematics – III Q.26 Solved University Question Paper (May 2017)

v
y =
u + v2
2

Equation of hyperbola is
x 2  y2 = 1
Hence its transformation on UV plane is
2 2
 2 4 2    2 v 2 = 1
(u + v ) u + v 
2 2
u v
(u2 + v2)2  (u2 + v2)2 = 1

 u 2  v2 = (u2 + v2)2



Gigatech Publication House


Igniting Minds
May 2018
2018 Solved University Questions Paper

Q. 1 (a) Solve any two : (8)


d3 y dy
i) dx3 + 4 dx = sin 2x
Solution : (D3 + 4D) y = sin 2x
AE is D3 + 4D = 0
D(D2 + 4) = 0
D (D − 2i) (D + 2i) = 0
D = 0, ± 2i
CF = c1 + c2 cos 2x + c3 sin 2x
1
PI = D3 +4D sin 2x
1
= sin 2x
− 4D +4D
1
= x 3D 2 +4 sin 2x
1
= x sin 2x
(3))(−4) +4
1
PI = − 8 x sin 2 x

C.S. is y = CF + PI
1
y = ci + c2 sin 2x + c2 sin 2 x − x sin 2x
8
1
ii) (D2 + 9) y = 1+sin 3x by variation of parameters method

Solution : AE is D2 + 9 = 0 ⇒ D = ± 3i
CF = c1 cos 3x + c2 sin 3x
PI = uy1 + vy2 = u cos 3x + v sin 3x
 y1 y2 
W= 
yʹ yʹ 
 1 2

Gigatech Publication House


Igniting Minds
Engineering Mathematics – III Q.2 Solved University Question Paper (May 2018)

 cos 3x sin 3x 
=  
 
−3 sin 3x 3 cos 3x
= 3 (cos2 3x + sin2 3x)
= 3

4 = − ∫
y2 F(x)
W dx
1
sin 3x 1+sin 3x
= − ∫ 3 dx
−1 sin 3x 1 − sin 3x
= ∫
3 1 + sin 3x 1 − sin 3x dx
−1 sin 3x − sin2 3x
= 3 ∫1 − sin2 3x
dx

−1 sin 3x − sin2 3x
= 3 ∫ cos2 3x dx
−1 ( sec 3x tan 3x − tan2 3x) dx
= 3 ∫
−1 (sec 3x tan 3x − sec2 3x + 1) dx
= 3 ∫
= −1 sec 3x tan 3x x
3  3 − 3 +3
− sec 3x tan 3x x
u = 9 + 9 −3

v = ∫ y1 F(x)
W dx
1
cos 3x 1+sin3x
= ∫ 3 dx

= ∫
1 3 cos 3x
9 1+ sin 3x dx
1
v = 9 log ( 1 + sin 3x)
− x cos 3x sin 3 x 1 1
PI = 3 + 9 − 9 + 9 sin3x log(1+sin 3x)

C.S. is y = CF + PI
2
dy dy
iii) (4x + 1)2 dx2 + 2 (4x + 1) dx + y = 2x + 1

Solution : Put 4x + 1 = ez
log (4x +1) = z

Gigatech Publication House


Igniting Minds
Engineering Mathematics – III Q.3 Solved University Question Paper (May 2018)

4x = ez − 1
1 z 1
2x = 2e – 2
1 z 1
2x +1 = 2 e − 2 +1
1 z 1
2x + 1 = 2e +2
d
Use D = dz so that
dy
(4x +1) dx = 4 Dy
d2y
(4x +1) dx2 = (4)2 D (D – 1) y

Given DE ⇒
1 1
16 D (D −1) y + 8 Dy + y = 2 ez + 2
1 1
(16 D2 − 16 D + 8D + 1) y = 2 ez + 2
1 1
(16 D2 − 8 D +1) y = 2 ez + 2 e0z

A E is 16 D2 − 8 D + 1 = 0
1 1
D = 4,4
3
4
CF = (c1 + c2) e
1 1 1
PI = [ ez + 2 e0z]
16D2 − 8D +1 2
1 1 z
= 2 16D2 − 8D +1 e
1 1
+2 e0z
16D − 8D +1
2

1 1 z 1 1 0z
= 2 16 − 8 +1 e + 2 0−0+1 e
1 z 1
PI = 18 e + 2
C.S. is y = C F + PI
z
4 1 z 1
y = (c1 + c2 z) e +
18 e + 2
1
4 1 1
y = [c1 + c2 log (4x +1) ] (4x +1) + (4x +1) +
18 2
Gigatech Publication House
Igniting Minds
Engineering Mathematics – III Q.4 Solved University Question Paper (May 2018)

(b) Solve by Laplace Transform method (4)


d2y
dt2 + 9y (t) = 18t
π
With y(0) = 0 , y 2 = 0
d2y
Solution : L  dt2  + 9 L [y (t)] = 18 L [t]
 
18
⇒ [s2 Y(s) − s [y′ (0) − y′ (0)] + 9 Y(s) = s2
As y′(0) is not known let y′(0) = A
18
[ s2 Y (s) − 0 − A] + 9 Y(s) = s2
18
(s2 + 9) Y(s) = A + s2
A 18
Y(s) = s2 + 9 + s2 (s2 +9)
A 18 + 2s2 −2s2
= 2
s +9 + s2 (s2 + 9)
A 2 2(s2 + 9)
= − +
s2 + g s2 + 9 s2(s2 + 9)
A −2 2
Y(s) = s2 + 9 + s2

y(t) = A −2 sin 3t + 2t


 3 
To find value of A, put
π
t = 2 and obtain
π A −2
y 2
3π 2π
= 3 sin 2 + 2
 
0 = A −2 (−1) + π
 3 
A −2
3 = π

y = π sin 3t + 2t
Q. 2 (a) An inductor of 0.5 henry is connected in series with resistor of 6 ohms. A capacitor
of 0.02 farad and generator having alternative voltage given by 24 sin 10t (t > 0)
with a switch K. Forming a differential equation find the current and charge at any
time t if charge is zero when switch is closed at t = 0. (4)
Solution : By Applying Kirchoff’s Voltage law to given circuit we get

Gigatech Publication House


Igniting Minds
Engineering Mathematics – III Q.5 Solved University Question Paper (May 2018)

dI Q
6I + 0.5 dt + 0.02 = 24 sin 10t
d2Q dQ
0.5 dt2 + 6 dt + 50 Q = 24 sin 10t
d2Q dQ
dt2 + 12 dt + 100 Q = 48 sin 10t

Given that Q = 0 , I = 0 at t = 0
d
Use D ≡ dt
(D2 + 12 D + 100) Q = 48 sin 10t
A.E. is D2 + 12D + 100 = 0
D = − 6 ± 8i
CF = e− 6t [ A cos 8t + B sin 8t]
1
PI = D2 + 12 D + 100 48 sin 10t
1
= 48 sin 10t
−100 + 12D + 100
4 4
= D sin 10t = − 10 cos 10t
2
PI = − 5 cos 10t

C.S. is
Q = CF + PI
2
Q = e −6t [A cos 8t + B sin 8t] − 5 cos 10t …. (1)
dQ −6t −6t
dt = −6e [ A cos 8t + B sin 8t] + e [−8A sin 8t + 8B cos 8t]+ 4 sin 10t ….(2)
2 2
Q = 0 at t = 0 so (1) ⇒ 0 = A + 0 − 5 ⇒ A = 5
dQ
dt = 0 at t=0 so …, (2) ⇒

0 = −6 [A + 0] + [ 0 + 8B] + 0
2
0 = −6A + 8B But A = 5
3
So we get B = 10

= e− 6t 5 cos 8t + 10 sin 8t − 5 cos 10 t


2 3 2
Q
 
I = − 6 e−6t  cos 8t + sin 8t + e−6t − sin 8t + cos 8t +4 sin 10t
2 3 16 24
5 10   5 10 

Gigatech Publication House


Igniting Minds
Engineering Mathematics – III Q.6 Solved University Question Paper (May 2018)

b) Solve any one: (4)

∫0e
t −4t
1) Find L [ t sin 3t dt]

3
Solution : L [sin 3t] = s2 +9
3
L [e−4t sin 3t] = (s + 4)2 + 9
t
1 3
L[ ∫ e − 4t sin 3t dt ] = s s2 + 8s+25
0
t
− ds s(s2 + 8s + 25)
d 3
L[t ∫e − 4t
sin 3t dt ] =
 
0

− ds s3 + 8s2 + 25s


d 3
=
 
(s + 8s + 25s) (0) − (3s2 + 16s + 25) (3)
3 2
= −
 (s3 + 8s2 + 25s)2 
9 s2 + 48s + 75
=
(s3 + 8s2 + 25s)2

Find L −1 (s2 +s +1)2


2s +1
2) (4)
 
L−1 (s2 +s +1)2
2s +1
Solution : Let = f(t)
 
∞ 2s +1 
L ∫ (s2 +s +1)2 ds
−1 f(t)
=
  t
s 
 ∞
L−1− (s2 +s +1) 
1 f(t)
=
  t
 s

L−1(s2 +s +1)
1 f(t)
=
  t

L−1 
1 f(t)
32
=
1 2

(s + 2) +  2  
2
t
  
t
−2 2 3 f(t)
e sin 2 t = t
3
t
2t −2 3
f(t) = e sin t
3 2

Gigatech Publication House


Igniting Minds
Engineering Mathematics – III Q.7 Solved University Question Paper (May 2018)

(c) Find Laplace transform of (1 + 2t − 3t2 + 4t3) U (t − 2) (4)


Solution : L [ 1 + 2t − 3t2 + 4t3) U(t −2)]
= L [f(t) U(t − 2)]
= e−2s L [ f( t +2)]
= e−2s L [ 1 + 2(t +2) − 3 ( t + 2)2 + 4 ( t +2)3]
= e−2s L [ 25 + 38t + 21 t2 + 4t3]

= e− 2s  + 2 + 3 + 4 
25 38 42 24
s s s s 
 x 0≤x≤1
Q. 3 (a) Find Fourier sine transform of f(x) =  2 −x 1 ≤ x ≤ 2 (4)
 0 x >2

Solution : Fs (λ) = ∫ f(u) sin λu du
0
1 2 ∞
= ∫ u sin λ u du + ∫ ( 2 − u) sin λ u du + ∫ (0) sin λu du
0 1 2
1 2
 − cos λu − sin λu  − cos λu  − sin λu
= (u) 
  − (1)  λ2  + (2 − u)  λ  − (−1)  λ2 
  
  λ   0     1
− cos λ sin λ − sin 2λ cosλ sin λ
= + 2 + + + 2 
 λ λ   λ2 λ λ 
2 sin λ − sin 2 λ
=
λ2
(b) Attempt any one : (4)
i) Find z – transform of f(k) = (k + 1) (k + 2) 2k , k ≥ 0.
Solution :
z
Z {2k} = = (1 − 2 z−1) −1
z−2
d
Z { k 2k} = −z dz [ (1 −2 z−1)

= − z [− (1− 2 z−1)−2 (2 z−2)]


= 2 z −1 ( 1 − 2 z−1) −2
k
Z{ (k +1) 2 } = Z {k2k}+ Z {2k}
= 2 z −1 (1 − 2z−1)−2 + (1 −2z−1)−1
= (2 z−1 + 1 −2z−1) ( 1 − 2 z −1)−2
= ( 1 − 2 z −1)−2
d
Z {k (k +1)2 k} = −z dz ( 1 −2z −1) −2

Gigatech Publication House


Igniting Minds
Engineering Mathematics – III Q.8 Solved University Question Paper (May 2018)

= −z [ −2(1−2z−1)−3 (2z −2)]


= 4 z −1 (1 − 2z −1) −3
Z {(k +1) (k +2)2k}
= Z {k (k +1) 2k} + Z {2 (k+1)2k}
= 4 z−1 (1 −2z−1)−3 + 2 (1 −2 z−1)−2
= (1 − 2z−1)−3 [4z−1 −42 (1 −2z−1)]
= 2 (1 − 2 z−1)−3
(ii) Show that (4)
Z−1   = {xk) for |z| > 1
1

z −  z− 
1 1 2
 2  3
Where xk = 6 2
1 k −1 1k−1
− 3 k≥1
    
1 6 6
Solution : X (z) = = −
z −1 z − 1 z −1 z −1
 2  3 2 3

|z| > 2 ⇒ 2z < 1 ⇒ 3z < 1


1 1 1
   
6 6
X(z) = −
z1 −2z z1 −3z
1 1
   
= z 1 + 2z + 2z + …. − z 1 + 3z + 3z + …. 
6 1 1 2 6 1 1 2
       
= 6  z + 2z2 + 22z3 + ---- + k −1 k 
1 1 1 1
 2 z 
− 6 z + 3z2 + 32z3 + …. + k −1 k + …. 
1 1 1 1
 3 z 
−1 −1
{ xk} = { 6 [ 2 − 3
1 k 1 k
]},
   
k ≥ 1
(c) Find directional derivative of φ = xy2 + yz3 at (2, −1,1) along the line
2(x −2) = (y + 1) = (z −1). (4)
Solution :
∂φ − ∂φ − ∂φ −
∇φ = i + j + k
∂x ∂y ∂z
− −
∇φ = y2 i + (2xy + z3) j + 3 yz2 k
− − −
(∇φ) (2,−1,1) = i − 3 j − 3 k
Given line is

Gigatech Publication House


Igniting Minds
Engineering Mathematics – III Q.9 Solved University Question Paper (May 2018)

x −2 y + 1 z −1
1 = 2 = 2
So given direction is
− − − −
a = i +2 j +2 k

|a | = 1+4+4=3

∧ a ∧ 1 − −
a = ⇒ a = 3 ( i + 2j + 2 k )

|a |

(Required Directional Derivative) = (∇ φ) . a
(2,−1,1)
1
= 3 [ (1) (1) + (−3) (2) + (−3) (2)]
−11
=
3
Q.4 (a) Prove any one : (4)
− −
−a ⋅∇  − 1  3(−a.‒r ) (b .‒r ) (−a . b)
i) b .∇  r  = −
   r 5
r 3

−1
1 r2
Solution : ∇ r = r ‒r
1
∇ r = − r−3 ‒r
− 1 −
b. ∇ r = − r−3 (b.‒r )
− −
∇ b ⋅ ∇ r  = ∇ [ (−r −3) (b⋅‒r )
1
 
− −
= [∇ (−r−3)] ( b⋅‒r ) − r −3 ∇ (b⋅‒r )
− −
= [3 r −5 ‒r ] (b⋅‒r ) − r−3 b
− − 1
a⋅⋅ ∇  b ⋅∇  =

3r − 5 (b ⋅ ‒r ) (−
a⋅⋅‒r ) − r −3 (−

a⋅⋅b)
 r 
which is required.
4
ii) ∇4 er = er + r er
4
Solution : We know ∇4 f (r) = f ′(iv) (r) + r f ′′′ (r)

Take f (r) = er

Gigatech Publication House


Igniting Minds
Engineering Mathematics – III Q.10 Solved University Question Paper (May 2018)

f ′ (r) = f ′′ (r) = f ′′′ (r) = f(iv) (r) = er


4
Use in above ∇4 er = er + er
r
− − − −
(b) Show that F = (6xy + z3) i + (3x2 – z) j + (3xz2 – y) k is irrotational and find φ

such that F = ∇φ. (4)
− − −
 i j k 
Solution :

∇×F =
 ∂ ∂ ∂ 
 ∂x ∂y ∂z 
 6xy + z3 3x2−z 3xz2 − y 
− − −
= i (– 1 + 1) + j (3z2 – 3z2) + k (6x – 6x)
= 0

Thus F is irrational dφ = ∇φ ⋅ d‒r

= F⋅ d‒r
= (6xy + z3) dx + (3x2 – z) dy + (3 × 22 −y) dz
= (6xy dx + 3x2 dy) + (z3 dx + 3xz2 dz) – (z dy + y dz)
dφ = d (3x2 y) + d (z3 x) – d (yz)
φ = 3x2 y + z3 x – yz + c

(c) Solve yk – 6 yk – 1 + 6 yk – 2 = 2 k ≥ 0.


5 1 1 k
(4)
 
 1 k
Solution : Z {yk} – 6 Z {yk – 1} + 6 Z {yk – 2} = Z 2  for k ≥ 0
5 1
  
5 1 z
Y (z) – 6 z– 1 Y (z) + 6 z– 2 Y (z) = 1
z– 2

⇒ 1 – 5 z– 1 + 1 z– 2 Y (z) = z
 6 6  1
z– 2

z2 – 5 z + 1
 6 6 z
  Y (z) =
 z2
 1
z– 2
z3
Y (z) =
z – 1  2 5 1
 2 z – 6 z + 6
z2
Y (z) =
z – 1 z – 12
 3  2

Gigatech Publication House


Igniting Minds
Engineering Mathematics – III Q.11 Solved University Question Paper (May 2018)

3
Y (z) 4 3 2
= 1 – 1+
z – 2 z – 2 
z 1 2
z– 3
 
z z 3 z
Y(z) = 4 – 3 +
1 2 
z – 2
1 1 2
z– 3 z– 2
 
y(k) = 1k – 3 1k + 3 k 1k – 1 k ≥ 0.
3 2 2 2
Q. 5 Attempt any two :
− ‒ − − −
(a) Evaluate ∫ F⋅ d r for F = (2x + y) i + (3y – x) j and C is the straight line joining
C
(0, 0) and (3, 2). (6)
Solution : The equation of straight line joining (0, 0) and (3, 2) is
x– 0 y– 0
3– 0 = 2– 0 =t
⇒ x = 3t, y = 2t
Limit of t (for C) are
t = 0 to t = 1
dx = 3dt, dy = 2dt
− ‒
F⋅ d r = F1 dx + F2 dy + F3 dz
= (2x + y) dx + (3y – x) dy + 0
= (6t + 2t) (3dt) + (6t – 3t) (2dt)
− ‒
F⋅ d r = 30 t dt
1 1
− ‒ 2
= ∫ 30 t dt =  2  =  2 – 0 = 15
30t 30
∫ F⋅ d r
   
C 0 0

(b) Apply Stokes Theorem to evaluate (7)

∫ (4y dx + 2z dy + 6y dz, where C is the curve of intersection of x2 + y2 + z2 = 6z


C
and z = x + 3.
− − − −
Solution : Take F = 4y i + 2z j + 6y k
and apply Stoke’s Theorem
− ‒ −
∫ F⋅ d r = ∫∫ (∇ × F) ⋅ n^ d−s …(1)
C S

where S is the surface of circle x2 + y2 + z2 = 6z , z = x + 3 n^ is a unit normal to the plane


Gigatech Publication House
Igniting Minds
Engineering Mathematics – III Q.12 Solved University Question Paper (May 2018)

x–z+3 = 0
Let φ = x–z+3
∂φ − ∂φ − ∂φ −
∇φ = i+ j+ k
∂x ∂y ∂z
− − −
∇φ = i +0 j +k
− −
∇φ i– k
n^ = =
|∇φ| 2
− − −
 i j k 

∇×F =
 
∂ ∂ ∂ = 4−i – 4− k
 ∂x ∂y ∂z 
 4y 2z 6y 
−i – −k 
(∇ × F−) ⋅ n^ = ( − −
)
4 i – 4k ⋅ 
 2 

1 8
= (4 + 4) = =4 2
2 2

∫∫ (∇ × F) ⋅ n^ ds = ∫∫ 4 2 ds
S S

= 4 2 (area of circle) = 4 2 (9π)


= 36π 2 ⋅.⋅ circle is a great circle (1) ⇒

∫ (4y dx + 2z dy + 6y dz) = π 2
36π
C

‒r ⋅ n^ ds over the surface of a sphere of radius 1 with centre at the


(c) Evaluate ∫∫
S
origin. (6)
Solution: S is a closed surface enclosing a volume say V.
By Gauss divergence Theorem

∫∫ F ⋅ n^ ds = ∫∫∫ (∇ ⋅ F‒) dV
S V

− −
Take F = r
− −
∫∫ r ⋅ n^ dS = ∫∫∫( ∇ ⋅ r ) d v = ∫∫∫ 3 dV
S V V

4
= 3 ∫∫∫ dV = 3 3 π (radius)3 = 4π
π
V

OR
Gigatech Publication House
Igniting Minds
Engineering Mathematics – III Q.13 Solved University Question Paper (May 2018)

Q. 6 Attempt any two : (6)

(a) Using Green’s Theorem evaluate


∫ − ‒
F⋅ d r where
C
− − −
F = (2x – cos y) i + x (4 + sin y) j
where c is the ellipse
x2 y2
a2 + b2 = 1, z = 0
Solution : Take P = 2x – cos y
Q = x (4 + sin y)
By Green’s Theorem
∂Q ∂P
∫ (P dx + Q dy) = ∫∫  ∂x – ∂y 
dx dy
C R

∫ (P dx + Q dy) = ∫∫ (4 + sin y – sin y) dx dy


C R

= ∫∫ 4 dx dy = 4 ∫∫ dx dy
R R

= 4 (area of ellipse) = 4 (π ab) = 4 π ab


− − − − −
(b) Evaluate ∫∫ ∇ × F ⋅ d−
( )
S for F = y i + z j + x k where S is the surface of paraboloid
S
z = 1 – x2 – y2 above the XOY plane. (7)
− − −
i j k  
Solution : ∇×F =
− ∂ 
∂ ∂ = – −i – −j – − k

∂x ∂y ∂z 
y z x  
Boundary of given surface S is the circle which is also the boundary of plane surface S1 of the
circle x2 + y2 = 1.

∫∫ (∇ × F) ⋅ d −S = ∫∫ (∇ × F−) ⋅ n^ d −S
S S1


For S1, n^ = k

(∇ × F−) ⋅ n^ = – 1

∫∫ (∇ × F) ⋅ d−s = ∫∫ (– 1) dx dy
S S1

= – (Area of circle x2 + y2 = 1)
= − π (radius)2 = – π
Gigatech Publication House
Igniting Minds
Engineering Mathematics – III Q.14 Solved University Question Paper (May 2018)

− −
(c) Use Gauss divergence Theorem to evaluate ∫∫ F⋅ dS over the cylindrical region
S
− − − −
bounded by x2 + y2 = 4 , z = 0, z = α where F = x i + y j + z2 k. (6)
2 2
Solution : S1 is the plane surface of the circle z = 0, x + y = 4 and S2 is the plane surface of
the circle
x2 + y2 = 4 ; z = α
S1 , S2 and S together become a closed surface and enclose the volume say V bounded by the
cylinder.
By Gauss divergence Theorem

∫∫ F⋅ d−S + ∫∫ −F⋅ d−S + ∫∫ −F⋅ d−S
S S1 S2


∫∫∫ (∇⋅F) dV …(1)
V

− − − −
Where F = x i + y j + z2 k

∇⋅F = 1 + 1 + 2z

∇⋅F = 2 + 2z

[RHS of (1)] = ∫∫∫ (2 + 2z) dx dy dz


V
z=α 2
α
(1 + z) dz dx dy = 2 ∫∫ z + 2  dx dy
z
= 2 ∫∫ ∫
z=0
 0

α2 α2
= 2 α + 2  ∫∫ dx dy = 2 α + 2  (4π)
 S 1
 
α 2
= 8π α + 2  …(2)
 

For S1 , n^ = −k
− − − ^ −
F ⋅ dS = F ⋅ n dS
− −
= F ⋅(– k) dx dy = – z2 dx dy

∫∫ −F ⋅ d−S = ∫∫ – z2 dx dy
S1 S1

= 0 (⋅.⋅ z = 0 for S1) …(3)



For S2, n^ = k

Gigatech Publication House


Igniting Minds
Engineering Mathematics – III Q.15 Solved University Question Paper (May 2018)

− ^
F ⋅ndS = z2 dx dy

∫∫ −F ⋅ d−S = ∫∫ z2 dx dy = α2 ∫∫ dx dy
S2 S2 S2

= α 4π
2
…(4)
Use (2), (3) and (4) in (1)
− α2
∫∫ F ⋅ d−S + 0 + 4π α2 = 8π α + 2 
S
 
− α2
∫∫ F ⋅ d−S = 8π α + 2  – 4π α2
S
 
= πα

Q. 7 (a) If v = sinh x cos y find u such that u + iv is analytic function. (4)
∂v
Solution : = cosh x cos y
∂x
∂v
= – sinh x sin y
∂y
Let f (z) = u + iv be analytic
∂u ∂u
du = dx + dy
∂x ∂y
∂v ∂v
= dx – dy (⋅.⋅ CR equation)
∂y ∂x
du = – sinh x siny dx – cosh x cos y dy

u = ∫ M dx + ∫ (Term of N not containing x) dy + c


y = const

= ∫ – sinh x sin y dx + c1 + c
y = const
u = – cosh x sin y + c2
1+z
(b) Evaluate o∫ z (z– 2) dz where c is the circle |z| = 1. (4)

Solution : From Cauchy’s integral formula


1 o∫ f (z)
f (a) = dz …(1)
2πi c z – a
1+z
Take f (z) = z– 2
Which is analytic within and on circle |z| = 1 , a = 0 is present in circle |z| = 1.
Put value of f (z) and a in equation (1)

Gigatech Publication House


Igniting Minds
Engineering Mathematics – III Q.16 Solved University Question Paper (May 2018)

1+z
1 o z– 2
f (0) = ∫
2πi z – 0
dz
1+0 1
f (0) = 0– 2 =–2
1 1 o∫ 1+z
−2 = z (z – 2) dz
2πi C

⇒ o∫ 1 + z dz = – πi
z (z – 2)
(c) Find the bilinear transformation which maps points 1, i, – 1 of z plane into i, 0 – i
of w– plane. (5)
Solution : Required bilinear transformation be
a + bz
w = c + dz …(1)
When z = 1, w = i,
a+b
(1) ⇒ i = c+d
Or a + b = ic + id …(2)
When z = i , w = 0 , (1) ⇒
a + ib = 0 or a = − ib …(3)
When z = – 1 , w = – i , (1) ⇒
a– b
i = c – d ⇒ − i c + id = a – b …(4)
Using (2) , (3) and (4)
we get
i– z
w = i+z
1 + iz
or w =
1 – iz
OR
Q. 8 (a) Find ‘a’ such that the function f(z) = r2 cos 2θ + ir2 sin (aθ) is an analytic function.
Solution : Compare given f (z) with
f(z) = u + iv we get
u = r2 cos 2θ , v = r2 sin aθ
∂u ∂v
= 2r cos 2θ , = 2r sin aθ
∂r ∂r
∂u ∂v
= – 2 r2 sin 2θ, = a r2 cos aθ
∂θ ∂θ
CR equations in polar form are
∂v ∂u ∂u ∂v
= r and = – r
∂θ ∂r ∂θ ∂r
Gigatech Publication House
Igniting Minds
Engineering Mathematics – III Q.17 Solved University Question Paper ((May
May 2018)
2018

ar2 cos aθ = 2r2 cos 22θ


and – 2r2 sin 2θ = – 2r2 sin aaθ
⇒ a = 2

(b) Evaluate ∫ z (zz + 3)


o 15z + 9
C 3 dz where C is the circle |z – 1| = 3. (4)

Solution :
Here z = 0 is present in c while z = – 3 is present outside C.
By Cauchy’s integral formula
1 o∫ f (z)
f (a) = dz …(1)
2πi C (zz – a)
Take a = 0 and
15z + 9
f(z) = z+3
We observe that f(z) is analytic within
wit in and on C. a = 0 is in C.
0+9
f(0) = 0 + 3
f(0) = 3
Putt value of a , f(z), f(0) in (1)
15z + 9
1 o∫ z + 3
3 = dz
2πi C z – 0
o∫ 15z + 9
⇒ ( + 3) dz = 6πi
C z (z Fig. : 1

i– z
(c) Show that under the transformation w = i + z , X – axis in z − plane is mapped onto
the circle |w| = 1. (5)
i– z
Solution : w = i+z …(1)
⇒ w (i + z) = i – z
wi + wz = i – z
wz + z = i – wi
z (w + 1) = i (1 – w)
1– w
z = i 1+w …(2)
Put z = x + iy and w = u + iv
1 – u – iv
x + iy = i 1 + u + iv
(1 – u)) – iv
= i (1 + u)) + iv

= i (11 + u) + iv (1


(1 – uu)−iv (1 u – iv
1 + u)
   1 + u – iv
u)
Gigatech Publication House
Igniting Minds
Engineering Mathematics – III Q.18 Solved University Question Paper (May 2018)

{(1 – u) (1 + u) – iv2} – i {v (1 + u) + v (1 – u)}


x + iy = i 
 (1 + u)2 + v2 
Equating imaginary parts, we get
1 – u2 – v2
y = (1 + u)2 + v2
Map of X– axis i.e. y = 0 is
u2 + v2 = 1 or |w| = 1

Gigatech Publication House


Igniting Minds
Notes

___________________________________________________
___________________________________________________
___________________________________________________
___________________________________________________
___________________________________________________
___________________________________________________
___________________________________________________
___________________________________________________
___________________________________________________
___________________________________________________
___________________________________________________
___________________________________________________
___________________________________________________
___________________________________________________
___________________________________________________
___________________________________________________
___________________________________________________
___________________________________________________
___________________________________________________
___________________________________________________
___________________________________________________
___________________________________________________
___________________________________________________

You might also like